Sie sind auf Seite 1von 377

Page 1 of 377

Table of Contents
RICARDO C. VALMONTE AND UNION OF LAWYERS AND ADVOCATES FOR PEOPLES’S RIGHTS (ULAP),
Petitioners, v. GEN.RENATO DE VILLA AND NATIONAL CAPITAL REGION DISTRICT COMMAND,
Respondents. ................................................................................................................................................ 4
PEOPLE OF THE PHILIPPINES, Plaintiff-Appellee, v. JULIAN DEEN ESCAÑO, VIRGILIO TOME USANA and
JERRY CASABAAN LOPEZ, Accused. ............................................................................................................... 9
MICROSOFT CORPORATION and LOTUS DEVELOPMENT CORPORATION, petitioners, vs. MAXICORP, INC.,
respondent. ................................................................................................................................................. 17
SONY MUSIC ENTERTAINMENT (PHILS.), INC. and IFPI (SOUTHEAST ASIA), LTD., Petitioners, vs HON.

ELENA S. LIM, SUSAN L. TAN, DAVID S. LIM, JAMES H. UY, WILSON ALEJANDRO, JR., JOSEPH DE LUNA,
MARIA A. VELA CRUZ, DAVID CHUNG, JAMES UY, JOHN DOES AND JANE DOES, AND SOLID LAGUNA
CORPORATION, Respondents. .................................................................................................................... 28
HARRY S. STONEHILL, ROBERT P. BROOKS, JOHN J. BROOKS and KARL BECK, petitioners, vs. HON. JOSE
W. DIOKNO, in his capacity as SECRETARY OF JUSTICE; JOSE LUKBAN, in his capacity as Acting Director,
National Bureau of Investigation; SPECIAL PROSECUTORS PEDRO D. CENZON, EFREN I. PLANA and
MANUEL VILLAREAL, JR. and ASST. FISCAL MANASES G. REYES; JUDGE AMADO ROAN, Municipal Court of
Manila; JUDGE ROMAN CANSINO, Municipal Court of Manila; JUDGE HERMOGENES CALUAG, Court of
First Instance of Rizal-Quezon City Branch, and JUDGE DAMIAN JIMENEZ, Municipal Court of Quezon
City, respondents. ....................................................................................................................................... 42
QUA CHEE GAN, JAMES UY, DANIEL DY alias DEE PAC, CHAN TIONG YU, CUA CHU TIAN, CHUA LIM PAO
alias JOSE CHUA and BASILIO KING, petitioners-appellants, vs. THE DEPORTATION BOARD, Respondent-
Appellee. ..................................................................................................................................................... 58
MAXIMO V. SOLIVEN, ANTONIO V. ROCES, FREDERICK K. AGCAOLI, and GODOFREDO L. MANZANAS,
petitioners, vs. THE HON. RAMON P. MAKASIAR, Presiding Judge of the Regional Trial Court of Manila,
Branch 35, UNDERSECRETARY SILVESTRE BELLO III, of the Department of Justice, LUIS C. VICTOR, THE
CITY FISCAL OF MANILA and PRESIDENT CORAZON C. AQUINO, respondents. ......................................... 65
VICENTE LIM, SR. and MAYOR SUSANA LIM, petitioners, vs. HON. NEMESIO S. FELIX and HON. ANTONIO
ALFANE, respondents.................................................................................................................................. 77
TEODORO C. BORLONGAN, JR., CORAZON M. BEJASA, ARTURO E. MANUEL, JR., ERIC L. LEE, P. SIERVO H.
DIZON, BENJAMIN DE LEON, DELFIN C. GONZALEZ, JR., and BEN YU LIM, JR., Petitioners, vs.
MAGDALENO M. PEÑA and HON. MANUEL Q. LIMSIACO, JR., as Judge Designate of the Municipal Trial
Court in Cities, Bago City, Respondents. ..................................................................................................... 89
THE PEOPLE OF THE PHILIPPINES ISLANDS, plaintiff-appellant, vs. JOSE MA. VELOSO, defendant-
appellant. .................................................................................................................................................. 100
HADJI IBRAHIM SOLAY PANGANDAMAN, MAGAMBAAN PANGANDAMAN, MACARIAN PANGANDAMAN,
MAMINTAL PANGANDAMAN, PACALUNDO PANGANDAMAN, MANGORAMAS PANGANDAMAN,
MACADAOB P. PANGORANGAN, KILATUN PANGANDAMAN, MARIO PANGANDAMAN, MACABIDAR
PANGANDAMAN, PUYAT P. ROMAMPAT, SANTORANI P. DIMAPENGEN, NASSER P. DIMAPENGEN and
DIAMA OPAO, Petitioners, v. DIMAPORO T. CASAR, AS MUNICIPAL CIRCUIT TRIAL JUDGE OF
Page 2 of 377

POONABAYABAO, TAMPARAN AND MASIU, LANAO DEL SUR and THE PEOPLE OF THE PHILIPPINES,
Respondents. ............................................................................................................................................ 111
COLUMBIA PICTURES, INC., ORION PICTURES CORP., PARAMOUNT PICTURES CORP., TWENTIETH
CENTURY FOX FILM CORP., UNITED ARTISTS CORP., UNIVERSAL CITY STUDIOS, INC., THE WALT DISNEY
COMPANY, and WARNER BROS., INC., Petitioners, v. HON. JUDGE ALFREDO C. FLORES, FGT VIDEO
NETWORK, INC., MANUEL MENDOZA, ALFREDO C. ONGYANCO, ERIC APOLONIO, SUSAN YANG and
EDUARDO A. YOTOKO, Respondents. ....................................................................................................... 121
PEOPLE OF THE PHILIPPINES, represented by Provincial Prosecutor FAUSTINO T. CHIONG, Petitioner, v.
COURT OF APPEALS, JUDGE CAESAR CASANOVA, Presiding Judge, Regional Trial Court, Branch 80,
Malolos, Bulacan, AZFAR HUSSAIN, MOHAMMAD SAGED, MUJAHID KHAN, MOHAMMAD ASLAM, and
MEHMOOD ALI, Respondents................................................................................................................... 130
JOSE BURGOS, SR., JOSE BURGOS, JR., BAYANI SORIANO and J. BURGOS MEDIA SERVICES, INC.,
petitioners, vs. THE CHIEF OF STAFF, ARMED FORCES OF THE PHILIPPINES, THE CHIEF, PHILIPPINE
CONSTABULARY, THE CHIEF LEGAL OFFICER, PRESIDENTIAL SECURITY COMMAND, THE JUDGE
ADVOCATE GENERAL, ET AL., respondents............................................................................................... 141
THE PEOPLE OF THE PHILIPPINES, plaintiff-appellee, vs. ROSA ARUTA y MENGUIN, accused-appellant.152
CYNTHIA D. NOLASCO, MILA AGUILAR-ROQUE and WILLIE C. TOLENTINO, Petitioners, v. HON. ERNANI
CRUZ PAÑO, Executive Judge, Regional Trial Court of Quezon City; HON. ANTONIO P. SANTOS, Presiding
Judge, Branch XLII, Metropolitan Trial Court of Quezon City; HON. SERGIO F. APOSTOL, City Fiscal,
Quezon City; HON. JUAN PONCE ENRILE, LT. GEN. FIDEL RAMOS and COL. JESUS ALTUNA, Respondents.
.................................................................................................................................................................. 170
HON. RICARDO G. PAPA, as Chief of Police of Manila; HON. JUAN PONCE ENRILE, as Commissioner of
Customs; PEDRO PACIS, as Collector of Customs of the Port of Manila; and MARTIN ALAGAO, as
Patrolman of the Manila Police Department, petitioners, vs. REMEDIOS MAGO and HILARION U.
JARENCIO, as Presiding Judge of Branch 23, Court of First Instance of Manila, respondents.................. 175
PEOPLE OF THE PHILIPPINES, Plaintiff-Appellee, v. IDEL AMINNUDIN y AHNI, Defendant-Appellant. .... 187
THE PEOPLE OF THE PHILIPPINES, Plaintiff-Appellee, v. MIKAEL MALMSTEDT, * defendant-appellant. . 197
PEOPLE OF THE PHILIPPINES, Plaintiff-Appellee, v. CARMELINA TABAR y CARMILOTES and ROMMEL
ARRIESGADO y TABAR, Accused. CARMELINA TABAR y CARMILOTES, Accused-Appellant. .................... 213
ARSENIO VERGARA VALDEZ, Petitioner, vs.PEOPLE OF THE PHILIPPINES, Respondent. .......................... 223
SPOUSES LEOPOLDO and MA. LUISA VEROY, petitioners, vs. THE HON. WILLIAM L. LAYAGUE, Presiding
Judge, Branch XIV, Regional Trial Court at Davao City; and BRIG. GEN. PANTALEON DUMLAO,
Commanding General, PC-Criminal Investigation Service, respondents. ................................................. 234
PEOPLE OF THE PHILIPPINES, Plaintiff-Appellee, v. ROLANDO DE GRACIA, CHITO HENSON and JOHN
DOES, Accused. ROLANDO DE GRACIA, Accused-Appellant. .................................................................... 242
ROMEO POSADAS y ZAMORA, petitioner, vs. THE HONORABLE COURT OF APPEALS and THE PEOPLE OF
THE PHILIPPINES, respondents. ................................................................................................................ 254
THE PEOPLE OF THE PHILIPPINES, Plaintiff-Appellee, v. ROGELIO MENGOTE Y TEJAS, Accused-Appellant.
.................................................................................................................................................................. 259
Page 3 of 377

PEOPLE OF THE PHILIPPINES, plaintiff-appellee, vs. RUBEN BURGOS y TITO, defendant-appellant........ 267
PEOPLE OF THE PHILIPPINES, plaintiff-appellee, vs. JUAN DE LA CRUZ y GONZALES and REYNALDO
BELTRAN y ANIBAN, accused-appellants. ................................................................................................. 284
PEOPLE OF THE PHILIPPINES, plaintiff-appellee, vs. FLORENCIO DORIA y BOLADO and VIOLETA GADDAO
y CATAMA @ "NENETH," accused-appellants. ......................................................................................... 291
IN THE MATTER OF THE PETITION FOR HABEAS CORPUS OF ROBERTO UMIL, ROLANDO DURAL and
RENATO VILLANUEVA. MANOLITA O. UMIL, and NICANOR P. DURAL, FELICITAS V. SESE, petitioners, vs.
FIDEL V. RAMOS, MAJ. GEN. RENATO DE VILLA, BRIG. GEN. RAMON MONTANO, BRIG. GEN. ALEXANDER
AGUIRRE, respondents.............................................................................................................................. 334
ROLITO GO y TAMBUNTING, petitioner, vs. THE COURT OF APPEALS, THE HON. BENJAMIN V. PELAYO,
Presiding Judge, Branch 168, Regional Trial Court, NCJR Pasig, M.M., and PEOPLE OF THE PHILIPPINES,
respondents. ............................................................................................................................................. 362
Page 4 of 377

G.R. No. 83988 May 24, 1990

RICARDO C. VALMONTE AND UNION OF LAWYERS AND ADVOCATES FOR


PEOPLES’S RIGHTS (ULAP), Petitioners, v. GEN.RENATO DE VILLA AND
NATIONAL CAPITAL REGION DISTRICT COMMAND, Respondents.

Ricardo C. Valmonte for and in his own behalf and co-petitioners.

PADILLA, J.:

In the Court’s decision dated 29 September 1989, petitioners’ petition for prohibition
seeking the declaration of the checkpoints as unconstitutional and their dismantling
and/or banning, was dismissed.

Petitioners have filed the instant motion and supplemental motion for reconsideration
of said decision. Before submission of the incident for resolution, the Solicitor General,
for the respondents, filed his comment, to which petitioners filed a reply.

It should be stated, at the outset, that nowhere in the questioned decision did this Court
legalize all checkpoints, i.e. at all times and under all circumstances. What the Court
declared is, that checkpoints are not illegal per se. Thus, under exceptional
circumstances, as where the survival of organized government is on the balance, or
where the lives and safety of the people are in grave peril, checkpoints may be allowed
and installed by the government. Implicit in this proposition is, that when the situation
clears and such grave perils are removed, checkpoints will have absolutely no reason to
remain.

Recent and on-going events have pointed to the continuing validity and need for
checkpoints manned by either military or police forces. The sixth (6th) attempted coup
d’etat (stronger than all previous ones) was staged only last 1 December 1989. Another
attempt at a coup d’etat is taken almost for granted. The NPA, through its sparrow units,
has not relented but instead accelerated its liquidation of armed forces and police
personnel. Murders, sex crimes, hold-ups and drug abuse have become daily
occurrences. Unlicensed firearms and ammunition have become favorite objects of
trade. Smuggling is at an all-time high. Whether or not effective as expected,
checkpoints have been regarded by the authorities as a security measure designed to
entrap criminals and insurgents and to constitute a dragnet for all types of articles in
illegal trade.

No one can be compelled, under our libertarian system, to share with the present
government its ideological beliefs and practices, or commend its political, social and
economic policies or performance. But, at least, one must concede to it the basic right
to defend itself from its enemies and, while in power, to pursue its program of
government intended for public welfare; and in the pursuit of those objectives, the
government has the equal right, under its police power, to select the reasonable means
Page 5 of 377

and methods for best achieving them. The checkpoint is evidently one of such means it
has selected.

Admittedly, the routine checkpoint stop does intrude, to a certain extent, on motorist’s
right to "free passage without interruption", but it cannot be denied that, as a rule, it
involves only a brief detention of travellers during which the vehicle’s occupants are
required to answer a brief question or two. 1 For as long as the vehicle is neither
searched nor its occupants subjected to a body search, and the inspection of the vehicle
is limited to a visual search, said routine checks cannot be regarded as violative of an
individual’s right against unreasonable search.

These routine checks, when conducted in a fixed area, are even less intrusive. As held
by the U.S. Supreme Court:

"Routine checkpoint stops do not intrude similarly on the motoring public. First, the
potential interference with legitimate traffic is minimal. Motorists using these highways
are not taken by surprise as they know, or may obtain knowledge of, the location of the
checkpoints and will not be stopped elsewhere. Second checkpoint operations both
appear to and actually involve less discretionary enforcement activity. The regularized
manner in which established checkpoints are operated is visible evidence, reassuring to
law-abiding motorists, that the stops are duly authorized and believed to serve the
public interest. The location of a fixed checkpoint is not chosen by officers in the field,
but by officials responsible for making overall decisions as to the most effective
allocation of limited enforcement resources. We may assume that such officials will be
unlikely to locate a checkpoint where it bears arbitrarily or oppressively on motorists as
a class, and since field officers may stop only those cars passing the checkpoint, there
is less room for abusive or harassing stops of individuals them there was in the case of
roving-patrol stops. Moreover, a claim that a particular exercise of discretion in locating
or operating a checkpoint is unreasonable is subject to post-stop judicial review." 2

The checkpoints are nonetheless attacked by the movants as a warrantless search and
seizure and, therefore, violative of the Constitution. 3

As already stated, vehicles are generally allowed to pass these checkpoints after a
routine inspection and a few questions. If vehicles are stopped and extensively searched,
it is because of some probable cause which justifies a reasonable belief of the men at
the checkpoints that either the motorist is a law-offender or the contents of the vehicle
are or have been instruments of some offense. Again, as held by the U.S. Supreme Court

"Automobiles, because of their mobility, may be searched without a warrant upon facts
not justifying a warrantless search of a residence or office. Brinegar v. United States,
338 US 160, 93 L Ed 1879, 69 S Ct 1302(1949); Carroll v. United States, 267 US 132,
69 L Ed 543, 45 S Ct 280, 39 ALR 790 (1925). The cases so holding have, however,
always insisted that the officers conducting the search have ‘reasonable or probable
cause’ to believe that they will find the instrumentality of a crime or evidence pertaining
to a crime before they begin their warrantless search. . . ." 4
Page 6 of 377

Besides these warrantless searches and seizures at the checkpoints are quite similar to
searches and seizures accompanying warrantless arrests during the commission of a
crime, or immediately thereafter. In People v. Kagui Malasuqui, it was held —

"To hold that no criminal can, in any case, be arrested and searched for the evidence
and tokens of his crime without a warrant, would be to leave society, to a large extent,
at the mercy of the shrewdest, the most expert, and the most depraved of criminals,
facilitating their escape in many instances." 5

By the same token, a warrantless search of incoming and outgoing passengers, at the
arrival and departure areas of an international airport, is a practice not constitutionally
objectionable because it is founded on public interest, safety, and necessity.

Lastly, the Court’s decision on checkpoints does not, in any way, validate nor condone
abuses committed by the military manning the checkpoints. The Court’s decision was
concerned with power, i.e. whether the government employing the military has the power
to install said checkpoints. Once that power is acknowledged, the Court’s inquiry
ceases. True, power implies the possibility of its abuse. But whether there is abuse in a
particular situation is a different "ball game" to be resolved in the constitutional arena.

The Court, like all other concerned members of the community, has become aware of
how some checkpoints have been used as points of thievery and extortion practiced
upon innocent civilians. Even the increased prices of foodstuffs coming from the
provinces, entering the Metro Manila area and other urban centers, are largely blamed
on the checkpoints, because the men manning them have reportedly become "experts"
in mulcting travelling traders. This, of course, is a national tragedy.

But the Court could not a priori regard in its now assailed decision that the men in
uniform are rascals or thieves. The Court had to assume that the men in uniform live
and act by the code of honor and they are assigned to the checkpoints to protect, and
not to abuse, the citizenry. 6 The checkpoint is a military "concoction." It behooves the
military to improve the QUALITY of their men assigned to these checkpoints. For no
system or institution will succeed unless the men behind it are honest, noble and
dedicated.

In any situation, where abuse marks the operation of a checkpoint, the citizen is not
helpless. For the military is not above but subject to the law. And the courts exist to see
that the law is supreme. Soldiers, including those who man checkpoints, who abuse
their authority act beyond the scope of their authority and are, therefore, liable
criminally and civilly for their abusive acts. 7 This tenet should be ingrained in the
soldiery in the clearest of terms by higher military authorities.

ACCORDINGLY, the Motion and Supplemental Motion for Reconsideration are DENIED.
This denial is FINAL.

SO ORDERED.

Fernan, C.J., Narvasa, Melencio-Herrera, Paras, Feliciano, Bidin, Cortes, Griño-Aquino,


Medialdea and Regalado, JJ., concur.
Page 7 of 377

Gancayco, J., is on leave.

Separate Opinions

GUTIERREZ, JR., J., concurring:

The problem we face in the resolution of this petition arises from our knowledge that
law enforcement officers use checkpoints as opportunities for mulcting, oppression, and
other forms of abuse. However, to completely ban checkpoints as unconstitutional is to
lose sight of the fact that the real objective behind their use is laudable and necessary.
If we say that ALL checkpoints are unconstitutional, we are banning a law enforcement
measure not because it is per se illegal but because it is being used for evil purposes by
the soldiers or police who man it.

This is another instance where the Supreme Court is urged to solve a problem of
discipline facing the executive and the military. My reluctant concurrence with the
majority opinion is premised on the hope that our top military and police officials will
devise effective measures which would insure that checkpoints are used only where
absolutely needed and that the officers who are assigned to these checkpoints discharge
their duties as professional soldiers or peace officers in the best traditions of the military
and the police. I repeat that this is a problem of enforcement and not legality.

CRUZ, J., dissenting:

I reiterate my original dissent and add the following observations.

The majority would justify the checkpoints on the ground of rampant criminality, e.g.,
the failed coup, the "sparrow" killings, murders, sex crimes, drug abuse, smuggling, etc.
I was not aware that the failure of the authorities to suppress crime was an excuse to
suspend the Bill of Rights. It has always been my impression that even criminals, and
more so the innocent, are entitled to the right against unreasonable searches and
seizures.

The protection of the security of the State is a convenient pretext of the police state to
suppress individual rights. Constitutional shortcuts should not be allowed in a free
regime where the highest function of authority is precisely to exalt liberty.

The ponencia says that the Constitution is not violated because "the inspection of the
vehicle is limited to a visual search." Assuming that this is all the search entails, it
suffers from the additional defect of inefficaciousness, making it virtually useless. It did
not prevent the staging of the December 1 coup, where the rebels used all kinds of high-
powered weapons that were not detected by "a visual search."

Under Article III, Section 2 of the Constitution, probable cause is determined "personally
by the judge," not by a soldier or a policeman. It is not for the peace officer to decide
when a warrantless search and seizure may be made save in the exceptional instances
allowed, as where a crime is being committed or before or after its commission. I can
hardly believe that the majority is seriously offering this exception as a continuing
situation to justify the regular warrantless searches at the checkpoints.chanrobles.com:
Page 8 of 377

It is easy to say that "where abuse marks the operation of a checkpoint, the citizen is
not helpless." Let us at least be realistic. This Court would be the first to dismiss the
complaint if not supported by hard evidence, which we know is not easily come by. The
remedy, in my view, is to remove the source of the evil instead of leaving it unchecked
and then simply suggesting a cure, which is not even effective. It is like inoculating a
patient after exposing him to contagion.

SARMIENTO, J., dissenting:

The majority states that checkpoints are justified by "grave peril." The question,
however, is whether or not the existence of such grave perils has the effect of suspending
the Bill of Rights, specifically, the right against unreasonable searches and seizures.

Under the Constitution," [a] state of martial law does not suspend the operation of the
Constitution." (CONST., art. VII, sec. 18). If not even martial law can suspend the
fundamental law, I do not see how a mere executive act can.

That the State has the light to defend itself is a proposition difficult to argue against.
The query, again, is whether or not it may defend itself against its enemies at the
expense of liberty. After fourteen years of authoritarian rule, I think by now we should
have learned our lesson, and known better.

Although "routine inspections" are another matter, I can not think that the checkpoints
in question have been meant to undertake routine inspections alone. As it is, no ground
rules have been given our law enforcers, which is to say that they have the carte blanche
to search vehicles and even persons without the benefit of a valid judicial warrant. I do
not believe that this can be done in a constitutional regime.

I find references to the case of People v. Kagui Malasugui [63 Phil. 221 (1936)] to be
inapt. In that case, there was a waiver of the right against unreasonable search and
secondly, there existed a clear probable cause for search and arrest. Certainly, there
was reason for excepting the case from the rule. Malasugui, however, is an exception.
And obviously, the majority would make a general rule out of it.

Certainly, it is different where the authority has probable cause to believe that a crime
has been committed by a suspect, in which case, it may place him under arrest or search
his person (Malasugui, supra). But I do not think that it may claim the existence of
probable cause for every vehicle or person stopped and searched at a checkpoint. And
precisely, checkpoints are intended to allow the authorities to fish for probable cause
even if in the beginning there was none. This makes, to my mind, the setting up of
checkpoints unconstitutional.
Page 9 of 377

G.R. Nos. 129756-58 January 28, 2000

PEOPLE OF THE PHILIPPINES, Plaintiff-Appellee, v. JULIAN DEEN ESCAÑO,


VIRGILIO TOME USANA and JERRY CASABAAN LOPEZ, Accused.

VIRGILIO TOME USANA and JERRY CASABAAN LOPEZ, Accused-Appellants.

DAVIDE, JR., C.J.:

Accused-appellants Virgilio T. Usana and Jerry C. Lopez, together with Julian D.


Escaño, were charged before the Regional Trial Court of Makati City, Branch 64, in
Criminal Case No. 95-936 with violation of Section 4, Article II of Republic Act No. 6425,
1 as amended. Escaño and Usana were also charged in Criminal Case No. 95-937 and
No. 95-938 with illegal possession of firearms and ammunition in violation of
Presidential Decree No. 1866.

The accusatory portion of the Information in Criminal Case No. 95-936 reads as follows:

That on or about the 5th day of April, 1995, in the City of Makati, Metro Manila,
Philippines and within the jurisdiction of this Honorable Court, the above-named
accused, conspiring and confederating together and all of them mutually helping and
aiding one another, without being authorized by law, did then and there willfully,
unlawfully and feloniously sell, distribute and transport 3.3143 kilograms of
"HASHISH", a prohibited drug, in violation of the above-cited law. 2

The charge against accused Julian D. Escaño in Criminal Case No. 95-937 reads as
follows:

That on or about the 5th day of April, 1995, in the City of Makati, Metro Manila,
Philippines and within the jurisdiction of this Honorable Court, the above-named
accused did then and there willfully, unlawfully and feloniously have in his possession,
direct custody and control one (1) pc. of cal. .45 pistol, government model with Serial
No. 990255, with magazine containing 7 live ammos and two (2) more magazines for cal.
.45 pistol containing 7 live ammos each, without first securing the necessary license or
permit from the proper government authorities and which firearm and ammunitions he
carried outside of his residence. 3

The accusatory portion of the information against Virgilio Usana in Criminal Case No.
95-938 reads:

That on or about the 5th day of April, 1995, in the City of Makati, Metro Manila,
Philippines and within the jurisdiction of this Honorable Court, the above-named
accused, did then and there willfully, unlawfully and feloniously have in his possession,
Page 10 of 377

direct custody and control One (1) pc. of rifle carbine with Serial No. 7176644 with a
banana type magazine loaded with 28 live ammunitions without first securing the
necessary license or permit from the proper government authorities and which firearms
and ammunitions he carried outside of his residence. 4

The cases were consolidated and jointly tried.

In its Decision of 30 May 1997, 5 which was promulgated on 17 June 1997, the trial
court convicted Escaño and herein appellants in Criminal Case No. 95-936, Escaño in
Criminal Case No. 95-937, and appellant Usana in Criminal Case No. 95-938.

Escaño filed on 19 June 1997 a Notice of Appeal, but on 16 July 1997, he filed a
Manifestation and Withdrawal of Appeal, 6 which was granted by the trial court in its
Order of 17 July 1997. 7

Usana and Lopez filed a Notice of Appeal on 30 June 1997, 8 manifesting therein that
they were appealing to this Court and to the Court of Appeals. Considering the penalties
imposed, the decision in Criminal Case No. 95-936 was appealed to this Court, while
the Court of Appeals took cognizance of the appeal from Criminal Case No. 95-938. In
its Order of 30 June 1997, 9 the trial court gave due course to the appeal and ordered
the transmittal of the record in Criminal Case No. 95-936 to this Court and the record
of Criminal Case No. 95-938 to the Court of Appeals.

Accordingly, it is only the appeal from the judgment in Criminal Case No. 95-936 that
is now before this Court.

Due to the differing versions of the parties, there is a need to narrate each of the
testimonies of the key players in this case.

The prosecution has this version of the events: On the 5th of April 1995 and during a
COMELEC gun ban, some law enforcers of the Makati Police, namely, PO3 Eduardo P.
Suba, PO3 Bernabe Nonato, SPO4 Juan de los Santos, and Inspector Ernesto Guico, 10
were manning a checkpoint at the corner of Senator Gil Puyat Ave. and the South Luzon
Expressway (SLEX). 11 They were checking the cars going to Pasay City, stopping those
they found suspicious, and imposing merely a running stop on the others. At about past
midnight, they stopped a Kia Pride car with Plate No. TBH 493. 12 P03 Suba saw a long
firearm on the lap of the person seated at the passenger seat, who was later identified
as Virgilio Usana. They asked the driver, identified as Escaño, to open the door. P03
Suba seized the long firearm, an M-1 US Carbine, from Usana. When Escaño, upon
order of the police, parked along Sen. Gil Puyat Ave., the other passengers were searched
for more weapons. Their search yielded a .45 caliber firearm which they seized from
Escaño. 13

The three passengers were thereafter brought to the police station Block 5 in the Kia
Pride driven by PO3 Nonato. 14 Upon reaching the precinct, Nonato turned over the key
to the desk officer. Since SPO4 de los Santos was suspicious of the vehicle, he requested
Escaño to open the trunk. 15 Escaño readily agreed and opened the trunk himself using
his key. 16 They noticed a blue bag inside it, 17 which they asked Escaño to open. The
bag contained a parcel wrapped in tape, 18 which, upon examination by National
Page 11 of 377

Bureau of Investigation Forensic Chemist Emilia A. Rosaldos, was found positive for
hashish weighing 3.3143 kilograms. 19

A certification was issued by the Firearms and Explosive Office of the National Police
Commission (NAPOLCOM) to the effect that Escaño was not a licensed/registered
firearms holder of any kind and caliber. Usana, however, according to the same
certification is a licensed/registered holder of a pistol Colt .45 caliber with license issued
on 14 October 1994 and to expire on April 1996. Usana also has an application for a
pistol Uzi Cal. 9mm. Neither of the two guns seized were licensed/registered with the
NAPOLCOM. 20

For his part, Escaño (or Jovy) testified that on the 4th of April 1995, between 11:00 and
11:30 in the morning, he was at the lobby of Legend Hotel, at Pioneer St., Mandaluyong
City, to meet with his business partners, including Usana and Lopez. He saw his friend
and erstwhile co-employee at Philippine Airlines, Ramon Cabrera, who had borrowed
his wife’s car. Since it was his wife’s birthday the following day, he asked Cabrera if he
could get back the car. Cabrera readily gave him the keys of the car. 21

He left the hotel at around 11:45 in the evening with Usana and Lopez. Using his wife’s
car, they cruised southward along Epifanio de los Santos Avenue (EDSA) and turned
right at Sen. Gil Puyat Avenue. They stopped before crossing SLEX because the traffic
light turned red. From the other side of SLEX, he could see a group of policemen. Upon
crossing SLEX, they were flagged down by one of the policemen, so he slowed down and
stopped. PO3 Nonato asked him to roll down the window and demanded to see his
license. He asked if he had committed any violation, but PO3 Nonato accused him of
being drunk, which he denied. The policemen persisted in asking for his license, but he
did not budge and instead reiterated that there was no reason for him to surrender his
license for he had not committed any violation. A verbal tussle ensued resulting in the
drawing of firearms by the policemen which prompted Usana to suggest that they go to
the police station because the policemen were carrying guns and they have not done
anything wrong. 22

He stated further that he was the one who drove to the police station along Dian St. with
his companions. He parked the car then they were brought to the office of the Deputy
Station Commander, Lieutenant Eco. 23 The policemen asked if they could search his
car. He then inquired if he was not entitled to a lawyer and why they needed to conduct
a search when they had not even told him what he had violated. Apparently, he thought
they were there only for verification purposes. Lt. Eco explained that that was the reason
why they were going to search his car, to see if he had done anything illegal. Although
the police were insistent in asking for the keys to his car, he continuously refused. Lt.
Eco asked his men to usher the trio into the detention cell. 24

After two hours, he was brought back to Lt. Eco’s office. Lt. Eco pointed to a bag, a rifle,
a pistol and a package wrapped in masking tape or packing tape on his desk, and said
these items constituted evidence of illegal possession of firearms and transporting of
drugs. He was surprised that they found those items from his car because his key had
been with him all the time. He was handcuffed, brought to his car, and again was
surprised to see its trunk open. 25
Page 12 of 377

On the other hand, Lopez had a different story. He claimed he was the mechanic of
Usana and they lived in the same subdivision. 26 On 4 April 1995, he was working on
Usana’s pick-up truck at the latter’s house when Escaño dropped by at around 4:30 in
the afternoon looking for Usana who was then working in Forbes Park. 27 At around
5:30 p.m., they left Usana’s house in Escaño’s metallic gray Kia Pride. Inside the car,
he saw a .45 caliber pistol and two spare magazines tucked in the right side and left
side of Escaño’s waist. He also saw a carbine under the right passenger seat. When he
inquired about the guns, Escaño replied that such did not pose any problem since they
were licensed. Before going to Usana, they went to Pasay City to see a certain Jerry. 28
They met Usana at the Sen. Gil Puyat Station of the LRT at around 9:00 p.m. He gave
his seat to Usana but was unaware if the latter noticed the rifle beneath the seat. 29

They went home via Sen. Gil Puyat Avenue but were stopped at a checkpoint after
crossing SLEX. The policemen directed their flashlights at them and one opened the
front passenger door. 30 The latter saw the rifle under Usana’s seat. Usana and Escaño
were ordered to get out of the car. PO3 Nonato immediately saw the gun tucked in
Escaño’s waist and asked if he was a policeman. Escaño replied that everything would
be explained at the police station. He was also asked to step out. No firearm was,
however, found in his possession. 31

When confronted about the guns, Escaño tried to intercede for his two companions and
said that." . . these two don’t know anything about it, I just took them for a drive." They
subsequently went to police station Block 5. A certain Toto, a policeman, drove the Kia
Pride to Block 5. 32

Upon reaching the police station, Escaño was immediately brought to the office of Lt.
Eco while he and Usana were asked to sit on the bench. After a few minutes, PO3 de los
Santos came out of the office of Lt. Eco to talk to him. He told him that all he knew
about Escaño is that he was a wealthy flight attendant with military connections. After
returning to Lt. Eco’s office, PO3 de los Santos went out of the police station with Lt.
Eco and Escaño. The three came back with a blue bag which he had never seen before.
The bag was opened before the three suspects. Escaño reiterated that his two
companions had nothing to do with the bag. 33chanroblesvirtuallawlibrary

He and Usana stayed overnight in their cell and only saw Escaño in the morning of April
5. At around 4:00 p.m., they were transferred to the CID and stayed in the office of a
certain Inspector Sipin. Escaño admitted he owned the bag/case. 34

For his part, Usana testified that he was a duly licensed architect who was gainfully
employed by Rolando de Asis and Taytay Management Corporation. 35 He admitted
owning a licensed .45 caliber pistol. 36 In March 1995, he hired as mechanic Lopez,
who lives in Bernabe Subdivision Phase II where he also lives. Escaño on the other hand,
was introduced to him by a certain Roberto Samparado, a neighbor of Lopez. Escaño,
an international flight attendant of Philippine Airlines and a businessman who owns
Verge Enterprises, also supplied materials to the Philippine Army and planned to engage
in a construction business. 37

On 4 April 1995, at around 7:30 p.m., he paged Escaño to talk about the materials for
the five prototype gunship helicopters they were supposed to supply. They talked on the
phone, agreeing to meet between 8:30 and 9:00 p.m. at the Sen. Gil Puyat Ave. Station
Page 13 of 377

of the Light Rail Transit, 38 and met at around a quarter past nine. Escaño was on
board a metallic Kia Pride with Lopez on the passenger seat. Lopez vacated the seat for
him. They went to Magallanes Village to meet a certain Norman Garcia and talk about
the documents 39 relating to the helicopter gunship of the Air Force. They arrived there
at 11:30 p.m. While they were talking with Garcia, he noticed a gun and magazines
tucked in Escaño’s waist. Upon inquiry, Escaño said it was not a problem and only for
his protection. 40 On their way to Roxas Boulevard, they were stopped at a checkpoint
along Sen. Gil Puyat Ave. Policemen knocked on the car windows so he and Escaño
rolled down their windows. A person in civilian clothes suddenly opened the right door,
took something from the side of his seat and shouted, "There’s a gun." He was surprised
because he did not carry anything when he boarded the car; neither did he see anything
inside the car because it was dark and he was not wearing his eyeglasses. 41 The person
who took the gun asked if he was a policeman, and he said he was an architect. He was
then asked to alight from the car, then frisked. Escaño was also asked to alight from a
car. They saw a gun tucked in his waist, so they asked if he was a policeman, and
Escaño answered in the negative. Lopez was then ordered to get out of the car by the
person in civilian clothes and was also searched. They rode the Anfra service vehicle of
the police. One of the policemen asked Lopez to handcuff him and Escaño. The
policeman who asked Escaño to get out of the car drove the Anfra van to Block 5 where
they arrived at 1:30 in the morning of 5 April. 42

He and Lopez waited outside the office of Lt. Eco while Escaño was inside with the
arresting officers. Lt. Eco came out of his office and urged Lopez to tell the truth. He
heard Lopez say that they were both just with Escaño and that they knew nothing about
the guns; neither do they own any. SPO4 de los Santos entered the office of Lt. Eco and
came out five minutes later with Escaño, Lt. Eco, and the other arresting officers,
Nonato, Suba and Erwin Eco, the person in civilian clothes. All six went out to the
parking area and returned after about five minutes. Lt. Eco was carrying a bag which
he placed on top his desk. Lopez and Escaño were asked about the contents of the bag.
The two replied it was the first time they saw that bag. Lt. Eco opened the bag before
them. They all saw something in brown paper. He and Lopez simultaneously exclaimed
that they knew nothing about the contents of the bag, and they implored Escaño to tell
the police that they had nothing to do with it. 43

The trial court found the prosecution’s version more credible than that of any one of the
accused, and ruled that the evidence presented by the prosecution was sufficient to
convict the accused as charged. It decreed:

WHEREFORE, in view of the foregoing judgment is hereby rendered as follows:

1. In Criminal Case No. 95-936, Accused JULIAN ESCAÑO y DEEN, VIRGILIO USANA
y TOME and JERRY LOPEZ y CASABAAN are GUILTY as charged and are sentenced to
suffer imprisonment of RECLUSION PERPETUA, and to pay a fine of P500,000.00.

The Branch Clerk of Court is directed to turn over to the Dangerous Drugs Board the
3.314 kilograms of Hashish (marijuana) for its appropriate disposition in accordance
with law; andchanroblesvirtuallawlibrary

2. In Criminal Cases Nos. 95-937 and 95-938, Accused JULIAN ESCAÑO y DEEN and
VIRGILIO USANA y TOME are GUILTY as charged in the two separate informations
Page 14 of 377

respectively filed against them and are sentenced to suffer the indeterminate prison
term from TEN (10) YEARS of PRISION MAYOR maximum, as minimum to SEVENTEEN
(17) YEARS, FOUR (4) MONTHS and ONE (1) DAY of RECLUSION TEMPORAL maximum
as maximum. 44

The firearms and ammunitions subject matter of these cases which are still with the
City Prosecutor’s Office are forfeited in favor of the Government are directed to be turned
over to the Firearms and Explosive Unit, PNP, Camp Crame, Quezon City for its
appropriate disposition.

SO ORDERED. 45

Accused-appellants Usana and Lopez anchor their appeal on the following arguments:

1. The trial court erred in admitting in evidence the hashish seized without search
warrant when the police officers already had the opportunity to secure a search warrant
before searching the bag found at the baggage compartment at the back of the car;

2. Assuming that the hashish is admissible in evidence, the trial court erred in finding
appellants to have conspired with Escaño in transporting the hashish when the evidence
clearly shows that the hashish was owned and possessed solely by Escaño;

3. The trial court erred in convicting appellants of illegal possession of hashish despite
the fact that they were neither in actual nor constructive possession of the illegal drug;
and

4. The trial court erred in not considering the exculpatory testimony of Julian Escaño
in favor of appellants.

Before going any further, some words are in order regarding the establishment of
checkpoints.

Accused-appellants assail the manner by which the checkpoint in question was


conducted. They contend that the checkpoint manned by elements of the Makati Police
should have been announced. They also complain of its having been conducted in an
arbitrary and discriminatory manner.

We take judicial notice of the existence of the COMELEC resolution 46 imposing a gun
ban during the election period issued pursuant to Section 52(c) in relation to Section
26(q) of the Omnibus Election Code (Batas Pambansa Blg. 881). The national and local
elections in 1995 were held on 8 May, the second Monday of the month. The incident,
which happened on 5 April 1995, was well within the election period.

This Court has ruled that not all checkpoints are illegal. Those which are warranted by
the exigencies of public order and are conducted in a way least intrusive to motorists
are allowed. 47 For, admittedly, routine checkpoints do intrude, to a certain extent, on
motorists’ right to "free passage without interruption," but it cannot be denied that, as
a rule, it involves only a brief detention of travelers during which the vehicle’s occupants
are required to answer a brief question or two. For as long as the vehicle is neither
searched nor its occupants subjected to a body search, and the inspection of the vehicle
Page 15 of 377

is limited to a visual search, said routine checks cannot be regarded as violative of an


individual’s right against unreasonable search. In fact, these routine checks, when
conducted in a fixed area, are even less intrusive. 48

The checkpoint herein conducted was in pursuance of the gun ban enforced by the
COMELEC. The COMELEC would be hard put to implement the ban if its deputized
agents were limited to a visual search of pedestrians. It would also defeat the purpose
for which such ban was instituted. Those who intend to bring a gun during said period
would know that they only need a car to be able to easily perpetrate their malicious
designs.

The facts adduced do not constitute a ground for a violation of the constitutional rights
of the accused against illegal search and seizure. PO3 Suba admitted that they were
merely stopping cars they deemed suspicious, such as those whose windows are heavily
tinted just to see if the passengers thereof were carrying guns. At best they would merely
direct their flashlights inside the cars they would stop, without opening the car’s doors
or subjecting its passengers to a body search. There is nothing discriminatory in this as
this is what the situation demands.

We see no need for checkpoints to be announced, as the accused have invoked. Not only
would it be impractical, it would also forewarn those who intend to violate the ban. Even
so, badges of legitimacy of checkpoints may still be inferred from their fixed location and
the regularized manner in which they are operated. 49

Usana and Lopez also question the validity of the search. The trial court, in convicting
the three accused for violation of R.A. No. 6425, accepted as aboveboard the search
done by the Makati Police of the trunk of the car. Jurisprudence recognizes six generally
accepted exceptions to the warrant requirement: (1) search incidental to an arrest; (2)
search of moving vehicles; (3) evidence in plain view; (4) customs searches; (5) consented
warrantless search; 50 and (6) stop-and-frisk situations. 51

Even though there was ample opportunity to obtain a search warrant, we cannot
invalidate the search of the vehicle, for there are indications that the search done on the
car of Escaño was consented to by him. Both Lopez and Usana testified that Escaño
was with the police officers when they searched the car. 52 There was no apparent
objection made by Escaño as he seemed to have freely accompanied the police officers
to the car. PO3 Suba, on the other hand, testified that "Escaño readily agreed to open
the trunk," upon request of SPO4 de los Santos. 53 But according to Escaño, he refused
the request of the police officers to search his car. 54 We must give credence to the
testimony of PO3 Suba. Not only is it buttressed by the testimony of Usana and Lopez
that Escaño freely accompanied the police officers to the car, it is also deemed admitted
by Escaño in failing to appeal the decision. The findings of fact of the trial court are thus
deemed final as against him.

Despite the validity of the search, we cannot affirm the conviction of Usana and Lopez
for violation of R.A. No. 6425, as amended. The following facts militate against a finding
of conviction: (1) the car belonged to Escaño; (2) the trunk of the car was not opened
soon after it was stopped and after the accused were searched for firearms; (3) the car
was driven by a policeman from the place where it was stopped until the police station;
(4) the car’s trunk was opened, with the permission of Escaño, without the presence of
Page 16 of 377

Usana and Lopez; and (5) after arrival at the police station and until the opening of the
car’s trunk, the car was in the possession and control of the police authorities. No fact
was adduced to link Usana and Lopez to the hashish found in the trunk of the car. Their
having been with Escaño in the latter’s car before the "finding" of the hashish sometime
after the lapse of an appreciable time and without their presence left much to be desired
to implicate them to the offense of selling, distributing, or transporting the prohibited
drug. In fact, there was no showing that Usana and Lopez knew of the presence of
hashish in the trunk of the car or that they saw the same before it was seized.

IN VIEW WHEREOF, that portion of the challenged decision of 30 May 1997 of the
Regional Trial Court, Makati, Branch 64, insofar as Criminal Case No. 95-936 is
concerned with regard to accused-appellants VIRGILIO T. USANA and JERRY C. LOPEZ,
holding them guilty of violation of Section 4, Article II of R.A. No 6425, as amended, is
hereby REVERSED and SET ASIDE and another is hereby rendered ACQUITTING them
therein on ground of reasonable doubt and ORDERING their immediate release from
confinement at the New Bilibid Prison, unless their further detention is justified for any
lawful ground. The Director of the Bureau of Corrections is hereby directed to report to
the Court the release of said accused-appellants within five (5) days from notice of this
decision.

SO ORDERED.

Puno, Kapunan, Pardo and Ynares-Santiago, JJ., concur.


Page 17 of 377

G.R. No. 140946 September 13, 2004

MICROSOFT CORPORATION and LOTUS DEVELOPMENT CORPORATION,


petitioners, vs. MAXICORP, INC., respondent.

CARPIO, J.:

The Case

This petition for review on certiorari1 seeks to reverse the Court of Appeals’ Decision2
dated 23 December 1998 and its Resolution dated 29 November 1999 in CA-G.R. SP No.
44777. The Court of Appeals reversed the Order3 of the Regional Trial Court, Branch
23, Manila ("RTC"), denying respondent Maxicorp, Inc.’s ("Maxicorp") motion to quash
the search warrant that the RTC issued against Maxicorp. Petitioners are the private
complainants against Maxicorp for copyright infringement under Section 29 of
Presidential Decree No. 49 ("Section 29 of PD 49")4 and for unfair competition under
Article 189 of the Revised Penal Code ("RPC").5

Antecedent Facts

On 25 July 1996, National Bureau of Investigation ("NBI") Agent Dominador Samiano,


Jr. ("NBI Agent Samiano") filed several applications for search warrants in the RTC
against Maxicorp for alleged violation of Section 29 of PD 49 and Article 189 of the RPC.
After conducting a preliminary examination of the applicant and his witnesses, Judge
William M. Bayhon issued Search Warrants Nos. 96-451, 96-452, 96-453 and 96-454,
all dated 25 July 1996, against Maxicorp.

Armed with the search warrants, NBI agents conducted on 25 July 1996 a search of
Maxicorp’s premises and seized property fitting the description stated in the search
warrants.

On 2 September 1996, Maxicorp filed a motion to quash the search warrants alleging
that there was no probable cause for their issuance and that the warrants are in the
form of "general warrants." The RTC denied Maxicorp’s motion on 22 January 1997. The
RTC also denied Maxicorp’s motion for reconsideration.
Page 18 of 377

The RTC found probable cause to issue the search warrants after examining NBI Agent
Samiano, John Benedict Sacriz ("Sacriz"), and computer technician Felixberto Pante
("Pante"). The three testified on what they discovered during their respective visits to
Maxicorp. NBI Agent Samiano also presented certifications from petitioners that they
have not authorized Maxicorp to perform the witnessed activities using petitioners’
products.

On 24 July 1997, Maxicorp filed a petition for certiorari with the Court of Appeals
seeking to set aside the RTC’s order. On 23 December 1998, the Court of Appeals
reversed the RTC’s order denying Maxicorp’s motion to quash the search warrants.
Petitioners moved for reconsideration. The Court of Appeals denied petitioners’ motion
on 29 November 1999.

The Court of Appeals held that NBI Agent Samiano failed to present during the
preliminary examination conclusive evidence that Maxicorp produced or sold the
counterfeit products. The Court of Appeals pointed out that the sales receipt NBI Agent
Samiano presented as evidence that he bought the products from Maxicorp was in the
name of a certain "Joel Diaz."

Hence, this petition.

The Issues

Petitioners seek a reversal and raise the following issues for resolution:

1. WHETHER THE PETITION RAISES QUESTIONS OF LAW;

2. WHETHER PETITIONERS HAVE LEGAL PERSONALITY TO FILE THE PETITION;

3. WHETHER THERE WAS PROBABLE CAUSE TO ISSUE THE SEARCH WARRANTS;

4. WHETHER THE SEARCH WARRANTS ARE "GENERAL WARRANTS."


Page 19 of 377

The Ruling of the Court

The petition has merit.

On Whether the Petition Raises Questions of Law

Maxicorp assails this petition as defective since it failed to raise questions of law.
Maxicorp insists that the arguments petitioners presented are questions of fact, which
this Court should not consider in a Rule 45 petition for review. Petitioners counter that
all the issues they presented in this petition involve questions of law. Petitioners point
out that the facts are not in dispute.

A petition for review under Rule 45 of the Rules of Court should cover questions of law.6
Questions of fact are not reviewable. As a rule, the findings of fact of the Court of Appeals
are final and conclusive and this Court will not review them on appeal,7 subject to
exceptions as when the findings of the appellate court conflict with the findings of the
trial court.8

The distinction between questions of law and questions of fact is settled. A question of
law exists when the doubt or difference centers on what the law is on a certain state of
facts. A question of fact exists if the doubt centers on the truth or falsity of the alleged
facts. Though this delineation seems simple, determining the true nature and extent of
the distinction is sometimes problematic. For example, it is incorrect to presume that
all cases where the facts are not in dispute automatically involve purely questions of
law.

There is a question of law if the issue raised is capable of being resolved without need
of reviewing the probative value of the evidence.9 The resolution of the issue must rest
solely on what the law provides on the given set of circumstances. Once it is clear that
the issue invites a review of the evidence presented, the question posed is one of fact.10
If the query requires a re-evaluation of the credibility of witnesses, or the existence or
relevance of surrounding circumstances and their relation to each other, the issue in
that query is factual.11 Our ruling in Paterno v. Paterno12 is illustrative on this point:

Such questions as whether certain items of evidence should be accorded probative value
or weight, or rejected as feeble or spurious, or whether or not the proofs on one side or
the other are clear and convincing and adequate to establish a proposition in issue, are
without doubt questions of fact. Whether or not the body of proofs presented by a party,
Page 20 of 377

weighed and analyzed in relation to contrary evidence submitted by adverse party, may
be said to be strong, clear and convincing; whether or not certain documents presented
by one side should be accorded full faith and credit in the face of protests as to their
spurious character by the other side; whether or not inconsistencies in the body of
proofs of a party are of such gravity as to justify refusing to give said proofs weight – all
these are issues of fact.

It is true that Maxicorp did not contest the facts alleged by petitioners. But this situation
does not automatically transform all issues raised in the petition into questions of law.
The issues must meet the tests outlined in Paterno.

Of the three main issues raised in this petition – the legal personality of the petitioners,
the nature of the warrants issued and the presence of probable cause – only the first
two qualify as questions of law. The pivotal issue of whether there was probable cause
to issue the search warrants is a question of fact. At first glance, this issue appears to
involve a question of law since it does not concern itself with the truth or falsity of certain
facts. Still, the resolution of this issue would require this Court to inquire into the
probative value of the evidence presented before the RTC. For a question to be one of
law, it must not involve an examination of the probative value of the evidence presented
by the litigants or any of them.13

Yet, this is precisely what the petitioners ask us to do by raising arguments requiring
an examination of the TSNs and the documentary evidence presented during the search
warrant proceedings. In short, petitioners would have us substitute our own judgment
to that of the RTC and the Court of Appeals by conducting our own evaluation of the
evidence. This is exactly the situation which Section 1, Rule 45 of the Rules of Court
prohibits by requiring the petition to raise only questions of law. This Court is not a trier
of facts. It is not the function of this court to analyze or weigh evidence.14 When we give
due course to such situations, it is solely by way of exception. Such exceptions apply
only in the presence of extremely meritorious circumstances.15

Indeed, this case falls under one of the exceptions because the findings of the Court of
Appeals conflict with the findings of the RTC.16 Since petitioners properly raised the
conflicting findings of the lower courts, it is proper for this Court to resolve such
contradiction.

On Whether Petitioners have the Legal Personality to File this Petition


Page 21 of 377

Maxicorp argues that petitioners have no legal personality to file this petition since the
proper party to do so in a criminal case is the Office of the Solicitor General as
representative of the People of the Philippines. Maxicorp states the general rule but the
exception governs this case.17 We ruled in Columbia Pictures Entertainment, Inc. v.
Court of Appeals18 that the petitioner-complainant in a petition for review under Rule
45 could argue its case before this Court in lieu of the Solicitor General if there is grave
error committed by the lower court or lack of due process. This avoids a situation where
a complainant who actively participated in the prosecution of a case would suddenly
find itself powerless to pursue a remedy due to circumstances beyond its control. The
circumstances in Columbia Pictures Entertainment are sufficiently similar to the
present case to warrant the application of this doctrine.

On Whether there was Probable Cause to Issue the Search Warrants

Petitioners argue that the Court of Appeals erred in reversing the RTC based on the fact
that the sales receipt was not in the name of NBI Agent Samiano. Petitioners point out
that the Court of Appeals disregarded the overwhelming evidence that the RTC
considered in determining the existence of probable cause. Maxicorp counters that the
Court of Appeals did not err in reversing the RTC. Maxicorp maintains that the entire
preliminary examination that the RTC conducted was defective.

The Court of Appeals based its reversal on two factual findings of the RTC. First, the
fact that the sales receipt presented by NBI Agent Samiano as proof that he bought
counterfeit goods from Maxicorp was in the name of a certain "Joel Diaz." Second, the
fact that petitioners’ other witness, John Benedict Sacriz, admitted that he did not buy
counterfeit goods from Maxicorp.

We rule that the Court of Appeals erred in reversing the RTC’s findings.

Probable cause means "such reasons, supported by facts and circumstances as will
warrant a cautious man in the belief that his action and the means taken in prosecuting
it are legally just and proper."19 Thus, probable cause for a search warrant requires
such facts and circumstances that would lead a reasonably prudent man to believe that
an offense has been committed and the objects sought in connection with that offense
are in the place to be searched.20

The judge determining probable cause must do so only after personally examining under
oath the complainant and his witnesses. The oath required must refer to "the truth of
the facts within the personal knowledge of the petitioner or his witnesses, because the
Page 22 of 377

purpose thereof is to convince the committing magistrate, not the individual making the
affidavit and seeking the issuance of the warrant, of the existence of probable cause."21
The applicant must have personal knowledge of the circumstances. "Reliable
information" is insufficient.22 Mere affidavits are not enough, and the judge must
depose in writing the complainant and his witnesses.23

The Court of Appeals’ reversal of the findings of the RTC centers on the fact that the two
witnesses for petitioners during the preliminary examination failed to prove conclusively
that they bought counterfeit software from Maxicorp. The Court of Appeals ruled that
this amounted to a failure to prove the existence of a connection between the offense
charged and the place searched.

The offense charged against Maxicorp is copyright infringement under Section 29 of PD


49 and unfair competition under Article 189 of the RPC. To support these charges,
petitioners presented the testimonies of NBI Agent Samiano, computer technician Pante,
and Sacriz, a civilian. The offenses that petitioners charged Maxicorp contemplate
several overt acts. The sale of counterfeit products is but one of these acts. Both NBI
Agent Samiano and Sacriz related to the RTC how they personally saw Maxicorp commit
acts of infringement and unfair competition.

During the preliminary examination, the RTC subjected the testimonies of the witnesses
to the requisite examination. NBI Agent Samiano testified that he saw Maxicorp display
and offer for sale counterfeit software in its premises. He also saw how the counterfeit
software were produced and packaged within Maxicorp’s premises. NBI Agent Samiano
categorically stated that he was certain the products were counterfeit because Maxicorp
sold them to its customers without giving the accompanying ownership manuals, license
agreements and certificates of authenticity.

Sacriz testified that during his visits to Maxicorp, he witnessed several instances when
Maxicorp installed petitioners’ software into computers it had assembled. Sacriz also
testified that he saw the sale of petitioners’ software within Maxicorp’s premises.
Petitioners never authorized Maxicorp to install or sell their software.

The testimonies of these two witnesses, coupled with the object and documentary
evidence they presented, are sufficient to establish the existence of probable cause.
From what they have witnessed, there is reason to believe that Maxicorp engaged in
copyright infringement and unfair competition to the prejudice of petitioners. Both NBI
Agent Samiano and Sacriz were clear and insistent that the counterfeit software were
not only displayed and sold within Maxicorp’s premises, they were also produced,
packaged and in some cases, installed there.
Page 23 of 377

The determination of probable cause does not call for the application of rules and
standards of proof that a judgment of conviction requires after trial on the merits. As
implied by the words themselves, "probable cause" is concerned with probability, not
absolute or even moral certainty. The prosecution need not present at this stage proof
beyond reasonable doubt. The standards of judgment are those of a reasonably prudent
man,24 not the exacting calibrations of a judge after a full-blown trial.

No law or rule states that probable cause requires a specific kind of evidence. No formula
or fixed rule for its determination exists.25 Probable cause is determined in the light of
conditions obtaining in a given situation.26 Thus, it was improper for the Court of
Appeals to reverse the RTC’s findings simply because the sales receipt evidencing NBI
Agent Samiano’s purchase of counterfeit goods is not in his name.

For purposes of determining probable cause, the sales receipt is not the only proof that
the sale of petitioners’ software occurred. During the search warrant application
proceedings, NBI Agent Samiano presented to the judge the computer unit that he
purchased from Maxicorp, in which computer unit Maxicorp had pre-installed
petitioners’ software.27 Sacriz, who was present when NBI Agent Samiano purchased
the computer unit, affirmed that NBI Agent Samiano purchased the computer unit.28
Pante, the computer technician, demonstrated to the judge the presence of petitioners’
software on the same computer unit.29 There was a comparison between petitioners’
genuine software and Maxicorp’s software pre-installed in the computer unit that NBI
Agent Sambiano purchased.30 Even if we disregard the sales receipt issued in the name
of "Joel Diaz," which petitioners explained was the alias NBI Agent Samiano used in the
operation, there still remains more than sufficient evidence to establish probable cause
for the issuance of the search warrants.

This also applies to the Court of Appeals’ ruling on Sacriz’s testimony. The fact that
Sacriz did not actually purchase counterfeit software from Maxicorp does not eliminate
the existence of probable cause. Copyright infringement and unfair competition are not
limited to the act of selling counterfeit goods. They cover a whole range of acts, from
copying, assembling, packaging to marketing, including the mere offering for sale of the
counterfeit goods. The clear and firm testimonies of petitioners’ witnesses on such other
acts stand untarnished. The Constitution and the Rules of Court only require that the
judge examine personally and thoroughly the applicant for the warrant and his
witnesses to determine probable cause. The RTC complied adequately with the
requirement of the Constitution and the Rules of Court.

Probable cause is dependent largely on the opinion and findings of the judge who
conducted the examination and who had the opportunity to question the applicant and
Page 24 of 377

his witnesses.31 For this reason, the findings of the judge deserve great weight. The
reviewing court should overturn such findings only upon proof that the judge
disregarded the facts before him or ignored the clear dictates of reason.32 Nothing in
the records of the preliminary examination proceedings reveal any impropriety on the
part of the judge in this case. As one can readily see, here the judge examined thoroughly
the applicant and his witnesses. To demand a higher degree of proof is unnecessary and
untimely. The prosecution would be placed in a compromising situation if it were
required to present all its evidence at such preliminary stage. Proof beyond reasonable
doubt is best left for trial.

On Whether the Search Warrants are in the Nature of General Warrants

A search warrant must state particularly the place to be searched and the objects to be
seized. The evident purpose for this requirement is to limit the articles to be seized only
to those particularly described in the search warrant. This is a protection against
potential abuse. It is necessary to leave the officers of the law with no discretion
regarding what articles they shall seize, to the end that no unreasonable searches and
seizures be committed.33

In addition, under Section 4, Rule 126 of the Rules of Criminal Procedure, a search
warrant shall issue "in connection with one specific offense." The articles described must
bear a direct relation to the offense for which the warrant is issued.34 Thus, this rule
requires that the warrant must state that the articles subject of the search and seizure
are used or intended for use in the commission of a specific offense.

Maxicorp argues that the warrants issued against it are too broad in scope and lack the
specificity required with respect to the objects to be seized. After examining the wording
of the warrants issued, the Court of Appeals ruled in favor of Maxicorp and reversed the
RTC’s Order thus:

Under the foregoing language, almost any item in the petitioner’s store can be seized on
the ground that it is "used or intended to be used" in the illegal or unauthorized copying
or reproduction of the private respondents’ software and their manuals.35

The Court of Appeals based its reversal on its perceived infirmity of paragraph (e) of the
search warrants the RTC issued. The appellate court found that similarly worded
warrants, all of which noticeably employ the phrase "used or intended to be used," were
previously held void by this Court.36 The disputed text of the search warrants in this
case states:
Page 25 of 377

a) Complete or partially complete reproductions or copies of Microsoft software bearing


the Microsoft copyrights and/or trademarks owned by MICROSOFT CORPORATION
contained in CD-ROMs, diskettes and hard disks;

b) Complete or partially complete reproductions or copies of Microsoft instruction


manuals and/or literature bearing the Microsoft copyrights and/or trademarks owned
by MICROSOFT CORPORATION;

c) Sundry items such as labels, boxes, prints, packages, wrappers, receptacles,


advertisements and other paraphernalia bearing the copyrights and/or trademarks
owned by MICROSOFT CORPORATION;

d) Sales invoices, delivery receipts, official receipts, ledgers, journals, purchase orders
and all other books of accounts and documents used in the recording of the
reproduction and/or assembly, distribution and sales, and other transactions in
connection with fake or counterfeit products bearing the Microsoft copyrights and/or
trademarks owned by MICROSOFT CORPORATION;

e) Computer hardware, including central processing units including hard disks, CD-
ROM drives, keyboards, monitor screens and diskettes, photocopying machines and
other equipment or paraphernalia used or intended to be used in the illegal and
unauthorized copying or reproduction of Microsoft software and their manuals, or which
contain, display or otherwise exhibit, without the authority of MICROSOFT
CORPORATION, any and all Microsoft trademarks and copyrights; and

f) Documents relating to any passwords or protocols in order to access all computer


hard drives, data bases and other information storage devices containing unauthorized
Microsoft software.37 (Emphasis supplied)

It is only required that a search warrant be specific as far as the circumstances will
ordinarily allow.38 The description of the property to be seized need not be technically
accurate or precise. The nature of the description should vary according to whether the
identity of the property or its character is a matter of concern.39 Measured against this
standard we find that paragraph (e) is not a general warrant. The articles to be seized
were not only sufficiently identified physically, they were also specifically identified by
stating their relation to the offense charged. Paragraph (e) specifically refers to those
articles used or intended for use in the illegal and unauthorized copying of petitioners’
software. This language meets the test of specificity.40
Page 26 of 377

The cases cited by the Court of Appeals are inapplicable. In those cases, the Court found
the warrants too broad because of particular circumstances, not because of the mere
use of the phrase "used or intended to be used." In Columbia Pictures, Inc. v. Flores,
the warrants ordering the seizure of "television sets, video cassette recorders, rewinders
and tape cleaners x x x" were found too broad since the defendant there was a licensed
distributor of video tapes.41 The mere presence of counterfeit video tapes in the
defendant’s store does not mean that the machines were used to produce the counterfeit
tapes. The situation in this case is different. Maxicorp is not a licensed distributor of
petitioners. In Bache & Co. (Phil.), Inc., et al. v. Judge Ruiz, et al., the Court voided the
warrants because they authorized the seizure of records pertaining to "all business
transactions" of the defendant.42 And in 20th Century Fox Film Corp. v. Court of
Appeals, the Court quashed the warrant because it merely gave a list of articles to be
seized, aggravated by the fact that such appliances are "generally connected with the
legitimate business of renting out betamax tapes."43

However, we find paragraph (c) of the search warrants lacking in particularity.


Paragraph (c) states:

c) Sundry items such as labels, boxes, prints, packages, wrappers, receptacles,


advertisements and other paraphernalia bearing the copyrights and/or trademarks
owned by MICROSOFT CORPORATION;

The scope of this description is all-embracing since it covers property used for personal
or other purposes not related to copyright infringement or unfair competition. Moreover,
the description covers property that Maxicorp may have bought legitimately from
Microsoft or its licensed distributors. Paragraph (c) simply calls for the seizure of all
items bearing the Microsoft logo, whether legitimately possessed or not. Neither does it
limit the seizure to products used in copyright infringement or unfair competition.

Still, no provision of law exists which requires that a warrant, partially defective in
specifying some items sought to be seized yet particular with respect to the other items,
should be nullified as a whole. A partially defective warrant remains valid as to the items
specifically described in the warrant.44 A search warrant is severable, the items not
sufficiently described may be cut off without destroying the whole warrant.45 The
exclusionary rule found in Section 3(2) of Article III of the Constitution renders
inadmissible in any proceeding all evidence obtained through unreasonable searches
and seizure. Thus, all items seized under paragraph (c) of the search warrants, not
falling under paragraphs a, b, d, e or f, should be returned to Maxicorp.
Page 27 of 377

WHEREFORE, we PARTIALLY GRANT the instant petition. The Decision of the Court of
Appeals dated 23 December 1998 and its Resolution dated 29 November 1999 in CA-
G.R. SP No. 44777 are REVERSED and SET ASIDE except with respect to articles seized
under paragraph (c) of Search Warrants Nos. 96-451, 96-452, 96-453 and 96-454. All
articles seized under paragraph (c) of the search warrants, not falling under paragraphs
a, b, d, e or f, are ordered returned to Maxicorp, Inc. immediately.

SO ORDERED.

Davide, Jr., Quisumbing, Ynares-Santiago, and Azcuna, JJ., concur.


Page 28 of 377

G.R. No. 156804 March 14, 2005

SONY MUSIC ENTERTAINMENT (PHILS.), INC. and IFPI (SOUTHEAST ASIA),

REGIONAL TRIAL COURT, BRANCH 90, DASMARIÑAS, CAVITE, ELENA S.


LIM, SUSAN L. TAN, DAVID S. LIM, JAMES H. UY, WILSON ALEJANDRO,
JR., JOSEPH DE LUNA, MARIA A. VELA CRUZ, DAVID CHUNG, JAMES UY,
JOHN DOES AND JANE DOES, AND SOLID LAGUNA CORPORATION,
Respondents.

GARCIA, J.:

Assailed and sought to be nullified in this petition for certiorari with application for
injunctive relief are the orders issued by the respondent judge on June 25, 20021 and
January 6, 2003,2 the first quashing Search Warrant No. 219-00, and the second,
denying reconsideration of the first.

From the petition, the comment thereon of private respondents, their respective
annexes, and other pleadings filed by the parties, the Court gathers the following
relevant facts:

In a criminal complaint filed with the Department of Justice (DOJ), the Videogram
Regulatory Board (VRB)3 charged herein private respondents James Uy, David Chung,
Elena Lim and another officer of respondent Solid Laguna Corporation (SLC) with
violation of Presidential Decree (PD) No. 1987.4 As alleged in the complaint, docketed
as I.S. No. 2000-1576, the four (4) were engaged in the replication, reproduction and
distribution of videograms without license and authority from VRB. On account of this
and petitioners’ own complaints for copyright infringement, the National Bureau of
Investigation (NBI), through Agent Ferdinand M. Lavin, applied on September 18, 2000,
with the Regional Trial Court at Dasmariñas, Cavite, Branch 80, presided by the
respondent judge, for the issuance of search warrants against private respondents
David Chung, James Uy, John and Jane Does, doing business under the name and style
"Media Group" inside the factory and production facility of SLC at Solid corner Camado
Sts., Laguna International Industrial Park, Biñan, Laguna.5

During the proceedings on the application, Agent Lavin presented, as witnesses, Rodolfo
Pedralvez, a deputized agent of VRB, and Rene C. Baltazar, an investigator retained by
the law firm R.V. Domingo & Associates, petitioners’ attorney-in-fact. In their sworn
statements, the three stated that petitioners sought their assistance, complaining about
the manufacture, sale and distribution of various titles of compact discs (CDs) in
Page 29 of 377

violation of petitioners’ right as copyright owners; that acting on the complaint, Agent
Lavin and the witnesses conducted an investigation, in the course of which unnamed
persons informed them that allegedly infringing or pirated discs were being
manufactured somewhere in an industrial park in Laguna; that in the process of their
operation, they were able to enter, accompanied by another unnamed source, the
premises of SLC and to see various replicating equipment and stacks of CDs; and that
they were told by their anonymous source that the discs were being manufactured in
the same premises. They also testified that private respondents were (1) engaged in the
reproduction or replication of audio and video compacts discs without the requisite
authorization from VRB, in violation of Section 6 of PD No. 1987, presenting a VRB
certification to such effect; and (2) per petitioners’ certification and a listing of Sony
music titles, infringing on petitioners’ copyrights in violation of Section 208 of Republic
Act (RA) No. 8293, otherwise known as Intellectual Property Code.6

On the basis of the foregoing sworn statements, the respondent judge issued Search
Warrant No. 219-007 for violation of Section 208 of R.A. No. 8293 and Search Warrant
No. 220-008 for violation of Section 6 of PD No. 1987.

The following day, elements of the Philippine National Police Criminal Investigation and
Detection Group, led by PO2 Reggie Comandante, enforced both warrants and brought
the seized items to a private warehouse of Carepak Moving and Storage at 1234 Villonco
Road, Sucat, Paranaque City and their custody turned over to VRB.9 An inventory of
seized items,10 as well as a "Return of Search Warrant" were later filed with the
respondent court.

Meanwhile, the respondents in I.S. No. 2000-1576 belabored to prove before the DOJ
Prosecutorial Service that, since 1998 and up to the time of the search, they were
licensed by VRB to operate as replicator and duplicator of videograms.

On the stated finding that "respondents can not . . . be considered an unauthorized


reproducers of videograms", being "licensed to engage in reproduction in videograms
under SLC in which they are the officers and/or or officials", the DOJ, via a resolution
dated January 15, 2001,11 dismissed VRB’s complaint in I.S. No. 2000-1576.

On February 6, 2001, private respondents, armed with the DOJ resolution adverted to,
moved to quash the search warrants thus issued.12 VRB interposed an opposition for
the reason that the DOJ has yet to resolve the motion for reconsideration it filed in I.S.
No. 2000-1576.
Page 30 of 377

Eventually, the DOJ denied VRB’s motion for reconsideration, prompting private
respondents to move anew for the quashal of the search warrants. In its supplement to
motion, private respondents attached copies of SLC’s license as videogram duplicator
and replicator.

In an order dated October 30, 2001,13 the respondent judge, citing the January 15,
2001 DOJ resolution in I.S. No. 2000-1576, granted private respondents’ motion to
quash, as supplemented, dispositively stating:

"Nonetheless, such being the case, the aforesaid Search Warrants are QUASHED"

Petitioners forthwith sought clarification on whether or not the quashal order referred
to both search warrants or to Search Warrant No. 220-00 alone, since it was the latter
that was based on the charge of violation of PD No. 1987.14 The respondent judge, in a
modificatory order dated January 29, 2002,15 clarified that her previous order quashed
only Search Warrant No. 220-00.

Meanwhile, or on November 22, 2001, petitioners filed with the DOJ an affidavit-
complaint, docketed thereat as I.S. No. 2001-1158, charging individual private
respondents with copyright infringement in violation of Sections 172 and 208 in relation
to other provisions of RA No. 8293.16 Attached to the affidavit-complaint were certain
documents and records seized from SLC’s premises, such as production and delivery
records.

Following their receipt of DOJ-issued subpoenas to file counter-affidavits, private


respondents moved, in the search warrant case, that they be allowed to examine the
seized items to enable them to intelligently prepare their defense.17 On January 30,
2002, respondent judge issued an order allowing the desired examination, provided it
is made under the supervision of the court’s sheriff and in the "presence of the applicant
of Search Warrant No. 219-00".18

On February 8, 2002, the parties, represented by their counsels, repaired to the Carepak
warehouse. An NBI agent representing Agent Lavin appeared. The examination,
however, did not push through on account of petitioners’ counsel insistence on Agent
Lavin’s physical presence.19 Private respondents were able to make an examination on
the following scheduled setting, February 15, 2002, albeit it was limited, as the minutes
of the inspection discloses, to inspecting only one (1) box containing 35 assorted CDs,
testing stampers, diskettes, a calendar, organizers and some folders and documents.
The minutes also contained an entry stating - "Other items/machines were not
Page 31 of 377

examined because they cannot be identified as they are not properly segregated from
other items/machines in the warehouse. The parties agreed to schedule another
examination on (to be agreed by the parties) after the items/machines subject of the
examination shall have been segregated from the other items/machines by Carepak
Moving and Storage , Inc."20

During the preliminary investigation conducted on February 26, 2002 in I.S. No. 2001-
1158, however, petitioners’ counsel objected to any further examination, claiming that
such exercise was a mere subterfuge to delay proceedings.21

On April 11, 2002, individual private respondents, through counsel, filed a "Motion To
Quash Search Warrant (And To Release Seized Properties)" grounded on lack of probable
cause to justify issuance of search warrant, it being inter alia alleged that the applicant
and his witnesses lacked the requisite personal knowledge to justify the valid issuance
of a search warrant; that the warrant did not sufficiently describe the items to be seized;
and that the warrant was improperly enforced.22 To this motion to quash, petitioners
interposed an opposition dated May 7, 2002 predicated on four (4) grounds.23 On June
26, 2002, respondent SLC filed a Manifestation joining its co-respondents in, and
adopting, their motion to quash.24

On June 25, 2002, the respondent judge issued the herein first assailed order quashing
Search Warrant No. 219-00 principally on the ground that the integrity of the seized
items as evidence had been compromised, commingled as they were with other articles.
Wrote the respondent judge:

Based on the report submitted, it appears that on February 15, 2002, an examination
was actually conducted. Unfortunately, the alleged seized items were commingled with
and not segregated from thousands of other items stored in the warehouse. Only one
box . . . were (sic) examined in the presence of both parties with the sheriff, such that
another date was set . . . . On February 22, 2002, during the hearing before the
Department of Justice (DOJ), [petitioners’ counsel] Atty. Arevalo manifested their
objection to the further examination on the alleged ground that all of the items subject
of the DOJ complaint have been examined.

Analyzing the report and the incidents relative thereto, it shows that the items subject
of the questioned Search Warrant were commingled with other items in the warehouse
of Carepak resulting in the failure to identify the machines and other items subject of
this Search Warrant, while the other items enumerated in the said Inventory of Seized
Items and Certification of Legality, Orderliness and Regularity in the Execution and
enforcement of Search Warrants were not examined, hence, the charge imputed against
Page 32 of 377

the respondents could not be established as the evidence to show such violation fails to
determine the culpability of said respondents, thus, violating their constitutional
rights.25

Excepting, petitioners moved for reconsideration, arguing on the main that the quashal
order was erroneously based on a ground outside the purview of a motion to quash.26
To this motion, private respondents interposed an opposition, against which petitioners
countered with a reply.

On January 6, 2003, respondent judge issued the second assailed order denying
petitioners’ motion for reconsideration on the strength of the following premises:

Careful scrutiny of the records of the case reveals that the application of the above-
entitled case stemmed from the application for Search Warrant alleging that the
respondent was not licensed to duplicate or replicate CDs and VCDs. The Court was
misled when the applicants declared that Solid Laguna Corporation (SLC) is not licensed
to engage in replicating/duplicating CDs and VCDs, when in truth and in fact, SLC was
still a holder of a valid and existing VRB license. Considering the fact that respondent
was duly licensed which facts (sic) was not laid bare to this Court when the application
for writ was filed by the private complainant through the National Bureau of
Investigation, this Court hereby recalls and quashes the above writ.

Lastly, taking into account that respondents were licensed to engage in


replicating/duplicating CDs and VCDs, the issuance of search warrant was of no force
and effect as there was absence of probable cause to justify said issuance. xxx27

Hence, petitioners’ present recourse.

In a Resolution dated February 19, 2003,28 the Court issued a temporary restraining
order enjoining the respondents from implementing and enforcing the respondent
judge’s questioned orders.

Petitioners ascribe on the respondent judge the commission of grave abuse of discretion
amounting to lack or in excess of jurisdiction in issuing the first assailed order in that:
Page 33 of 377

1. It was based on a ground that is not a basis for quashal of a search warrant, i.e.,
private respondents’ failure to examine the seized items, which ground is extraneous to
the determination of the validity of the issuance of the search warrant.

2. Public respondent, in effect, conducted a "preliminary investigation" that absolved


the private respondents from any liability for copyright infringement.

3. Public respondent recognized the motion to quash search warrant filed by persons
who did not have any standing to question the warrant.

Petitioners also deplore the issuance of the second assailed order which they tag as
predicated on a ground immaterial to Search Warrant No. 219-00.

Private respondents filed their Comment on May 13, 2003, essentially reiterating their
arguments in the "Motion To Quash Search Warrant (And To Release Seized Properties)".
Apart therefrom, they aver that petitioners violated the rule on hierarchy of courts by
filing the petition directly with this Court. As to be expected, petitioners’ reply to
comment traversed private respondents’ position.

Owing to their inability to locate respondent David Chung, petitioners moved and the
Court subsequently approved the dropping, without prejudice, of said respondent from
the case.29

On February 20, 2004, private respondents filed their Rejoinder, therein inviting
attention to petitioner IFPI’s failure to execute the certification on non-forum shopping
as required by Rule 7, Section 5 of the Rules of Court and questioning the validity of the
Special Powers of Attorney of petitioners’ attorney-in-fact to file this case.

In Resolution of March 31, 2004, the Court gave due course to the petition and directed
the submission of memoranda which the parties, after each securing an extension, did
submit.

The underlying issue before Us revolves on the propriety of the quashal of Search
Warrant No. 219-00 which, in turn, resolves itself into question of the propriety of the
warrant’s issuance in the first place.
Page 34 of 377

It has repeatedly been said that one’s house, however, humble is his castle where his
person, papers and effects shall be secured and whence he shall enjoy undisturbed
privacy except, to borrow from Villanueva vs. Querubin,30 "in case of overriding social
need and then only under the stringent procedural safeguards." The protection against
illegal searches and seizure has found its way into our 1935 and 1973 Constitutions
and is now embodied in Article III, Section 2 of the 1987 Constitution, thus -

The right of the people to be secure in their persons, houses, papers and effects against
unreasonable searches and seizures of whatever nature and for any purpose shall be
inviolable, and no search warrant or warrant of arrest shall issue except upon probable
cause to be determined personally by the judge after examination under oath or
affirmation of the complainant and the witnesses he may produce, and particularly
describing the place to be searched and the persons or things to be seized,

and in Section 4, Rule 126 of the Rules of Court, viz -

Sec. 4. Requisites for issuing search warrant. – A search warrant shall not issue but
upon probable cause . . . to be determined personally by the judge after examination
under oath or affirmation of the complainant and the witnesses he may produce, and
particularly describing the place to be searched and the things to be seized.

Complementing the aforequoted provisions is Section 5 of the same Rule, reading:

SEC. 5. Examination of the complainant; record. The judge must, before issuing the
warrant, personally examine in form of searching questions and answers, in writing and
under oath, the complainant and any witnesses he may produce on facts personally
known to them and attach to the record their sworn statements together with any
affidavits submitted.

To prevent stealthy encroachment upon, or gradual depreciation of the right to privacy,


a liberal construction in search and seizure cases is given in favor of the individual.
Consistent with this postulate, the presumption of regularity is unavailing in aid of the
search process when an officer undertakes to justify it.31 For, the presumption juris
tantum of regularity cannot, by itself, prevail against the constitutionally protected
rights of an individual because zeal in the pursuit of criminals cannot ennoble the use
of arbitrary methods that the Constitution itself detests.32
Page 35 of 377

A core requisite before a warrant shall validly issue is the existence of a probable cause,
meaning "the existence of such facts and circumstances which would lead a reasonably
discreet and prudent man to believe that an offense has been committed and that the
objects sought in connection with the offense are in the place to be searched".33 And
when the law speaks of facts, the reference is to facts, data or information personally
known to the applicant and the witnesses he may present. Absent the element of
personal knowledge by the applicant or his witnesses of the facts upon which the
issuance of a search warrant may be justified, the warrant is deemed not based on
probable cause and is a nullity, its issuance being, in legal contemplation, arbitrary, as
held by us in Columbia Pictures, Inc. vs. Court of Appeals.34 Testimony based on what
is supposedly told to a witness, being patent hearsay and, as rule, of no evidentiary
weight35 or probative value, whether objected to or not,36 would, alone, not suffice
under the law on the existence of probable cause.

In our view, the issuance of the search warrant in question did not meet the
requirements of probable cause. The respondent judge did not accordingly err in
quashing the same, let alone gravely abuse her discretion.

Petitioners argue that the instant petition is on all fours with Columbia,37 wherein the
en banc Court upheld the validity of search warrants based on the testimonies of the
applicant and his witnesses who conducted an investigation on the unlawful
reproduction and distribution of video tapes of copyrighted films.

We are not persuaded.

In Columbia, the issuing court probed the applicant’s and his witnesses’ personal
knowledge of the fact of infringement. It was, however, determined by this Court that
during the application hearing, therein petitioner’s attorney-in-fact, a witness of the
applicant, "stated in his affidavit and further expounded in his deposition that he
personally knew of the fact that private respondents had never been authorized by his
clients to reproduce, lease and possess for the purposes of selling any of the copyrighted
films."38 Significantly, the Court, in upholding the validity of the writ issued upon
complaint of Columbia Pictures, Inc., et al., stated that "there is no allegation of
misrepresentation, much less finding thereof by the lower court, on the part of
petitioners’ witnesses."39

Therein lies the difference with the instant case.


Page 36 of 377

Here, applicant Agent Lavin and his witnesses, Pedralvez and Baltazar, when queried
during the application hearing how they knew that audio and video compact discs were
infringing or pirated, relied for the most part on what alleged unnamed sources told
them and/or on certifications or lists made by persons who were never presented as
witnesses. In net effect, they testified under oath as to the truth of facts they had no
personal knowledge of. The following excerpts of the depositions of applicant Lavin and
his witnesses suggest as much:

A. Deposition of Agent Lavin

28. Question: What happened next?

Answer: We then went to the Laguna Industrial Park, your Honor . . . We then verified
from an informant that David Chung, James Uy . . . under the name and style Media
Group were the ones replicating the infringing CDs.

xxx xxx xxx

36. Question: How do you know that all of these VCDs and CDs you purchased or are
indeed infringing?

Answer: I have with me the VRB certification that the VCDs are unauthorized copies. I
also have with me the Complaint-Affidavit of Sony Music and IFPI that certified that
these are infringing copies, as well as the title list of Sony Music wherein some of the
CDs purchased are indicated. (Annex "10", Comment, Rollo, p. 841)

B. Deposition of Baltazar

18. Question: What did you see in that address?

Answer: We saw that they had in stock several infringing, pirated and unauthorized
CDs. They also had videograms without VRB labels, aside from artworks and labels.
John Doe gave us a "Wholesome" CD while Jane Doe gave us "Kenny Rogers Videoke"
and "Engelbert Humperdinck Videoke" which the informant told us were being
reproduced in that facility. The informant further showed us the rooms where the
replicating and/or stamping machine was located.
Page 37 of 377

19. Question: How did you determine that the CDs you purchased are counterfeit,
pirated or unauthorized?

Answer: The Attorney-in-fact of Sony Music and IFPI certified in his Complaint-Affidavit
that they are unauthorized copies. I also have with me a listing of Sony Music titles and
some of the CDs I purchased are in that list.40

C. Deposition of Pedralvez

27. Question: What proof do you have they are producing infringing materials?

Answer: We were given some samples by John Doe and Jane Doe. These are Kenny
Rogers Videoke, Engelbert Humperdinck Videoke, and Andrew E. Wholesome CD. The
informant told us that the said samples were being reproduced in the facility.

28. Question: How do you know that all of these VCDs you purchased or got are indeed
unauthorized?

Answer: The VRB has certified that they are unauthorized copies. (Annex "12",
Comment, Rollo, pp. 849-852).

Unlike their counterparts in Columbia who were found to be personally knowledgeable


about their facts, Agent Lavin and his witnesses, judging from their above quoted
answers, had no personal knowledge that the discs they saw, purchased or received
were, in fact, pirated or infringing on petitioners’ copyrights. To us, it is not enough that
the applicant and his witnesses testify that they saw stacks of several allegedly
infringing, pirated and unauthorized discs in the subject facility. The more decisive
consideration determinative of whether or not a probable cause obtains to justify the
issuance of a search warrant is that they had personal knowledge that the discs were
actually infringing, pirated or unauthorized copies.41

Moreover, unlike in Columbia, misrepresentation on the part of the applicant and his
witnesses had been established in this case.
Page 38 of 377

This is not to say that the master tapes should have been presented in evidence during
the application hearing, as private respondents, obviously having in mind the holding
in 20th Century Fox Film Corp. vs. Court of Appeals,42 would have this Court believe.
It is true that the Court, in 20th Century Fox, underscored the necessity, in determining
the existence of probable cause in copyright infringement cases, of presenting the
master tapes of the copyrighted work. But, as emphatically clarified in Columbia "such
auxiliary procedure, however, does not rule out the use of testimonial or documentary
evidence, depositions, admissions or other classes of evidence xxx especially where the
production in court of object evidence would result in delay, inconvenience or expenses
out of proportion to its evidentiary value.43 What this Court is saying is that any
evidence presented in lieu of the master tapes, if not readily available, in similar
application proceedings must be reliable, and, if testimonial, it must, at the very least,
be based on the witness’ personal knowledge.

Petitioners argue, citing People v. Chua Uy,44 that Agent Lavin’s informants’ testimonies
are not indispensable as they would only be corroborative.45 Like Columbia, Chua Uy
is not a winning card for petitioners, for, in the latter case, there was a reliable testimony
to corroborate what the applicant testified to, i.e., the testimony of the police poseur-
buyer in a buy-bust operation involving prohibited drugs. The circumstances are
different in this case wherein the applicant and his witnesses had no personal
knowledge that the discs they purchased were infringing or pirated copies. It cannot be
overemphasized that not one of them testified seeing the pirated discs being
manufactured at SLC’s premises. What they stated instead was that they were given
copies of "Kenny Rogers Videoke", "Engelbert Humperdinck Videoke" and "Andrew E.
Wholesome CD" by two anonymous sources, while yet another informant told them that
the discs were manufactured at said premises.

Initial hearsay information or tips from confidential informants could very well serve as
basis for the issuance of a search warrant, if followed up personally by the recipient and
validated,46 as what transpired in Columbia. Unfortunately, the records show that such
is not the case before us.

On the issue that the public respondent gravely abused her discretion in conducting
what petitioners perceived amounted to a "preliminary investigation", this Court has
already ruled in Solid Triangle Sales Corp. vs. Sheriff of RTC Quezon City, Branch 93,47
that "in the determination of probable cause, the court must necessarily resolve whether
or not an offense exists to justify the issuance or quashal of the warrant". In the exercise
of this mandate - which we can allow as being akin to conducting a preliminary
investigation - abuse of discretion cannot plausibly be laid at the doorstep of the issuing
court on account of its prima facie holding that no offense has been committed, even if
consequent to such holding a warrant is recalled and the private complainant is
incidentally deprived of vital evidence to prove his case. Solid Triangle succinctly
explains why:
Page 39 of 377

The proceedings for the issuance/quashal of a search warrant before a court on the one
hand, and the preliminary investigation before an authorized officer on the other, are
proceedings entirely independent of each other. One is not bound by the other’s finding
as regards the existence of a crime. The first is to determine whether a warrant should
issue or be quashed, and the second, whether an information should be filed in court.

When the court, in determining probable cause for issuing or quashing a search
warrant, finds that no offense has been committed, it does not interfere with or encroach
upon the proceedings in the preliminary investigation. The court does not oblige the
investigating officer not to file the information for the court’s ruling that no crime exists
is only of purposes of issuing or quashing the warrant. This does not, as petitioners
would like to believe, constitute a usurpation of the executive function. Indeed, to shirk
from this duty would amount to an abdication of a constitutional obligation.48

While the language of the first questioned Order may be viewed as encroaching on
executive functions, nonetheless, it remains that the order of quashal is entirely
independent of the proceedings in I.S. No. 2001-1158. And needless to stress, the DOJ
is by no means concluded by the respondent judge’s findings as regards the existence,
or the non-existence, of a crime.

We can, to a point, accord merit to petitioners’ lament that the basis of the first
questioned order, i.e., the mingling of the seized items with other items, is extraneous
to the determination of the validity of the issuance of the search warrant. It is to be
pointed out, though, that public respondent corrected her error when it was raised in
petitioners’ motion for reconsideration. There can really be no serious objection to a
judge correcting or altogether altering his case disposition on a motion for
reconsideration, it being the purpose of such recourse to provide the court an
opportunity to cleanse itself of an error unwittingly committed, or, with like effect, to
allow the aggrieved party the chance to convince the court that its ruling is erroneous.49
A motion for reconsideration before resort to certiorari is required precisely "to afford
the public respondent an opportunity to correct any actual or fancied error attributed
to it by way of re-examination of the legal and factual aspects of the case".50

Similarly, as to the matter of the respondent judge’s recognizing the April 11, 2002
motion to quash search warrant51 filed by the individual private respondents, instead
of by SLC, as presumptive owner of the seized items, such error was properly addressed
when respondent SLC, represented throughout the proceedings below by the same
counsel of its co-respondents, formally manifested that it was adopting the same motion
as its own.52
Page 40 of 377

It is apropos to point out at this juncture that petitioners have imputed on individual
private respondents criminal liability, utilizing as tools of indictment the very articles
and papers seized from the premises of SLC. Be that as it may, petitioners should be
deemed in estoppel to raise the personality of individual private respondents to interpose
a motion to quash. To be sure, it would be unsporting for petitioners to prosecute
individual private respondents on the basis of seized articles but on the same breath
deny the latter standing to question the legality of the seizure on the postulate that only
the party whose rights have been impaired thereby, meaning SLC, can raise that
challenge. There can be no quibbling that individual private respondents stand to be
prejudiced or at least be inconvenient by any judgment in any case based on the seized
properties. In a very real sense, therefore, they are real parties in interest who ought not
to be prevented from assailing the validity of Search Warrant 219-00, albeit they cannot
plausibly asked for the release and appropriate as their own the seized articles.

Petitioners’ related argument that SLC could not have validly adopted individual private
respondents’ motion to quash due to laches is untenable.

The records show that the seizure in question was effected on September 19, 2000. The
complaint in I.S. No. 2000-1576 was filed against the officers of SLC, all of whom, except
for one, are also private respondents in the instant petition. I.S. No. 2000-1576 was only
resolved on January 15, 2001 when the DOJ dismissed the complaint on the ground
that SLC was, in fact, duly licensed by the VRB. Shortly thereafter, or on February 6,
2001, less than five (5) months after the seizure, private respondents moved to quash
both search warrants.53 The motion clearly indicates private respondents’ desire for the
return of the seized items, and there is nothing in the records showing that petitioners
objected to the motion on the ground that the movants had no standing to question the
warrants.

This bring Us to the second assailed order. As earlier stated, DOJ, in I.S. No. 2000-
1576, found respondent SLC to be licensed by VRB to engage in the business of
replicating or duplicating videograms.

Petitioners would have the Court believe that the second questioned order was based on
a ground immaterial to the charge of infringement. A scrutiny of the text of the said
order, however, shows that the respondent judge denied petitioners’ motion for
reconsideration because she was misled by the applicant’s and his witnesses’ testimony.
It may be that a VRB license is no defense to a charge of violating Section 208 of R.A.
No. 8293. It must be stressed in this regard, however, that the core issue here is the
validity of the warrant which applicant secured on the basis of, among others, his
representation which turned out to be false.
Page 41 of 377

As above discussed, the answers of Agent Lavin and his witnesses to the public
respondent’s searching questions, particularly those relating to how they knew that the
compact discs they purchased or received were illegal, unauthorized or infringing, were
based on certifications and not personal knowledge. The subject warrant, as well as
Search Warrant No. 220-00, was issued nonetheless. It may well have been that the
issuing judge was, in the end, convinced to issue the warrants by means of the
erroneous VRB certification presented during the joint application hearing, overriding
whatever misgivings she may have had with the applicant’s and his witnesses’ other
answers. This Court, however, cannot engage in such speculation and sees no need to.

Summing up, the issuance of Search Warrant No. 219-00 was, at bottom, predicated on
the sworn testimonies of persons without personal knowledge of facts they were
testifying on and who relied on a false certification issued by VRB. Based as it were on
hearsay and false information, its issuance was without probable cause and, therefore,
invalid.

Given the foregoing perspective, the peripheral issues of (a) whether or not petitioner
IFPI (South East Asia), Ltd. failed to comply with the rules requiring the filing of a
certification on non-forum shopping; and (b) whether or not IFPI’s board of directors
ratified its conditional authorization for its attorney-in-fact to represent IFPI in this
petition, need not detain us long. In our review of the records, R.V. Domingo &
Associates, whose authority to represent the petitioners in this petition continues, had
duly executed the sworn certification on non- forum shopping.

In the same manner, this Court, having taken cognizance of this petition, need not
belabor the issue of whether or not petitioners have cavalierly breached the rule on
hierarchy of courts. Suffice it to state that, while the Court looks with disfavor on utter
disregard of its rules,54 it is within its power to suspend its own rules or to except a
particular case from its operation whenever the ends of justice so requires, as here.

WHEREFORE, the instant petition is hereby DISMISSED and the temporary restraining
order issued on February 19, 2003 is consequently RECALLED.
Costs against petitioners.

SO ORDERED.
Panganiban, (Chairman), Sandoval-Gutierrez, and Corona, JJ., concur.
Carpio-Morales, J., on leave.
Page 42 of 377

G.R. No. L-19550 June 19, 1967

HARRY S. STONEHILL, ROBERT P. BROOKS, JOHN J. BROOKS and KARL


BECK, petitioners, vs. HON. JOSE W. DIOKNO, in his capacity as
SECRETARY OF JUSTICE; JOSE LUKBAN, in his capacity as Acting
Director, National Bureau of Investigation; SPECIAL PROSECUTORS PEDRO
D. CENZON, EFREN I. PLANA and MANUEL VILLAREAL, JR. and ASST.
FISCAL MANASES G. REYES; JUDGE AMADO ROAN, Municipal Court of
Manila; JUDGE ROMAN CANSINO, Municipal Court of Manila; JUDGE
HERMOGENES CALUAG, Court of First Instance of Rizal-Quezon City
Branch, and JUDGE DAMIAN JIMENEZ, Municipal Court of Quezon City,
respondents.

Paredes, Poblador, Cruz and Nazareno and Meer, Meer and Meer and Juan T. David for
petitioners.
Office of the Solicitor General Arturo A. Alafriz, Assistant Solicitor General Pacifico P. de
Castro, Assistant Solicitor General Frine C. Zaballero, Solicitor Camilo D. Quiason and
Solicitor C. Padua for respondents.

CONCEPCION, C.J.:

Upon application of the officers of the government named on the margin1 — hereinafter
referred to as Respondents-Prosecutors — several judges2 — hereinafter referred to as
Respondents-Judges — issued, on different dates,3 a total of 42 search warrants against
petitioners herein4 and/or the corporations of which they were officers,5 directed to the
any peace officer, to search the persons above-named and/or the premises of their
offices, warehouses and/or residences, and to seize and take possession of the following
personal property to wit:

Books of accounts, financial records, vouchers, correspondence, receipts, ledgers,


journals, portfolios, credit journals, typewriters, and other documents and/or papers
showing all business transactions including disbursements receipts, balance sheets and
profit and loss statements and Bobbins (cigarette wrappers).

as "the subject of the offense; stolen or embezzled and proceeds or fruits of the offense,"
or "used or intended to be used as the means of committing the offense," which is
described in the applications adverted to above as "violation of Central Bank Laws, Tariff
and Customs Laws, Internal Revenue (Code) and the Revised Penal Code."
Page 43 of 377

Alleging that the aforementioned search warrants are null and void, as contravening the
Constitution and the Rules of Court — because, inter alia: (1) they do not describe with
particularity the documents, books and things to be seized; (2) cash money, not
mentioned in the warrants, were actually seized; (3) the warrants were issued to fish
evidence against the aforementioned petitioners in deportation cases filed against them;
(4) the searches and seizures were made in an illegal manner; and (5) the documents,
papers and cash money seized were not delivered to the courts that issued the warrants,
to be disposed of in accordance with law — on March 20, 1962, said petitioners filed
with the Supreme Court this original action for certiorari, prohibition, mandamus and
injunction, and prayed that, pending final disposition of the present case, a writ of
preliminary injunction be issued restraining Respondents-Prosecutors, their agents and
/or representatives from using the effects seized as aforementioned or any copies
thereof, in the deportation cases already adverted to, and that, in due course, thereafter,
decision be rendered quashing the contested search warrants and declaring the same
null and void, and commanding the respondents, their agents or representatives to
return to petitioners herein, in accordance with Section 3, Rule 67, of the Rules of Court,
the documents, papers, things and cash moneys seized or confiscated under the search
warrants in question.

In their answer, respondents-prosecutors alleged, 6 (1) that the contested search


warrants are valid and have been issued in accordance with law; (2) that the defects of
said warrants, if any, were cured by petitioners' consent; and (3) that, in any event, the
effects seized are admissible in evidence against herein petitioners, regardless of the
alleged illegality of the aforementioned searches and seizures.

On March 22, 1962, this Court issued the writ of preliminary injunction prayed for in
the petition. However, by resolution dated June 29, 1962, the writ was partially lifted or
dissolved, insofar as the papers, documents and things seized from the offices of the
corporations above mentioned are concerned; but, the injunction was maintained as
regards the papers, documents and things found and seized in the residences of
petitioners herein.7

Thus, the documents, papers, and things seized under the alleged authority of the
warrants in question may be split into two (2) major groups, namely: (a) those found
and seized in the offices of the aforementioned corporations, and (b) those found and
seized in the residences of petitioners herein.

As regards the first group, we hold that petitioners herein have no cause of action to
assail the legality of the contested warrants and of the seizures made in pursuance
thereof, for the simple reason that said corporations have their respective personalities,
separate and distinct from the personality of herein petitioners, regardless of the
Page 44 of 377

amount of shares of stock or of the interest of each of them in said corporations, and
whatever the offices they hold therein may be.8 Indeed, it is well settled that the legality
of a seizure can be contested only by the party whose rights have been impaired
thereby,9 and that the objection to an unlawful search and seizure is purely personal
and cannot be availed of by third parties. 10 Consequently, petitioners herein may not
validly object to the use in evidence against them of the documents, papers and things
seized from the offices and premises of the corporations adverted to above, since the
right to object to the admission of said papers in evidence belongs exclusively to the
corporations, to whom the seized effects belong, and may not be invoked by the
corporate officers in proceedings against them in their individual capacity. 11 Indeed, it
has been held:

. . . that the Government's action in gaining possession of papers belonging to the


corporation did not relate to nor did it affect the personal defendants. If these papers
were unlawfully seized and thereby the constitutional rights of or any one were invaded,
they were the rights of the corporation and not the rights of the other defendants. Next,
it is clear that a question of the lawfulness of a seizure can be raised only by one whose
rights have been invaded. Certainly, such a seizure, if unlawful, could not affect the
constitutional rights of defendants whose property had not been seized or the privacy of
whose homes had not been disturbed; nor could they claim for themselves the benefits
of the Fourth Amendment, when its violation, if any, was with reference to the rights of
another. Remus vs. United States (C.C.A.)291 F. 501, 511. It follows, therefore, that the
question of the admissibility of the evidence based on an alleged unlawful search and
seizure does not extend to the personal defendants but embraces only the corporation
whose property was taken. . . . (A Guckenheimer & Bros. Co. vs. United States, [1925]
3 F. 2d. 786, 789, Emphasis supplied.)

With respect to the documents, papers and things seized in the residences of petitioners
herein, the aforementioned resolution of June 29, 1962, lifted the writ of preliminary
injunction previously issued by this Court, 12 thereby, in effect, restraining herein
Respondents-Prosecutors from using them in evidence against petitioners herein.

In connection with said documents, papers and things, two (2) important questions need
be settled, namely: (1) whether the search warrants in question, and the searches and
seizures made under the authority thereof, are valid or not, and (2) if the answer to the
preceding question is in the negative, whether said documents, papers and things may
be used in evidence against petitioners herein.1äwphï1.ñët

Petitioners maintain that the aforementioned search warrants are in the nature of
general warrants and that accordingly, the seizures effected upon the authority there of
are null and void. In this connection, the Constitution 13 provides:
Page 45 of 377

The right of the people to be secure in their persons, houses, papers, and effects against
unreasonable searches and seizures shall not be violated, and no warrants shall issue
but upon probable cause, to be determined by the judge after examination under oath
or affirmation of the complainant and the witnesses he may produce, and particularly
describing the place to be searched, and the persons or things to be seized.

Two points must be stressed in connection with this constitutional mandate, namely:
(1) that no warrant shall issue but upon probable cause, to be determined by the judge
in the manner set forth in said provision; and (2) that the warrant shall particularly
describe the things to be seized.

None of these requirements has been complied with in the contested warrants. Indeed,
the same were issued upon applications stating that the natural and juridical person
therein named had committed a "violation of Central Ban Laws, Tariff and Customs
Laws, Internal Revenue (Code) and Revised Penal Code." In other words, no specific
offense had been alleged in said applications. The averments thereof with respect to the
offense committed were abstract. As a consequence, it was impossible for the judges
who issued the warrants to have found the existence of probable cause, for the same
presupposes the introduction of competent proof that the party against whom it is
sought has performed particular acts, or committed specific omissions, violating a given
provision of our criminal laws. As a matter of fact, the applications involved in this case
do not allege any specific acts performed by herein petitioners. It would be the legal
heresy, of the highest order, to convict anybody of a "violation of Central Bank Laws,
Tariff and Customs Laws, Internal Revenue (Code) and Revised Penal Code," — as
alleged in the aforementioned applications — without reference to any determinate
provision of said laws or

To uphold the validity of the warrants in question would be to wipe out completely one
of the most fundamental rights guaranteed in our Constitution, for it would place the
sanctity of the domicile and the privacy of communication and correspondence at the
mercy of the whims caprice or passion of peace officers. This is precisely the evil sought
to be remedied by the constitutional provision above quoted — to outlaw the so-called
general warrants. It is not difficult to imagine what would happen, in times of keen
political strife, when the party in power feels that the minority is likely to wrest it, even
though by legal means.

Such is the seriousness of the irregularities committed in connection with the disputed
search warrants, that this Court deemed it fit to amend Section 3 of Rule 122 of the
former Rules of Court 14 by providing in its counterpart, under the Revised Rules of
Court 15 that "a search warrant shall not issue but upon probable cause in connection
Page 46 of 377

with one specific offense." Not satisfied with this qualification, the Court added thereto
a paragraph, directing that "no search warrant shall issue for more than one specific
offense."

The grave violation of the Constitution made in the application for the contested search
warrants was compounded by the description therein made of the effects to be searched
for and seized, to wit:

Books of accounts, financial records, vouchers, journals, correspondence, receipts,


ledgers, portfolios, credit journals, typewriters, and other documents and/or papers
showing all business transactions including disbursement receipts, balance sheets and
related profit and loss statements.

Thus, the warrants authorized the search for and seizure of records pertaining to all
business transactions of petitioners herein, regardless of whether the transactions were
legal or illegal. The warrants sanctioned the seizure of all records of the petitioners and
the aforementioned corporations, whatever their nature, thus openly contravening the
explicit command of our Bill of Rights — that the things to be seized be particularly
described — as well as tending to defeat its major objective: the elimination of general
warrants.

Relying upon Moncado vs. People's Court (80 Phil. 1), Respondents-Prosecutors
maintain that, even if the searches and seizures under consideration were
unconstitutional, the documents, papers and things thus seized are admissible in
evidence against petitioners herein. Upon mature deliberation, however, we are
unanimously of the opinion that the position taken in the Moncado case must be
abandoned. Said position was in line with the American common law rule, that the
criminal should not be allowed to go free merely "because the constable has blundered,"
16 upon the theory that the constitutional prohibition against unreasonable searches
and seizures is protected by means other than the exclusion of evidence unlawfully
obtained, 17 such as the common-law action for damages against the searching officer,
against the party who procured the issuance of the search warrant and against those
assisting in the execution of an illegal search, their criminal punishment, resistance,
without liability to an unlawful seizure, and such other legal remedies as may be
provided by other laws.

However, most common law jurisdictions have already given up this approach and
eventually adopted the exclusionary rule, realizing that this is the only practical means
of enforcing the constitutional injunction against unreasonable searches and seizures.
In the language of Judge Learned Hand:
Page 47 of 377

As we understand it, the reason for the exclusion of evidence competent as such, which
has been unlawfully acquired, is that exclusion is the only practical way of enforcing
the constitutional privilege. In earlier times the action of trespass against the offending
official may have been protection enough; but that is true no longer. Only in case the
prosecution which itself controls the seizing officials, knows that it cannot profit by their
wrong will that wrong be repressed.18

In fact, over thirty (30) years before, the Federal Supreme Court had already declared:

If letters and private documents can thus be seized and held and used in evidence
against a citizen accused of an offense, the protection of the 4th Amendment, declaring
his rights to be secure against such searches and seizures, is of no value, and, so far as
those thus placed are concerned, might as well be stricken from the Constitution. The
efforts of the courts and their officials to bring the guilty to punishment, praiseworthy
as they are, are not to be aided by the sacrifice of those great principles established by
years of endeavor and suffering which have resulted in their embodiment in the
fundamental law of the land.19

This view was, not only reiterated, but, also, broadened in subsequent decisions on the
same Federal Court. 20 After reviewing previous decisions thereon, said Court held, in
Mapp vs. Ohio (supra.):

. . . Today we once again examine the Wolf's constitutional documentation of the right
of privacy free from unreasonable state intrusion, and after its dozen years on our books,
are led by it to close the only courtroom door remaining open to evidence secured by
official lawlessness in flagrant abuse of that basic right, reserved to all persons as a
specific guarantee against that very same unlawful conduct. We hold that all evidence
obtained by searches and seizures in violation of the Constitution is, by that same
authority, inadmissible in a State.

Since the Fourth Amendment's right of privacy has been declared enforceable against
the States through the Due Process Clause of the Fourteenth, it is enforceable against
them by the same sanction of exclusion as it used against the Federal Government.
Were it otherwise, then just as without the Weeks rule the assurance against
unreasonable federal searches and seizures would be "a form of words," valueless and
underserving of mention in a perpetual charter of inestimable human liberties, so too,
without that rule the freedom from state invasions of privacy would be so ephemeral
and so neatly severed from its conceptual nexus with the freedom from all brutish means
of coercing evidence as not to permit this Court's high regard as a freedom "implicit in
Page 48 of 377

the concept of ordered liberty." At the time that the Court held in Wolf that the
amendment was applicable to the States through the Due Process Clause, the cases of
this Court as we have seen, had steadfastly held that as to federal officers the Fourth
Amendment included the exclusion of the evidence seized in violation of its provisions.
Even Wolf "stoutly adhered" to that proposition. The right to when conceded operatively
enforceable against the States, was not susceptible of destruction by avulsion of the
sanction upon which its protection and enjoyment had always been deemed dependent
under the Boyd, Weeks and Silverthorne Cases. Therefore, in extending the substantive
protections of due process to all constitutionally unreasonable searches — state or
federal — it was logically and constitutionally necessarily that the exclusion doctrine —
an essential part of the right to privacy — be also insisted upon as an essential
ingredient of the right newly recognized by the Wolf Case. In short, the admission of the
new constitutional Right by Wolf could not tolerate denial of its most important
constitutional privilege, namely, the exclusion of the evidence which an accused had
been forced to give by reason of the unlawful seizure. To hold otherwise is to grant the
right but in reality to withhold its privilege and enjoyment. Only last year the Court itself
recognized that the purpose of the exclusionary rule to "is to deter — to compel respect
for the constitutional guaranty in the only effectively available way — by removing the
incentive to disregard it" . . . .

The ignoble shortcut to conviction left open to the State tends to destroy the entire
system of constitutional restraints on which the liberties of the people rest. Having once
recognized that the right to privacy embodied in the Fourth Amendment is enforceable
against the States, and that the right to be secure against rude invasions of privacy by
state officers is, therefore constitutional in origin, we can no longer permit that right to
remain an empty promise. Because it is enforceable in the same manner and to like
effect as other basic rights secured by its Due Process Clause, we can no longer permit
it to be revocable at the whim of any police officer who, in the name of law enforcement
itself, chooses to suspend its enjoyment. Our decision, founded on reason and truth,
gives to the individual no more than that which the Constitution guarantees him to the
police officer no less than that to which honest law enforcement is entitled, and, to the
courts, that judicial integrity so necessary in the true administration of justice.
(emphasis ours.)

Indeed, the non-exclusionary rule is contrary, not only to the letter, but also, to the
spirit of the constitutional injunction against unreasonable searches and seizures. To
be sure, if the applicant for a search warrant has competent evidence to establish
probable cause of the commission of a given crime by the party against whom the
warrant is intended, then there is no reason why the applicant should not comply with
the requirements of the fundamental law. Upon the other hand, if he has no such
competent evidence, then it is not possible for the Judge to find that there is probable
cause, and, hence, no justification for the issuance of the warrant. The only possible
explanation (not justification) for its issuance is the necessity of fishing evidence of the
Page 49 of 377

commission of a crime. But, then, this fishing expedition is indicative of the absence of
evidence to establish a probable cause.

Moreover, the theory that the criminal prosecution of those who secure an illegal search
warrant and/or make unreasonable searches or seizures would suffice to protect the
constitutional guarantee under consideration, overlooks the fact that violations thereof
are, in general, committed By agents of the party in power, for, certainly, those belonging
to the minority could not possibly abuse a power they do not have. Regardless of the
handicap under which the minority usually — but, understandably — finds itself in
prosecuting agents of the majority, one must not lose sight of the fact that the
psychological and moral effect of the possibility 21 of securing their conviction, is
watered down by the pardoning power of the party for whose benefit the illegality had
been committed.

In their Motion for Reconsideration and Amendment of the Resolution of this Court
dated June 29, 1962, petitioners allege that Rooms Nos. 81 and 91 of Carmen
Apartments, House No. 2008, Dewey Boulevard, House No. 1436, Colorado Street, and
Room No. 304 of the Army-Navy Club, should be included among the premises
considered in said Resolution as residences of herein petitioners, Harry S. Stonehill,
Robert P. Brook, John J. Brooks and Karl Beck, respectively, and that, furthermore, the
records, papers and other effects seized in the offices of the corporations above referred
to include personal belongings of said petitioners and other effects under their exclusive
possession and control, for the exclusion of which they have a standing under the latest
rulings of the federal courts of federal courts of the United States. 22

We note, however, that petitioners' theory, regarding their alleged possession of and
control over the aforementioned records, papers and effects, and the alleged "personal"
nature thereof, has Been Advanced, not in their petition or amended petition herein, but
in the Motion for Reconsideration and Amendment of the Resolution of June 29, 1962.
In other words, said theory would appear to be readjustment of that followed in said
petitions, to suit the approach intimated in the Resolution sought to be reconsidered
and amended. Then, too, some of the affidavits or copies of alleged affidavits attached
to said motion for reconsideration, or submitted in support thereof, contain either
inconsistent allegations, or allegations inconsistent with the theory now advanced by
petitioners herein.

Upon the other hand, we are not satisfied that the allegations of said petitions said
motion for reconsideration, and the contents of the aforementioned affidavits and other
papers submitted in support of said motion, have sufficiently established the facts or
conditions contemplated in the cases relied upon by the petitioners; to warrant
application of the views therein expressed, should we agree thereto. At any rate, we do
Page 50 of 377

not deem it necessary to express our opinion thereon, it being best to leave the matter
open for determination in appropriate cases in the future.

We hold, therefore, that the doctrine adopted in the Moncado case must be, as it is
hereby, abandoned; that the warrants for the search of three (3) residences of herein
petitioners, as specified in the Resolution of June 29, 1962, are null and void; that the
searches and seizures therein made are illegal; that the writ of preliminary injunction
heretofore issued, in connection with the documents, papers and other effects thus
seized in said residences of herein petitioners is hereby made permanent; that the writs
prayed for are granted, insofar as the documents, papers and other effects so seized in
the aforementioned residences are concerned; that the aforementioned motion for
Reconsideration and Amendment should be, as it is hereby, denied; and that the petition
herein is dismissed and the writs prayed for denied, as regards the documents, papers
and other effects seized in the twenty-nine (29) places, offices and other premises
enumerated in the same Resolution, without special pronouncement as to costs.

It is so ordered.

Reyes, J.B.L., Dizon, Makalintal, Bengzon, J.P., Zaldivar and Sanchez, JJ., concur.

CASTRO, J., concurring and dissenting:

From my analysis of the opinion written by Chief Justice Roberto Concepcion and from
the import of the deliberations of the Court on this case, I gather the following distinct
conclusions:

1. All the search warrants served by the National Bureau of Investigation in this case
are general warrants and are therefore proscribed by, and in violation of, paragraph 3
of section 1 of Article III (Bill of Rights) of the Constitution;

2. All the searches and seizures conducted under the authority of the said search
warrants were consequently illegal;

3. The non-exclusionary rule enunciated in Moncado vs. People, 80 Phil. 1, should be,
and is declared, abandoned;
Page 51 of 377

4. The search warrants served at the three residences of the petitioners are expressly
declared null and void the searches and seizures therein made are expressly declared
illegal; and the writ of preliminary injunction heretofore issued against the use of the
documents, papers and effect seized in the said residences is made permanent; and

5. Reasoning that the petitioners have not in their pleadings satisfactorily demonstrated
that they have legal standing to move for the suppression of the documents, papers and
effects seized in the places other than the three residences adverted to above, the opinion
written by the Chief Justice refrains from expressly declaring as null and void the such
warrants served at such other places and as illegal the searches and seizures made
therein, and leaves "the matter open for determination in appropriate cases in the
future."

It is precisely the position taken by the Chief Justice summarized in the immediately
preceding paragraph (numbered 5) with which I am not in accord.

I do not share his reluctance or unwillingness to expressly declare, at this time, the
nullity of the search warrants served at places other than the three residences, and the
illegibility of the searches and seizures conducted under the authority thereof. In my
view even the exacerbating passions and prejudices inordinately generated by the
environmental political and moral developments of this case should not deter this Court
from forthrightly laying down the law not only for this case but as well for future cases
and future generations. All the search warrants, without exception, in this case are
admittedly general, blanket and roving warrants and are therefore admittedly and
indisputably outlawed by the Constitution; and the searches and seizures made were
therefore unlawful. That the petitioners, let us assume in gratia argumente, have no
legal standing to ask for the suppression of the papers, things and effects seized from
places other than their residences, to my mind, cannot in any manner affect, alter or
otherwise modify the intrinsic nullity of the search warrants and the intrinsic illegality
of the searches and seizures made thereunder. Whether or not the petitioners possess
legal standing the said warrants are void and remain void, and the searches and seizures
were illegal and remain illegal. No inference can be drawn from the words of the
Constitution that "legal standing" or the lack of it is a determinant of the nullity or
validity of a search warrant or of the lawfulness or illegality of a search or seizure.

On the question of legal standing, I am of the conviction that, upon the pleadings
submitted to this Court the petitioners have the requisite legal standing to move for the
suppression and return of the documents, papers and effects that were seized from
places other than their family residences.
Page 52 of 377

Our constitutional provision on searches and seizures was derived almost verbatim from
the Fourth Amendment to the United States Constitution. In the many years of judicial
construction and interpretation of the said constitutional provision, our courts have
invariably regarded as doctrinal the pronouncement made on the Fourth Amendment
by federal courts, especially the Federal Supreme Court and the Federal Circuit Courts
of Appeals.

The U.S. doctrines and pertinent cases on standing to move for the suppression or
return of documents, papers and effects which are the fruits of an unlawful search and
seizure, may be summarized as follows; (a) ownership of documents, papers and effects
gives "standing;" (b) ownership and/or control or possession — actual or constructive
— of premises searched gives "standing"; and (c) the "aggrieved person" doctrine where
the search warrant and the sworn application for search warrant are "primarily" directed
solely and exclusively against the "aggrieved person," gives "standing."

An examination of the search warrants in this case will readily show that, excepting
three, all were directed against the petitioners personally. In some of them, the
petitioners were named personally, followed by the designation, "the President and/or
General Manager" of the particular corporation. The three warrants excepted named
three corporate defendants. But the "office/house/warehouse/premises" mentioned in
the said three warrants were also the same "office/house/warehouse/premises"
declared to be owned by or under the control of the petitioners in all the other search
warrants directed against the petitioners and/or "the President and/or General
Manager" of the particular corporation. (see pages 5-24 of Petitioners' Reply of April 2,
1962). The searches and seizures were to be made, and were actually made, in the
"office/house/warehouse/premises" owned by or under the control of the petitioners.

Ownership of matters seized gives "standing."

Ownership of the properties seized alone entitles the petitioners to bring a motion to
return and suppress, and gives them standing as persons aggrieved by an unlawful
search and seizure regardless of their location at the time of seizure. Jones vs. United
States, 362 U.S. 257, 261 (1960) (narcotics stored in the apartment of a friend of the
defendant); Henzel vs. United States, 296 F. 2d. 650, 652-53 (5th Cir. 1961), (personal
and corporate papers of corporation of which the defendant was president), United
States vs. Jeffers, 342 U.S. 48 (1951) (narcotics seized in an apartment not belonging to
the defendant); Pielow vs. United States, 8 F. 2d 492, 493 (9th Cir. 1925) (books seized
from the defendant's sister but belonging to the defendant); Cf. Villano vs. United States,
310 F. 2d 680, 683 (10th Cir. 1962) (papers seized in desk neither owned by nor in
exclusive possession of the defendant).
Page 53 of 377

In a very recent case (decided by the U.S. Supreme Court on December 12, 1966), it was
held that under the constitutional provision against unlawful searches and seizures, a
person places himself or his property within a constitutionally protected area, be it his
home or his office, his hotel room or his automobile:

Where the argument falls is in its misapprehension of the fundamental nature and scope
of Fourth Amendment protection. What the Fourth Amendment protects is the security
a man relies upon when he places himself or his property within a constitutionally
protected area, be it his home or his office, his hotel room or his automobile. There he
is protected from unwarranted governmental intrusion. And when he puts some thing
in his filing cabinet, in his desk drawer, or in his pocket, he has the right to know it will
be secure from an unreasonable search or an unreasonable seizure. So it was that the
Fourth Amendment could not tolerate the warrantless search of the hotel room in
Jeffers, the purloining of the petitioner's private papers in Gouled, or the surreptitious
electronic surveilance in Silverman. Countless other cases which have come to this
Court over the years have involved a myriad of differing factual contexts in which the
protections of the Fourth Amendment have been appropriately invoked. No doubt, the
future will bring countless others. By nothing we say here do we either foresee or
foreclose factual situations to which the Fourth Amendment may be applicable. (Hoffa
vs. U.S., 87 S. Ct. 408 (December 12, 1966). See also U.S. vs. Jeffers, 342 U.S. 48, 72
S. Ct. 93 (November 13, 1951). (Emphasis supplied).

Control of premises searched gives "standing."

Independent of ownership or other personal interest in the records and documents


seized, the petitioners have standing to move for return and suppression by virtue of
their proprietary or leasehold interest in many of the premises searched. These
proprietary and leasehold interests have been sufficiently set forth in their motion for
reconsideration and need not be recounted here, except to emphasize that the
petitioners paid rent, directly or indirectly, for practically all the premises searched
(Room 91, 84 Carmen Apts; Room 304, Army & Navy Club; Premises 2008, Dewey
Boulevard; 1436 Colorado Street); maintained personal offices within the corporate
offices (IBMC, USTC); had made improvements or furnished such offices; or had paid
for the filing cabinets in which the papers were stored (Room 204, Army & Navy Club);
and individually, or through their respective spouses, owned the controlling stock of the
corporations involved. The petitioners' proprietary interest in most, if not all, of the
premises searched therefore independently gives them standing to move for the return
and suppression of the books, papers and affects seized therefrom.

In Jones vs. United States, supra, the U.S. Supreme Court delineated the nature and
extent of the interest in the searched premises necessary to maintain a motion to
Page 54 of 377

suppress. After reviewing what it considered to be the unduly technical standard of the
then prevailing circuit court decisions, the Supreme Court said (362 U.S. 266):

We do not lightly depart from this course of decisions by the lower courts. We are
persuaded, however, that it is unnecessarily and ill-advised to import into the law
surrounding the constitutional right to be free from unreasonable searches and seizures
subtle distinctions, developed and refined by the common law in evolving the body of
private property law which, more than almost any other branch of law, has been shaped
by distinctions whose validity is largely historical. Even in the area from which they
derive, due consideration has led to the discarding of those distinctions in the homeland
of the common law. See Occupiers' Liability Act, 1957, 5 and 6 Eliz. 2, c. 31, carrying
out Law Reform Committee, Third Report, Cmd. 9305. Distinctions such as those
between "lessee", "licensee," "invitee," "guest," often only of gossamer strength, ought
not be determinative in fashioning procedures ultimately referable to constitutional
safeguards. See also Chapman vs. United States, 354 U.S. 610, 616-17 (1961).

It has never been held that a person with requisite interest in the premises searched
must own the property seized in order to have standing in a motion to return and
suppress. In Alioto vs. United States, 216 F. Supp. 48 (1963), a Bookkeeper for several
corporations from whose apartment the corporate records were seized successfully
moved for their return. In United States vs. Antonelli, Fireworks Co., 53 F. Supp. 870,
873 (W D. N. Y. 1943), the corporation's president successfully moved for the return and
suppression is to him of both personal and corporate documents seized from his home
during the course of an illegal search:

The lawful possession by Antonelli of documents and property, "either his own or the
corporation's was entitled to protection against unreasonable search and seizure. Under
the circumstances in the case at bar, the search and seizure were unreasonable and
unlawful. The motion for the return of seized article and the suppression of the evidence
so obtained should be granted. (Emphasis supplied).

Time was when only a person who had property in interest in either the place searched
or the articles seize had the necessary standing to invoke the protection of the
exclusionary rule. But in MacDonald vs. Unite States, 335 U.S. 461 (1948), Justice
Robert Jackson joined by Justice Felix Frankfurter, advanced the view that "even a guest
may expect the shelter of the rooftree he is under against criminal intrusion." This view
finally became the official view of the U.S. Supreme Court and was articulated in United
States vs. Jeffers, 432 U.S 48 (1951). Nine years later, in 1960, in Jones vs. Unite States,
362 U.S. 257, 267, the U.S. Supreme Court went a step further. Jones was a mere guest
in the apartment unlawfully searched but the Court nonetheless declared that the
exclusionary rule protected him as well. The concept of "person aggrieved by an unlawful
Page 55 of 377

search and seizure" was enlarged to include "anyone legitimately on premise where the
search occurs."

Shortly after the U.S. Supreme Court's Jones decision the U.S. Court of Appeals for the
Fifth Circuit held that the defendant organizer, sole stockholder and president of a
corporation had standing in a mail fraud prosecution against him to demand the return
and suppression of corporate property. Henzel vs. United States, 296 F 2d 650, 652 (5th
Cir. 1961), supra. The court conclude that the defendant had standing on two
independent grounds: First — he had a sufficient interest in the property seized, and
second — he had an adequate interest in the premises searched (just like in the case at
bar). A postal inspector had unlawfully searched the corporation' premises and had
seized most of the corporation's book and records. Looking to Jones, the court observed:

Jones clearly tells us, therefore, what is not required qualify one as a "person aggrieved
by an unlawful search and seizure." It tells us that appellant should not have been
precluded from objecting to the Postal Inspector's search and seizure of the corporation's
books and records merely because the appellant did not show ownership or possession
of the books and records or a substantial possessory interest in the invade premises . .
. (Henzel vs. United States, 296 F. 2d at 651). .

Henzel was soon followed by Villano vs. United States, 310 F. 2d 680, 683, (10th Cir.
1962). In Villano, police officers seized two notebooks from a desk in the defendant's
place of employment; the defendant did not claim ownership of either; he asserted that
several employees (including himself) used the notebooks. The Court held that the
employee had a protected interest and that there also was an invasion of privacy. Both
Henzel and Villano considered also the fact that the search and seizure were "directed
at" the moving defendant. Henzel vs. United States, 296 F. 2d at 682; Villano vs. United
States, 310 F. 2d at 683.

In a case in which an attorney closed his law office, placed his files in storage and went
to Puerto Rico, the Court of Appeals for the Eighth Circuit recognized his standing to
move to quash as unreasonable search and seizure under the Fourth Amendment of the
U.S. Constitution a grand jury subpoena duces tecum directed to the custodian of his
files. The Government contended that the petitioner had no standing because the books
and papers were physically in the possession of the custodian, and because the
subpoena was directed against the custodian. The court rejected the contention, holding
that

Schwimmer legally had such possession, control and unrelinquished personal rights in
the books and papers as not to enable the question of unreasonable search and seizure
Page 56 of 377

to be escaped through the mere procedural device of compelling a third-party naked


possessor to produce and deliver them. Schwimmer vs. United States, 232 F. 2d 855,
861 (8th Cir. 1956).

Aggrieved person doctrine where the search warrant s primarily directed against said
person gives "standing."

The latest United States decision squarely in point is United States vs. Birrell, 242 F.
Supp. 191 (1965, U.S.D.C. S.D.N.Y.). The defendant had stored with an attorney certain
files and papers, which attorney, by the name of Dunn, was not, at the time of the
seizing of the records, Birrell's attorney. * Dunn, in turn, had stored most of the records
at his home in the country and on a farm which, according to Dunn's affidavit, was
under his (Dunn's) "control and management." The papers turned out to be private,
personal and business papers together with corporate books and records of certain
unnamed corporations in which Birrell did not even claim ownership. (All of these type
records were seized in the case at bar). Nevertheless, the search in Birrell was held
invalid by the court which held that even though Birrell did not own the premises where
the records were stored, he had "standing" to move for the return of all the papers and
properties seized. The court, relying on Jones vs. U.S., supra; U.S. vs. Antonelli
Fireworks Co., 53 F. Supp. 870, Aff'd 155 F. 2d 631: Henzel vs. U.S., supra; and
Schwimmer vs. U.S., supra, pointed out that

It is overwhelmingly established that the searches here in question were directed solely
and exclusively against Birrell. The only person suggested in the papers as having
violated the law was Birrell. The first search warrant described the records as having
been used "in committing a violation of Title 18, United States Code, Section 1341, by
the use of the mails by one Lowell M. Birrell, . . ." The second search warrant was
captioned: "United States of America vs. Lowell M. Birrell. (p. 198)

Possession (actual or constructive), no less than ownership, gives standing to move to


suppress. Such was the rule even before Jones. (p. 199)

If, as thus indicated Birrell had at least constructive possession of the records stored
with Dunn, it matters not whether he had any interest in the premises searched. See
also Jeffers v. United States, 88 U.S. Appl. D.C. 58, 187 F. 2d 498 (1950), affirmed 432
U.S. 48, 72 S. Ct. 93, 96 L. Ed. 459 (1951).

The ruling in the Birrell case was reaffirmed on motion for reargument; the United States
did not appeal from this decision. The factual situation in Birrell is strikingly similar to
Page 57 of 377

the case of the present petitioners; as in Birrell, many personal and corporate papers
were seized from premises not petitioners' family residences; as in Birrell, the searches
were "PRIMARILY DIRECTED SOLETY AND EXCLUSIVELY" against the petitioners. Still
both types of documents were suppressed in Birrell because of the illegal search. In the
case at bar, the petitioners connection with the premises raided is much closer than in
Birrell.

Thus, the petitioners have full standing to move for the quashing of all the warrants
regardless whether these were directed against residences in the narrow sense of the
word, as long as the documents were personal papers of the petitioners or (to the extent
that they were corporate papers) were held by them in a personal capacity or under their
personal control.

Prescinding a from the foregoing, this Court, at all events, should order the return to
the petitioners all personal and private papers and effects seized, no matter where these
were seized, whether from their residences or corporate offices or any other place or
places. The uncontradicted sworn statements of the petitioners in their, various
pleadings submitted to this Court indisputably show that amongst the things seized
from the corporate offices and other places were personal and private papers and effects
belonging to the petitioners.

If there should be any categorization of the documents, papers and things which where
the objects of the unlawful searches and seizures, I submit that the grouping should be:
(a) personal or private papers of the petitioners were they were unlawfully seized, be it
their family residences offices, warehouses and/or premises owned and/or possessed
(actually or constructively) by them as shown in all the search and in the sworn
applications filed in securing the void search warrants and (b) purely corporate papers
belonging to corporations. Under such categorization or grouping, the determination of
which unlawfully seized papers, documents and things are personal/private of the
petitioners or purely corporate papers will have to be left to the lower courts which
issued the void search warrants in ultimately effecting the suppression and/or return
of the said documents.

And as unequivocally indicated by the authorities above cited, the petitioners likewise
have clear legal standing to move for the suppression of purely corporate papers as
"President and/or General Manager" of the corporations involved as specifically
mentioned in the void search warrants. Finally, I must articulate my persuasion that
although the cases cited in my disquisition were criminal prosecutions, the great clauses
of the constitutional proscription on illegal searches and seizures do not withhold the
mantle of their protection from cases not criminal in origin or nature.
Page 58 of 377

G.R. No. L-10280 September 30, 1963

QUA CHEE GAN, JAMES UY, DANIEL DY alias DEE PAC, CHAN TIONG YU,
CUA CHU TIAN, CHUA LIM PAO alias JOSE CHUA and BASILIO KING,
petitioners-appellants, vs. THE DEPORTATION BOARD, Respondent-
Appellee.

BARRERA, J.:

This is an appeal from the decision of the Court of First Instance of Manila (in Sp.
Proc. No. 20037) denying the petition for writs of habeas corpus and/or prohibition,
certiorari, and mandamus filed by Qua Chee Gan, James Uy, Daniel Dy alias Dee Pac,
Chan Tiong Yu, Chua Chu Tian, Chua Lim Pao alias Jose Chua, and Basilio King. The
facts of the case, briefly stated, are as follows:.

On May 12, 1952, Special Prosecutor Emilio L. Galang charged the above-named
petitioners before the Deportation Board, with having purchased U.S. dollars in the total
sum of $130,000.00, without the necessary license from the Central Bank of the
Philippines, and of having clandestinely remitted the same to Hongkong and petitioners,
Qua Chee Gan, Chua Lim Pao alias Jose Chua, and Basilio King, with having attempted
to bribe officers of the Philippine and United States Governments (Antonio Laforteza,
Chief of the Intelligence Division of the Central Bank, and Capt. A. P. Charak of the OSI,
U.S. Air Force) in order to evade prosecution for said unauthorized purchase of U.S.
dollars.1

Following the filing of said deportation charges, a warrant for the arrest of said
aliens was issued by the presiding member of the Deportation Board. Upon their filing
surety bond for P10,000.00 and cash bond for P10,000.00, herein petitioners-appellants
were provisionally set at liberty.

On September 22, 1952, petitioners-appellants filed a joint motion to dismiss the


charges presented against them in the Deportation Board for the reason, among others,
that the same do not constitute legal ground for deportation of aliens from this country,
and that said Board has no jurisdiction to entertain such charges. This motion to
dismiss having been denied by order of the Board of February 9, 1953, petitioners-
appellants filed in this Court a petition for habeas corpus and/or prohibition, which
petition was given due course in our resolution of July 7, 1953, but made returnable to
the Court of First Instance of Manila (G.R. No. L-6783). The case was docketed in the
lower court as Special Proceeding No. 20037.
Page 59 of 377

At the instance of petitioners and upon their filing a bond for P5,000.00 each, a
writ of preliminary injunction was issued by the lower court, restraining the respondent
Deportation Board from hearing Deportation charges No. R-425 against petitioners,
pending final termination of the habeas corpus and/or prohibition proceedings.

On July 29, 1953, the respondent Board filed its answer to the original petition,
maintaining among others, that the Deportation Board, as an agent of the President,
has jurisdiction over the charges filed against petitioners and the authority to order their
arrest; and that, while petitioner Qua Chee Gan was acquitted of the offense of
attempted bribery of a public official, he was found in the same decision of the trial
court that he did actually offer money to an officer of the United States Air Force in order
that the latter may abstain from assisting the Central Bank official in the investigation
of the purchase of $130,000.00 from the Clark Air Force Base, wherein said petitioner
was involved.

After due trial, the court rendered a decision on January 18, 1956, upholding the
validity of the delegation by the president to the Deportation Board of his power to
conduct investigations for the purpose of determining whether the stay of an alien in
this country would be injurious to the security, welfare and interest of the State. The
court, likewise, sustained the power of the deportation Board to issue warrant of arrest
and fix bonds for the alien's temporary release pending investigation of charges against
him, on the theory that the power to arrest and fix the amount of the bond of the arrested
alien is essential to and complement the power to deport aliens pursuant to Section 69
of the Revised Administrative Code. Consequently, the petitioners instituted the present
appeal. .

It may be pointed out at the outset that after they were provisionally released on
bail, but before the charges filed against them were actually investigated, petitioners-
appellant raised the question of jurisdiction of the Deportation Board, first before said
body, then in the Court of First Instance, and now before us. Petitioners-appellants
contest the power of the President to deport aliens and, consequently, the delegation to
the Deportation Board of the ancillary power to investigate, on the ground that such
power is vested in the Legislature. In other words, it is claimed, for the power to deport
to be exercised, there must be a legislation authorizing the same.

Under Commonwealth Act No. 613 (Immigration Act of 1940), the Commissioner
of Immigration was empowered to effect the arrest and expulsion of an alien, after
previous determination by the Board of Commissioners of the existence of ground or
grounds therefor (Sec- 37). With the enactment of this law, however, the legislature did
not intend to delimit or concentrate the exercise of the power to deport on the
Immigration Commissioner alone, because in its Section 52, it provides:.
Page 60 of 377

SEC. 52. This Act is in substitution for and supersedes all previous laws relating
to the entry of aliens into the Philippines, and their exclusion, deportation, and
repatriation therefrom, with the exception of section sixty-nine of Act Numbered Twenty-
seven hundred and eleven which shall continue in force and effect: ..." (Comm. Act No.
613).

Section 69 of Act No. 2711 (Revised Administrative Code) referred to above reads:.

SEC. 69 Deportation of subject to foreign power. - A subject of a foreign power


residing in the Philippines shall not be deported, expelled, or excluded from said Islands
or repatriated to his own country by the President of the Philippines except upon prior
investigation, conducted by said Executive or his authorized agent, of the ground upon
which Such action is contemplated. In such case the person concerned shall be
informed of the charge or charges against him and he shall be allowed not less than
these days for the preparation of his defense. He shall also have the right to be heard
by himself or counsel, to produce witnesses in his own behalf, and to cross-examine the
opposing witnesses."

While it may really be contended that the aforequoted provision did not expressly
confer on the President the authority to deport undesirable aliens, unlike the express
grant to the Commissioner of Immigration under Commonwealth Act No. 613, but
merely lays down the procedure to be observed should there be deportation proceedings,
the fact that such a procedure was provided for before the President can deport an alien-
which provision was expressly declared exempted from the repealing effect of the
Immigration Act of 1940-is a clear indication of the recognition, and inferentially a
ratification, by the legislature of the existence of such power in the Executive. And the,
exercise of this power by the chief Executive has been sanctioned by this Court in several
decisions.2

Under the present and existing laws, therefore, deportation of an undesirable


alien may be effected in two ways: by order of the President, after due investigation,
pursuant to Section 69 of the Revised Administrative Code, and by the Commissioner
of Immigration, upon recommendation by the Board of Commissioners, under Section
37 of Commonwealth Act No. 613.

Petitioners contend, however, that even granting that the President is invested
with power to deport, still he may do so only upon the grounds enumerated in
Commonwealth Act No. 613, as amended, and on no other, as it would be unreasonable
and undemocratic to hold that an alien may be deported upon an unstated or undefined
Page 61 of 377

ground depending merely on the unlimited discretion of the Chief Executive. This
contention is not without merit, considering that whenever the legislature believes a
certain act or conduct to be a just cause for deportation, it invariably enacts a law to
that effect. Thus, in a number of amendatory acts, grounds have been added to those
originally contained in Section 37 of Commonwealth Act No. 613, as justifying
deportation of an alien, as well as other laws which provide deportation as part of the
penalty imposed on aliens committing violation thereof.

Be this as it may, the charges against the herein petitioners constitute in effect
an act of profiteering, hoarding or blackmarketing of U.S. dollars, in violation of the
Central Bank regulations - an economic sabotage - which is a ground for deportation
under the provisions of Republic Act 503 amending Section 37 of the Philippine
Immigration Act of 1940. The President may therefore order the deportation of these
petitioners if after investigation they are shown to have committed the act charged.

There seems to be no doubt that the President's power of investigation may be


delegated. This is clear from a reading of Section 69 of the Revised Administrative Code
which provides for a "prior investigation, conducted by said Executive (the President) or
his authorized agent." The first executive order on the subject was that of Governor
General Frank Murphy (No. 494, July 26, 1934), constituting a board to take action on
complaints against foreigners, to conduct investigations and thereafter make
recommendations. By virtue of Executive Order No. 33 dated May 29, 1936, President
Quezon created the Deportation Board primarily to receive complaints against aliens
charged to be undesirable, to conduct investigation pursuant to Section 69 of the
Revised Administrative Code and the rules and regulations therein provided, and make
the corresponding recommendation. 3 Since then, the Deportation Board has been
conducting the investigation as the authorized agent of the President.

This gives rise to the question regarding the extent of the power of the President
to conduct investigation, i.e., whether such authority carries with it the power to order
the arrest of the alien complained of, since the Administrative Code is silent on the
matter, and if it does, whether the same may be delegated to the respondent Deportation
Board.

Let it be noted that Section 69 of the Revised Administrative Code, unlike


Commonwealth Act No. 613 wherein the Commissioner of Immigration was specifically
granted authority, among others, to make arrests, fails to provide the President with
like specific power to be exercised in connection with such investigation. It must be for
this reason that President Roxas for the first time, saw it necessary to issue his
Executive Order No. 69, dated July 29, 1947, providing -
Page 62 of 377

For the purpose of insuring the appearance of aliens charged before the
Deportation Board created under Executive Order No. 37, dated January 4, 1947, and
facilitating the execution of the order of deportation whenever the President decides the
case against the respondent. I, Manuel Roxas, President of the Philippines, by virtue of
the powers vested in me by law, do hereby order that all respondents in deportation
proceedings shall file a bond with the Commissioner of Immigration in such amount
and containing such conditions as he may prescribe. .
xxx xxx xxx
Note that the executive order only required the filing of a bond to secure
appearance of the alien under investigation. It did not authorize the arrest of the
respondent.

It was only on January 5, 1951, when President Quirino reorganized the


Deportation Board by virtue of his Executive Order No. 398, that the Board was
authorized motu proprio or upon the filing of formal charges by the Special Prosecutor
of the Board, to issue the warrant for the arrest of the alien complained of and to hold
him under detention during the investigation unless he files a bond for his provisional
release in such amount and under such conditions as may be prescribed by the
Chairman of the Board.

As has been pointed out elsewhere, Section 69 of the Revised Administrative


Code, upon whose authority the President's power to deport is predicated, does not
provide for the exercise of the power to arrest. But the Solicitor General argues that the
law could not have denied to the Chief Executive acts which are absolutely necessary to
carry into effect the power of deportation granted him, such as the authority to order
the arrest of the foreigner charged as undesirable.

In this connection, it must be remembered that the right of an individual to be


secure in his person is guaranteed by the Constitution in the following language:.

3. The right of the People to be secure in their persons, houses, papers and effects
against unreasonable searches and seizures shall not be violated, and no warrants shall
issue but upon probable cause, to be determined by the judge after examination under
oath or affirmation of the complainant and the witnesses he may produce, and
particularly describing the place to be searched, and the persons or things to be seized."
(Sec 1, Art. III, Bill of Rights, Philippine Constitution).

As observed by the late Justice Laurel in his concurring opinion in the case of
Rodriguez, et al. v. Villamiel, et al. (65 Phil. 230, 239), this provision is not the same as
Page 63 of 377

that contained in the Jones Law wherein this guarantee is placed among the rights of
the accused. Under our Constitution, the same is declared a popular right of the people
and, of course, indisputably it equally applies to both citizens and foreigners in this
country. Furthermore, a notable innovation in this guarantee is found in our
Constitution in that it specifically provides that the probable cause upon which a
warrant of arrest may be issued, must be determined by the judge after examination
under oath, etc., of the complainant and the witnesses he may produce. This
requirement - "to be determined by the judge" - is not found in the Fourth Amendment
of the U.S. Constitution, in the Philippine Bill or in the Jones Act, all of which do not
specify who will determine the existence of a probable cause. Hence, under their
provisions, any public officer may be authorized by the Legislature to make such
determination, and thereafter issue the warrant of arrest. Under the express terms of
our Constitution, it is, therefore, even doubtful whether the arrest of an individual may
be ordered by any authority other than the judge if the purpose is merely to determine
the existence of a probable cause, leading to an administrative investigation. The
Constitution does not distinguish between warrants in a criminal case and
administrative warrants in administrative proceedings. And, if one suspected of having
committed a crime is entitled to a determination of the probable cause against him, by
a judge, why should one suspected of a violation of an administrative nature deserve
less guarantee? Of course it is different if the order of arrest is issued to carry out a final
finding of a violation, either by an executive or legislative officer or agency duly
authorized for the purpose, as then the warrant is not that mentioned in the
Constitution which is issuable only on probable cause. Such, for example, would be a
warrant of arrest to carry out a final order of deportation, or to effect compliance of an
order of contempt.

The contention of the Solicitor General that the arrest of a foreigner is necessary
to carry into effect the power of deportation is valid only when, as already stated, there
is already an order of deportation. To carry out the order of deportation, the President
obviously has the power to order the arrest of the deportee. But, certainly, during the
investigation, it is not indispensable that the alien be arrested. It is enough, as was true
before the executive order of President Quirino, that a bond be required to insure the
appearance of the alien during the investigation, as was authorized in the executive
order of President Roxas. Be that as it may, it is not imperative for us to rule, in this
proceeding - and nothing herein said is intended to so decide - on whether or not the
President himself can order the arrest of a foreigner for purposes of investigation only,
and before a definitive order of deportation has been issued. We are merely called upon
to resolve herein whether, conceding without deciding that the President can personally
order the arrest of the alien complained of, such power can be delegated by him to the
Deportation Board.

Unquestionably, the exercise of the power to order the arrest of an individual


demands the exercise of discretion by the one issuing the same, to determine whether
under specific circumstances, the curtailment of the liberty of such person is warranted.
Page 64 of 377

The fact that the Constitution itself, as well as the statute relied upon, prescribe the
manner by which the warrant may be issued, conveys the intent to make the issuance
of such warrant dependent upon conditions the determination of the existence of which
requires the use of discretion by the person issuing the same. In other words, the
discretion of whether a warrant of arrest shall issue or not is personal to the one upon
whom the authority devolves. And authorities are to the effect that while ministerial
duties may be delegated, official functions requiring the exercise of discretion and
judgment, may not be so delegated. Indeed, an implied grant of power, considering that
no express authority was granted by the law on the matter under discussion, that would
serve the curtailment or limitation on the fundamental right of a person, such as his
security to life and liberty, must be viewed with caution, if we are to give meaning to the
guarantee contained in the Constitution. If this is so, then guarantee a delegation of
that implied power, nebulous as it is, must be rejected as inimical to the liberty of the
people. The guarantees of human rights and freedom can not be made to rest
precariously on such a shaky foundation.

We are not unaware of the statements made by this Court in the case of Tan Sin
v. Deportation Board (G.R. No. L-11511, Nov. 28,1958). It may be stated, however, that
the power of arrest was not squarely raised in that proceeding, but only as a
consequence of therein petitioner's proposition that the President had no inherent power
to deport and that the charges filed against him did not constitute ground for
deportation. .

IN VIEW OF THE FOREGOING, Executive Order No. 398, series of 1951, insofar
as it empowers the Deportation Board to issue warrant of arrest upon the filing of formal
charges against an alien or aliens and to fix bond and prescribe the conditions for the
temporary release of said aliens, is declared illegal. As a consequence, the order of arrest
issued by the respondent Deportation Board is declared null and void and the bonds
filed pursuant to such order of arrest, decreed cancelled. With the foregoing
modification, the decision appealed from is hereby affirmed. No costs. So ordered.

Padilla, Bautista Angelo, Labrador, Concepcion, Paredes, Dizon, Regala and Makalintal,
JJ., concur.
Bengzon, C.J., reserved his vote.
Reyes, J.B.L., J., took no part.
Page 65 of 377

G.R. No. 82585 November 14, 1988

MAXIMO V. SOLIVEN, ANTONIO V. ROCES, FREDERICK K. AGCAOLI, and


GODOFREDO L. MANZANAS, petitioners, vs. THE HON. RAMON P.
MAKASIAR, Presiding Judge of the Regional Trial Court of Manila, Branch
35, UNDERSECRETARY SILVESTRE BELLO III, of the Department of
Justice, LUIS C. VICTOR, THE CITY FISCAL OF MANILA and PRESIDENT
CORAZON C. AQUINO, respondents.

G.R. No. 82827 November 14, 1988

LUIS D. BELTRAN, petitioner, vs. THE HON. RAMON P. MAKASIAR, Presiding Judge of
Branch 35 of the Regional Trial Court, at Manila, THE HON. LUIS VICTOR, CITY FISCAL
OF MANILA, PEOPLE OF THE PHILIPPINES, SUPERINTENDENT OF THE WESTERN
POLICE DISTRICT, and THE MEMBERS OF THE PROCESS SERVING UNIT AT THE
REGIONAL TRIAL COURT OF MANILA, respondents.

G.R. No. 83979 November 14, 1988.

LUIS D. BELTRAN, petitioner, vs. EXECUTIVE SECRETARY CATALINO MACARAIG,


SECRETARY OF JUSTICE SEDFREY ORDOÑEZ, UNDERSECRETARY OF JUSTICE
SILVESTRE BELLO III, THE CITY FISCAL OF MANILA JESUS F. GUERRERO, and
JUDGE RAMON P. MAKASIAR, Presiding Judge of Branch 35 of the Regional Trial Court,
at Manila, respondents.

Angara, Abello, Concepcion, Regala and Cruz for petitioners in G.R. No. 82585.
Perfecto V. Fernandez, Jose P. Fernandez and Cristobal P. Fernandez for petitioner in
G.R. Nos. 82827 and 83979.

RESOLUTION
PER CURIAM:

In these consolidated cases, three principal issues were raised: (1) whether or not
petitioners were denied due process when informations for libel were filed against them
although the finding of the existence of a prima facie case was still under review by the
Secretary of Justice and, subsequently, by the President; (2) whether or not the
constitutional rights of Beltran were violated when respondent RTC judge issued a
warrant for his arrest without personally examining the complainant and the witnesses,
if any, to determine probable cause; and (3) whether or not the President of the
Page 66 of 377

Philippines, under the Constitution, may initiate criminal proceedings against the
petitioners through the filing of a complaint-affidavit.

Subsequent events have rendered the first issue moot and academic. On March 30,
1988, the Secretary of Justice denied petitioners' motion for reconsideration and upheld
the resolution of the Undersecretary of Justice sustaining the City Fiscal's finding of a
prima facie case against petitioners. A second motion for reconsideration filed by
petitioner Beltran was denied by the Secretary of Justice on April 7, 1988. On appeal,
the President, through the Executive Secretary, affirmed the resolution of the Secretary
of Justice on May 2, 1988. The motion for reconsideration was denied by the Executive
Secretary on May 16, 1988. With these developments, petitioners' contention that they
have been denied the administrative remedies available under the law has lost factual
support.

It may also be added that with respect to petitioner Beltran, the allegation of denial of
due process of law in the preliminary investigation is negated by the fact that instead of
submitting his counter- affidavits, he filed a "Motion to Declare Proceedings Closed," in
effect waiving his right to refute the complaint by filing counter-affidavits. Due process
of law does not require that the respondent in a criminal case actually file his counter-
affidavits before the preliminary investigation is deemed completed. All that is required
is that the respondent be given the opportunity to submit counter-affidavits if he is so
minded.

The second issue, raised by petitioner Beltran, calls for an interpretation of the
constitutional provision on the issuance of warrants of arrest. The pertinent provision
reads:

Art. III, Sec. 2. The right of the people to be secure in their persons, houses, papers and
effects against unreasonable searches and seizures of whatever nature and for any
purpose shall be inviolable, and no search warrant or warrant of arrest shall issue
except upon probable cause to be determined personally by the judge after examination
nder oath or affirmation of the complainant and the witnesses he may produce, and
particularly describing the place to be searched and the persons or things to be seized.

The addition of the word "personally" after the word "determined" and the deletion of the
grant of authority by the 1973 Constitution to issue warrants to "other responsible
officers as may be authorized by law," has apparently convinced petitioner Beltran that
the Constitution now requires the judge to personally examine the complainant and his
witnesses in his determination of probable cause for the issuance of warrants of arrest.
This is not an accurate interpretation.
Page 67 of 377

What the Constitution underscores is the exclusive and personal responsibility of the
issuing judge to satisfy himself of the existence of probable cause. In satisfying himself
of the existence of probable cause for the issuance of a warrant of arrest, the judge is
not required to personally examine the complainant and his witnesses. Following
established doctrine and procedure, he shall: (1) personally evaluate the report and the
supporting documents submitted by the fiscal regarding the existence of probable cause
and, on the basis thereof, issue a warrant of arrest; or (2) if on the basis thereof he finds
no probable cause, he may disregard the fiscal's report and require the submission of
supporting affidavits of witnesses to aid him in arriving at a conclusion as to the
existence of probable cause.

Sound policy dictates this procedure, otherwise judges would be unduly laden with the
preliminary examination and investigation of criminal complaints instead of
concentrating on hearing and deciding cases filed before their courts.

On June 30, 1987, the Supreme Court unanimously adopted Circular No. 12, setting
down guidelines for the issuance of warrants of arrest. The procedure therein provided
is reiterated and clarified in this resolution.

It has not been shown that respondent judge has deviated from the prescribed
procedure. Thus, with regard to the issuance of the warrants of arrest, a finding of grave
abuse of discretion amounting to lack or excess of jurisdiction cannot be sustained.

Anent the third issue, petitioner Beltran argues that "the reasons which necessitate
presidential immunity from suit impose a correlative disability to file suit." He contends
that if criminal proceedings ensue by virtue of the President's filing of her complaint-
affidavit, she may subsequently have to be a witness for the prosecution, bringing her
under the trial court's jurisdiction. This, continues Beltran, would in an indirect way
defeat her privilege of immunity from suit, as by testifying on the witness stand, she
would be exposing herself to possible contempt of court or perjury.

The rationale for the grant to the President of the privilege of immunity from suit is to
assure the exercise of Presidential duties and functions free from any hindrance or
distraction, considering that being the Chief Executive of the Government is a job that,
aside from requiring all of the office holder's time, also demands undivided attention.

But this privilege of immunity from suit, pertains to the President by virtue of the office
and may be invoked only by the holder of the office; not by any other person in the
Page 68 of 377

President's behalf. Thus, an accused in a criminal case in which the President is


complainant cannot raise the presidential privilege as a defense to prevent the case from
proceeding against such accused.

Moreover, there is nothing in our laws that would prevent the President from waiving
the privilege. Thus, if so minded the President may shed the protection afforded by the
privilege and submit to the court's jurisdiction. The choice of whether to exercise the
privilege or to waive it is solely the President's prerogative. It is a decision that cannot
be assumed and imposed by any other person.

As regards the contention of petitioner Beltran that he could not be held liable for libel
because of the privileged character or the publication, the Court reiterates that it is not
a trier of facts and that such a defense is best left to the trial court to appreciate after
receiving the evidence of the parties.

As to petitioner Beltran's claim that to allow the libel case to proceed would produce a
"chilling effect" on press freedom, the Court finds no basis at this stage to rule on the
point.

The petitions fail to establish that public respondents, through their separate acts,
gravely abused their discretion as to amount to lack of jurisdiction. Hence, the writs of
certiorari and prohibition prayed for cannot issue.

WHEREFORE, finding no grave abuse of discretion amounting to excess or lack of


jurisdiction on the part of the public respondents, the Court Resolved to DISMISS the
petitions in G. R. Nos. 82585, 82827 and 83979. The Order to maintain the status quo
contained in the Resolution of the Court en banc dated April 7, 1988 and reiterated in
the Resolution dated April 26, 1988 is LIFTED.

Fernan, C.J., Narvasa, Melencio-Herrera, Cruz, Paras, Feliciano, Gancayco, Padilla,


Bidin, Sarmiento, Cortes, Griño-Aquino Medialdea and Regalado, JJ., concur.

Separate Opinions

GUTIERREZ, JR., J., concurring:


Page 69 of 377

I concur with the majority opinion insofar as it involves the three principal issues
mentioned in its opening statement. However, as to the more important issue on
whether or not the prosecution of the libel case would produce a "chilling effect" on press
freedom, I beg to reserve my vote. I believe this is the more important issue in these
petitions and it should be resolved now rather that later.

Consistent with our decision in Salonga v. Cruz Pano (134 SCRA 438 [1985]), the Court
should not hesitate to quash a criminal prosecution in the interest of more enlightened
and substantial justice where it is not alone the criminal liability of an accused in a
seemingly minor libel case which is involved but broader considerations of governmental
power versus a preferred freedom.

We have in these four petitions the unusual situation where the highest official of the
Republic and one who enjoys unprecedented public support asks for the prosecution of
a newspaper columnist, the publisher and chairman of the editorial board, the
managing editor and the business manager in a not too indubitable a case for alleged
libel.

I am fully in accord with an all out prosecution if the effect will be limited to punishing
a newspaperman who, instead of observing accuracy and fairness, engages in
unwarranted personal attacks, irresponsible twisting of facts, of malicious distortions
of half-truths which tend to cause dishonor, discredit, or contempt of the complainant.
However, this case is not a simple prosecution for libel. We have as complainant a
powerful and popular President who heads the investigation and prosecution service
and appoints members of appellate courts but who feels so terribly maligned that she
has taken the unorthodox step of going to court inspite of the invocations of freedom of
the press which would inevitably follow.

I believe that this Court should have acted on this issue now instead of leaving the
matter to fiscals and defense lawyers to argue before a trial judge.

There is always bound to be harassment inherent in any criminal prosecution. Where


the harassment goes beyond the usual difficulties encountered by any accused and
results in an unwillingness of media to freely criticize government or to question
government handling of sensitive issues and public affairs, this Court and not a lower
tribunal should draw the demarcation line.

As early as March 8, 1918, the decision in United States v. Bustos (37 Phil. 731) stated
that "(c)omplete liberty to comment on the conduct of public men is a scalpel in the case
Page 70 of 377

of free speech. The sharp incision of its probe relieves the abscesses of officialdom. Men
in public life may suffer under a hostile and unjust accusation; the wound can be
assuaged with the balm of a clear conscience." The Court pointed out that while
defamation is not authorized, criticism is to be expected and should be borne for the
common good.

In People v. Perfecto (43 Phil. 887 [1922]), the Court stated:

xxx xxx xxx

... No longer is there a Minister of the Crown own or a person in authority of such exalted
position that the citizen must speak of him only with bated breath. "In the eye of our
Constitution and laws, every man is a sovereign, a ruler and a freeman, and has equal
rights with every other man." (at p. 900)

In fact, the Court observed that high official position, instead of affording immunity from
slanderous and libelous charges, would actually invite attacks by those who desire to
create sensation. It would seem that what would ordinarily be slander if directed at the
typical person should be examined from various perspectives if directed at a high
government official. Again, the Supreme Court should draw this fine line instead of
leaving it to lower tribunals.

This Court has stressed as authoritative doctrine in Elizalde v. Gutierrez (76 SCRA 448
[1977]) that a prosecution for libel lacks justification if the offending words find
sanctuary within the shelter of the free press guaranty. In other words, a prosecution
for libel should not be allowed to continue, where after discounting the possibility that
the words may not be really that libelous, there is likely to be a chilling effect, a patently
inhibiting factor on the willingness of newspapermen, especially editors and publishers
to courageously perform their critical role in society. If, instead of merely reading more
carefully what a columnist writes in his daily column, the editors tell their people to lay
off certain issues or certain officials, the effect on a free press would be highly injurious.

Because many questions regarding press freedom are left unanswered by our resolution,
I must call attention to our decisions which caution that "no inroads on press freedom
should be allowed in the guise of punitive action visited on what otherwise should be
characterized as libel." (Lopez v. Court of Appeals, 34 SCRA 117 [1970]; See also the
citations in Elizalde v. Gutierrez, supra).
Page 71 of 377

The United States Supreme Court is even more emphatic, to wit:

In deciding the question now, we are compelled by neither precedent nor policy to give
any more weight to the epithet "libel" than we have to other "mere labels" of state law.
N. A. A. C. P. v. Button, 371 US 415, 429, 9L ed 2d 405, 415, 83 S Ct 328. Like
insurrection, contempt, advocacy of unlawful acts, breach of the peace, obscenity,
solicitation of legal business, and the other various other formulae for the repression of
expression that have been challenged in this Court, libel can claim no talismanic
immunity from constitutional limitations. It must be measured by standards that satisfy
the First Amendment.

xxx xxx xxx

Those who won our independence believed ... that public discussion is a political duty;
and that this should be a fundamental principle of the American government. They
recognized the risk to which all human institutions are subject. But they knew that
order cannot be secured merely through fear of punishment for its infraction; that it is
hazardous to discourage thought, hope and imagination; that fear breeds repression;
that repression breeds hate; that hate menaces stable government; that the path of
safety lies in the opportunity to discuss freely supposed grievances and proposed
remedies; and that the fitting remedy for evil counsel is good ones. Believing in the power
of reason as applied through public discussion, they eschewed silence coerced by law—
the argument of force in its worst form. ...

Thus we consider this case against the background of a profound national commitment
to the principle that debate on public issues should be uninhibited, robust, and wide
open, and that it may well include vehement, caustic, and sometimes unpleasantly
sharp attacks on government and public officials. ... (at pp. 700-701)

Shunting aside the individual liability of Mr. Luis Beltran, is there a prima facie showing
that Messrs. Maximo Soliven, Antonio V. Roces, Frederick K. Agcaoili, and Godofredo L.
Manzanas knowingly participated in a wilful purveying of falsehood? Considering the
free speech aspects of these petitions, should not a differentiated approach to their
particular liabilities be taken instead of lumping up everybody with the offending
columnist? I realize that the law includes publishers and editors but perhaps the
"chilling effect" issue applies with singular effectivity to publishers and editors vis-a-vis
newspaper columnists. There is no question that, ordinarily, libel is not protected by
the free speech clause but we have to understand that some provocative words, which
if taken literally may appear to shame or disparage a public figure, may really be
intended to provoke debate on public issues when uttered or written by a media
Page 72 of 377

personality. Will not a criminal prosecution in the type of case now before us dampen
the vigor and limit the variety of public debate? There are many other questions arising
from this unusual case which have not been considered.

I, of course, concur with the Court's opinion because it has decided to limit the issues
to narrowly drawn ones. I see no reason to disagree with the way the Court has resolved
them. The first issue on prematurity is moot. The second issue discusses a procedure
now embodied in the recently amended Rules of Court on how a Judge should proceed
before he issues a warrant of arrest. Anent the third issue, considerations of public
policy dictate that an incumbent President should not be sued. At the same time, the
President cannot stand by helplessly bereft of legal remedies if somebody vilifies or
maligns him or her.

The Court has decided to defer the "chilling effect" issue for a later day. To this, I take
exception. I know that most of our fiscals and judges are courageous individuals who
would not allow any considerations of possible consequences to their careers to stand
in the way of public duty. But why should we subject them to this problem? And why
should we allow the possibility of the trial court treating and deciding the case as one
for ordinary libel without bothering to fully explore the more important areas of concern,
the extremely difficult issues involving government power and freedom of expression.

However, since we have decided to defer the "chilling effect" issue for a later day, I limit
myself to reiterating the dissenting words of Mr. Justice Jackson in the American case
of Beaurnhais v. Illinois (343 U. S. 250) when he said:

If one can claim to announce the judgment of legal history on any subject, it is that
criminal libel laws are consistent with the concept of ordered liberty only when applied
with safeguards evolved to prevent their invasion of freedom of expression.

In the trial of the libel case against the petitioners, the safeguards in the name of
freedom of expression should be faithfully applied.

Separate Opinions

GUTIERREZ, JR., J., concurring:

I concur with the majority opinion insofar as it involves the three principal issues
mentioned in its opening statement. However, as to the more important issue on
Page 73 of 377

whether or not the prosecution of the libel case would produce a "chilling effect" on press
freedom, I beg to reserve my vote. I believe this is the more important issue in these
petitions and it should be resolved now rather that later.

Consistent with our decision in Salonga v. Cruz Pano (134 SCRA 438 [1985]), the Court
should not hesitate to quash a criminal prosecution in the interest of more enlightened
and substantial justice where it is not alone the criminal liability of an accused in a
seemingly minor libel case which is involved but broader considerations of governmental
power versus a preferred freedom.

We have in these four petitions the unusual situation where the highest official of the
Republic and one who enjoys unprecedented public support asks for the prosecution of
a newspaper columnist, the publisher and chairman of the editorial board, the
managing editor and the business manager in a not too indubitable a case for alleged
libel.

I am fully in accord with an all out prosecution if the effect will be limited to punishing
a newspaperman who, instead of observing accuracy and fairness, engages in
unwarranted personal attacks, irresponsible twisting of facts, of malicious distortions
of half-truths which tend to cause dishonor, discredit, or contempt of the complainant.
However, this case is not a simple prosecution for libel. We have as complainant a
powerful and popular President who heads the investigation and prosecution service
and appoints members of appellate courts but who feels so terribly maligned that she
has taken the unorthodox step of going to court inspite of the invocations of freedom of
the press which would inevitably follow.

I believe that this Court should have acted on this issue now instead of leaving the
matter to fiscals and defense lawyers to argue before a trial judge.

There is always bound to be harassment inherent in any criminal prosecution. Where


the harassment goes beyond the usual difficulties encountered by any accused and
results in an unwillingness of media to freely criticize government or to question
government handling of sensitive issues and public affairs, this Court and not a lower
tribunal should draw the demarcation line.

As early as March 8, 1918, the decision in United States v. Bustos (37 Phil. 731) stated
that "(c)omplete liberty to comment on the conduct of public men is a scalpel in the case
of free speech. The sharp incision of its probe relieves the abscesses of officialdom. Men
in public life may suffer under a hostile and unjust accusation; the wound can be
Page 74 of 377

assuaged with the balm of a clear conscience." The Court pointed out that while
defamation is not authorized, criticism is to be expected and should be borne for the
common good.

In People v. Perfecto (43 Phil. 887 [1922]), the Court stated:

xxx xxx xxx

... No longer is there a Minister of the Crown own or a person in authority of such exalted
position that the citizen must speak of him only with bated breath. "In the eye of our
Constitution and laws, every man is a sovereign, a ruler and a freeman, and has equal
rights with every other man." (at p. 900)

In fact, the Court observed that high official position, instead of affording immunity from
slanderous and libelous charges, would actually invite attacks by those who desire to
create sensation. It would seem that what would ordinarily be slander if directed at the
typical person should be examined from various perspectives if directed at a high
government official. Again, the Supreme Court should draw this fine line instead of
leaving it to lower tribunals.

This Court has stressed as authoritative doctrine in Elizalde v. Gutierrez (76 SCRA 448
[1977]) that a prosecution for libel lacks justification if the offending words find
sanctuary within the shelter of the free press guaranty. In other words, a prosecution
for libel should not be allowed to continue, where after discounting the possibility that
the words may not be really that libelous, there is likely to be a chilling effect, a patently
inhibiting factor on the willingness of newspapermen, especially editors and publishers
to courageously perform their critical role in society. If, instead of merely reading more
carefully what a columnist writes in his daily column, the editors tell their people to lay
off certain issues or certain officials, the effect on a free press would be highly injurious.

Because many questions regarding press freedom are left unanswered by our resolution,
I must call attention to our decisions which caution that "no inroads on press freedom
should be allowed in the guise of punitive action visited on what otherwise should be
characterized as libel." (Lopez v. Court of Appeals, 34 SCRA 117 [1970]; See also the
citations in Elizalde v. Gutierrez, supra).<äre||anº•1àw>

The United States Supreme Court is even more emphatic, to wit:


Page 75 of 377

In deciding the question now, we are compelled by neither precedent nor policy to give
any more weight to the epithet "libel" than we have to other "mere labels" of state law.
N. A. A. C. P. v. Button, 371 US 415, 429, 9L ed 2d 405, 415, 83 S Ct 328. Like
insurrection, contempt, advocacy of unlawful acts, breach of the peace, obscenity,
solicitation of legal business, and the other various other formulae for the repression of
expression that have been challenged in this Court, libel can claim no talismanic
immunity from constitutional limitations. It must be measured by standards that satisfy
the First Amendment.

xxx xxx xxx

Those who won our independence believed ... that public discussion is a political duty;
and that this should be a fundamental principle of the American government. They
recognized the risk to which all human institutions are subject. But they knew that
order cannot be secured merely through fear of punishment for its infraction; that it is
hazardous to discourage thought, hope and imagination; that fear breeds repression;
that repression breeds hate; that hate menaces stable government; that the path of
safety lies in the opportunity to discuss freely supposed grievances and proposed
remedies; and that the fitting remedy for evil counsel is good ones. Believing in the power
of reason as applied through public discussion, they eschewed silence coerced by law—
the argument of force in its worst form. ...

Thus we consider this case against the background of a profound national commitment
to the principle that debate on public issues should be uninhibited, robust, and wide
open, and that it may well include vehement, caustic, and sometimes unpleasantly
sharp attacks on government and public officials. ... (at pp. 700-701)

Shunting aside the individual liability of Mr. Luis Beltran, is there a prima facie showing
that Messrs. Maximo Soliven, Antonio V. Roces, Frederick K. Agcaoili, and Godofredo L.
Manzanas knowingly participated in a wilful purveying of falsehood? Considering the
free speech aspects of these petitions, should not a differentiated approach to their
particular liabilities be taken instead of lumping up everybody with the offending
columnist? I realize that the law includes publishers and editors but perhaps the
"chilling effect" issue applies with singular effectivity to publishers and editors vis-a-vis
newspaper columnists. There is no question that, ordinarily, libel is not protected by
the free speech clause but we have to understand that some provocative words, which
if taken literally may appear to shame or disparage a public figure, may really be
intended to provoke debate on public issues when uttered or written by a media
personality. Will not a criminal prosecution in the type of case now before us dampen
Page 76 of 377

the vigor and limit the variety of public debate? There are many other questions arising
from this unusual case which have not been considered.

I, of course, concur with the Court's opinion because it has decided to limit the issues
to narrowly drawn ones. I see no reason to disagree with the way the Court has resolved
them. The first issue on prematurity is moot. The second issue discusses a procedure
now embodied in the recently amended Rules of Court on how a Judge should proceed
before he issues a warrant of arrest. Anent the third issue, considerations of public
policy dictate that an incumbent President should not be sued. At the same time, the
President cannot stand by helplessly bereft of legal remedies if somebody vilifies or
maligns him or her.

The Court has decided to defer the "chilling effect" issue for a later day. To this, I take
exception. I know that most of our fiscals and judges are courageous individuals who
would not allow any considerations of possible consequences to their careers to stand
in the way of public duty. But why should we subject them to this problem? And why
should we allow the possibility of the trial court treating and deciding the case as one
for ordinary libel without bothering to fully explore the more important areas of concern,
the extremely difficult issues involving government power and freedom of expression.

However, since we have decided to defer the "chilling effect" issue for a later day, I limit
myself to reiterating the dissenting words of Mr. Justice Jackson in the American case
of Beaurnhais v. Illinois (343 U. S. 250) when he said:

If one can claim to announce the judgment of legal history on any subject, it is that
criminal libel laws are consistent with the concept of ordered liberty only when applied
with safeguards evolved to prevent their invasion of freedom of expression.

In the trial of the libel case against the petitioners, the safeguards in the name of
freedom of expression should be faithfully applied.
Page 77 of 377

G.R. Nos. 94054-57 February 19, 1991

VICENTE LIM, SR. and MAYOR SUSANA LIM, petitioners, vs. HON. NEMESIO
S. FELIX and HON. ANTONIO ALFANE, respondents.

G.R. Nos. 94266-69 February 19, 1991

JOLLY T. FERNANDEZ, FLORENCIO T. FERNANDEZ, JR., NONILON A. BAGALIHOG,


MAYOR NESTOR C. LIM and MAYOR ANTONIO KHO, petitioners, vs. HON. NEMESIO
S. FELIX and PROSECUTOR ANTONIO C. ALFANE, respondents.

Francisco R. Llamas for petitioners in G.R. Nos. 94054-57.

Jolly T. Fernandez, Elenito Bagalihog, Orlando M. Danao and Hechanova, Ballicid &
Associates for petitioners in G.R. Nos. 94266-69.

GUTIERREZ, JR., J.:

May a Judge without ascertaining the facts through his own personal determination
and relying solely on the certification or recommendation of a prosecutor that a probable
cause exists issue a warrant of arrest?

On March 17, 1989, at about 7:30 o'clock in the morning, at the vicinity of the airport
road of the Masbate Domestic Airport, located at the municipality of Masbate province
of Masbate, Congressman Moises Espinosa, Sr. and his security escorts, namely
Provincial Guards Antonio Cortes, Gaspar Amaro, and Artemio Fuentes were attacked
and killed by a lone assassin. Dante Siblante another security escort of Congressman
Espinosa, Sr. survived the assassination plot, although, he himself suffered a gunshot
wound.

An investigation of the incident then followed.

Thereafter, and for the purpose of preliminary investigation, the designated investigator,
Harry O. Tantiado, TSg, of the PC Criminal Investigation Service at Camp Bagong Ibalon
Legazpi City filed an amended complaint with the Municipal Trial Court of Masbate
accusing, among others, Vicente Lim, Sr., Mayor Susana Lim of Masbate (petitioners in
G.R. Nos. 9405457), Jolly T. Fernandez, Florencio T. Fernandez, Jr., Nonilon A.
Bagalihog, Mayor Nestor C. Lim and Mayor Antonio Kho (petitioners in G.R. Nos. 94266-
Page 78 of 377

69) of the crime of multiple murder and frustrated murder in connection with the airport
incident. The case was docketed as Criminal Case No. 9211.

After conducting the preliminary investigation, the court issued an order dated July 31,
1989 stating therein that:

. . . after weighing the affidavits and answers given by the witnesses for the prosecution
during the preliminary examination in searching questions and answers, concludes that
a probable cause has been established for the issuance of a warrant of arrest of named
accused in the amended complaint, namely, Jimmy Cabarles, Ronnie Fernandez,
Nonilon Bagalihog, Jolly Fernandez, Florencio Fernandez, Jr., Vicente Lim, Sr., Susana
Lim, Nestor Lim, Antonio Kho, Jaime Liwanag, Zaldy Dumalag and Rene Tualla alias
Tidoy. (Rollo, p. 58, G.R. Nos. 94054-57)

xxx xxx xxx

In the same Order, the court ordered the arrest of the petitioners and recommended the
amount of P200,000.00 as bail for the provisional liberty of each of the accused.

Petitioners Jolly Fernandez and Nonilon Bagalihog filed a motion for the reduction of
bail which was granted by the court and they were allowed to post bail in the amount
of P150,000.00 each. Except for Jimmy Cabarles, all the rest of the accused posted bail
at P200,000.00 each.

On August 29, 1989, the entire records of the case consisting of two hundred sixty one
(261) pages were transmitted to the Provincial Prosecutor of Masbate. Respondent
Acting Fiscal Antonio C. Alfane was designated to review the case.

On September 22, 1989, Fiscal Alfane issued a Resolution which affirmed the finding of
a prima facie case against the petitioners but differed in the designation of the crime in
that the ruled that ". . . all of the accused should not only be charged with Multiple
Murder With Frustrated Murder" but for a case of MURDER for each of the killing of the
four victims and a physical injuries case for inflicting gunshot wound on the buttocks
of Dante Siblante." (Annex "H", Comment of Fiscal Alfane, p. 186, Rollo, G.R. Nos.
94054-57) A motion to reconsider the Resolution filed by petitioners Vicente Lim, Sr.
and Mayor Susana Lim was denied.
Page 79 of 377

On October 30, 1989, Fiscal Alfane filed with the Regional Trial Court of Masbate, four
(4) separate informations of murder against the twelve (12) accused with a
recommendation of no bail.

On November 21, 1989, petitioners Vicente Lim, Sr. and Susana Lim filed with us a
verified petition for change of venue. (Case No. A.M. No. 89-11-1270-MTC, formerly,
G.R. Nos. 90587-90)

On December 14, 1989, we issued an en banc Resolution authorizing the change of


venue from the Regional Trial Court of Masbate to the Regional Trial Court of Makati to
avoid a miscarriage of justice, to wit:

Acting on the petition for change of venue of the trial of Criminal Cases Nos. 5811, 5812,
5813, and 5814 from the Regional Trial Court, Masbate, Masbate to any of the Regional
Trial Courts at Quezon City or Makati, the Court Resolved to (a) GRANT the aforesaid
petition for transfer of venue in order to avoid miscarriage of justice (Article VIII, Section
5(4) of the Philippine Constitution); (b) DIRECT the Clerk of Court, Regional Trial Court,
Masbate, Masbate to transmit the records of the aforesaid cases to the Executive Judge,
Regional Trial Court, Makati, for raffling among the other branches of the court; and (c)
ORDER the Regional Trial Court of Masbate, Masbate to desist from further taking
cognizance of the said cases until such time that the petition is finally resolved.

The cases were raffled to Branch 56 presided by respondent Judge Nemesio S. Felix.

Petitioners Vicente Lim, Sr. and Susana Lim filed with the respondent court several
motions and manifestations which in substance prayed for the following:

1. An order be issued requiring the transmittal of the initial records of the preliminary
inquiry or investigation conducted by the Municipal Judge Barsaga of Masbate for the
best enlightenment of this Honorable Court in its personal determination of the
existence of a probable cause or prima facie evidence as well as its determination of the
existence of guilt, pursuant to the mandatory mandate of the constitution that no
warrant shall issue unless the issuing magistrate shall have himself been personally
convinced of such probable cause.

2. Movants be given ample opportunity to file their motion for preliminary investigation
as a matter of right; and
Page 80 of 377

3. In the event that this court may later be convinced of the existence of a probable
cause, to be allowed to file a motion for reduction of bail or for admission of bail. (p. 17,
Rollo, G.R. Nos. 94054-57)

In another manifestation, the Lims reiterated that the court conduct a hearing to
determine if there really exists a prima facie case against them in the light of documents
which are recantations of some witnesses in the preliminary investigation. The motions
and manifestations were opposed by the prosecution.

On July 5, 1990, the respondent court issued an order denying for lack of merit the
motions and manifestations and issued warrants of arrest against the accused including
the petitioners herein. The respondent Judge said:

In the instant cases, the preliminary investigation was conducted by the Municipal Trial
Court of Masbate, Masbate which found the existence of probable cause that the offense
of multiple murder was committed and that all the accused are probably guilty thereof,
which was affirmed upon review by the Provincial Prosecutor who properly filed with the
Regional Trial Court four separate informations for murder. Considering that both the
two competent officers to whom such duty was entrusted by law have declared the
existence of probable cause, each information is complete in form and substance, and
there is no visible defect on its face, this Court finds it just and proper to rely on the
prosecutor's certification in each information which reads: (pp. 19-20, Rollo, G.R Nos.
94054-57; Emphasis supplied)

xxx xxx xxx

The petitioners then filed these consolidated petitions questioning the July 5, 1990
Order.

In a Resolution dated July 17, 1990 in G.R. Nos. 94054-57, we issued ". . . a
TEMPORARY RESTRAINING ORDER, effective immediately and continuing until further
orders from this Court, ordering the respondent judge or his duly authorized
representatives or agents to CEASE and DESIST from enforcing or implementing the
warrant of arrest without bail issued against the petitioners in his Order dated July 5,
1990 in Criminal Cases Nos. 5811-14.
Page 81 of 377

In another Resolution dated July 31, 1990 in G.R. Nos. 94266-69, we resolved:

xxx xxx xxx

. . . To ISSUE writs of (1) PRELIMINARY MANDATORY INJUNCTION, ordering and


directing the respondent judge to recall/set aside and/or annul the legal effects of the
warrants of arrest without bail issued against and served upon herein petitioners Jolly
T. Fernandez, Florencio T. Fernandez, Jr. and Nonilon Bagalihog and release them from
confinement at PC-CIS Detention Center, Camp Crame, Quezon City; and (2)
TEMPORARY RESTRAINING ORDER, effective immediately and continuing until further
orders from this Court, ordering the respondent judge or his duly authorized
representatives or agents, to CEASE AND DESIST from enforcing or implementing the
warrants of arrest without bail issued against petitioners Mayors Nestor C. Lim and
Antonio T. Kho.

The primary issue in these consolidated petitions centers on whether or not a judge may
issue a warrant of arrest without bail by simply relying on the prosecution's certification
and recommendation that a probable cause exists.

This is not a novel question. In the case of Placer v. Villanueva (126 SCRA 463 [1983]),
we ruled that a judge may rely upon the fiscal's certification of the existence of probable
cause and, on the basis thereof, issue a warrant of arrest. However, the certification
does not bind the judge to come out with the warrant of arrest. This decision interpreted
the "search and seizure" provision of the 1973 Constitution which provides:

. . . no search warrant or warrant of arrest shall issue except upon probable cause to be
determined by the judge, or such other responsible officer as may be authorized by law,
after examination under oath or affirmation of the complainant and the witnesses he
may produce . . .

We ruled:

. . . The issuance of a warrant is not a mere ministerial function; it calls for the exercise
of judicial discretion on the part of the issuing magistrate. This is clear from the
following provisions of Section 6, Rule 112 of the Rules of Court.
Page 82 of 377

Warrant of arrest, when issued. — If the judge be satisfied from the preliminary
examination conducted by him or by the investigating officer that the offense
complained of has been committed and that there is reasonable ground to believe that
the accused has committed it, he must issue a warrant or order for his arrest.

Under this section, the judge must satisfy himself of the existence of probable cause
before issuing a warrant or order of arrest. If on the face of the information the judge
finds no probable cause, he may disregard the fiscal's certification and require the
submission of the affidavits of witnesses to aid him in arriving at a conclusion as to the
existence of a probable cause. This has been the rule since U.S. v. Ocampo (18 Phil. 1)
and Amarga v. Abbas (98 Phil. 739). And this evidently is the reason for the issuance by
respondent of the questioned orders of April 13, 15, 16, 19, 1982 and July 13, 1982.
Without the affidavits of the prosecution witnesses and other evidence which, as a
matter of long-standing practice had been attached to the information filed in his sala,
respondent found the informations inadequate bases for the determination of probable
cause. For as the ensuing events would show, after petitioners had submitted the
required affidavits, respondent wasted no time in issuing the warrants of arrest in the
case where he was satisfied that probable cause existed.

The case of Soliven v. Makasiar (167 SCRA 393 [19881) was decided after the effectivity
of the 1987 Constitution. We stated:

The second issue, raised by petitioner Beltran, calls for an interpretation of the
constitutional provision on the issuance of warrants of arrest. The pertinent provision
reads:

Art. III, Sec. 2. The right of the people to be secure in their persons, houses, papers and
effects against unreasonable searches and seizures of whatever nature and for any
purpose shall be inviolable, and no search warrant or warrant of arrest shall issue
except upon probable cause to be determined personally by the judge after examination
under oath or affirmation of the complainant and the witnesses he may produce, and
particularly describing the place to be searched and the persons or things to be seized.

The addition of the word "personally" after the word "determined" and the deletion of the
grant of authority by the 1973 Constitution to issue warrants to "other respondent
officers as may be authorized by law", has apparently convinced petitioner Beltran that
the Constitution now requires the judge to personally examine the complainant and his
witnesses in his determination of probable cause for the issuance of arrest. This is not
an accurate interpretation.
Page 83 of 377

What the Constitution underscores is the exclusive and personal responsibility of the
issuing judge to satisfy himself of the existence of probable cause. In satisfying himself
of the existence of probable cause for the issuance of a warrant of arrest, the judge is
not required to personally examine the complainant and his witnesses. Following
established doctrine and procedures, he shall: (1) personally evaluate the report and the
supporting documents submitted by the fiscal regarding the existence of probable cause
and, on the basis thereof, issue a warrant of arrest; or (2) if on the basis thereof he finds
no probable cause, he may disregard the fiscal's report and require the submission of
supporting affidavits of witnesses to aid him in arriving at a conclusion as to the
existence of probable cause.

Sound policy dictates this procedure, otherwise judges would be unduly laden with the
preliminary examinations and investigation of criminal complaints instead of
concentrating on hearing and deciding cases filed before their courts.

The decision in People v. Honorable Enrique B. Inting, et al. (G.R. No. 88919, July 25,
1990), reiterated the above interpretation of "personal" determination by the Judge:

We emphasize important features of the constitutional mandate that ". . . no search


warrant or warrant of arrest shall issue except upon probable cause to be determined
personally by the judge . . ." (Article III, Section 2, Constitution)

First, the determination of probable cause is a function of the Judge. It is not for the
Provincial Fiscal or Prosecutor nor for the Election Supervisor to ascertain. Only the
Judge and the Judge alone makes this determination.

Second, the preliminary inquiry made by a Prosecutor does not bind the Judge. It merely
assists him to make the determination of probable cause. The Judge does not have to
follow what the Prosecutor presents to him. By itself, the Prosecutor's certification of
probable cause is ineffectual. It is the report, the affidavits, the transcripts of
stenographic notes (if any), and all other supporting documents behind the Prosecutor's
certification which are material in assisting the Judge to make his determination.

And third, Judges and Prosecutors alike should distinguish the preliminary inquiry
which determines probable cause for the issuance of a warrant of arrest from the
preliminary investigation proper which ascertains whether the offender should be held
for trial or released. Even if the two inquiries are conducted in the course of one and the
same proceeding, there should be no confusion about the objectives. The determination
of probable cause for the warrant of arrest is made by the Judge. The preliminary
Page 84 of 377

investigation proper –– whether or not there is reasonable ground to believe that the
accused is guilty of the offense charged and, therefore, whether or not he should be
subjected to the expense, rigors and embarrassment of trial –– is the function of the
Prosecutor.

The Court made this clear in the case of Castillo v. Villaluz (171 SCRA 39 [19891):

Judges of Regional Trial Courts (formerly Courts of First Instance) no longer have
authority to conduct preliminary investigations. That authority, at one time reposed in
them under Sections 13, 14 and 16, Rule 112 of the Rules of Court of 1964, (See Sec.
4, Rule 108, Rules of Court of 1940; People v. Solon, 47 Phil. 443, cited in Moran,
Comments on the Rules, 1980 ed., Vol. 4, pp. 115-116) was removed from them by the
1985 Rules on Criminal Procedure, effective on January 1, 1985, (Promulgated on
November 11, 1984) which deleted all provisions granting that power to said Judges.
We had occasion to point tills out in Salta v. Court of Appeals, 143 SCRA 228, and to
stress as well certain other basic propositions, namely: (1) that the conduct of a
preliminary investigation is "not a judicial function . . . (but) part of the prosecution's
job, a function of the executive," (2) that whenever "there are enough his or prosecutors
to conduct preliminary investigations, courts are counseled to leave this job which is
essentially executive to them," and the fact "that a certain power is granted does not
necessary mean that it should be indiscriminately exercised.

The 1988 Amendments to the 1985 Rules on Criminal Procedure, declared effective on
October 1, 1988, (The 1988 Amendments were published in the issue of Bulletin Today
of October 29, 1988) did not restore that authority to Judges of Regional Trial Courts;
said amendments did not in fact deal at all with the officers or courts having authority
to conduct preliminary investigations.

This is not to say, however, that somewhere along the line RTC Judges also lost the
power to make a preliminary examination for the purpose of determining whether
probable cause exists to justify the issuance of a warrant of arrest (or search warrant).
Such a power –– indeed, it is as much a duty as it is a power –– has been and remains
vested in every judge by the provisions in the Bill of Rights in the 1935, the 1973 and
the present [1987] Constitutions securing the people against unreasonable searches
and seizures, thereby placing it beyond the competence of mere Court Rule or Statute
to revoke. The distinction must, therefore, be made clear while an RTC Judge may no
longer conduct preliminary investigations to ascertain whether there is sufficient ground
for the filing of a criminal complaint or information, he retains the authority, when such
a pleading is filed with his court, to determine whether there is probable cause justifying
the issuance of a warrant of arrest. It might be added that this distinction accords,
Page 85 of 377

rather than conflicts, with the rationale of Salta because both law and rule, in restricting
to judges the authority to order arrest, recognize the function to be judicial in nature.

We reiterate that preliminary investigation should be distinguished as to whether it is


an investigation for the determination of a sufficient ground for the filing of the
information or it is an investigation for the determination of a probable cause for the
issuance of a warrant of arrest. The first kind of preliminary investigation is executive
in nature. It is part of the prosecution's job. The second kind of preliminary investigation
which is more properly called preliminary examination is judicial in nature and is lodged
with the Judge. . . .

Finally in the recent case of People v. Delgado, et al. (G.R. Nos. 93419-32, September
18, 1990) there is a statement that the judge may rely on the resolution of COMELEC
to file the information by the same token that it may rely on the certification made by
the prosecutor who conducted the preliminary investigation in the issuance of the
warrant of arrest. We, however, also reiterated that ". . . the court may require that the
record of the preliminary investigation be submitted to it to satisfy itself that there is
probable cause which will warrant the issuance of a warrant of arrest." (Section 2, Article
III, Constitution). Reliance on the COMELEC resolution or the Prosecutor's certification
presupposes that the records of either the COMELEC or the Prosecutor have been
submitted to the Judge and he relies on the certification or resolution because the
records of the investigation sustain the recommendation. The warrant issues not on the
strength of the certification standing alone but because of the records which sustain it.

It is obvious from the present petition that notwithstanding the above decisions, some
Judges are still bound by the inertia of decisions and practice under the 1935 and 1973
Constitutions and are sadly confused or hesitant. Prosecutors are also interested in a
clear cut ruling. We will, therefore, restate the rule in greater detail and hopefully clearer
terms.

There is no problem with search warrants which are relatively fewer and far between
and where there is no duplication of work between the Judge and the Prosecutor. The
problem lies with warrants of arrest especially in metropolitan or highly urban areas. If
a Judge has to personally question each complainant and witness or go over the records
of the Prosecutor's investigation page by page and word for word before he acts on each
of a big pile of applications for arrest warrants on his desk, he or she may have no more
time for his or her more important judicial functions.
Page 86 of 377

At the same time, the Judge cannot ignore the clear words of the 1987 Constitution
which requires ". . . probable cause to be personally determined by the judge . . .", not
by any other officer or person.

If a Judge relies solely on the certification of the Prosecutor as in this case where all the
records of the investigation are in Masbate, he or she has not personally determined
probable cause. The determination is made by the Provincial Prosecutor. The
constitutional requirement has not been satisfied. The Judge commits a grave abuse of
discretion.

The records of the preliminary investigation conducted by the Municipal Court of


Masbate and reviewed by the respondent Fiscal were still in Masbate when the
respondent Fiscal issued the warrants of arrest against the petitioners. There was no
basis for the respondent Judge to make his own personal determination regarding the
existence of a probable cause for the issuance of a warrant of arrest as mandated by the
Constitution. He could not possibly have known what transpired in Masbate as he had
nothing but a certification. Significantly, the respondent Judge denied the petitioners'
motion for the transmittal of the records on the ground that the mere certification and
recommendation of the respondent Fiscal that a probable cause exists is sufficient for
him to issue a warrant of arrest.

We reiterate the ruling in Soliven v. Makasiar that the Judge does not have to personally
examine the complainant and his witnesses. The Prosecutor can perform the same
functions as a commissioner for the taking of the evidence. However, there should be a
report and necessary documents supporting the Fiscal's bare certification. All of these
should be before the Judge.

The extent of the Judge's personal examination of the report and its annexes depends
on the circumstances of each case.1âwphi1 We cannot determine beforehand how
cursory or exhaustive the Judge's examination should be. The Judge has to exercise
sound discretion for, after all, the personal determination is vested in the Judge by the
Constitution. It can be as brief or as detailed as the circumstances of each case require.
To be sure, the Judge must go beyond the Prosecutor's certification and investigation
report whenever necessary. He should call for the complainant and witnesses
themselves to answer the court's probing questions when the circumstances of the case
so require.

It is worthy to note that petitioners Vicente Lim, Sr. and Susana Lim presented to the
respondent Judge documents of recantation of witnesses whose testimonies were used
to establish a prima facie case against them. Although, the general rule is that
Page 87 of 377

recantations are not given much weight in the determination of a case and in the
granting of a new trial (Tan Ang Bun v. Court of Appeals, et al. G.R. No. L-47747,
February 15, 1990, People v. Lao Wan Sing, 46 SCRA 298 [1972]) the respondent Judge
before issuing his own warrants of arrest should, at the very least, have gone over the
records of the preliminary examination conducted earlier in the light of the evidence
now presented by the concerned witnesses in view of the "political undertones"
prevailing in the cases. Even the Solicitor General recognized the significance of the
recantations of some witnesses when he recommends a reinvestigation of the cases, to
wit:

It must be pointed out, however, that among the documents attached to this Petition
are affidavits of recantation subsequently executed by Jimmy Cabarles and Danilo
Lozano and an affidavit executed by one, Camilo Sanano, father of the complainant's
witnesses, Renato and Romeo Sanano. It was precisely on the strength of these earlier
written statements of these witnesses that the Municipal Trial Court of Masbate found
the existence of a prima facie case against petitioners and accordingly recommended
the filing of a Criminal Information. Evidently, the same written statements were also
the very basis of the "Fiscal's Certification", since the attached affidavits of recantation
were not yet then available. Since the credibility of the prosecution witnesses is now
assailed and put in issue and, since the petitioners have not yet been arraigned, it would
be to the broader interest of justice and fair play if a reinvestigation of this case be had
to secure the petitioners against hasty prosecution and to protect them from an open
and public accusation of crime, from the trouble, expense and anxiety of a public trial,
and also to protect the State from useless and expensive trials (Salonga v. Paño G.R.
No. 59524, February 18,1985). (Rollo of G.R. Nos. 94054-56, pp. 200-201)

We reiterate that in making the required personal determination, a Judge is not


precluded from relying on the evidence earlier gathered by responsible officers. The
extent of the reliance depends on the circumstances of each case and is subject to the
Judge's sound discretion. However, the Judge abuses that discretion when having no
evidence before him, he issues a warrant of arrest.

Indubitably, the respondent Judge committed a grave error when he relied solely on the
Prosecutor's certification and issued the questioned Order dated July 5, 1990 without
having before him any other basis for his personal determination of the existence of a
probable cause.

WHEREFORE, the instant petitions are hereby GRANTED. The questioned Order of
respondent Judge Nemesio S. Felix of Branch 56, Regional Trial Court of Makati dated
July 5, 1990 is declared NULL and VOID and SET ASIDE. The Temporary Restraining
Page 88 of 377

Orders and Preliminary Mandatory Injunction issued in the instant Petitions are made
PERMANENT.

SO ORDERED.

Fernan, C.J., Narvasa, Melencio-Herrera, Cruz, Paras, Feliciano, Gancayco, Padilla,


Bidin, Griño-Aquino, Medialdea and Regalado, JJ., concur.
Sarmiento, J., took no part.
Page 89 of 377

G.R. No. 143591 November 23, 2007

TEODORO C. BORLONGAN, JR., CORAZON M. BEJASA, ARTURO E.


MANUEL, JR., ERIC L. LEE, P. SIERVO H. DIZON, BENJAMIN DE LEON,
DELFIN C. GONZALEZ, JR., and BEN YU LIM, JR., Petitioners, vs.
MAGDALENO M. PEÑA and HON. MANUEL Q. LIMSIACO, JR., as Judge
Designate of the Municipal Trial Court in Cities, Bago City, Respondents.

NACHURA, J.:

For review is the Decision1 of the Court of Appeals (CA) dated June 20, 2000 in CA-G.R.
SP No. 49666 dismissing the petition for certiorari filed by petitioners Teodoro C.
Borlongan, Jr., Corazon M. Bejasa, Arturo Manuel, Jr., Benjamin de Leon, P. Siervo
Dizon, Delfin C. Gonzalez, Jr., Eric Lee and Ben T. Lim, Jr.

The factual and procedural antecedents of the case are as follows:

Respondent Magdaleno Peña instituted a civil case for recovery of agent’s compensation
and expenses, damages, and attorney’s fees,2 against Urban Bank and the petitioners,
before the Regional Trial Court (RTC) of Negros Occidental, Bago City. The case was
raffled to Branch 62 and was docketed as Civil Case No. 754. Respondent anchored his
claim for compensation on the contract of agency3 allegedly entered into with the
petitioners wherein the former undertook to perform such acts necessary to prevent any
intruder and squatter from unlawfully occupying Urban Bank’s property located along
Roxas Boulevard, Pasay City. Petitioners filed a Motion to Dismiss4 arguing that they
never appointed the respondent as agent or counsel. Attached to the motion were the
following documents: 1) a letter5 dated December 19, 1994 signed by Herman Ponce
and Julie Abad on behalf of Isabela Sugar Company, Inc. (ISCI), the original owner of
the subject property; 2) an unsigned letter6 dated December 7, 1994 addressed to
Corazon Bejasa from Marilyn G. Ong; 3) a letter7 dated December 9, 1994 addressed to
Teodoro Borlongan and signed by Marilyn G. Ong; and 4) a Memorandum8 dated
November 20, 1994 from Enrique Montilla III. Said documents were presented in an
attempt to show that the respondent was appointed as agent by ISCI and not by Urban
Bank or by the petitioners.

In view of the introduction of the above-mentioned documents, respondent Peña filed


his Complaint-Affidavit9 with the Office of the City Prosecutor, Bago City.10 He claimed
that said documents were falsified because the alleged signatories did not actually affix
their signatures, and the signatories were neither stockholders nor officers and
employees of ISCI.11 Worse, petitioners introduced said documents as evidence before
the RTC knowing that they were falsified.
Page 90 of 377

In a Resolution12 dated September 23, 1998, the City Prosecutor concluded that the
petitioners were probably guilty of four (4) counts of the crime of Introducing Falsified
Documents penalized by the second paragraph of Article 172 of the Revised Penal Code
(RPC). The City Prosecutor concluded that the documents were falsified because the
alleged signatories untruthfully stated that ISCI was the principal of the respondent;
that petitioners knew that the documents were falsified considering that the signatories
were mere dummies; and that the documents formed part of the record of Civil Case No.
754 where they were used by petitioners as evidence in support of their motion to
dismiss, adopted in their answer and later, in their Pre-Trial Brief.13 Subsequently, the
corresponding Informations14 were filed with the Municipal Trial Court in Cities
(MTCC), Bago City. The cases were docketed as Criminal Cases Nos. 6683, 6684, 6685,
and 6686. Thereafter, Judge Primitivo Blanca issued the warrants15 for the arrest of
the petitioners.

On October 1, 1998, petitioners filed an Omnibus Motion to Quash, Recall Warrants of


Arrest and/or For Reinvestigation.16 Petitioners insisted that they were denied due
process because of the non-observance of the proper procedure on preliminary
investigation prescribed in the Rules of Court. Specifically, they claimed that they were
not afforded the right to submit their counter-affidavit. They then argued that since no
such counter-affidavit and supporting documents were submitted by the petitioners,
the trial judge merely relied on the complaint-affidavit and attachments of the
respondent in issuing the warrants of arrest, also in contravention of the Rules.
Petitioners further prayed that the information be quashed for lack of probable cause.
Lastly, petitioners posited that the criminal case should have been suspended on the
ground that the issue being threshed out in the civil case is a prejudicial question.

In an Order17 dated November 13, 1998, the court denied the omnibus motion primarily
on the ground that preliminary investigation was not available in the instant case ---
which fell within the jurisdiction of the MTCC. The court, likewise, upheld the validity
of the warrant of arrest, saying that it was issued in accordance with the Rules. Besides,
the court added, petitioners could no longer question the validity of the warrant since
they already posted bail. The court also believed that the issue involved in the civil case
was not a prejudicial question, and thus, denied the prayer for suspension of the
criminal proceedings. Lastly, the court was convinced that the Informations contained
all the facts necessary to constitute an offense.

Petitioners subsequently instituted a special civil action for Certiorari and Prohibition
with Prayer for Writ of Preliminary Injunction and TRO, before the CA ascribing grave
abuse of discretion amounting to lack or excess of jurisdiction on the part of the MTCC
in issuing and not recalling the warrants of arrest, reiterating the arguments in their
Page 91 of 377

omnibus motion.18 They, likewise, questioned the court’s conclusion that by posting
bail, petitioners already waived their right to assail the validity of the warrant of arrest.

On June 20, 2000, the CA dismissed the petition.19 Hence, the instant petition for
review on certiorari under Rule 45 of the Rules of Court. Petitioners now raise before us
the following issues:

A.

Where the offense charged in a criminal complaint is not cognizable by the Regional
Trial Court and not covered by the Rule on Summary Procedure, is the finding of
probable cause required for the filing of an Information in court?

If the allegations in the complaint-affidavit do not establish probable cause, should not
the investigating prosecutor dismiss the complaint, or at the very least, require the
respondent to submit his counter-affidavit?

B.

Can a complaint-affidavit containing matters which are not within the personal
knowledge of the complainant be sufficient basis for the finding of probable cause?

C.

Where the offense charged in a criminal complaint is not cognizable by the Regional
Trial Court and not covered by the Rule on Summary Procedure, and the record of the
preliminary investigation does not show the existence of probable cause, should not the
judge refuse to issue a warrant of arrest and dismiss the criminal case, or at the very
least, require the accused to submit his counter-affidavit in order to aid the judge in
determining the existence of probable cause?

D.

Can a criminal prosecution be restrained?


Page 92 of 377

E.

Can this Honorable Court itself determine the existence of probable cause?20

On August 2, 2000, this Court issued a Temporary Restraining Order (TRO)21 enjoining
the judge of the MTCC from proceeding in any manner with Criminal Cases Nos. 6683
to 6686, effective during the entire period that the case is pending before, or until further
orders of, this Court.

With the MTCC proceedings suspended, we now proceed to resolve the issues raised.

Respondents contend that the foregoing issues had become moot and academic when
the petitioners posted bail and were arraigned.

We do not agree.

It appears that upon the issuance of the warrant of arrest, petitioners immediately
posted bail as they wanted to avoid embarrassment being then the officers of Urban
Bank. On the scheduled date for the arraignment, despite the petitioners’ refusal to
enter a plea, the court entered a plea of "Not Guilty."

The earlier ruling of this Court that posting of bail constitutes a waiver of the right to
question the validity of the arrest has already been superseded by Section 26,22 Rule
114 of the Revised Rules of Criminal Procedure. Furthermore, the principle that the
accused is precluded from questioning the legality of his arrest after arraignment is true
only if he voluntarily enters his plea and participates during trial, without previously
invoking his objections thereto.23

Records reveal that petitioners filed the omnibus motion to quash the information and
warrant of arrest, and for reinvestigation, on the same day that they posted bail. Their
bail bonds likewise expressly contained a stipulation that they were not waiving their
right to question the validity of their arrest.24 On the date of the arraignment, the
petitioners refused to enter their plea, obviously because the issue of the legality of the
information and their arrest was yet to be settled by the Court. This notwithstanding,
the court entered a plea of "Not Guilty." From these circumstances, we cannot
Page 93 of 377

reasonably infer a valid waiver on the part of the petitioners, as to preclude them from
raising the issue of the validity of the arrest before the CA and eventually before this
Court.

In their petition filed before this Court, petitioners prayed for a TRO to restrain the
MTCC from proceeding with the criminal cases (which the Court eventually issued on
August 2, 2000). Thus, we confront the question of whether a criminal prosecution can
be restrained, to which we answer in the affirmative.

As a general rule, the Court will not issue writs of prohibition or injunction, preliminary
or final, to enjoin or restrain criminal prosecution. However, the following exceptions to
the rule have been recognized: 1) when the injunction is necessary to afford adequate
protection to the constitutional rights of the accused; 2) when it is necessary for the
orderly administration of justice or to avoid oppression or multiplicity of actions; 3) when
there is a prejudicial question which is sub judice; 4) when the acts of the officer are
without or in excess of authority; 5) where the prosecution is under an invalid law,
ordinance or regulation; 6) when double jeopardy is clearly apparent; 7) where the Court
has no jurisdiction over the offense; 8) where it is a case of persecution rather than
prosecution; 9) where the charges are manifestly false and motivated by the lust for
vengeance; and 10) when there is clearly no prima facie case against the accused and a
motion to quash on that ground has been denied.25

Considering that the issues for resolution involve the validity of the information and
warrant of arrest, and considering further that no waiver of rights may be attributed to
the petitioners as earlier discussed, we issued a TRO on August 2, 2000 to give the
Court the opportunity to resolve the case before the criminal prosecution is allowed to
continue. The nature of the crime and the penalty involved (which is less than 4 years
of imprisonment), likewise, necessitate the suspension of the case below in order to
prevent the controversy from being mooted.

We now proceed with the main issues, viz.: 1) whether petitioners were deprived of their
right to due process of law because of the denial of their right to preliminary
investigation and to submit their counter-affidavit; 2) whether the Informations charging
the petitioners were validly filed and the warrants for their arrest were properly issued;
and 3) whether this Court can, itself, determine probable cause.

As will be discussed below, the petitioners could not validly claim the right to
preliminary investigation. Still, petitioners insist that they were denied due process
because they were not afforded the right to submit counter-affidavits which would have
aided the court in determining the existence of probable cause.26 Petitioners also claim
Page 94 of 377

that the respondent’s complaint-affidavit was not based on the latter’s personal
knowledge; hence, it should not have been used by the court as basis in its finding of
probable cause.27 Moreover, petitioners aver that there was no sufficient evidence to
prove the elements of the crime. Specifically, it was not established that the documents
in question were falsified; that petitioners were the ones who presented the documents
as evidence; and that petitioners knew that the documents were indeed falsified.28
Petitioners likewise assert that at the time of the filing of the complaint-affidavit, they
had not yet formally offered the documents as evidence; hence, they could not have
"introduced" the same in court.29 Considering the foregoing, petitioners pray that this
Court, itself, determine whether or not probable cause exists.30

The pertinent provisions of the 1985 Rules of Criminal Procedure,31 namely, Sections
1, 3 (a) and 9(a) of Rule 112, are relevant to the resolution of the aforesaid issues:

SECTION 1. Definition. – Preliminary investigation is an inquiry or proceeding for the


purpose of determining whether there is sufficient ground to engender a well-founded
belief that a crime cognizable by the Regional Trial Court has been committed and that
the respondent is probably guilty thereof, and should be held for trial.32

SEC. 3. Procedure. – Except as provided for in Section 7 hereof, no complaint or


information for an offense cognizable by the Regional Trial Court shall be filed without
a preliminary investigation having been first conducted in the following manner:

(a) The complaint shall state the known address of the respondent and be accompanied
by affidavits of the complainant and his witnesses as well as other supporting
documents, in such number of copies as there are respondents, plus two (2) copies of
the official file. The said affidavits shall be sworn to before any fiscal, state prosecutor
or government official authorized to administer oath, or, in their absence or
unavailability, a notary public, who must certify that he personally examined the
affiants and that he is satisfied that they voluntarily executed and understood their
affidavits.33

SEC. 9. Cases not falling under the original jurisdiction of the Regional Trial Courts not
covered by the Rule on Summary Procedure. –

(a) Where filed with the fiscal. – If the complaint is filed directly with the fiscal or state
prosecutor, the procedure outlined in Section 3 (a) of this Rule shall be observed. The
Fiscal shall take appropriate action based on the affidavits and other supporting
documents submitted by the complainant.34
Page 95 of 377

Petitioners were charged with the offense defined and penalized by the second paragraph
of Article 17235 of the Revised Penal Code. The penalty imposable is arresto mayor in
its maximum period to prision correccional in its minimum period, or four (4) months
and one (1) day to two (2) years and four (4) months. Clearly, the case is cognizable by
the Municipal Trial Court and preliminary investigation is not mandatory.36

Records show that the prosecutor relied merely on the complaint-affidavit of the
respondent and did not require the petitioners to submit their counter-affidavits. The
prosecutor should not be faulted for taking this course of action, because it is
sanctioned by the Rules. To reiterate, upon the filing of the complaint and affidavit with
respect to cases cognizable by the MTCC, the prosecutor shall take the appropriate
action based on the affidavits and other supporting documents submitted by the
complainant. It means that the prosecutor may either dismiss the complaint if he does
not see sufficient reason to proceed with the case, or file the information if he finds
probable cause. The prosecutor is not mandated to require the submission of counter-
affidavits. Probable cause may then be determined on the basis alone of the affidavits
and supporting documents of the complainant, without infringing on the constitutional
rights of the petitioners.

On the other hand, for the issuance of a warrant of arrest, the judge must personally
determine the existence of probable cause. Again, the petitioners insist that the trial
judge erred in issuing the warrant of arrest without affording them their right to submit
their counter-affidavits.

Section 2, Article III of the Constitution provides:

SEC. 2. The right of the people to be secure in their persons, houses, papers, and effects
against unreasonable searches and seizures of whatever nature and for any purpose
shall be inviolable, and no search warrant or warrant of arrest shall issue except upon
probable cause to be determined personally by the judge after examination under oath
or affirmation of the complainant and the witnesses he may produce, and particularly
describing the place to be searched and the persons or things to be seized.

What the Constitution underscores is the exclusive and personal responsibility of the
issuing judge to satisfy himself of the existence of probable cause. But the judge is not
required to personally examine the complainant and his witnesses. Following
established doctrine and procedure, he shall (1) personally evaluate the report and the
supporting documents submitted by the prosecutor regarding the existence of probable
cause, and on the basis thereof, he may already make a personal determination of the
Page 96 of 377

existence of probable cause; and (2) if he is not satisfied that probable cause exists, he
may disregard the prosecutor’s report and require the submission of supporting
affidavits of witnesses to aid him in arriving at a conclusion as to the existence of
probable cause.37

In determining probable cause for the issuance of the warrant of arrest in the case at
bench, we find nothing wrong with the procedure adopted by the trial judge --- he relied
on the resolution of the prosecutor, as well as the supporting documents submitted by
the respondent. There is no provision of law or procedural rule which makes the
submission of counter-affidavits mandatory before the judge can determine whether or
not there exists probable cause to issue the warrant.

In light of the foregoing, it appears that the proper procedure was followed by the
prosecutor in determining probable cause for the filing of the informations, and by the
trial court judge in determining probable cause for the issuance of the warrants of
arrest. To reiterate, preliminary investigation was not mandatory, and the submission
of counter-affidavit was not necessary.1âwphi1

However, notwithstanding the proper observance of the procedure laid down by the
Rules, a closer scrutiny of the records reveals that the Informations should not have
been filed and the warrants of arrest should not have been issued, because of lack of
probable cause.

Probable cause, for purposes of filing a criminal information, has been defined as such
facts as are sufficient to engender a well-founded belief that a crime has been committed
and that the accused is probably guilty thereof.38 It is the existence of such facts and
circumstances as would excite the belief in a reasonable mind, acting on the facts within
the knowledge of the prosecutor, that the person charged was guilty of the crime for
which he is to be prosecuted.39 A finding of probable cause needs only to rest on
evidence showing that, more likely than not, a crime has been committed and that it
was committed by the accused.40

On the other hand, we have defined probable cause for the issuance of a warrant of
arrest as the existence of such facts and circumstances that would lead a reasonably
discreet and prudent person to believe that an offense has been committed by the person
sought to be arrested.41

To accord respect to the discretion granted to the prosecutor and for reasons of
practicality, this Court, as a rule, does not interfere with the prosecutor’s determination
Page 97 of 377

of probable cause. Otherwise, courts would be swamped with petitions to review the
prosecutor’s findings in such investigations.42 In the same way, the general rule is that
this Court does not review the factual findings of the trial court, which include the
determination of probable cause for the issuance of a warrant of arrest.43 It is only in
exceptional cases when this Court may set aside the conclusions of the prosecutor and
the trial judge on the existence of probable cause, that is, when it is necessary to prevent
the misuse of the strong arm of the law or to protect the orderly administration of
justice.44 The facts obtaining in the present case warrant the application of the
exception.

Petitioners were charged with violation of par. 2, Article 172 of the RPC or Introduction
of Falsified Document in a Judicial Proceeding. The elements of the offense are as
follows:

1. That the offender knew that a document was falsified by another person.

2. That the false document is embraced in Article 171 or in any subdivisions No. 1 or 2
of Article 172.

3. That he introduced said document in evidence in any judicial proceeding.45

The falsity of the document and the defendant’s knowledge of its falsity are essential
elements of the offense.46

The Office of the City Prosecutor filed the Informations against the petitioners on the
basis of the complaint-affidavit of the respondent, together with the following attached
documents: the motion to dismiss and answer filed by the petitioners in Civil Case No.
754; petitioners’ pre-trial brief in said case; the alleged falsified documents; a copy of
the minutes of the regular meeting of ISC during the election of the board; and the list
of stockholders of ISC.47 On the basis of these documents and on the strength of the
affidavit executed by the respondent, the prosecutor concluded that probable cause
exists. These same affidavit and documents were used by the trial court in issuing the
warrant of arrest.

Contrary to the findings of the MTCC, as affirmed by the Court of Appeals, we find the
complaint-affidavit and attachments insufficient to support the existence of probable
cause. Specifically, the respondent failed to sufficiently establish prima facie that the
alleged documents were falsified. In support of his claim of falsity of the documents, the
Page 98 of 377

private respondent stated in his complaint-affidavit that Herman Ponce, Julie Abad and
Marilyn Ong, the alleged signatories of the questioned letters, did not actually affix their
signatures; and that they were not actually officers or stockholders of ISCI.48 He further
claimed that Enrique Montilla’s signature appearing in another memorandum
addressed to respondent was forged.49 These are mere assertions, insufficient to
warrant the filing of the complaint or the issuance of the warrant of arrest.

It must be emphasized that the affidavit of the complainant, or any of his witnesses,
shall allege facts within their (affiants) personal knowledge. The allegation of the
respondent that the signatures of Ponce, Abad, Ong and Montilla were falsified does not
qualify as personal knowledge. Nowhere in said affidavit did respondent state that he
was present at the time of the execution of the documents. Neither did he claim that he
was familiar with the signatures of the signatories. He simply made a bare assertion
that the signatories were mere dummies of ISCI and they were not in fact officers,
stockholders or representatives of the corporation. At the very least, the affidavit was
based on respondent’s "personal belief" and not "personal knowledge."50 Considering
the lack of personal knowledge on the part of the respondent, he could have submitted
the affidavit of other persons who are qualified to attest to the falsity of the signatures
appearing in the questioned documents. One cannot just claim that a certain document
is falsified without further stating the basis for such claim, i.e., that he was present at
the time of the execution of the document or he is familiar with the signatures in
question. Otherwise, this could lead to abuse and malicious prosecution. This is actually
the reason for the requirement that affidavits must be based on the personal knowledge
of the affiant. The requirement assumes added importance in the instant case where
the accused were not made to rebut the complainant’s allegation through counter-
affidavits.

Neither can the respondent find support in the documents attached to his complaint-
affidavit. The minutes of the regular meeting, as well as the list of stockholders, could
have possibly shown that the signatories were not officers or stockholders of the
corporation. However, they did not at all show that the questioned documents were
falsified. In the letter allegedly signed by Ponce and Abad, there was no representation
that they were the president and corporate secretary of ISCI. Besides, the mere fact that
they were not officers or stockholders of ISCI does not necessarily mean that their
signatures were falsified. They still could have affixed their signatures as authorized
representatives of the corporation.

True, a finding of probable cause need not be based on clear and convincing evidence,
or on evidence beyond reasonable doubt. It does not require that the evidence would
justify conviction. Nonetheless, although the determination of probable cause requires
less than evidence which would justify conviction, it should at least be more than mere
suspicion.51 While probable cause should be determined in a summary manner, there
is a need to examine the evidence with care to prevent material damage to a potential
Page 99 of 377

accused’s constitutional right to liberty and the guarantees of freedom and fair play,
and to protect the State from the burden of unnecessary expenses in prosecuting alleged
offenses and holding trials arising from false, fraudulent or groundless charges.52 It is,
therefore, imperative for the prosecutor to relieve the accused from the pain and
inconvenience of going through a trial once it is ascertained that no probable cause
exists to form a sufficient belief as to the guilt of the accused.53

Considering that the respondent failed to adduce sufficient evidence to support his claim
that the documents were falsified, it follows that the introduction of the questioned
documents in Civil Case No. 754 is not an offense punished by any provision of the
Revised Penal Code or any other law. The petitioners should not be burdened with court
proceedings, more particularly a criminal proceeding, if in the first place, there is no
evidence sufficient to engender a well-founded belief that an offense was committed.

WHEREFORE, the petition is GRANTED. The Decision of the Court of Appeals, dated
June 20, 2000, in CA-G.R. SP No. 49666 is REVERSED and SET ASIDE. The Temporary
Restraining Order dated August 2, 2000 is hereby made permanent. Accordingly, the
Municipal Trial Court in Cities, City of Bago, is ORDERED to DISMISS Criminal Case
Nos. 6683-86.

SO ORDERED.
Page 100 of 377

G.R. No. L-23051 October 20, 1925

THE PEOPLE OF THE PHILIPPINES ISLANDS, plaintiff-appellant, vs. JOSE


MA. VELOSO, defendant-appellant.

Claro M. Recto for appellant.


Attorney-General Villa-Real for appellee.

MALCOLM, J.:

This is an appeal from a judgment of the Court of First Instance of Manila finding the
accused, Jose Ma. Veloso, guilty of the crime of resistance of the agents of the authority,
in violation of article 252 of the Penal Code, and sentencing him to four months and one
day imprisonment, arresto mayor, with the accessory penalties, to pay a fine of P200,
with the corresponding subsidiary imprisonment in case of insolvency, and to pay the
costs. The errors assigned by counsel for the accused as appellant, go to the proposition
that the resistance of the police was justifiable on account of the illegality of the John
Doe search warrant.

In May, 1923, the building located at No. 124 Calle Arzobispo, City of Manila, was used
by an organization known as the Parliamentary Club. Jose Ma. Veloso was at that time
a member of the House of Representative of the Philippine Legislature. He was also the
manager of the club.

The police of Manila had reliable information that the so-called Parliamentary Club was
nothing more than a gambling house. Indeed, on May 19, 1923, J. F. Townsend, the
chief of the gambling squad, had been to the club and verified this fact. As a result, on
May 25, 1923, Detective Andres Geronimo of the secret service of the City of Manila,
applied for, and obtained a search warrant from Judge Garduño of the municipal court.
Thus provided, the police attempted to raid the Parliamentary Club a little after three in
the afternoon of the date above- mentioned. They found the doors to the premises closed
and barred. Accordingly, one band of police including policeman Rosacker, ascended a
telephone pole, so as to enter a window of the house. Other policemen, headed by
Townsend, broke in the outer door.

Once inside the Parliamentary Club, nearly fifty persons were apprehended by the
police. One of them was the defendant Veloso. Veloso asked Townsend what he wanted,
and the latter showed him the search warrant. Veloso read it and told Townsend that
he was Representative Veloso and not John Doe, and that the police had no right to
Page 101 of 377

search the house. Townsend answered that Veloso was considered as John Doe. As
Veloso's pocket was bulging, as if it contained gambling utensils, Townsend required
Veloso to show him the evidence of the game. About five minutes was consumed in
conversation between the policemen and the accused the policemen insisting on
searching Veloso, and Veloso insisting in his refusal to submit to the search.

At last the patience of the officers was exhausted. So policeman Rosacker took hold of
Veloso only to meet with his resistance. Veloso bit Rosacker in the right forearm, and
gave him a blow in another part of the body, which injured the policeman quite severely.
Through the combined efforts of Townsend and Rosacker, Veloso was finally laid down
on the floor, and long sheets of paper, of reglas de monte, cards, cardboards, and chips
were taken from his pockets.

All of the persons arrested were searched and then conducted to the patrol wagons.
Veloso again refused to obey and shouted offensive epithets against the police
department. It was necessary for the policemen to conduct him downstairs. At the door,
Veloso resisted so tenaciously that three policemen were needed to place him in the
patrol wagon. 1awph!l.net

In the municipal court of the City of Manila, the persons arrest in the raid were accused
of gambling. All of them were eventually acquitted in the Court of First Instance for lack
of proof, with the sole exception of Veloso, who was found guilty of maintaining a
gambling house. This case reached the appellate court where the accused was finally
sentenced to pay a fine of P500. (No. 22163. 1 )

The foregoing are the principal facts taken mainly from the findings of the trial judge,
the Honorable Vicente Nepomuceno. Counsel for the appellant makes no effort to
impugn these findings, except that he stresses certain points as more favorable to the
case of his client. The defense, as previously indicated, is planted squarely on the
contention that since the name of Veloso did not appear in the search warrant, but
instead the pseudonym John Doe was used, Veloso had a legal right to resist the police
by force. The nature of this defense makes it advisable to set forth further facts, relating
particularly to the search warrant, before passing to the law.

There are found in the record the application for search warrant, the affidavit for search
warrant, and the search warrant. The application reads:

UNITED STATES OF AMERICA


PHILIPPINE ISLANDS
Page 102 of 377

IN THE MUNICIPAL COURT OF THE CITY OF MANILA

THE PEOPLE OF THE PHILIPPINE ISLANDS, plaintiff, vs. JOHN DOE, Defendant.

APPLICATION FOR (G)


SEARCH WARRANT

Testimony taken before Hon. L. Garduño, Judge, Municipal Court, Manila.

Andres Geronimo, being duly sworn, testifies as follows:

Q. What is your name, residence and occupation? — A. Andres Geronimo, No. 47


Revellin, detective.

Q. Are you the applicant of this search warrant? — A. Yes, sir.

Q. Do you know the premises situated at No. 124 Calle Arzobispo, District of W. C., City
of Manila? — A. Yes. sir.

Q. Do you know who occupies said premises? — A. I do not know. According to the best
of my information the house is occupied by John Doe.

Q . What are your reasons for applying for this search warrant? — A. It has been reported
to me by a person whom I consider to be reliable that in said premises there are
instruments and devices used in gambling games, such as cards, dice, chips, lottery
tickets, lists of drawing and lists used in prohibited games kept. It has been reported to
me by a person whom I consider to be reliable that there are or there will be gambling
conducted in said premises. The aforesaid premises are known as gambling house. I
have watched the foregoing premises and believed it to be a gambling house and a place
where instruments and devices used in gambling games, such as cards, dice, chips,
lottery tickets, lists of drawing and lists used in prohibited games are kept.
Page 103 of 377

I, Andres Geronimo, being duly sworn, depose and say that I have read the foregoing
questions and answers and that I find the same to correct and true to the best of my
knowledge and belief.

(Sgd.) ANDRES GERONIMO

Subscribed and sworn to before me this 25th day of May, 1923.

(Sgd.) L. GARDUÑO Judge, Municipal Court

The affidavit and the search warrant are so nearly alike that it will suffice to copy the
search warrant alone. This document reads:

UNITED STATES OF AMERICA


PHILIPPINE ISLANDS

IN THE MUNICIPAL COURT OF THE CITY OF MANILA

THE PEOPLE OF THE PHILIPPINE ISLANDS, Plaintiff,

vs.

JOHN DOE, Defendant.

SEARCH WARRANT (G)

The People of the Philippine Islands, to any member of the

Police Force of the City of Manila.

GREETING:
Page 104 of 377

Proof by affidavit having this day been made before me by Andres Geronimo that he has
good reason to believe and does believe that John Doe has illegally in his possession in
the building occupied by him and which is under his control, namely in the building
numbered 124 Calle Arzobispo, City of Manila, Philippines Islands, certain devices and
effects used in violation of the Gambling Law, to wit: money, cards, chips, reglas, pintas,
tables and chairs and other utensils used in connection with the game commonly known
as monte and that the said John Doe keeps and conceals said devices and effects with
the illegal and criminal intention of using them in violation of the Gambling Law.

Now therefore, you are hereby commanded that at any time in the day or night within
ten (10) days on or after this date to make a search on the person of said John Doe and
in the house situated at No. 124 Calle Arzobispo, City of Manila, Philippine Islands, in
quest of the above described devices and effects and if you find the same or any part
thereof, you are commanded to bring it forthwith before me as provided for by law.

Given under my hand, this 25th day of May, 1923.

(Sgd.) L. GARDUÑO
Judge, Municipal Court

Coming now to the legal aspects of the case it is first worthy of mention that by reason
of the Fourth Amendment to the United States Constitution and the eleventh and
eighteenth paragraphs of the Philippine Bill of Rights, as found in the present Organic
Act, the security of the dwelling and the person is guaranteed. The organic act provides
"that the right to be secured against unreasonable searches and seizures shall not be
violated." It further provides "that no warrant shall issue but upon probable cause,
supported by oath or affirmation and particularly describing the place to be searched
and the person or things to be seized."

In the Philippine Code of Criminal Procedure are found provisions of the same import
although naturally entering more into detail. It is therein provided, among other things,
that "a search warrant shall not issue except for probable cause and upon application
supported by oath particularly describing the place to be searched and the person of
thing to be seized." (Section 97.) After the judge or justice shall have examined on oath
the complainant and any witnesses he may produce, and shall have taken their
depositions in writing (section 98), and after the judge or justice is satisfied of the
existence of facts upon which the application is based, or that there is probable cause
Page 105 of 377

to believe that they exist, he must issue the warrant which must be substantially in the
following form:

. . . You are, therefore, commanded, . . . to make immediate search on the person of


............................, or in the house situated ...................................... (describing it or
any other place to be searched with reasonable particularity, as the case may be) for the
following property: . . . ." (Section 99.) It is finally provided that "a person charged with
a crime may be searched for dangerous weapons or anything which may be used as
proof of the commission of the crime. (Section 105).

A search warrant must conform strictly to the requirements of the constitutional and
statutory provisions under which it is issued. Otherwise it has rightly been held, must
be absolutely legal, "for there is not a description of process known to the law, the
execution of which is more distressing to the citizen. Perhaps there is none which excites
such intense feeling in consequence of its humiliating and degrading effect." The warrant
will always be construed strictly without, however, going the full length of requiring
technical accuracy. No presumptions of regularity are to be invoked in aid of the process
when an officer undertakes to justify under it. (24 R. C. L., pp. 711, et seq.; Reed vs.
Rice [1829], 2 J. J. Marshall [Ky.] 44; 19 Am. Dec., 122; Smith vs. McDuffee [1914], 72
Ore., 276; Ann. Cas. 1916 D, 947.)

The search warrant has been likened to a warrant of arrest. Although apprehending that
there are material differences between the two, in view of the paucity of authority
pertaining to John Doe search warrants we propose to take into consideration the
authorities relied upon by the appellant, thus following the precedent of Uy Kheytin vs.
Villareal ([1920], 42 Phil., 886), where the regularity of the issuance of the search
warrant was also questioned.

In the lower court, and again in this court, the attorneys for the defense quoted from
Wharton's Criminal Procedure. In that text at pages 51, 52, 54, 55, and 56 of volume 1
of the Tenth Edition, is found the following:

Form and Sufficiency of Warrant. Technical accuracy is not required. . . .

xxx xxx xxx


Page 106 of 377

Name and description of the accused should be inserted in the body of the warrant and
where the name is unknown there must be such a description of the person accused as
will enable the officer to identify him when found.

xxx xxx xxx

Warrant for apprehension of unnamed party, or containing a wrong name for the party
to be apprehended is void, except in those cases where it contains a descriptio personae
such as will enable the officer to identify the accused.

xxx xxx xxx

John Doe' Warrants. It follows, on principle, from what has already been said regarding
the essential requirements of warrants for the apprehension of persons accused, and
about blank warrants, that a warrant for the apprehension of a person whose true name
is unknown, by the name of "John Doe" or "Richard Roe," "whose other or true name in
unknown," is void, without other and further descriptions of the person to be
apprehended, and such warrant will not justify the officer in acting under it. Such a
warrant must, in addition, contain the best descriptio personae possible to be obtained
of the person or persons to be apprehended, and this description must be sufficient to
indicate clearly the proper person or persons upon whom the warrant is to be served;
and should state his personal appearance and peculiarities, give his occupation and
place of residence, and any other circumstances by means of which he can be identified.

Person apprehended in act of committing a crime, under a "John Doe" warrant, on the
other hand, the apprehension will not be illegal, or the officer liable, because under such
circumstances it is not necessary that a warrant should have been issued.

The authority most often cited to sustain the text, and quoted with approval by the
United States Supreme Court, is the case of Commonwealth vs. Crotty ([1865], 10 Allen
[Mass.], 403). It there appeared that one Peaslee had made a complaint to the police
court Lee, charging that "John Doe or Richard Roe, whose other or true name is to your
complainant unknown," had committed an assault and battery upon him; upon which
complaint a warrant was issued against "John Doe or Richard Roe, whose other or true
name is to your complainant unknown, named in the foregoing complaint." Neither the
complaint nor the warrant contained any further description or means of identification
of the person to be arrested. Crotty resisted the arrest upon the ground that the warrant
was invalid. Mr. Chief Justice Bigelow, as the organ of the Supreme Court of
Massachusetts, said:
Page 107 of 377

We cannot entertain a doubt that the warrant on which the officer attempted to arrest
one of the defendant at the time of the alleged riot was insufficient, illegal and void. It
did not contain the name of the defendant, nor any description or designation by which
he could be known and identified as the person against whom it was issued. It was in
effect a general warrant, upon which any other individual might as well have been
arrested, as being included in the description, as the defendant himself. Such a warrant
was contrary to elementary principles, and in direct violation of the constitutional right
of the citizen, as set forth in the Declaration of Rights, article 14, which declares that
every subject has a right to be secure from all unreasonable searches and seizures of
his person, and that all warrants, therefore, are contrary to this right, if the order in the
warrant to a civil officer to arrest one or more suspected persons or to seize their
property be not accompanied with a special designation of the persons or objects of
search, arrest or seizure. This is in fact only a declaration of an ancient common law
right. It was always necessary to express the name or give some description of a party
to be arrested on a warrant; and if one was granted with the name in blank, and without
other designation of the person to be arrested, it was void. (1 Hale P. C. 577. 2 Ib. 119.
Foster, 312. 7 Dane Ab. 248. 1 Chit. Crim. Law, 39. Mead vs. Haws, 7 Cow., 332, and
cases cited.)

This rule or principle does not prevent the issue and service of a warrant against a party
whose name is unknown. In such case the best description possible of the person to be
arrested is to be given in the warrant; but it must be sufficient to indicate clearly on
whom it is to be served, by stating his occupation, his personal appearance and
peculiarities, the place of his residence, or other circumstances by which he can be
identified. (1 Chit. Crim. Law, 39, 40.)

The warrant being defective and void on its face, the officer had no right to arrest the
person on whom he attempted to serve it. He acted without warrant and was a
trespasser. The defendant whom he sought to arrest had a right to resist by force, using
no more than was necessary to resist the unlawful acts of the officer . . .

The defendants, therefore, in resisting the officer in making an arrest under the warrant
in question, if they were guilty of no improper or excessive force or violence, did not do
an unlawful act by lawful means, or a lawful act by unlawful means, and so could not
be convicted of the misdemeanor of a riot, with which they are charged in the indictment.

Appellant's argument, as based on these authorities, runs something like this. The law,
constitutional and statutory, requires that the search warrant shall not issue unless the
application "particularly" describe the person to be seized. A failure thus to name the
Page 108 of 377

person is fatal to the validity of the search warrant. To justify search and arrest, the
process must be legal. Illegal official action may be forcibly resisted.

For the prosecution, however, as the arguments are advanced by the Attorney-General,
and as the law was summarized by the trial judge, there is much to be said. Careful and
logical reflection brings forth certain points of paramount force and exercising a decisive
influence. We will now make mention of them by correlating the facts and the law.

In the first place, the affidavit for the search warrant and the search warrant itself
described the building to be searched as "the building No. 124 Calle Arzobispo, City of
Manila, Philippine Islands." This, without doubt, was a sufficient designation of the
premises to be searched. It is the prevailing rule that a description of a place to be
searched is sufficient if the officer with the warrant can, with reasonable effort, ascertain
and identify the place intended. (Steele vs. U. S. [1925], U. S. Supreme Court Advance
Opinions 1924-1925; 69 Law. ed., 757). The police officers were accordingly authorized
to break down the door and enter the premises of the building occupied by the so-called
Parliamentary Club. When inside, they then had the right to arrest the persons
presumably engaged in a prohibited game, and to confiscate the evidence of the
commission of the crime. It has been held that an officer making an arrest may take
from the person arrested any money or property found upon his person, which was used
in the commission of the crime or was the fruit of the crime, or which may furnish the
person arrested with the means of committing violence or of escaping, or which may be
used as evidence on the trial of the cause, but not otherwise. (Moreno vs. Ago Chi [1909],
12 Phil., 439.)

Proceeding along a different line of approach, it is undeniable that the application for
the search warrant, the affidavit, and the search warrant failed to name Jose Ma. Veloso
as the person to be seized. But the affidavit and the search warrant did state that "John
Doe has illegally in his possession in the building occupied by him, and which is under
his control, namely, in the building numbered 124 Calle Arzobispo, City of Manila,
Philippine Islands, certain devices and effects used in violation of the Gambling Law."
Now, in this connection, it must not be forgotten that the Organic Act requires a
particular description of the place to be searched, and the person or things to be seized,
and that the warrant in this case sufficiently described the place and the gambling
apparatus, and, in addition, contained a description of the person to be seized. Under
the authorities cited by the appellant, it is invariably recognized that the warrant for the
apprehension of an unnamed party is void, "except in those cases where it contains a
description personae such as will enable the officer to identify the accused." The
description must be sufficient to indicate clearly the proper person upon whom the
warrant is to be served. As the search warrant stated that John Doe had gambling
apparatus in his possession in the building occupied by him at No. 124 Calle Arzobispo,
City of Manila, and as this John Doe was Jose Ma. Veloso, the manager of the club, the
police could identify John Doe as Jose Ma. Veloso without difficulty.
Page 109 of 377

Again, it must be remembered that No. 124 Calle Arzobispo was supposed to be used
for club purposes. It was not the home of Veloso; not the place of abode of the family,
which the law carefully protects in all of its sanctity. It was a club partially public in
nature. It was, moreover, a camouflaged club with a high sounding name calculated to
mislead the police, but intended for nefarious practices. In a club of such a character,
unlike in the home, there would commonly be varying occupancy, a number of John
Does and Richard Roes whose names would be unknown to the police.

It is also borne out by the authorities that, in defense of himself, any member of his
family or his dwelling, a man has a right to employ all necessary violence. But even in
the home, and much less so in a club or public place, the person sought to be arrested
or to be searched should use no more force than is necessary to repel the unlawful act
of the officers. To authorize resistance to the agents of the authority, the illegality of the
invasion must be clearly manifest. Here, there was possibly a proper case for protest.
There was no case for excessive violence to enforce the defendant's idea of a debatable
legal question. (Commonwealth vs. Crotty, supra; People vs. Chan Fook [1921], 42 Phil.,
230; 3 Groizard, Codigo Penal, pp. 456, 457.)

The trial judge deduced from the searched warrant that the accused Veloso was
sufficiently identified therein. Mention was made by his Honor of the code provision
relating to a complaint or information, permitting a fictitious name to be inserted in the
complaint or information, in lieu of the true name. The Attorney-General adds to this
the argument that the police were authorized to arrest without a warrant since a crime
was being committed. We find it unnecessary to comment on this contention.

John Doe search warrants should be the exception and not the rule. The police should
particularly describe the place to be searched and the person or things to be seized,
wherever and whenever it is feasible. The police should not be hindered in the
performance of their duties, which are difficult enough of performance under the best
of conditions, by superficial adherence to technicality or far fetched judicial interference.

We agree with the trial judge and with the Attorney-General in their conclusions to the
effect that the search warrant was valid, and that the defendant has been proved guilty
beyond a reasonable doubt, of the crime of resistance of the agents of the authority.

The information alleges that at the time of the commission of the crime, the accused
was a member of the House of Representatives. The trial court was led to consider this
allegation in relation with the facts as an aggravating circumstance, and to sentence the
accused accordingly. We doubt, however, that advantage was taken by the offender of
Page 110 of 377

his public position when he resisted the officers of the law. The offender did not
necessarily make use of the prestige of his office as a means to commit a crime.
Undoubtedly, Jose Ma. Veloso, as Juan de la Cruz, would have resisted the police just
as stoutly, as the Honorable Jose Ma. Veloso did. The penalty, accordingly, falls within
the medium of that provided by the Penal Code.

Finding present no reversible error, agreeing in all respects with the findings of facts as
made by the trial judge, and concurring with the trial judge in his legal conclusion, with
one exception, it results that the judgment appealed from must be, as it is hereby,
affirmed, with the sole modification that the defendant and appellant shall be sentenced
to two months and one day imprisonment, arresto mayor, with the costs of this instance
against him. Let the corresponding order to carry this judgment into effect issue.

Avanceña, C.J., Street, Villamor, Ostrand, Johns, and Romualdez, JJ., concur.
Villa-Real, JJ., took no part.
Page 111 of 377

G.R. No. L-71782 April 14, 1988

HADJI IBRAHIM SOLAY PANGANDAMAN, MAGAMBAAN PANGANDAMAN,


MACARIAN PANGANDAMAN, MAMINTAL PANGANDAMAN, PACALUNDO
PANGANDAMAN, MANGORAMAS PANGANDAMAN, MACADAOB P.
PANGORANGAN, KILATUN PANGANDAMAN, MARIO PANGANDAMAN,
MACABIDAR PANGANDAMAN, PUYAT P. ROMAMPAT, SANTORANI P.
DIMAPENGEN, NASSER P. DIMAPENGEN and DIAMA OPAO, Petitioners, v.
DIMAPORO T. CASAR, AS MUNICIPAL CIRCUIT TRIAL JUDGE OF
POONABAYABAO, TAMPARAN AND MASIU, LANAO DEL SUR and THE
PEOPLE OF THE PHILIPPINES, Respondents.

SYLLABUS

1. REMEDIAL LAW; CRIMINAL PROCEDURE; PRELIMINARY INVESTIGATION; STAGES


TO BE OBSERVED BY A JUDGE OF AN INFERIOR COURT AUTHORIZED TO CONDUCT
PRELIMINARY INVESTIGATION OVER CRIMES COGNIZABLE BY REGIONAL TRIAL
COURT. — A preliminary investigation of any crime cognizable by the Regional Trial
Courts, a judge of an inferior court (other than in Metro-Manila or the chartered cities,
where no authority to conduct preliminary investigation is vested in such officials) must
observe the procedure prescribed in Section 3 of Rule 112, 1985 Rules on Criminal
Procedure. And although not specifically so declared, the procedure mandated by the
Rule actually consists of two phases or stages. The first phase consists of an ex-parte
inquiry into the sufficiency of the complaint and the affidavits and other documents
offered in support thereof. The second phase which gives the respondent opportunity to
present evidence concludes with the Judge rendering his resolution, either for dismissal
of the complaint or holding the respondent for trial, to the provincial fiscal for
appropriate action. (Rule 112, 1985 Criminal Procedure)

2. ID.; ID.; WARRANT OF ARREST; COMPLETION OF ENTIRE PROCEDURE OF


PRELIMINARY INVESTIGATION, NOT INDISPENSABLE TO ISSUANCE THEREOF. —
While Rule 20 provides is that no complaint or information for an offense cognizable by
the Regional Trial Court may be filed without completing the procedure, nowhere is it
provided that the entire procedure must be completed before a warrant of arrest may
issue. The rule is and has always been that such issuance need only await a finding of
probable cause as provided by Sec. 6 of Rule 112, not the completion of the entire
procedure of preliminary investigation.

3. ID.; ID.; ID.; MEANING OF "SEARCHING QUESTIONS AND ANSWERS." — In Luna v.


Plaza, this Court ruled that the term "searching questions and answers" means —." . .
only, taking into consideration the purpose of the preliminary examination which is to
Page 112 of 377

determine "whether there is a reasonable ground to believe that an offense has been
committed and the accused is probably guilty thereof so that a warrant of arrest may
be issued and the accused held for trial," such questions as have tendency to show the
commission of a crime and the perpetuator thereof. What would be searching questions
would depend on what is sought to be inquired into, such as: the nature of the offense,
the date, time, and place of its commission, the possible motives for its commission; the
subject, his age, education, status, financial and social circumstances, his attitude
toward the investigation, social attitudes, opportunities to commit the offense; the
victim, his age, status, family responsibilities, financial and social circumstances,
characteristics, etc. The points that are the subject of inquiry may differ from case to
case. The questions, therefore must to a great degree depend upon the Judge making
the investigation . . ."

4. ID.; ID.; ID.; WARRANT ISSUE AGAINST FIFTY JOHN DOES, VOID FOR BEING
GENERAL IN NATURE. — A warrant is issued against fifty (50) "John Does" not one of
whom the witnesses to the complaint could or would identify, it is of the nature of a
general warrant, one of a class of writs long proscribed as unconstitutional and once
anathematized as "totally subversive of the liberty of the subject." Clearly violative of the
constitutional injunction that warrants of arrest should particularly describe the person
or persons to be seized, the warrant must, as regards its unidentified subjects, be
voided.

5. ID.; ID.; PRELIMINARY INVESTIGATION; WHERE THE PROVINCIAL FISCAL


ANNOUNCED HIS INTENTION TO INVESTIGATE THE INCIDENT, INVESTIGATING
JUDGE SHOULD ENDORSE THE SAME TO THE FORMER. — In a case where the Fiscal
announced his intention to conduct his own inquiry, the judge although he is not legally
inhibited should as a courtesy endorse to the former the investigation of the case filed
with him. The action and final resolution of the respondent Judge after completing the
second stage of the preliminary investigation are subject to review by the Provincial
Fiscal. Practical considerations of expediency and the avoidance of the duplication of
work dictate that the latter official be permitted to take over the investigation even before
the Municipal Judge completes his own inquiry.

NARVASA, J.:
The petitioners ask this Court:

1) to annul the warrant for their arrest issued by respondent Judge Dimaporo T. Casar
of the Municipal Circuit Court of Masiu, Lanao del Sur, in Criminal Case No. 1748
entitled "People v. Hadji Ibrahim Solay Pangandaman, Et. Al.;"
Page 113 of 377

"2) to prohibit the Judge from taking further cognizance of said Criminal Case No. 1748;
and

3) to compel the Judge to forward the entire record of Criminal Case No. 1748 to the
Provincial Fiscal of Lanao del Sur for proper disposition. 1

Their plea is essentially grounded on the claim that the warrant for their arrest was
issued by the respondent Judge without a proper preliminary investigation. 2 The
Solicitor General agrees and recommends that their petition be granted and the warrant
of arrest voided. 3

On July 27, 1985, a shooting incident occurred in Pantao, Masiu, Lanao del Sur, which
left at least five persons dead and two others wounded. What in fact transpired is still
unclear. According to one version, armed men had attacked a residence in Pantao,
Masiu, with both attackers and defenders suffering casualties. 4 Another version has it
that a group that was on its way to another place, Lalabuan, also in Masiu, had been
ambushed. 5

On the following day, Atty. Mangurun Batuampar, claiming to represent the widow of
one of the victims, filed a letter-complaint with the Provincial Fiscal at Marawi City,
asking for a "full blast preliminary investigation" of the incident. 6 The letter adverted
to the possibility of innocent persons being implicated by the parties involved on both
sides — none of whom was, however, identified — and promised that supporting
affidavits would shortly be filed. Immediately the Provincial Fiscal addressed a "1st
indorsement" to the respondent Judge, transmitting Atty. Batuampar’s letter and
requesting that "all cases that may be filed relative . . . (to the incident) that happened
in the afternoon of July 27, 1985," be forwarded to his office, which "has first taken
cognizance of said cases." 7

No case relative to the incident was, however, presented to the respondent Judge until
Saturday, August 10, 1985, when a criminal complaint for multiple murder was filed
before him by P.C. Sgt. Jose L. Laru-an, which was docketed as Case No. 1748. 8 On
that same day, the respondent Judge "examined personally all (three) witnesses
(brought by the sergeant) under oath thru . . . (his) closed and direct supervision,"
reducing to writing the questions to the witnesses and the latter’s answers. 9 Thereafter
the Judge "approved the complaint and issued the corresponding warrant of arrest"
against the fourteen (14) petitioners (who were named by the witnesses) and fifty (50)
"John Does." 10
Page 114 of 377

An "ex-parte" motion for reconsideration was filed on August 14, 1985 by Atty.
Batuampar (joined by Atty. Pama L. Muti), seeking recall of the warrant of arrest and
subsequent holding of a "thorough investigation" on the ground that the Judge’s initial
investigation had been "hasty and manifestly haphazard" with "no searching questions"
having been propounded. 11 The respondent Judge denied the motion for "lack of basis;"
12 hence the present petition.chanrobles law library

While they concede the authority of the respondent Judge to conduct a preliminary
investigation of the offenses involved, which are cognizable by Regional Trial Courts, the
petitioners and the Solicitor General argue that the Judge in the case at bar failed to
conduct the investigation in accordance with the procedure prescribed in Section 3,
Rule 112 of the Rules of Court; 13 and that failure constituted a denial to petitioners of
due process which nullified the proceedings leading to the issuance of the warrant for
the petitioners’ arrest. 14 It is further contended that August 10, 1985 was a Saturday
during which "Municipal Trial Courts are open from 8:00 a.m. to 1:00 p.m. only, . . ."
and." . . it would hardly have been possible for respondent Judge to determine the
existence of probable cause against sixty-four (64) persons whose participations were of
varying nature and degree in a matter of hours and issue the warrant of arrest in the
same day; 15 and that there was undue haste and an omission to ask searching
questions by the Judge who relied "mainly on the supporting affidavits which were
obviously prepared already when presented to him by an enlisted PC personnel as
investigator." 16

The petitioners further assert that the respondent Judge conducted the preliminary
investigation of the charges." . . in total disregard of the Provincial Fiscal . . ." who, as
said respondent well knew, had already taken cognizance of the matter twelve (12) days
earlier and was poised to conduct his own investigation of the same; 17 and that
issuance of a warrant of arrest against fifty (50) "John Does" transgressed the
Constitutional provision requiring that such warrants should particularly describe the
persons or things to be seized. 18

There can be no debate about the proposition that in conducting a preliminary


investigation of any crime cognizable by the Regional Trial Courts, a judge of an inferior
court (other than in Metro-Manila or the chartered cities, where no authority to conduct
preliminary investigation is vested in such officials) must observe the procedure
prescribed in Section 3 of Rule 112, 1985 Rules on Criminal Procedure. And although
not specifically so declared, the procedure mandated by the Rule actually consists of
two phases or stages.

The first phase consists of an ex-parte inquiry into the sufficiency of the complaint and
the affidavits and other documents offered in support thereof. And it ends with the
Page 115 of 377

determination by the Judge either: (1) that there is no ground to continue with the
inquiry, in which case he dismisses the complaint and transmits the order of dismissal,
together with the records of the case, to the provincial fiscal; or (2) that the complaint
and the supporting documents show sufficient cause to continue with the inquiry and
this ushers in the second phase.

This second phase is designed to give the respondent notice of the complaint, access to
the complainant’s evidence and an opportunity to submit counter-affidavits and
supporting documents. At this stage also, the Judge may conduct a hearing and
propound to the parties and their witnesses questions on matters that, in his view, need
to be clarified. The second phase concludes with the Judge rendering his resolution,
either for dismissal of the complaint or holding the respondent for trial, which shall be
transmitted, together with the record, to the provincial fiscal for appropriate action.

The procedure above described must be followed before the complaint or information is
filed in the Regional Trial Court. Failure to do so will result in a denial of due process.
19

Here, no information is filed in the Regional Trial Court. There is no pretense that the
preliminary investigation has been completed, insofar as the respondent Judge is
concerned, and that he does not intend to undertake the second phase. In this situation,
it cannot be said that he has failed to observe the prescribed procedure. What has
happened is simply that after receiving the complaint and examining the complainant’s
witnesses, and having come to believe, on the basis thereof, that the offenses charged
had been committed, the respondent Judge issued the warrant now complained of
against the fourteen (14) respondents (now petitioners) named and identified by the
witnesses as the perpetrators of the killings and injuries, as well as against 50 "John
Does."

The real question, therefore, is whether or not the respondent Judge had the power to
issue the warrant of arrest without completing the entire prescribed procedure for
preliminary investigation. Stated otherwise, is completion of the procedure laid down in
Section 3 of Rule 112 a condition sine qua non for the issuance of a warrant of arrest?

There is no requirement that the entire procedure for preliminary investigation must be
completed before a warrant of arrest may be issued. What the Rule 20 provides is that
no complaint or information for an offense cognizable by the Regional Trial Court may
be filed without completing the procedure. But nowhere is it provided that the procedure
must be completed before a warrant of arrest may issue. Indeed, it is the contrary that
is true. The present Section 6 of the same Rule 112 clearly authorizes the municipal
Page 116 of 377

trial court to order the respondent’s arrest even before opening the second phase of the
investigation if said court is satisfied that a probable cause exists and there is a
necessity to place the respondent under immediate custody in order not to frustrate the
ends of justice.

"SECTION. 6. When warrant of arrest may issue. —

x x x

(b) By the Municipal Trial Court. — If the municipal trial judge conducting the
preliminary investigation is satisfied after an examination in writing and under oath of
the complainant and his witnesses in the form of searching questions and answers, that
a probable came exists and that there is a necessity of placing the respondent under
immediate custody in order not to frustrate the ends of justice, he shall issue a warrant
of arrest." 21

This was equally true under the former rules, where the first phase of the investigation
was expressly denominated "preliminary examination" to distinguish it from the second
phase, or preliminary investigation proper. Thus, the former Section 6 of Rule 112
provided:

"SECTION 6. Warrant of arrest, when issued. — If the judge be satisfied from the
preliminary examination conducted by him or by the investigating officer that the
offense complained of has been committed and that there is reasonable ground to believe
that the accused has committed it, he must issue a warrant or order for his arrest."

In Mayuga v. Maravilla, 22 this Court found occasion to dwell in some detail on the
process of preliminary investigation and, incidentally, to affirm the power of a justice of
the peace or municipal judge conducting a preliminary investigation to order the arrest
of the accused after the first stage (preliminary examination), saying:

"Appellant should bear in mind that a preliminary investigation such as was conducted
by the Justice of the Peace has for its purpose only the determination of whether a crime
has been committed and whether there is probable cause to believe the accused guilty
thereof, and if so, the issuance of a warrant of arrest. And it should not be forgotten
that a preliminary investigation has two stages: First, a preliminary examination of the
Page 117 of 377

complainant and his witnesses prior to the arrest of the accused; and, second, the
reading to the accused after his arrest of the complaint or information filed against him,
and his being informed of the substance of the evidence against him, after which he is
allowed to present evidence in his favor, if he so desires. Probable cause, in regard to
the first stage of preliminary investigation, depends on the discretion of the judge or
magistrate empowered to issue the warrant of arrest. It suffices that facts are presented
to him to convince him, not that a person has committed the crime, but that there is
probable cause to believe that such person committed the crime charged. The
proceeding is generally ex parte unless the defendant desires to be present and while
under the old Rules the Justice of the Peace or investigating officer must take the
testimony of the complainant and the latter’s witnesses under oath, only the testimony
of the complainant shall be in writing and only an abstract of the testimony of the other
is required. Regarding preliminary investigation, it has thus been ruled that ‘the
occasion is not for the full and exhaustive display of the parties’ evidence; it is for the
presentation of such evidence only as may engender well-grounded belief that an offense
has been committed and that the accused is probably guilty thereof.’ . . ." 23

The rule on arrest after preliminary examination has, of course, been modified
somewhat since the occurrence of the facts upon which Mayuga was decided, but not
to abrogate the authority of the investigating judge to order such arrest, and only to
prescribe the requirement that before he may do so, he must examine the witnesses to
the complaint, the examination to be under oath and reduced to writing in the form of
searching questions and answers. This modification was introduced by Republic Act
3838, approved June 22, 1963, amending Section 87 of the Judiciary Act of 1948, and
the "searching questions and answers" requirement is incorporated in the present
Section 6 of Rule 112 already quoted.chanrobles.com : virtual law library

The argument, therefore, must be rejected that the respondent Judge acted with grave
abuse of discretion in issuing the warrant of arrest against petitioners without first
completing the preliminary investigation in accordance with the prescribed procedure.
The rule is and has always been that such issuance need only await a finding of probable
cause, not the completion of the entire procedure of preliminary investigation.

Also without appreciable merit is petitioners’ other argument that there was scarcely
time to determine probable cause against sixty-four persons (the fourteen petitioners
and fifty "Does") within a matter of hours on a Saturday when municipal trial courts are
open only from 8:00 a.m. to 1:00 p.m. That argument founders upon the respondent
Judge’s positive affirmations that he had personally and closely examined under oath
the three witnesses to the complaint 24 and that he had issued the warrant of arrest
"believing that the offense thus filed had been committed." 25 Nothing in the record
before this Court belies or discredits those affirmations which have, besides, the benefit
of the legal presumption that official duty has been regularly performed. 26 The
contention that the witnesses to the complaint had merely sworn before the respondent
Page 118 of 377

Judge to statements prepared beforehand and submitted by a military investigator 27


must, in view of the foregoing considerations and for lack of any support in the record,
be dismissed as mere speculation.

The same argument also unwarrantedly assumes that the respondent Judge limited the
proceedings on preliminary examination to the usual Saturday office hours of 8:00 a.m.
to 1:00 p.m., in addition to not making any persuasive showing that such proceedings
could not have been completed within that time-frame. For all that appears, said
respondent could have put off the 1:00 p.m. adjournment until he had finished
interrogating the witnesses to his satisfaction. And there is really nothing unusual in
completing within a three-hour period the questioning of three witnesses in a
preliminary examination to determine the existence of probable cause.

The record which, lacking proof to the contrary, must be accepted as an accurate
chronicle of the questioned proceedings, shows prima facie that the respondent Judge
had personally examined the witnesses to the complaint, and a consideration of the
latter’s sworn answers to his questions satisfies this Court that the finding of probable
cause against the petitioners was neither arbitrary nor unfounded.

The three witnesses to the complaint, Misandoning Monasprang, a student, Lawandato


Ripors, an engineering graduate, and Sanny Monib, a farmer, gave mutually
corroborative accounts of the incident. Under separate questioning, they declared that
they were members of a party that was passing by Pantao on its way to Lalabuan from
Talaguian, all in Masiu, Lanao del Sur, at about 10:00 a.m. on July 27, 1985, when
they were ambushed and fired upon by an armed group which included the petitioners
and about fifty other unidentified persons; that five of the party had been killed and two
(the witnesses Lawandato Ripors and Sanny Monib) wounded; that even after they had
killed their victims, the ambushers had continued to fire at the dead bodies; that the
witnesses managed to escape their attackers and return to Talaguian, where they
informed their relatives about what had happened, and thence went to the municipal
hall in Masiu to report to the authorities; that the dead victims were recovered only late
in the afternoon of that day because the authorities could not "penetrate" the area and
the ambushers refused to release the bodies; and that the ambush was an offshoot of a
grudge between the families of the ambushers and those of the victims. 28

The witnesses named and identified the dead victims as Cadar Monasprang, Macacrao
Guiling, Macrang Hadji Alawi, Alicman Ripors and Malabato Diator. All of them also
identified by name each of the fourteen petitioners as members of the ambush group.
The respondent Judge can hardly be faulted for finding enough cause to hold the
petitioners named in the statements of three eyewitnesses to killings perpetrated in
broad daylight.
Page 119 of 377

In Luna v. Plaza, 29 this Court ruled that the term "searching questions and answers"
means —

". . . only, taking into consideration the purpose of the preliminary examination which
is to determine "whether there is a reasonable ground to believe that an offense has
been committed and the accused is probably guilty thereof so that a warrant of arrest
may be issued and the accused held for trial," such questions as have tendency to show
the commission of a crime and the perpetuator thereof. What would be searching
questions would depend on what is sought to be inquired into, such as: the nature of
the offense, the date, time, and place of its commission, the possible motives for its
commission; the subject, his age, education, status, financial and social circumstances,
his attitude toward the investigation, social attitudes, opportunities to commit the
offense; the victim, his age, status, family responsibilities, financial and social
circumstances, characteristics, etc. The points that are the subject of inquiry may differ
from case to case. The questions, therefore must to a great degree depend upon the
Judge making the investigation . . ."

Upon this authority, and considering what has already been stated above, this Court is
not prepared to question the propriety of the respondent Judge’s finding of probable
cause or substitute its judgment for his in the matter of what questions to put to the
witnesses during the preliminary examination.

Upon the facts and the law, therefore, the warrant of arrest in question validly issued
against the petitioners, such issuance having been ordered after proceedings, to which
no irregularity has been shown to attach, in which the respondent Judge found
sufficient cause to commit the petitioners to answer for the crime complained of.

Insofar, however, as said warrant is issued against fifty (50) "John Does" not one of
whom the witnesses to the complaint could or would identify, it is of the nature of a
general warrant, one of a class of writs long proscribed as unconstitutional and once
anathematized as "totally subversive of the liberty of the subject." 30 Clearly violative of
the constitutional injunction that warrants of arrest should particularly describe the
person or persons to be seized, 31 the warrant must, as regards its unidentified
subjects, be voided.

The fact that the Provincial Fiscal may have announced his intention of investigating
the incident himself did not, in the view of the Court, legally inhibit the respondent
Judge from conducting his own inquiry into the matter if, as is made to appear here, it
was regularly brought before him and no formal complaint was filed before the Fiscal.
Page 120 of 377

Courtesy may have dictated that in those circumstances he leave the investigation to
the Fiscal and simply endorse to the latter the complaint filed with him; duty did not,
and if he nonetheless chose to conduct his own investigation, nothing in the rules states
or implies that he could not do so.

Be that as it may, since the action and final resolution of the respondent Judge after
completing the second stage of the preliminary investigation are subject to review by the
Provincial Fiscal, practical considerations of expediency and the avoidance of
duplication of work dictate that the latter official be permitted to take over the
investigation even in its present stage.

WHEREFORE, the warrant complained of is upheld and declared valid insofar as it


orders the arrest of the petitioners. Said warrant is voided to the extent that it is issued
against fifty (50) "John Does." The respondent Judge is directed to forward to the
Provincial Fiscal of Lanao del Sur the record of the preliminary investigation of the
complaint in Criminal Case No. 1728 of his court for further appropriate action. Without
pronouncement as to costs.

SO ORDERED.
Page 121 of 377

G.R. No. 78631 June 29, 1993

COLUMBIA PICTURES, INC., ORION PICTURES CORP., PARAMOUNT


PICTURES CORP., TWENTIETH CENTURY FOX FILM CORP., UNITED
ARTISTS CORP., UNIVERSAL CITY STUDIOS, INC., THE WALT DISNEY
COMPANY, and WARNER BROS., INC., Petitioners, v. HON. JUDGE
ALFREDO C. FLORES, FGT VIDEO NETWORK, INC., MANUEL MENDOZA,
ALFREDO C. ONGYANCO, ERIC APOLONIO, SUSAN YANG and EDUARDO A.
YOTOKO, Respondents.

Siguion Reyna, Montecillo & Ongsiako Law Office, for Petitioners.


Santos & Associates and San Jose, Enrique, Lucas, Santos & Borje Law Offices for
Respondents.

SYLLABUS

1. CONSTITUTIONAL LAW; BILL OF RIGHTS; RIGHT AGAINST UNREASONABLE


SEARCHES AND SEIZURES; ISSUANCE OF SEARCH WARRANT; REQUISITES. — In
issuing a search warrant, the judge must strictly comply with the constitutional and
statutory requirements. He must determine the existence of probable cause by
personally examining the applicant and his witnesses in the form of searching questions
(Silva v. Presiding Judge, RTC of Negros Oriental, Br. XXXIII (203 SCRA 140 [1991]).
The search warrant must contain a specific description of the place to be searched and
the articles sought to be seized must be described with particularity (Pendon v. Court of
Appeals, 191 SCRA 429 [1990]).

2. ID.; ID.; ID.; SEARCH WARRANT IN NATURE OF GENERAL ONE


CONSTITUTIONALLY OBJECTIONABLE; CASE AT BAR. — In 20th Century Fox Film
Corp. v. Court of Appeals (164 SCRA 655 [1988]), wherein therein petitioner is also one
of the petitioners herein, we upheld the legality of the order of the lower court lifting the
search warrant issued under circumstances similar to those obtaining in the case at
bar. A striking similarity between the case at bar and 20th Century Fox is the fact that
Search Warrant No. 45, specifically paragraph (c) thereof describing the articles to be
seized, contains an almost identical description as the warrant issued in the 20th
Century Fox case, to wit: (c) Television sets, Video Cassettes Recorders, rewinders, tape
head cleaners, accessories, equipments and other machines used or intended to be used
in the unlawful reproduction, sale, rental/lease, distribution of the above-mentioned
video tapes which she is keeping and concealing in the premises above-described. On
the propriety of the seizure of the articles above-described, we held in said case:
Television sets, video cassette recorders, rewinders and tape cleaners are articles which
can be found in a video tape store engaged in the legitimate business of lending or
Page 122 of 377

renting out betamax tapes. In short, these articles and appliances are generally
connected with, or related to a legitimate business not necessarily involving piracy of
intellectual property or infringement of copyright laws, Hence, including these articles
without specification and/or particularity that they were really instruments in violating
an Anti-Piracy law makes the search warrant too general which could result in the
confiscation of all items found in any video store. The language used in paragraph (c) of
Search Warrant No. 45 is thus too all-embracing as to include all the paraphernalia of
FGT in the operation of its business. As the search warrant is in the nature of a general
one, it is constitutionally objectionable (Corro v. Lising, 137 SCRA 541 [1985]).

3. ID.; ID.; ZEAL IN PURSUIT OF CRIMINALS CANNOT ENNOBLE USE OF ARBITRARY


METHODS. — Much has been said in the media about piracy of films and videotapes
and that violators of the law must be brought to the courts but, as the Court said in
Bagalihog v. Fernandez(198 SCRA 614 [1991])," [z]eal in the pursuit of criminals cannot
ennoble the use of arbitrary methods that the Constitution itself abhors."

MELO, J.:

Before us is a petition for certiorari seeking to set aside the order dated May 29, 1987
of the Regional Trial Court of the National Capital Region (Branch 167, Pasig) directing
the immediate release and return of television sets, video cassette recorders, rewinders,
tape head cleaners, accessories, equipment, and other paraphernalia or pieces of
machinery which had been seized by operatives of the National Bureau of Investigation
by virtue of a search warrant.

Petitioners herein are all foreign corporations organized and existing under the laws of
the United States of America and represented in the Philippines by their attorney-in-
fact, Rebecca Benitez-Cruz of the Motion Picture Association of America, Inc. (MPAA for
brevity). Private respondent FGT Video Network, Inc. is a merger of Fox, Galactic, and
Technica Video. It is registered with and licensed by the Videogram Regulatory Board
as a distributor under License No. 1333 VMM. Technica Video, Inc. which is part of the
merger, is registered with and licensed as a reproducer by the said board under License
No. 967 VMM (p. 11, Rollo).

In a letter dated April 20, 1987, the MPAA, through counsel Rico V. Domingo, lodged a
complaint before then Director Antonio Carpio of the National Bureau of Investigation
(NBI) against certain video establishments for violation of Presidential Decree No. 49
(Protection of Intellectual Property), as amended by Presidential Decree No. 1988, in
connection with its anti-piracy campaign. Specifically complaining of the "unauthorized
sale, rental, reproduction and/or disposition of copyrighted film", the MPAA sought the
Page 123 of 377

NBI’s "urgent assistance in the conduct of search and seizure operations in Metro Manila
and elsewhere." (p. 29, Rollo.)

On the basis of said letter, NBI and private agents conducted discreet surveillance
operations on certain video establishments, among them private respondent FGT Video
Network, Inc. (FGT). Thus, on April 20, 1987, Danilo Manalang, a.k.a. Ronaldo Lim,
allegedly an NBI agent, went to the office of FGT to have the copyrighted motion pictures
"Cleopatra" owned by Twentieth Century Fox Film Corp. and "The Ten Commandments"
owned by Paramount Pictures, Inc. reproduced or retaped in video format. For the
reproduction services, FGT issued Order Slip No. 3482 dated April 20, 1987 and Delivery
Slip No. 118667 dated April 22, 1987, for which services Danilo Manalang paid P45.00.
On May 5, 1987, Manalang also had MGM’s copyrighted film "Walk Like a Man"
reproduced or retaped by FGT for P15.00 (p. 5, Rollo).

Consequently, on May 14, 1987, NBI Agent III Lauro C. Reyes, with Manalang and
Rebecca Benitez-Cruz as witnesses, applied for a search warrant with the Regional Trial
Court in Pasig. Introduced as evidence in support of the application were the following:
the letter dated April 20, 1987 of the MPAA through Rico V. Domingo (Exh. A): FGT’s
Order Slip No. 3842 (Exh. B); FGT’s Delivery Slip No. 118667 (Exh. B-1); video cassettes
containing the film "The Ten Commandments" (Exh. B-1-A, B-1-B); video cassette
containing the film "Cleopatra" (Exh. B-1-C); video cassette containing the film "Walk
Like a Man" (Exh. B-1-D); FGT’s Order Slip No. 3923 dated May 5, 1987 (Exh. B-2);
FGT’s Delivery Slip No. 123321 dated May 6, 1987 (Exh. B-3); list of copyrighted MPAA
member company titles (Exh. C); sketch of location of FGT’s office or premises (Exh. D);
affidavit of Rebecca Benitez-Cruz (Exh. E); special power of attorney designating Ms.
Benitez-Cruz as petitioners’ attorney-in-fact (Exh. F to F-8); and affidavit of Danilo
Manalang (Exh. G).

Upon the offer of these pieces of evidence, Judge Alfredo C. Flores of the aforesaid court,
issued Search Warrant No. 45 which reads:

TO ANY PEACE OFFICER:

GREETINGS:

It appearing to the satisfaction of the Undersigned after examining under oath NBI
Senior Agent Lauro C. Reyes and his witnesses Mr. Danilo Manalang and Ms. Rebecca
Benitez-Cruz, that there is a probable cause to believe that Violation of Section 56 P.D.
No. 49 as amended by P.D. No. 1988 (otherwise known as the Decree on Protection of
Page 124 of 377

Intellectual Property) has been committed and that there are good and sufficient reasons
to believe that FGT Video Network, Inc., Manuel Mendoza, Alfredo C. Ongyanco, Eric
Apolonio, Susan Yang and Eduardo Yotoko are responsible and have in
control/possession at No. 4 Epifanio de los Santos corner Connecticut, Greenhills, San
Juan, Metro Manila (per attached sketch and list of MPAA member Company Titles) the
following properties to wit:

(a) Pirated video tapes of the copyrighted motion pictures/films the titles of which are
mentioned in the attached list;

(b) Posters, advertising leaflets, flyers, brochures, invoices, lists of titles being
reproduced or retaped, journals, ledgers, jon (sic) order slips, delivery slips and books
of accounts bearing and/or mentioning the pirated films with titles (as per attached list),
or otherwise used in the reproduction/repating business of the defendants;

(c) Television sets, video cassette recorders, rewinders, tape head cleaners, accessories,
equipment and other machines and paraphernalia or materials used or intended to be
used in the unlawful sale, lease, distribution, or possession for purpose of sale, lease,
distribution, circulation or public exhibition of the above-mentioned pirated video tapes
which they are keeping and concealing in the premises above-described, which should
be seized and brought to the Undersigned.

You are hereby commanded to make an immediate search at any time in the day
between 8:00 A.M. to 5:00 P.M. of the premises above-described and forthwith seize and
take possession of the above-enumerated personal properties, and bring said properties
to the undersigned immediately upon implementation to be dealt with as the law directs.

WITNESS MY HAND this 14th day of May, 1987, at Pasig, Metro Manila. (pp. 30-31,
Rollo; Emphasis supplied.)

At or about high noon of the same day, agents from the NBI, led by Lauro C. Reyes and
Mamerto Espartero, with the assistance of the personnel of the Videogram Regulatory
Board headed by Elmer San Pascual, duly served Search Warrant No. 45 on the
operators or representatives of FGT. In the course of the search of the premises of FGT,
the NBI agents found and seized various video tapes of duly copyrighted motion pictures
or films owned and exclusively distributed by petitioners. Also seized were machines
and equipment, television sets, paraphernalia, materials, accessories, rewinders, tape
head cleaners, statements of order, return slips, video prints, flyers, production orders,
and posters. Inventories of these seized articles were then prepared and copies thereof
Page 125 of 377

were furnished Jess Ayson, production manager of FGT. On May 18, 1987, the NBI
agents filed a return of the search warrant with a motion to retain custody of the seized
items (p. 32, Rollo).

Meanwhile, FGT filed an urgent motion for the immediate release of equipment and
accessories "not covered" by the search warrant, without prejudice to the filing of a
motion to quash the said search warrant (p. 101, Rollo). It argued that as a licensed
video reproducer, FGT had the right to maintain possession of the seized reproduction
equipment and paraphernalia which are not contraband or illegal per se, but are rather
"exclusively used and intended to be used for reproduction" and not in the "sale, lease,
distribution or possession for purposes of sale, lease distribution, circulation or public
exhibition of pirated video tapes." (p. 102, Rollo.)chanrobles law library

Petitioners opposed the motion, asserting that the seized articles were all lawfully taken.
They explained that since FGT was a videogram distributor and not a reproducer, "it
may be logically concluded that such 634 VCRs, accessories, etc." were "used or
intended to be used in the unlawful sale, lease, distribution or possession for purposes
of sale, lease, distribution, circulation or public exhibition of, at the very least, the 310
videocassette tapes containing the copyrighted films/motion pictures." They asserted
that Search Warrant No. 45 was issued upon the proper determination of probable cause
and that, therefore, it is not for FGT "to second-guess the wisdom" of the court’s directive
to seize the questioned VCRs and accessories "as an inquiry thereon would involve
evidentiary matters which are better ventilated in the criminal prosecution proper." (pp.
107-116, Rollo.)

Finding that FGT was a "registered and duly licensed distributor and in certain
instances and under special instructions and conditions . reproducer of videograms"
and that, therefore, its right to possess and use the seized equipment had been "placed
in serious doubt", the lower court resolved the doubt "against the Government and in
favor of a lawful business enterprise." Applying the constitutional precept of
presumption of innocence and considering that the seized articles are not contraband,
respondent court ruled that to allow the Government "to keep possession of the
equipment(s) and machines where there is no actual criminal charge" would amount to
a "confiscation in violation of the due process clause of the constitution,
notwithstanding the filing by the Director of the NBI of a letter to the Department of
Justice recommending that the defendants be charged with violation of Section 56 of
P.D. No. 49, as amended by P.D. No. 1988." (pp. 131-132, Rollo.)

Thus, in its order on May 29, 1987, the lower court granted FGT’s motion and ordered
the immediate release and return of the "television sets, video cassette recorders,
rewinders, tape head cleaners, accessories, equipment and other machines or
Page 126 of 377

paraphernalias, as reflected in the ‘Receipt for Properties Seized’ attached to the records
of the case beginning from page 84 to page 130, to the defendants, excluding video
cassette tapes reflected in the ‘Receipts for Properties Seized’, beginning from page 132
to page 146 of the records." Respondent court also ordered the inventory of all articles
returned with individual descriptions "to evidence their existence" copies of which
inventory should be furnished the NBI and the court (p. 132, Rollo).

Hence, the present recourse.

As prayed for by petitioners, on June 17, 1987, the Court issued a temporary restraining
order enjoining respondents from implementing the lower court’s order of May 29, 1987
upon a bond in the amount of P750,000.00 which petitioners accordingly posted on
June 19, 1987. (pp. 138-141, Rollo.)chanrobles.com:

The sole issue to be resolved is whether or not the lower court acted with grave abuse
of discretion amounting to lack of jurisdiction in ordering the immediate release and
return of some of the items seized by virtue of the search warrant.

Petitioners insist that the search warrant was issued upon due determination of
probable cause. They argue that FGT’s act of illegally reproducing copyrighted films had
been clearly established by evidence on record and that FGT’s principal ground in
praying for the immediate release of the seized articles is a matter of defense which
should be ventilated at the trial of the case on the merits.

Private respondents, on the other hand, claim that the issuance of Search Warrant No.
45 is tainted with illegality as no particular or specific acts or omissions constituting
the offense charged had been alleged in the application for its issuance.

The right to security against unreasonable searches and seizures is guaranteed under
Section 2, Article III of the 1987 Constitution which provides:

Sec. 2. The right of the people to be secure in their persons, houses, papers and effects
against unreasonable searches and seizures of whatever nature and for any purpose
shall be inviolable, and no search warrant or warrant of arrest shall issue except upon
probable cause to be determined by the judge after examination under oath or
affirmation of the complainant and the witnesses he may produce, and particularly
describing the place to be searched and the persons or things to be seized.
Page 127 of 377

Thus, Sections 3 and 4 of Rule 126 of the Rules of Court provide for the requisites in
the issuance of search warrants:

SEC. 3. Requisites for issuing search warrant. — A search warrant shall not issue but
upon probable cause in connection with one specific offense to be determined personally
by the judge after examination under oath or affirmation of the complainant and the
witnesses he may produce, and particularly describing the place to be searched and the
things to be seized.

SEC. 4. Examination of complainant; record. — The judge must, before issuing the
warrant, personally examine in the form of searching questions and answers, in writing
and under oath the complainant and the witnesses he may produce on facts personally
known to them and attach to the record their sworn statements together with any
affidavits submitted.

In issuing a search warrant, the judge must strictly comply with the constitutional and
statutory requirements. He must determine the existence of probable cause by
personally examining the applicant and his witnesses in the form of searching questions
(Silva v. Presiding Judge, RTC of Negros Oriental, Br. XXXIII (203 SCRA 140 [1991]).
The search warrant must contain a specific description of the place to be searched and
the articles sought to be seized must be described with particularity (Pendon v. Court of
Appeals, 191 SCRA 429 [1990]).

Withal, measured by the foregoing constitutional and legal provisions as well as the
existing jurisprudence on the matter, we find that Search Warrant No. 45 fails to satisfy
the test of legality. More so because the Court has previously decided a case dealing
with virtually the same search warrant.

In 20th Century Fox Film Corp. v. Court of Appeals (164 SCRA 655 [1988]), wherein
therein petitioner is also one of the petitioners herein, we upheld the legality of the order
of the lower court lifting the search warrant issued under circumstances similar to those
obtaining in the case at bar.

A striking similarity between the case at bar and 20th Century Fox is the fact that
Search Warrant No. 45, specifically paragraph (c) thereof describing the articles to be
seized, contains an almost identical description as the warrant issued in the 20th
Century Fox case, to wit:
Page 128 of 377

(c) Television sets, Video Cassettes Recorders, rewinders, tape head cleaners,
accessories, equipments and other machines used or intended to be used in the
unlawful reproduction, sale, rental/lease, distribution of the above-mentioned video
tapes which she is keeping and concealing in the premises above-described. (at p. 664.)

On the propriety of the seizure of the articles above-described, we held in said case:

Television sets, video cassette recorders, rewinders and tape cleaners are articles which
can be found in a video tape store engaged in the legitimate business of lending or
renting out betamax tapes. In short, these articles and appliances are generally
connected with, or related to a legitimate business not necessarily involving piracy of
intellectual property or infringement of copyright laws. Hence, including these articles
without specification and/or particularity that they were really instruments in violating
an Anti-Piracy law makes the search warrant too general which could result in the
confiscation of all items found in any video store. (at p. 665.)

The language used in paragraph (c) of Search Warrant No. 45 is thus too all-embracing
as to include all the paraphernalia of FGT in the operation of its business. As the search
warrant is in the nature of a general one, it is constitutionally objectionable (Corro v.
Lising, 137 SCRA 541 [1985]).

In consequence, respondent court was merely correcting its own erroneous conclusions
in issuing Search Warrant No. 45 when it ordered the return of the seized television sets
and other paraphernalia specified in the motion filed by FGT. This can be gleaned from
its statement that" .. the machines and equipment could have been used or intended to
be used in the illegal reproduction of tapes of the copyrighted motion pictures/films,
yet, it cannot be said with moral certainty that the machines or equipment(s) were used
in violating the law by the mere fact that pirated video tapes of the copyrighted motion
pictures/films were reproduced. As already stated, FGT Video Network, Inc. is a
registered and duly licensed distributor and in certain instances and under special
instructions .. reproducer of videograms, and as such, it has the right to keep in its
possession, maintain and operate reproduction equipment(s) and paraphernalia(s)." (pp.
131-132, Rollo.)

Far from being despotic or arbitrary, respondent judge must be commended for
rectifying his error when he found that his initial conclusions were inaccurate and
erroneous, colliding as they did with the constitutional rights of private Respondent.
Page 129 of 377

Much has been said in the media about piracy of films and videotapes and that violators
of the law must be brought to the courts but, as the Court said in Bagalihog v. Fernandez
(198 SCRA 614 [1991])," [z]eal in the pursuit of criminals cannot ennoble the use of
arbitrary methods that the Constitution itself abhors." (at p. 622.)

WHEREFORE, the petition is DISMISSED, the assailed order of May 29, 1987
AFFIRMED, and the temporary restraining order issued on June 18, 1987, vacated and
lifted.

SO ORDERED.

Feliciano, Bidin, Davide, Jr. and Romero, JJ., concur.


Page 130 of 377

G.R. No. 126379 June 26, 1998

PEOPLE OF THE PHILIPPINES, represented by Provincial Prosecutor


FAUSTINO T. CHIONG, Petitioner, v. COURT OF APPEALS, JUDGE CAESAR
CASANOVA, Presiding Judge, Regional Trial Court, Branch 80, Malolos,
Bulacan, AZFAR HUSSAIN, MOHAMMAD SAGED, MUJAHID KHAN,
MOHAMMAD ASLAM, and MEHMOOD ALI, Respondents.

NARVASA, C.J.:

In behalf of the People, the Solicitor General has perfected the appeal at bar under Rule
45 of the Rules of Court from the Decision promulgated on September 11, 1996 of the
Fourteenth Division of the Court of Appeals.1 Said judgment dismissed the Peoples
petition for certiorari to invalidate (i) the order of Judge Caesar A Casanova of Branch
80 of the Regional Trial Court dated February 9 1996,2 as well as (ii) that dated May 28,
1996 denying the Peoples motion for reconsideration.3 Those orders were handed down
in Criminal Case No. 43-M-96, a case of illegal possession of explosives after the accused
had been arraigned and entered a plea of not guilty to the charge. More particularly, the
Order of February 9, 1996:

1) quashed a search warrant (No. 1068 [95]) issued by Judge Marciano I. Bacalla of
Branch 216 of the Regional Trial Court at Quezon City on December 15, 1995,4

2) declared inadmissible for any purpose the items seized under the warrant, and

3) directed the turnover of the amount of U.S. $5,750.00 to the Court within five (5) days
to be released thereafter in favor of the lawful owner considering that said amount was
not mentioned in the Search Warrant."

The antecedents, culled from the records by the Appellate Court, are hereunder set out.

1. On December 14, 1995, S/Insp PNP James Brillantes applied for search warrant
before Branch 261, RTC of Quezon City against Mr. Azfar Hussain, who had allegedly
in his possession firearms and explosives at Abigail Variety Store, Apt. 1207 Area F,
Bagong Buhay Ave. Sapang Palay, San Jose del Monte Bulacan.
Page 131 of 377

2. The following day, December 15, 1995, Search Warrant No. 1068 (95) against Mr.
Hussain was issued not at Abigail Variety Store but at Apt. No. 1, immediately adjacent
9to0 Abigail Variety Store resulting in the arrest of four (4) Pakistani nationals and in
the seizure of their personal belongings, papers and effects such as wallet, wrist
watches, pair of shoes, jackets, t-shirts, belts, sunglasses and travelling bags including
cash amounting to $3,550.00 and P1,500.00 aside from US $5,175.00 (receipted) which
were never mentioned in the warrant. The sum of $5,175.00 was however returned to
the respondents upon order of the court on respondents motion or request. Included
allegedly are one piece of dynamite stick; two pieces of plastic explosives C-4 type and
one (1) fragmentation grenade. But without the items described in the search warrant
are: (a) three (3) Ingram machine pistols; (b) four (4) gmm pistol; (c) blasting caps; (d)
fuse; (e) assorted chemical ingredients for explosives; and (f) assorted magazine assg
and ammunitions.

3. On December 19, 1995, three days after the warrant was served, a return was made
without mentioning the personal belongings, papers and effects including cash
belonging to the private respondents. There was no showing that lawful occupants were
made to witness the search.

4. On January 22,1996, private respondents upon arraignment, pleaded not guilty to


the offense charged; ** and on the same date, submitted their Extremely Urgent Motion
(To Quash Search Warrant and to Declare Evidence Obtained Inadmissible), dated
January 15, 1996;

5. ** According to the private respondents in their pleading (consolidated comment on


petition for certiorari **): On January 29, 1996, an ocular inspection of the premises
searched was conducted by respondent Judge and the following facts had been
established as contained in the order dated January 30, 1996** to wit:

1) That the residence of all the accused is at Apartment No. 1 which is adjacent to the
Abigails Variety Store;

2) That there is no such number as 1207 found in the building as it is correspondingly


called only Apartment No. 1, 2, 3, and 4;

3) That Apartment No. 1 is separate from the Abigails Variety Store;


Page 132 of 377

4) That there are no connecting doors that can pass from Abigails Variety Store to
Apartment No. 1;

5) That Abigails Variety Store and Apartment No. 1 have its own respective doors used
for ingress and egress.

That there being no objection on the said observation of the Court, let the same be
reduced on the records.

SO ORDERED.

6. On February 9, 1996, respondent Judge ** issued its order duly granting the motion
to quash search warrant **;5

7. On February 12, 1996, private respondents filed the concomitant motion to dismiss
**;

8. On February 19, 1996, Asst. Provincial Prosecutor Rolando Bulan filed a motion for
reconsideration and supplemental motion on the order quashing the search warrant**;

9. On February 27, 1996 and March 12, 1996, private respondent filed
opposition/comment and supplemental opposition/comment on the motion for
reconsideration **:

10. On May 28, 1996, respondent Judge ** issued its order denying the motion for
reconsideration **; (and on) June 11, 1996, private respondents filed extremely urgent
reiterated motion to dismiss**.

Chiefly to nullify Judge Casanovas quashal Order of February 9, 1996 above referred
to, the Solicitor General forthwith commenced a special civil action of certiorari in the
Court of Appeals. The action did not prosper, however. As earlier mentioned, the
Fourteenth Division of the Appellate Tribunal promulgated judgment on September 11,
1996, dismissing the case for lack of merit.

The judgment was grounded on the following propositions, to wit:6


Page 133 of 377

1. The place actually searched was different and distinct from the place described in the
search warrant. This fact was ascertained by the Trial Judge through an ocular
inspection, the findings wherein, not objected to by the People, were embodied in an
order dated January 30, 1996. The place searched, in which the accused (herein
petitioners) were then residing, was Apartment No. 1. It is a place other than and
separate from, and in no way connected with, albeit and adjacent to, Abigails Variety
Store, the place stated in the search warrant.

2. The public prosecutors claim -- that the sketch submitted to Judge Bacalla relative
to the application for a search warrant, actually depicted the particular place to be
searched -- was effectively confuted by Judge Casanova who pointed out that said
SKETCH was not dated, not signed by the person who made it and not even mentioned
in the Search Warrant by the Honorable Judge (Bacalla, who) instead ** directed them
to search Abigail Variety Store Apartment 1207 ** in the Order ** dated December 15,
1995 -- this, too, being the address given in the Application for Search Warrant dated
December 14, 1995 requested by P/SR INSP. Roger James Brillantes, the Team Leader.
The untenability of the claim is made more patent by the Peoples admission, during the
hearing of its petition for certiorari in the Court of Appeals, that said sketch was in truth
not attached to the application for search warrant ** (but) merely attached to the motion
for reconsideration.7

Quoted with approval by the Appellate Court were the following observations of Judge
Casanova contained in his Order of May 28, 1996, viz.:8

(d)** ** it is very clear that the place searched is different from the place mentioned in
the Search Warrant, that is the reason why even P/SR. INSP Roger James Brillantes,
SPO1 Prisco Bella and SPO4 Cesar D. Santiago, who were all EDUCATED, CULTURED
and ADEPT to their tasks of being RAIDERS and who were all STATIONED IN BULACAN
were not even able to OPEN THEIR MOUTH to say in TAGALOG with Honorable Judge
who issued the Search Warrant the words KATABI, or KADIKIT or KASUNOD NG
ABIGAIL VARIETY STORE ang papasukin namin or if they happen to be an ENGLISH
speaking POLICEMEN, they were not able to open their mouth even to WHISPER the
ENGLISH WORDS RESIDE or ADJACENT or BEHIND or NEXT to ABIGAIL VARIETY
STORE, the place they are going to raid.**.

3. The search was not accomplished in the presence of the lawful occupants of the place
(herein private respondents) or any member of the family, said occupants being
handcuffed and immobilized in the living room at the time. The search was thus done
in violation of the law.9
Page 134 of 377

4. The articles seized were not brought to the court within 48 hours as required by the
warrant itself; (i)n fact the return was done after 3 days or 77 hours from service, in
violation of Section 11, Rule 126 of the Rules of Court.10

5. Judge Casanova correctly took cognizance of the motion to quash search warrant,
pursuant to the doctrinal tenets laid down in Nolasco vs. Pao (139 SCRA 152) which
overhauled the previous ruling of the Supreme Court in Templo vs. dela Cruz (60 SCRA
295). It is now the prevailing rule that whenever a search warrant has been issued by
one court or branch thereof and a criminal case is initiated in another court or branch
thereof as a result of the search of the warrant, that search warrant is deemed
consolidated with the criminal case for orderly procedure. The criminal case is more
substantial than the search warrant proceedings, and the presiding Judge in the
criminal case has the right to rule on the search warrant and to exclude evidence
unlawfully obtained (Nolasco & Sans cases).

6. Grave abuseof discretion cannot be imputed to the respondent Judge, in light of


Article III, Section 2 of the Constitution and Rule 126 of the Rules of Court.

7. The proper remedy against the challenged Order is an appeal, not the special civil
aciton of certiorari.

The Solicitor General now seeks reversal of the foregoing verdict ascribing to the Court
of Appeals the following errors, to wit:

1) sanctioning the lower Courts precipitate act of disregarding the proceedings before
the issuing Court and overturning the latters determination of probable cause and
particularity of the place to be searched;

2) sanctioning the lower Courts conclusion that the sketch was not attached to the
application for warrant despite the clear evidence ** to the contrary;

3) ignoring the very issues raised in the petition before it:

4) holding that the validity of an otherwise valid warrant could be diminished by the
tardiness by which the return is made;
Page 135 of 377

5) hastly applying the general rule that certiorari cannot be made a substitute for appeal
although the circumstances attending the case at bar clearly fall within the exceptions
to that rule; and

6) depriving petitioner of the opportunity to present evidence to prove the validity of the
warrant when the petition before it was abruptly resolved without informing petitioner
thereof.

The whole case actually hinges on the question of whether or not a search warrant was
validly issued as regards the apartment in which private respondents were then actually
residing, or more explicitly, whether or not that particular apartment had been
specifically described in the warrant.

The Government insists that the police officers who applied to the Quezon City RTC for
the search warrant had direct, personal knowledge of the place to be searched and the
things to be seized. It claims tha tone of said officers, infact, had been able to
surreptitiously enter the place to be searched prior to the search: this being the first of
four (4) separate apartments behind the Abigail Variety Store; and they were also the
same police officers who eventually effected the search and seizure. They thus had
personal knowledge of the place to be searched and had the competence to make a
sketch thereof; they knew exactly what objects should be taken therefrom; and they had
presented evidence sufficient to establish probable cause. That may be so; but
unfortunately, the place they had in mind -- the first of four (4) separate apartment
units (No. 1) at the rear of Abigail Variety Store -- was not what the Judge who issued
the warrant himself had in mind, and was not what was ultimately described in the
search warrant.

The discrepancy appears to have resulted from the officers own faulty depiction of the
premises to be searched. For in their application and in the affidavit thereto appended,
they wrote down a description of the place to be searched, which is exactly what the
Judge reproduced in the search warrant: premises located at Abigail Variety Store Apt
1207, Area-F, Bagong Buhay Avenue, Sapang Palay, San Jose Del Monte, Bulacan. And
the scope of the search was made more particular -- and more restrictive -- by the
Judges admonition in the warrant that the search be limited only to the premises herein
described.

Now, at the time of the application for a search warrant, there were at least five (5)
distinct places in the area involved: the store known as Abigails Variety Store, and four
(4) separate and independent residential apartment units. These are housed in a single
Page 136 of 377

structure and are contiguous to each other although there are no connecting doors
through which a person could pass from the interior of one to any of the others. Each
of the five (5) places is independent of the others, and may be entered only through its
individual front door. Admittedly, the police officers did not intend a search of all five (5)
places, but only one of the residential units at the rear of Abigails Variety Store: that
immediately next to the store (Number 1).

However, despite having personal and direct knowledge of the physical configuration of
the store and the apartments behind the store, the police officers failed to make Judge
Bacalla understand the need to pinpoint Apartment No. 1 in the warrant. Even after
having received the warrant -- which directs that the search be limited only to the
premises herein described, Abigail Variety Store Apt 1207 -- thus literally excluding the
apartment units at the rear of the store -- they did not ask the Judge to correct said
description. They seem to have simply assumed that their own definite idea of the place
to be searched -- clearly indicated, according to them, in the sketch they claim to have
submitted to Judge Bacalla in support of their application -- was sufficient
particularization of the general identification of the place in the search warrant.

The Solicitor General argues that this assumption is sanctioned by Burgos, Sr. v. Chief
of Staff, AFP,11 allegedly to the effect that the executing officers prior knowledge as to
the place intended in the warrant is relevant, and he may, in case of any ambiguity in
the warrant as to the place to be searched, look to the affidavit in the official court file.

Burgos is inapplicable. That case concerned two (2) search warrants which, upon
perusal, immediately disclosed an obvious typographical error. The application in said
case was for seizure of subversive material allegedly concealed in two places: one at No.
19. Road 3, Project 6, Quezon City; and the other, at "784 Units C & D. RMS Building,
Quezon Avenue, Quezon City;" Two (2) warrants issued -- No. 20-82 [a] and No. 20-82
[b]). Objection was made to the execution of Warrant No. 20-82 (b) at 784 Units C & D,
RMS Building, Quezon Avenue, Quezon City because both search warrants apparently
indicated the same address (No. 19, Road 3, Project 6, Quezon City) as the place where
the supposedly subversive material was hidden. This was error, of course but, as this
Court there ruled, the error was obviously typographical, for it was absurd to suppose
that the Judge had issued two warrants for the search of only one place. Adverting to
the fact that the application for the search warrants specified two (2) distinct addresses,
and that in fact the address, 784 Units C&D, RMS Building, Quezon Avenue, Quezon
City appeared in the opening paragraph of Warrant 20-82 (b), this Court concluded that
evidently, this was the address the judge intended to be searched when he issued the
second warrant (No. 20-82 [b]); and to clear up the ambiguity caused by the obviously
typographical error, the officer executing the warrant could consult the records in the
official court file.12cräläwvirtualibräry
Page 137 of 377

The case at bar, however, does not deal with the correction of an obvious typographical
erro involving ambiguous descriptions of the place to be searched, as in Burgos, but the
search of a place different from that clearly and without ambiguity identified in the
search warrant. In Burgos, the inconsistency calling for clarification was immediately
perceptible on the face of the warrants in question. In the instant case, there is no
ambiguity at all in the warrant. The ambiguity lies outside the instrument, arising from
the absence of a meeting of minds as to the place to be searched between the applicants
for the warrant and the Judge issuing the same; and what was done was to substitute
for the place that the judge had written down in the warrant, the premises that the
executing officers had in their mind. This should not have been done. It is neither fair
nor licit to allow police officers to search a place different from that stated in the warrant
on the claim that the place actually searched -- although not that specified in the
warrant -- is exactly what they had in view when they applied for the warrant and had
demarcated in their supporting evidence. What is material in determining the validity of
a search is the place stated in the warrant itself, not what the applicants had in their
thoughts, or had represented in the proofs they submitted to the court issuing the
warrant. Indeed, following the officers theory, in the context of the facts of this case, all
four (4) apartment units at the rear of Abigails Variety Store would have been fair game
for a search.

The place to be searched, as set out in the warrant, cannot be amplified or modified by
the officers own personal knowledge of the premises, or the evidence they adduced in
support of their application for the warrant. Such a change is proscribed by the
Constitution which requires inter alia the search warrant to particularly describe the
place to be searched as well as the persons or things to be seized. It would concede to
police officers the power of choosing the place to be searched, even if not be that
delineated in the warrant. It would open wide the door to abuse of search process, and
grant to officers executing a search warrant that discretion which the Constitution has
precisely removed from them. The particularization of the description of the place to be
searched may properly be done only by the Judge, and only in the warrant itself; it
cannot be left to the discretion of the police officers conducting the search.

The Government faults Judge Casanova for having undertaken a review of Judge
Bacallas finding of probable cause, as if he were an appellate court. A perusal of the
record however shows that all that Judge Casanova did was merely to point out
inconsistencies between Judge Bacalla' Order of December 15, 1995 and the warrant
itself, as regards the identities of the police officers examined by Judge Bacalla.13 In
Judge Casanovas view, said inconsistencies, being quite apparent in the record, put in
doubt the sufficiency of the determination of the facts on which the search warrant was
founded.

The Government alleges that the officers had satisfactorily established probable cause
before Judge Bacalla for the issuance of a search warrant. While this may be conceded,
Page 138 of 377

the trouble is, to repeat, that the place described in the search warrant -- which, of
course, is the only place that may be legitimately searched in virtue thereof -- was not
that which the police officers who applied for the warrant had in mind, with the result
that what they actually subjected to search-and-seizure operations was a place other
than that stated in the warrant. In fine, while there was a search warrant more or less
properly issued as regards Abigails Variety Store, there was none for Apartment No. 1 -
- the first of the four (4) apartment units at the rear of said store, and precisely the place
in which the private respondents were then residing.

It bears stressing that under Section 2, Article III of the Constitution, providing that:14

The right of the people to be secure in their persons, houses, papers, and effects against
unreasonable searches and seizures of whatever nature and for any purpose shall be
inviolable, and no search warrant or warrant of arrest shall issue except upon probable
cause to be determined personally by the judge after examination under oath or
affirmation of the complainant and the witnesses he may produce, and particularly
describing the place to be serched, and the things to be seized.,

it does not suffice, for a search warrant to be deemed valid, that it be based on probable
cause, personally determined by the judge after examination under oath, or affirmation
of the complainant and the witnesses he may produce; it is essential, too, that it
particularly describe the place to be searched,15 the manifest intention being that the
search be confined strictly to the place so described.

There was therefore in this case an infringement of the constitutional requirement that
a search warrant particularly describe the place to be searched; and that infringement
necessarily brought into operation the concomitant provision that (a)ny evidence
obtained in violation ** (inter alia of the search-and-seizure provision) shall be
inadmissible for any purpose in any proceeding.16cräläwvirtualibräry

In light of what has just been discussed, it is needless to discuss such other points
sought to be made by the Office of the Solicitor General as whether or not (1) the sketch
of the building housing the store and the residential apartment units -- the place to be
searched being plainly marked -- was in fact attached to the application for the search
warrant; or (2) the search had been conducted in the presence of the occupants of the
place (herein petitioners), among others; or (3) the validity of the search warrant was
diminished by the tardiness by which the return was made, or (4) the Court of Appeals
had improperly refused to receive evidence which ** (the People) had earlier been denied
opportunity to present before the trial court; or (5) the remedy of the special civil action
of certiorari in the Court of Appeals had been erroneously availed of. The resolution of
Page 139 of 377

these issues would not affect the correctness of the conclusion that the search and
seizure proceedings are void because the place set forth in the search warrant is different
from that which the officers actually searched, or the speciousness of their argument
that anyway, the premises searched were precisely what they had described to the
Judge, and originally and at all times had in mind.

Only one other matter merits treatment. The Solicitor Generals Office opines that where
a search warrant has been issued by the court other than the one trying the main
criminal case, the proper recourse of persons wishing to quash the warrant is to assail
it before the issuing court and not before that in which the criminal case involving the
subject of the warrant is afterwards filed.17 In support, it cites the second of five (5)
policy guidelines laid down by this Court in Malaloan v. Court of Appeals18 concerning
possible conflicts of jurisdiction (or, more accurately, in the exercise of jurisdiction)
where the criminal case is pending in one court and the search warrant is issued by
another court for the seizure of personal property intended to be used as evidence in
said criminal case. Said second guideline reads:19

2. When the latter court (referring to the court which does not try the main criminal
case) issues the search warrant, a motion to quash the same may be filed in and shall
be resolved by said court, without prejudice to any proper recourse to the appropriate
higher court by the party aggrieved by the resolution of the issuing court. All grounds
and objections then available, existent or known shall be raised in the original or
subsequent proceedings for the quashal of the warrant, other they shall be deemed
waived.

The guidelines have been misconstrued. Where a search warrant is issued by one court
and the criminal action based on the results of the search is afterwards commenced in
another court, it is not the rule that a motion to quash the warrant (or to retrieve things
thereunder seized) may be filed only with the issuing Court. Such a motion may be filed
for the first time for the first time in either the issuing Court or that in which the criminal
action is pending. However, the remedy is alternative, not cumulative. The Court first
taking cognizance of the motion does so to the exclusion of the other, and the
proceedings thereon are subject to the Omnibus Motion Rule and the rule against
forum-shopping. This is clearly stated in the third policy guidelines which indeed is
what properly applies to the case at bar, to wit:

3. Where no motion to quash the search warrant was filed in or resolved by the issuing
court, the interested party may move in the court where the criminal case is pending for
the suppression as evidence of the personal property seized under the warrant if the
same is offered therein for said purpose. Since two separate courts with different
participations are involved in this situation, a motion to quash a search warrant and a
Page 140 of 377

motion to supress evidence are alternative and not cummulative remedies. In order to
prevent forum shopping, a motion to quash shall consequently be governed by the
omnibus motion rule, provided however, that objections not available, existent or known
during the proceedings for the quashal of the warrant may be raised in the hearing of
the motion to suppress. The resolution of the court on the motion to suppress shall
likewise be subject to any proper remedy in the appopriate higher court.

In this case, the search warrant was applied for in, and issued by, Branch 216 of the
Regional Trial Court at Quezon City, and the return was made to said court. On the
other hand, the criminal action in connection with the explosives subject of the warrant
was filed in Branch 80 of the Regional Trial Court of Bulacan. In this situation, a motion
to quash the search warrant, or for the return of the personal property seized (not
otherwise contraband) could have properly been presented in the QC RTC. No such
motion was ever filed. It was only after the criminal action had been commenced in the
Bulacan RTC that the motion to quash and to suppress evidence was submitted to the
latter. The case thus falls within guideline No. 3 above quoted in accordance with which
the latter court must be deemed to have acted within its competence.

WHEREFORE, the judgment of the Fourteenth Division of the Court of Appeals of


September 11, 1996 -- which dismissed the Peoples petition for certiorari seeking
nullification of the Orders of Branch 80 of the Regional Trial Court dated February 9,
1996 and May 28, 1996 in the Criminal Case No. 43-M-96 -- is, for the reasons set out
in the foregoing opinion, hereby AFFIRMED without pronouncement as to costs.

SO ORDERED.

Romero, Kapunan, and Purisima, JJ., concur.


Page 141 of 377

G.R. No. L-64261 December 26, 1984

JOSE BURGOS, SR., JOSE BURGOS, JR., BAYANI SORIANO and J. BURGOS
MEDIA SERVICES, INC., petitioners, vs. THE CHIEF OF STAFF, ARMED
FORCES OF THE PHILIPPINES, THE CHIEF, PHILIPPINE CONSTABULARY,
THE CHIEF LEGAL OFFICER, PRESIDENTIAL SECURITY COMMAND, THE
JUDGE ADVOCATE GENERAL, ET AL., respondents.

Lorenzo M. Tañada, Wigberto E. Tañada, Martiniano Vivo, Augusto Sanchez, Joker P.


Arroyo, Jejomar Binay and Rene Saguisag for petitioners.
The Solicitor General for respondents.

ESCOLIN, J.:

Assailed in this petition for certiorari prohibition and mandamus with preliminary
mandatory and prohibitory injunction is the validity of two [2] search warrants issued
on December 7, 1982 by respondent Judge Ernani Cruz-Pano, Executive Judge of the
then Court of First Instance of Rizal [Quezon City], under which the premises known as
No. 19, Road 3, Project 6, Quezon City, and 784 Units C & D, RMS Building, Quezon
Avenue, Quezon City, business addresses of the "Metropolitan Mail" and "We Forum"
newspapers, respectively, were searched, and office and printing machines, equipment,
paraphernalia, motor vehicles and other articles used in the printing, publication and
distribution of the said newspapers, as well as numerous papers, documents, books
and other written literature alleged to be in the possession and control of petitioner Jose
Burgos, Jr. publisher-editor of the "We Forum" newspaper, were seized.

Petitioners further pray that a writ of preliminary mandatory and prohibitory injunction
be issued for the return of the seized articles, and that respondents, "particularly the
Chief Legal Officer, Presidential Security Command, the Judge Advocate General, AFP,
the City Fiscal of Quezon City, their representatives, assistants, subalterns,
subordinates, substitute or successors" be enjoined from using the articles thus seized
as evidence against petitioner Jose Burgos, Jr. and the other accused in Criminal Case
No. Q- 022782 of the Regional Trial Court of Quezon City, entitled People v. Jose Burgos,
Jr. et al. 1

In our Resolution dated June 21, 1983, respondents were required to answer the
petition. The plea for preliminary mandatory and prohibitory injunction was set for
hearing on June 28, 1983, later reset to July 7, 1983, on motion of the Solicitor General
in behalf of respondents.
Page 142 of 377

At the hearing on July 7, 1983, the Solicitor General, while opposing petitioners' prayer
for a writ of preliminary mandatory injunction, manifested that respondents "will not
use the aforementioned articles as evidence in the aforementioned case until final
resolution of the legality of the seizure of the aforementioned articles. ..." 2 With this
manifestation, the prayer for preliminary prohibitory injunction was rendered moot and
academic.

Respondents would have this Court dismiss the petition on the ground that petitioners
had come to this Court without having previously sought the quashal of the search
warrants before respondent judge. Indeed, petitioners, before impugning the validity of
the warrants before this Court, should have filed a motion to quash said warrants in
the court that issued them. 3 But this procedural flaw notwithstanding, we take
cognizance of this petition in view of the seriousness and urgency of the constitutional
issues raised not to mention the public interest generated by the search of the "We
Forum" offices, which was televised in Channel 7 and widely publicized in all
metropolitan dailies. The existence of this special circumstance justifies this Court to
exercise its inherent power to suspend its rules. In the words of the revered Mr. Justice
Abad Santos in the case of C. Vda. de Ordoveza v. Raymundo, 4 "it is always in the
power of the court [Supreme Court] to suspend its rules or to except a particular case
from its operation, whenever the purposes of justice require it...".

Respondents likewise urge dismissal of the petition on ground of laches. Considerable


stress is laid on the fact that while said search warrants were issued on December 7,
1982, the instant petition impugning the same was filed only on June 16, 1983 or after
the lapse of a period of more than six [6] months.

Laches is failure or negligence for an unreasonable and unexplained length of time to


do that which, by exercising due diligence, could or should have been done earlier. It is
negligence or omission to assert a right within a reasonable time, warranting a
presumption that the party entitled to assert it either has abandoned it or declined to
assert it. 5

Petitioners, in their Consolidated Reply, explained the reason for the delay in the filing
of the petition thus:

Respondents should not find fault, as they now do [p. 1, Answer, p. 3, Manifestation]
with the fact that the Petition was filed on June 16, 1983, more than half a year after
the petitioners' premises had been raided.
Page 143 of 377

The climate of the times has given petitioners no other choice. If they had waited this
long to bring their case to court, it was because they tried at first to exhaust other
remedies. The events of the past eleven fill years had taught them that everything in this
country, from release of public funds to release of detained persons from custody, has
become a matter of executive benevolence or largesse

Hence, as soon as they could, petitioners, upon suggestion of persons close to the
President, like Fiscal Flaminiano, sent a letter to President Marcos, through counsel
Antonio Coronet asking the return at least of the printing equipment and vehicles. And
after such a letter had been sent, through Col. Balbino V. Diego, Chief Intelligence and
Legal Officer of the Presidential Security Command, they were further encouraged to
hope that the latter would yield the desired results.

After waiting in vain for five [5] months, petitioners finally decided to come to Court. [pp.
123-124, Rollo]

Although the reason given by petitioners may not be flattering to our judicial system,
We find no ground to punish or chastise them for an error in judgment. On the contrary,
the extrajudicial efforts exerted by petitioners quite evidently negate the presumption
that they had abandoned their right to the possession of the seized property, thereby
refuting the charge of laches against them.

Respondents also submit the theory that since petitioner Jose Burgos, Jr. had used and
marked as evidence some of the seized documents in Criminal Case No. Q- 022872, he
is now estopped from challenging the validity of the search warrants. We do not follow
the logic of respondents. These documents lawfully belong to petitioner Jose Burgos, Jr.
and he can do whatever he pleases with them, within legal bounds. The fact that he has
used them as evidence does not and cannot in any way affect the validity or invalidity
of the search warrants assailed in this petition.

Several and diverse reasons have been advanced by petitioners to nullify the search
warrants in question.

1. Petitioners fault respondent judge for his alleged failure to conduct an examination
under oath or affirmation of the applicant and his witnesses, as mandated by the above-
quoted constitutional provision as wen as Sec. 4, Rule 126 of the Rules of Court .6 This
objection, however, may properly be considered moot and academic, as petitioners
Page 144 of 377

themselves conceded during the hearing on August 9, 1983, that an examination had
indeed been conducted by respondent judge of Col. Abadilla and his witnesses.

2. Search Warrants No. 20-82[a] and No. 20- 82[b] were used to search two distinct
places: No. 19, Road 3, Project 6, Quezon City and 784 Units C & D, RMS Building,
Quezon Avenue, Quezon City, respectively. Objection is interposed to the execution of
Search Warrant No. 20-82[b] at the latter address on the ground that the two search
warrants pinpointed only one place where petitioner Jose Burgos, Jr. was allegedly
keeping and concealing the articles listed therein, i.e., No. 19, Road 3, Project 6, Quezon
City. This assertion is based on that portion of Search Warrant No. 20- 82[b] which
states:

Which have been used, and are being used as instruments and means of committing
the crime of subversion penalized under P.D. 885 as amended and he is keeping and
concealing the same at 19 Road 3, Project 6, Quezon City.

The defect pointed out is obviously a typographical error. Precisely, two search warrants
were applied for and issued because the purpose and intent were to search two distinct
premises. It would be quite absurd and illogical for respondent judge to have issued two
warrants intended for one and the same place. Besides, the addresses of the places
sought to be searched were specifically set forth in the application, and since it was Col.
Abadilla himself who headed the team which executed the search warrants, the
ambiguity that might have arisen by reason of the typographical error is more apparent
than real. The fact is that the place for which Search Warrant No. 20- 82[b] was applied
for was 728 Units C & D, RMS Building, Quezon Avenue, Quezon City, which address
appeared in the opening paragraph of the said warrant. 7 Obviously this is the same
place that respondent judge had in mind when he issued Warrant No. 20-82 [b].

In the determination of whether a search warrant describes the premises to be searched


with sufficient particularity, it has been held "that the executing officer's prior
knowledge as to the place intended in the warrant is relevant. This would seem to be
especially true where the executing officer is the affiant on whose affidavit the warrant
had issued, and when he knows that the judge who issued the warrant intended the
building described in the affidavit, And it has also been said that the executing officer
may look to the affidavit in the official court file to resolve an ambiguity in the warrant
as to the place to be searched." 8

3. Another ground relied upon to annul the search warrants is the fact that although
the warrants were directed against Jose Burgos, Jr. alone, articles b belonging to his
Page 145 of 377

co-petitioners Jose Burgos, Sr., Bayani Soriano and the J. Burgos Media Services, Inc.
were seized.

Section 2, Rule 126 of the Rules of Court, enumerates the personal properties that may
be seized under a search warrant, to wit:

Sec. 2. Personal Property to be seized. — A search warrant may be issued for the search
and seizure of the following personal property:

[a] Property subject of the offense;

[b] Property stolen or embezzled and other proceeds or fruits of the offense; and

[c] Property used or intended to be used as the means of committing an offense.

The above rule does not require that the property to be seized should be owned by the
person against whom the search warrant is directed. It may or may not be owned by
him. In fact, under subsection [b] of the above-quoted Section 2, one of the properties
that may be seized is stolen property. Necessarily, stolen property must be owned by
one other than the person in whose possession it may be at the time of the search and
seizure. Ownership, therefore, is of no consequence, and it is sufficient that the person
against whom the warrant is directed has control or possession of the property sought
to be seized, as petitioner Jose Burgos, Jr. was alleged to have in relation to the articles
and property seized under the warrants.

4. Neither is there merit in petitioners' assertion that real properties were seized under
the disputed warrants. Under Article 415[5] of the Civil Code of the Philippines,
"machinery, receptables, instruments or implements intended by the owner of the
tenement for an industry or works which may be carried on in a building or on a piece
of land and which tend directly to meet the needs of the said industry or works" are
considered immovable property. In Davao Sawmill Co. v. Castillo9 where this legal
provision was invoked, this Court ruled that machinery which is movable by nature
becomes immobilized when placed by the owner of the tenement, property or plant, but
not so when placed by a tenant, usufructuary, or any other person having only a
temporary right, unless such person acted as the agent of the owner.
Page 146 of 377

In the case at bar, petitioners do not claim to be the owners of the land and/or building
on which the machineries were placed. This being the case, the machineries in question,
while in fact bolted to the ground remain movable property susceptible to seizure under
a search warrant.

5. The questioned search warrants were issued by respondent judge upon application
of Col. Rolando N. Abadilla Intelligence Officer of the P.C. Metrocom.10 The application
was accompanied by the Joint Affidavit of Alejandro M. Gutierrez and Pedro U. Tango,
11 members of the Metrocom Intelligence and Security Group under Col. Abadilla which
conducted a surveillance of the premises prior to the filing of the application for the
search warrants on December 7, 1982.

It is contended by petitioners, however, that the abovementioned documents could not


have provided sufficient basis for the finding of a probable cause upon which a warrant
may validly issue in accordance with Section 3, Article IV of the 1973 Constitution which
provides:

SEC. 3. ... and no search warrant or warrant of arrest shall issue except upon probable
cause to be determined by the judge, or such other responsible officer as may be
authorized by law, after examination under oath or affirmation of the complainant and
the witnesses he may produce, and particularly describing the place to be searched and
the persons or things to be seized.

We find petitioners' thesis impressed with merit. Probable cause for a search is defined
as such facts and circumstances which would lead a reasonably discreet and prudent
man to believe that an offense has been committed and that the objects sought in
connection with the offense are in the place sought to be searched. And when the search
warrant applied for is directed against a newspaper publisher or editor in connection
with the publication of subversive materials, as in the case at bar, the application
and/or its supporting affidavits must contain a specification, stating with particularity
the alleged subversive material he has published or is intending to publish. Mere
generalization will not suffice. Thus, the broad statement in Col. Abadilla's application
that petitioner "is in possession or has in his control printing equipment and other
paraphernalia, news publications and other documents which were used and are all
continuously being used as a means of committing the offense of subversion punishable
under Presidential Decree 885, as amended ..." 12 is a mere conclusion of law and does
not satisfy the requirements of probable cause. Bereft of such particulars as would
justify a finding of the existence of probable cause, said allegation cannot serve as basis
for the issuance of a search warrant and it was a grave error for respondent judge to
have done so.
Page 147 of 377

Equally insufficient as basis for the determination of probable cause is the statement
contained in the joint affidavit of Alejandro M. Gutierrez and Pedro U. Tango, "that the
evidence gathered and collated by our unit clearly shows that the premises above-
mentioned and the articles and things above-described were used and are continuously
being used for subversive activities in conspiracy with, and to promote the objective of,
illegal organizations such as the Light-a-Fire Movement, Movement for Free Philippines,
and April 6 Movement." 13

In mandating that "no warrant shall issue except upon probable cause to be determined
by the judge, ... after examination under oath or affirmation of the complainant and the
witnesses he may produce; 14 the Constitution requires no less than personal
knowledge by the complainant or his witnesses of the facts upon which the issuance of
a search warrant may be justified. In Alvarez v. Court of First Instance, 15 this Court
ruled that "the oath required must refer to the truth of the facts within the personal
knowledge of the petitioner or his witnesses, because the purpose thereof is to convince
the committing magistrate, not the individual making the affidavit and seeking the
issuance of the warrant, of the existence of probable cause." As couched, the quoted
averment in said joint affidavit filed before respondent judge hardly meets the test of
sufficiency established by this Court in Alvarez case.

Another factor which makes the search warrants under consideration constitutionally
objectionable is that they are in the nature of general warrants. The search warrants
describe the articles sought to be seized in this wise:

1] All printing equipment, paraphernalia, paper, ink, photo (equipment, typewriters,


cabinets, tables, communications/recording equipment, tape recorders, dictaphone and
the like used and/or connected in the printing of the "WE FORUM" newspaper and any
and all documents communication, letters and facsimile of prints related to the "WE
FORUM" newspaper.

2] Subversive documents, pamphlets, leaflets, books, and other publication to promote


the objectives and piurposes of the subversive organization known as Movement for Free
Philippines, Light-a-Fire Movement and April 6 Movement; and,

3] Motor vehicles used in the distribution/circulation of the "WE FORUM" and other
subversive materials and propaganda, more particularly,

1] Toyota-Corolla, colored yellow with Plate No. NKA 892;


Page 148 of 377

2] DATSUN pick-up colored white with Plate No. NKV 969

3] A delivery truck with Plate No. NBS 524;

4] TOYOTA-TAMARAW, colored white with Plate No. PBP 665; and,

5] TOYOTA Hi-Lux, pick-up truck with Plate No. NGV 427 with marking "Bagong Silang."

In Stanford v. State of Texas 16 the search warrant which authorized the search for
"books, records, pamphlets, cards, receipts, lists, memoranda, pictures, recordings and
other written instruments concerning the Communist Party in Texas," was declared void
by the U.S. Supreme Court for being too general. In like manner, directions to "seize any
evidence in connectionwith the violation of SDC 13-3703 or otherwise" have been held
too general, and that portion of a search warrant which authorized the seizure of any
"paraphernalia which could be used to violate Sec. 54-197 of the Connecticut General
Statutes [the statute dealing with the crime of conspiracy]" was held to be a general
warrant, and therefore invalid. 17 The description of the articles sought to be seized
under the search warrants in question cannot be characterized differently.

In the Stanford case, the U.S. Supreme Courts calls to mind a notable chapter in English
history: the era of disaccord between the Tudor Government and the English Press,
when "Officers of the Crown were given roving commissions to search where they pleased
in order to suppress and destroy the literature of dissent both Catholic and Puritan
Reference herein to such historical episode would not be relevant for it is not the policy
of our government to suppress any newspaper or publication that speaks with "the voice
of non-conformity" but poses no clear and imminent danger to state security.

As heretofore stated, the premises searched were the business and printing offices of
the "Metropolitan Mail" and the "We Forum newspapers. As a consequence of the search
and seizure, these premises were padlocked and sealed, with the further result that the
printing and publication of said newspapers were discontinued.

Such closure is in the nature of previous restraint or censorship abhorrent to the


freedom of the press guaranteed under the fundamental law, 18 and constitutes a
virtual denial of petitioners' freedom to express themselves in print. This state of being
is patently anathematic to a democratic framework where a free, alert and even militant
press is essential for the political enlightenment and growth of the citizenry.
Page 149 of 377

Respondents would justify the continued sealing of the printing machines on the ground
that they have been sequestered under Section 8 of Presidential Decree No. 885, as
amended, which authorizes "the sequestration of the property of any person, natural or
artificial, engaged in subversive activities against the government and its duly
constituted authorities ... in accordance with implementing rules and regulations as
may be issued by the Secretary of National Defense." It is doubtful however, if
sequestration could validly be effected in view of the absence of any implementing rules
and regulations promulgated by the Minister of National Defense.

Besides, in the December 10, 1982 issue of the Daily Express, it was reported that no
less than President Marcos himself denied the request of the military authorities to
sequester the property seized from petitioners on December 7, 1982. Thus:

The President denied a request flied by government prosecutors for sequestration of the
WE FORUM newspaper and its printing presses, according to Information Minister
Gregorio S. Cendana.

On the basis of court orders, government agents went to the We Forum offices in Quezon
City and took a detailed inventory of the equipment and all materials in the premises.

Cendaña said that because of the denial the newspaper and its equipment remain at
the disposal of the owners, subject to the discretion of the court. 19

That the property seized on December 7, 1982 had not been sequestered is further
confirmed by the reply of then Foreign Minister Carlos P. Romulo to the letter dated
February 10, 1983 of U.S. Congressman Tony P. Hall addressed to President Marcos,
expressing alarm over the "WE FORUM " case. 20 In this reply dated February 11, 1983,
Minister Romulo stated:

2. Contrary to reports, President Marcos turned down the recommendation of our


authorities to close the paper's printing facilities and confiscate the equipment and
materials it uses. 21

IN VIEW OF THE FOREGOING, Search Warrants Nos. 20-82[a] and 20-82[b] issued by
respondent judge on December 7, 1982 are hereby declared null and void and are
accordingly set aside. The prayer for a writ of mandatory injunction for the return of the
Page 150 of 377

seized articles is hereby granted and all articles seized thereunder are hereby ordered
released to petitioners. No costs.

SO ORDERED.

Fernando, C.J., Teehankee, Makasiar, Concepcion, Jr., Melencio-Herrera, Plana,


Relova, Gutierrez, Jr., De la Fuente and Cuevas, JJ., concur.

Aquino, J., took no part.

Separate Opinions
ABAD SANTOS, J., concurring

I am glad to give my concurrence to the ponencia of Mr. Justice Escolin At the same
time I wish to state my own reasons for holding that the search warrants which are the
subject of the petition are utterly void.

The action against "WE FORUM" was a naked suppression of press freedom for the
search warrants were issued in gross violation of the Constitution.

The Constitutional requirement which is expressed in Section 3, Article IV, stresses two
points, namely: "(1) that no warrant shall issue but upon probable cause, to be
determined by the judge in the manner set forth in said provision; and (2) that the
warrant shall particularly describe the things to be seized." (Stonehill vs. Diokno, 126
Phil. 738, 747: 20 SCRA 383 [1967].)

Any search warrant is conducted in disregard of the points mentioned above will result
in wiping "out completely one of the most fundamental rights guaranteed in our
Constitution, for it would place the sanctity of the domicile and the privacy of
communication and correspondence at the mercy of the whims caprice or passion of
peace officers." (Ibid, p. 748.)

The two search warrants were issued without probable cause. To satisfy the requirement
of probable cause a specific offense must be alleged in the application; abstract
averments will not suffice. In the case at bar nothing specifically subversive has been
Page 151 of 377

alleged; stated only is the claim that certain objects were being used as instruments and
means of committing the offense of subversion punishable under P.D. No. 885, as
amended. There is no mention of any specific provision of the decree. I n the words of
Chief Justice C Concepcion, " It would be legal heresy of the highest order, to convict
anybody" of violating the decree without reference to any determinate provision thereof.

The search warrants are also void for lack of particularity. Both search warrants
authorize Col. Rolando Abadilla to seize and take possession, among other things, of the
following:

Subversive documents, pamphlets, leaflets, books and other publication to promote the
objectives and purposes of the subversive organizations known as Movement for Free
Philippines, Light-a-Fire Movement and April 6 Movement.

The obvious question is: Why were the documents, pamphlets, leaflets, books, etc.
subversive? What did they contain to make them subversive? There is nothing in the
applications nor in the warrants which answers the questions. I must, therefore,
conclude that the warrants are general warrants which are obnoxious to the
Constitution.

In point of fact, there was nothing subversive published in the WE FORUM just as there
is nothing subversive which has been published in MALAYA which has replaced the
former and has the same content but against which no action has been taken.

Conformably with existing jurisprudence everything seized pursuant to the warrants


should be returned to the owners and all of the items are subject to the exclusionary
rule of evidence.

Teehankee, J., concur.


Page 152 of 377

G.R. No. 120915 April 3, 1998

THE PEOPLE OF THE PHILIPPINES, plaintiff-appellee, vs. ROSA ARUTA y


MENGUIN, accused-appellant.

ROMERO, J.:

With the pervasive proliferation of illegal drugs and its pernicious effects on our society,
our law enforcers tend at times to overreach themselves in apprehending drug offenders
to the extent of failing to observe well-entrenched constitutional guarantees against
illegal searches and arrests. Consequently, drug offenders manage to evade the clutches
of the law on mere technicalities.

Accused-appellant Rosa Aruta y Menguin was arrested and charged with violating
Section 4, Article II of Republic Act No. 6425 or the Dangerous Drugs Act. The
information reads:

That on or about the fourteenth (14th) day of December, 1988, in the City of Olongapo,
Philippines, and within the jurisdiction of this Honorable Court, the above-named
accused, without being lawfully authorized, did then and there willfully, unlawfully and
knowingly engage in transporting approximately eight (8) kilos and five hundred (500)
grams of dried marijuana packed in plastic bag marked "Cash Katutak" placed in a
traveling bag, which are prohibited drugs.

Upon arraignment, she pleaded "not guilty." After trial on the merits, the Regional Trial
Court of Olongapo City convicted and sentenced her to suffer the penalty of life
imprisonment and to pay a fine of twenty thousand (P20,000.00) pesos.1

The prosecution substantially relied on the testimonies of P/Lt. Ernesto Abello, Officer-
in-Charge of the Narcotics Command (NARCOM) of Olongapo City and P/Lt. Jose
Domingo. Based on their testimonies, the court a quo found the following:

On December 13, 1988, P/Lt. Abello was tipped off by his informant, known only as
Benjie, that a certain "Aling Rosa" would be arriving from Baguio City the following day,
December 14, 1988, with a large volume of marijuana. Acting on said tip, P/Lt. Abello
assembled a team composed of P/Lt. Jose Domingo, Sgt. Angel Sudiacal, Sgt. Oscar
Imperial, Sgt. Danilo Santiago and Sgt. Efren Quirubin.
Page 153 of 377

Said team proceeded to West Bajac-Bajac, Olongapo City at around 4:00 in the
afternoon of December 14, 1988 and deployed themselves near the Philippine National
Bank (PNB) building along Rizal Avenue and the Caltex gasoline station. Dividing
themselves into two groups, one group, made up of P/Lt. Abello, P/Lt. Domingo and the
informant posted themselves near the PNB building while the other group waited near
the Caltex gasoline station.

While thus positioned, a Victory Liner Bus with body number 474 and the letters BGO
printed on its front and back bumpers stopped in front of the PNB building at around
6:30 in the evening of the same day from where two females and a male got off. It was
at this stage that the informant pointed out to the team "Aling Rosa" who was then
carrying a traveling bag.

Having ascertained that accused-appellant was "Aling Rosa," the team approached her
and introduced themselves as NARCOM agents. When P/Lt. Abello asked "Aling Rosa"
about the contents of her bag, the latter handed it to the former.

Upon inspection, the bag was found to contain dried marijuana leaves packed in a
plastic bag marked "Cash Katutak." The team confiscated the bag together with the
Victory Liner bus ticket to which Lt. Domingo affixed his signature. Accused-appellant
was then brought to the NARCOM office for investigation where a Receipt of Property
Seized was prepared for the confiscated marijuana leaves.

Upon examination of the seized marijuana specimen at the PC/INP Crime Laboratory,
Camp Olivas, Pampanga, P/Maj. Marlene Salangad, a Forensic Chemist, prepared a
Technical Report stating that said specimen yielded positive results for marijuana, a
prohibited drug.

After the presentation of the testimonies of the arresting officers and of the above
technical report, the prosecution rested its case.

Instead of presenting its evidence, the defense filed a "Demurrer to Evidence" alleging
the illegality of the search and seizure of the items thereby violating accused-appellant's
constitutional right against unreasonable search and seizure as well as their
inadmissibility in evidence.
Page 154 of 377

The said "Demurrer to Evidence" was, however, denied without the trial court ruling on
the alleged illegality of the search and seizure and the inadmissibility in evidence of the
items seized to avoid pre-judgment. Instead, the trial court continued to hear the case.

In view of said denial, accused-appellant testified on her behalf. As expected, her version
of the incident differed from that of the prosecution. She claimed that immediately prior
to her arrest, she had just come from Choice Theater where she watched the movie
"Balweg." While about to cross the road, an old woman asked her help in carrying a
shoulder bag. In the middle of the road, Lt. Abello and Lt. Domingo arrested her and
asked her to go with them to the NARCOM Office.

During investigation at said office, she disclaimed any knowledge as to the identity of
the woman and averred that the old woman was nowhere to be found after she was
arrested. Moreover, she added that no search warrant was shown to her by the arresting
officers.

After the prosecution made a formal offer of evidence, the defense filed a "Comment
and/or Objection to Prosecution's Formal Offer of Evidence" contesting the admissibility
of the items seized as they were allegedly a product of an unreasonable search and
seizure.

Not convinced with her version of the incident, the Regional Trial Court of Olongapo City
convicted accused-appellant of transporting eight (8) kilos and five hundred (500) grams
of marijuana from Baguio City to Olongapo City in violation of Section 4, Article 11 of
R.A. No. 6425, as amended, otherwise known as the Dangerous Drugs Act of 1972 and
sentenced her to life imprisonment and to pay a fine of twenty thousand (P20,000.00)
pesos without subsidiary imprisonment in case of insolvency.2

In this appeal, accused-appellant submits the following:

1. The trial court erred in holding that the NARCOM agents could not apply for a warrant
for the search of a bus or a passenger who boarded a bus because one of the
requirements for applying a search warrant is that the place to be searched must be
specifically designated and described.

2. The trial court erred in holding or assuming that if a search warrant was applied for
by the NARCOM agents, still no court would issue a search warrant for the reason that
the same would be considered a general search warrant which may be quashed.
Page 155 of 377

3. The trial court erred in not finding that the warrantless search resulting to the arrest
of accused-appellant violated the latter's constitutional rights.

4. The trial court erred in not holding that although the defense of denial is weak yet
the evidence of the prosecution is even weaker.

These submissions are impressed with merit.

In People v. Ramos,3 this Court held that a search may be conducted by law enforcers
only on the strength of a search warrant validly issued by a judge as provided in Article
III, Section 2 of the Constitution which provides:

Sec. 2. The right of the people to be secure in their persons, houses, papers, and effects
against unreasonable searches and seizures of whatever nature and for any purpose
shall be inviolable, and no search warrant or warrant of arrest shall issue except upon
probable cause to be determined personally by the judge after examination under oath
or affirmation of the complainant and the witnesses he may produce, and particularly
describing the place to be searched and the persons or things to be seized.

This constitutional guarantee is not a blanket prohibition against all searches and
seizures as it operates only against "unreasonable" searches and seizures. The plain
import of the language of the Constitution, which in one sentence prohibits
unreasonable searches and seizures and at the same time prescribes the requisites for
a valid warrant, is that searches and seizures are normally unreasonable unless
authorized by a validly issued search warrant or warrant of arrest. Thus, the
fundamental protection accorded by the search and seizure clause is that between
person and police must stand the protective authority of a magistrate clothed with power
to issue or refuse to issue search warrants or warrants of arrest.4

Further, articles which are the product of unreasonable searches and seizures are
inadmissible as evidence pursuant to the doctrine pronounced in Stonehill v. Diokno.5
This exclusionary rule was later enshrined in Article III, Section 3(2) of the Constitution,
thus:

Sec. 3(2). Any evidence obtained in violation of this or the preceding section shall be
inadmissible in evidence for any purpose in any proceeding.
Page 156 of 377

From the foregoing, it can be said that the State cannot simply intrude indiscriminately
into the houses, papers, effects, and most importantly, on the person of an individual.
The constitutional provision guaranteed an impenetrable shield against unreasonable
searches and seizures. As such, it protects the privacy and sanctity of the person himself
against unlawful arrests and other forms of restraint.6

Therewithal, the right of a person to be secured against any unreasonable seizure of his
body and any deprivation of his liberty is a most basic and fundamental one. A statute,
rule or situation which allows exceptions to the requirement of a warrant of arrest or
search warrant must perforce be strictly construed and their application limited only to
cases specifically provided or allowed by law. To do otherwise is an infringement upon
personal liberty and would set back a right so basic and deserving of full protection and
vindication yet often violated.7

The following cases are specifically provided or allowed by law:

1. Warrantless search incidental to a lawful arrest recognized under Section 12, Rule
126 of the Rules of Court8 and by prevailing jurisprudence;

2. Seizure of evidence in "plain view," the elements of which are:

(a) a prior valid intrusion based on the valid warrantless arrest in which the police are
legally present in the pursuit of their official duties;

(b) the evidence was inadvertently discovered by the police who had the right to be where
they are;

(c) the evidence must be immediately apparent, and

(d) "plain view" justified mere seizure of evidence without further search;

3. Search of a moving vehicle. Highly regulated by the government, the vehicle's inherent
mobility reduces expectation of privacy especially when its transit in public
Page 157 of 377

thoroughfares furnishes a highly reasonable suspicion amounting to probable cause


that the occupant committed a criminal activity;

4. Consented warrantless search;

5. Customs search;9

6. Stop and Frisk;10 and

7. Exigent and Emergency Circumstances.11

The above exceptions, however, should not become unbridled licenses for law
enforcement officers to trample upon the constitutionally guaranteed and more
fundamental right of persons against unreasonable search and seizures. The essential
requisite of probable cause must still be satisfied before a warrantless search and
seizure can be lawfully conducted.

Although probable cause eludes exact and concrete definition, it generally signifies a
reasonable ground of suspicion supported by circumstances sufficiently strong in
themselves to warrant a cautious man to believe that the person accused is guilty of the
offense with which he is charged. It likewise refers to the existence of such facts and
circumstances which could lead a reasonably discreet and prudent man to believe that
an offense has been committed and that the item(s), article(s) or object(s) sought in
connection with said offense or subject to seizure and destruction by law is in the place
to be searched.12

It ought to be emphasized that in determining probable cause, the average man weighs
facts and circumstances without resorting to the calibrations of our rules of evidence of
which his knowledge is technically nil. Rather, he relies on the calculus of common
sense which all reasonable men have in abundance. The same quantum of evidence is
required in determining probable cause relative to search. Before a search warrant can
be issued, it must be shown by substantial evidence that the items sought are in fact
seizable by virtue of being connected with criminal activity, and that the items will be
found in the place to be searched.13

In searches and seizures effected without a warrant, it is necessary for probable cause
to be present. Absent any probable cause, the article(s) seized could not be admitted
Page 158 of 377

and used as evidence against the person arrested. Probable cause, in these cases, must
only be based on reasonable ground of suspicion or belief that a crime has been
committed or is about to be committed.

In our jurisprudence, there are instances where information has become a sufficient
probable cause to effect a warrantless search and seizure.

In People v. Tangliben,14 acting on information supplied by informers, police officers


conducted a surveillance at the Victory Liner Terminal compound in San Fernando,
Pampanga against persons who may commit misdemeanors and also on those who may
be engaging in the traffic of dangerous drugs. At 9:30 in the evening, the policemen
noticed a person carrying a red traveling bag who was acting suspiciously. They
confronted him and requested him to open his bag but he refused. He acceded later on
when the policemen identified themselves. Inside the bag were marijuana leaves
wrapped in a plastic wrapper. The police officers only knew of the activities of Tangliben
on the night of his arrest.

In instant case, the apprehending officers already had prior knowledge from their
informant regarding Aruta's alleged activities. In Tangliben policemen were confronted
with an on-the-spot tip. Moreover, the policemen knew that the Victory Liner compound
is being used by drug traffickers as their "business address". More significantly,
Tangliben was acting suspiciously. His actuations and surrounding circumstances led
the policemen to reasonably suspect that Tangliben is committing a crime. In instant
case, there is no single indication that Aruta was acting suspiciously.

In People v. Malmstedt,15 the Narcom agents received reports that vehicles coming from
Sagada were transporting marijuana. They likewise received information that a
Caucasian coming from Sagada had prohibited drugs on his person. There was no
reasonable time to obtain a search warrant, especially since the identity of the suspect
could not be readily ascertained. His actuations also aroused the suspicion of the
officers conducting the operation. The Court held that in light of such circumstances,
to deprive the agents of the ability and facility to act promptly, including a search
without a warrant, would be to sanction impotence and ineffectiveness in law
enforcement, to the detriment of society.

Note, however, the glaring differences of Malmstedt to the instant case. In present case,
the police officers had reasonable time within which to secure a search warrant. Second,
Aruta's identity was priorly ascertained. Third, Aruta was not acting suspiciously.
Fourth, Malmstedt was searched aboard a moving vehicle, a legally accepted exception
Page 159 of 377

to the warrant requirement. Aruta, on the other hand, was searched while about to cross
a street.

In People v. Bagista,16 the NARCOM officers had probable cause to stop and search all
vehicles coming from the north to Acop, Tublay, Benguet in view of the confidential
information they received from their regular informant that a woman having the same
appearance as that of accused-appellant would be bringing marijuana from up north.
They likewise had probable cause to search accused-appellant's belongings since she
fitted the description given by the NARCOM informant. Since there was a valid
warrantless search by the NARCOM agents, any evidence obtained in the course of said
search is admissible against accused-appellant. Again, this case differs from Aruta as
this involves a search of a moving vehicle plus the fact that the police officers erected a
checkpoint. Both are exceptions to the requirements of a search warrant.

In Manalili v. Court of Appeals and People,17 the policemen conducted a surveillance


in an area of the Kalookan Cemetery based on information that drug addicts were
roaming therein. Upon reaching the place, they chanced upon a man in front of the
cemetery who appeared to be "high" on drugs. He was observed to have reddish eyes
and to be walking in a swaying manner. Moreover, he appeared to be trying to avoid the
policemen. When approached and asked what he was holding in his hands, he tried to
resist. When he showed his wallet, it contained marijuana. The Court held that the
policemen had sufficient reason to accost accused-appellant to determine if he was
actually "high" on drugs due to his suspicious actuations, coupled with the fact that
based on information, this area was a haven for drug addicts.

In all the abovecited cases, there was information received which became the bases for
conducting the warrantless search. Furthermore, additional factors and circumstances
were present which, when taken together with the information, constituted probable
causes which justified the warrantless searches and seizures in each of the cases.

In the instant case, the determination of the absence or existence of probable cause
necessitates a reexamination of the facts. The following have been established: (1) In the
morning of December 13, 1988, the law enforcement officers received information from
an informant named "Benjie" that a certain "Aling Rosa" would be leaving for Baguio
City on December 14, 1988 and would be back in the afternoon of the same day carrying
with her a large volume of marijuana; (2) At 6:30 in the evening of December 14, 1988,
accused-appellant alighted from a Victory Liner Bus carrying a traveling bag even as the
informant pointed her out to the law enforcement officers; (3) The law enforcement
officers approached her and introduced themselves as NARCOM agents; (4) When asked
by Lt. Abello about the contents of her traveling bag, she gave the same to him; (5) When
Page 160 of 377

they opened the same, they found dried marijuana leaves; (6) Accused-appellant was
then brought to the NARCOM office for investigation.

This case is similar to People v. Aminnudin where the police received information two
days before the arrival of Aminnudin that the latter would be arriving from Iloilo on
board the M/V Wilcon 9. His name was known, the vehicle was identified and the date
of arrival was certain. From the information they had received, the police could have
persuaded a judge that there was probable cause, indeed, to justify the issuance of a
warrant. Instead of securing a warrant first, they proceeded to apprehend Aminnudin.
When the case was brought before this Court, the arrest was held to be illegal; hence
any item seized from Aminnudin could not be used against him.

Another recent case is People v. Encinada where the police likewise received confidential
information the day before at 4:00 in the afternoon from their informant that Encinada
would be bringing in marijuana from Cebu City on board M/V Sweet Pearl at 7:00 in
the morning of the following day. This intelligence information regarding the culprit's
identity, the particular crime he allegedly committed and his exact whereabouts could
have been a basis of probable cause for the lawmen to secure a warrant. This Court held
that in accordance with Administrative Circular No. 13 and Circular No. 19, series of
1987, the lawmen could have applied for a warrant even after court hours. The failure
or neglect to secure one cannot serve as an excuse for violating Encinada's
constitutional right.

In the instant case, the NARCOM agents were admittedly not armed with a warrant of
arrest. To legitimize the warrantless search and seizure of accused-appellant's bag,
accused-appellant must have been validly arrested under Section 5 of Rule 113 which
provides inter alia:

Sec. 5. Arrest without warrant; when lawful. — A peace officer or a private person may,
without a warrant, arrest a person:

(a) When in his presence, the person to be arrested has committed, is actually
committing, or is attempting to commit an offense;

xxx xxx xxx

Accused-appellant Aruta cannot be said to be committing a crime. Neither was she


about to commit one nor had she just committed a crime. Accused-appellant was merely
Page 161 of 377

crossing the street and was not acting in any manner that would engender a reasonable
ground for the NARCOM agents to suspect and conclude that she was committing a
crime. It was only when the informant pointed to accused-appellant and identified her
to the agents as the carrier of the marijuana that she was singled out as the suspect.
The NARCOM agents would not have apprehended accused-appellant were it not for the
furtive finger of the informant because, as clearly illustrated by the evidence on record,
there was no reason whatsoever for them to suspect that accused-appellant was
committing a crime, except for the pointing finger of the informant. This the Court could
neither sanction nor tolerate as it is a clear violation of the constitutional guarantee
against unreasonable search and seizure. Neither was there any semblance of any
compliance with the rigid requirements of probable cause and warrantless arrests.

Consequently, there was no legal basis for the NARCOM agents to effect a warrantless
search of accused-appellant's bag, there being no probable cause and the accused-
appellant not having been lawfully arrested. Stated otherwise, the arrest being
incipiently illegal, it logically follows that the subsequent search was similarly illegal, it
being not incidental to a lawful arrest. The constitutional guarantee against
unreasonable search and seizure must perforce operate in favor of accused-appellant.
As such, the articles seized could not be used as evidence against accused-appellant for
these are "fruits of a poisoned tree" and, therefore, must be rejected, pursuant to Article
III, Sec. 3(2) of the Constitution.

Emphasis is to be laid on the fact that the law requires that the search be incidental to
a lawful arrest, in order that the search itself may likewise be considered legal.
Therefore, it is beyond cavil that a lawful arrest must precede the search of a person
and his belongings. Where a search is first undertaken, and an arrest effected based on
evidence produced by the search, both such search and arrest would be unlawful, for
being contrary to law.18

As previously discussed, the case in point is People v. Aminnudin19 where, this Court
observed that:

. . . accused-appellant was not, at the moment of his arrest, committing a crime nor was
it shown that he was about to do so or that he had just done so. What he was doing was
descending the gangplank of the M/V Wilcon 9 and there was no outward indication
that called for his arrest. To all appearances, he was like any of the other passengers
innocently disembarking from the vessel. It was only when the informer pointed to him
as the carrier of the marijuana that he suddenly became suspect and so subject to
apprehension. It was the furtive finger that triggered his arrest. The identification by the
informer was the probable cause as determined by the officers (and not a judge) that
authorized them to pounce upon Aminnudin and immediately arrest him.
Page 162 of 377

In the absence of probable cause to effect a valid and legal warrantless arrest, the search
and seizure of accused-appellant's bag would also not be justified as seizure of evidence
in "plain view" under the second exception. The marijuana was obviously not
immediately apparent as shown by the fact that the NARCOM agents still had to request
accused-appellant to open the bag to ascertain its contents.

Neither would the search and seizure of accused-appellant's bag be justified as a search
of a moving vehicle. There was no moving vehicle to speak of in the instant case as
accused-appellant was apprehended several minutes after alighting from the Victory
Liner bus. In fact, she was accosted in the middle of the street and not while inside the
vehicle.

People v. Solayao,20 applied the stop and frisk principle which has been adopted in
Posadas v. Court of Appeals.21 In said case, Solayao attempted to flee when he and his
companions were accosted by government agents. In the instant case, there was no
observable manifestation that could have aroused the suspicion of the NARCOM agents
as to cause them to "stop and frisk" accused-appellant. To reiterate, accused-appellant
was merely crossing the street when apprehended. Unlike in the abovementioned cases,
accused-appellant never attempted to flee from the NARCOM agents when the latter
identified themselves as such. Clearly, this is another indication of the paucity of
probable cause that would sufficiently provoke a suspicion that accused-appellant was
committing a crime.

The warrantless search and seizure could not likewise be categorized under exigent and
emergency circumstances, as applied in People v. De
Gracia.22 In said case, there were intelligence reports that the building was being used
as headquarters by the RAM during a coup d' etat. A surveillance team was fired at by
a group of armed men coming out of the building and the occupants of said building
refused to open the door despite repeated requests. There were large quantities of
explosives and ammunitions inside the building. Nearby courts were closed and general
chaos and disorder prevailed. The existing circumstances sufficiently showed that a
crime was being committed. In short, there was probable cause to effect a warrantless
search of the building. The same could not be said in the instant case.

The only other exception that could possibly legitimize the warrantless search and
seizure would be consent given by the accused-appellant to the warrantless search as
to amount to a waiver of her constitutional right. The Solicitor General argues that
accused-appellant voluntarily submitted herself to search and inspection citing People
v. Malasugui23 where this Court ruled:
Page 163 of 377

When one voluntarily submits to a search or consents to have it made on his person or
premises, he is precluded from complaining later thereof. (Cooley, Constitutional
Limitations, 8th ed., [V]ol. I, p. 631.) The right to be secure from unreasonable search
may, like every right, be waived and such waiver may be made either expressly or
impliedly.

In support of said argument, the Solicitor General cited the testimony of Lt. Abello, thus:

Q When this informant by the name of alias Benjie pointed to Aling Rosa, what happened
after that?

A We followed her and introduced ourselves as NARCOM agents and confronted her with
our informant and asked her what she was carrying and if we can see the bag she was
carrying.

Q What was her reaction?

A She gave her bag to me.

Q So what happened after she gave the bag to you?

A I opened it and found out plastic bags of marijuana inside.24

This Court cannot agree with the Solicitor General's contention for the Malasugui case
is inapplicable to the instant case. In said case, there was probable cause for the
warrantless arrest thereby making the warrantless search effected immediately
thereafter equally lawful.25 On the contrary, the most essential element of probable
cause, as expounded above in detail, is wanting in the instant case making the
warrantless arrest unjustified and illegal. Accordingly, the search which accompanied
the warrantless arrest was likewise unjustified and illegal. Thus, all the articles seized
from the accused-appellant could not be used as evidence against her.

Aside from the inapplicability of the abovecited case, the act of herein accused-appellant
in handing over her bag to the NARCOM agents could not be construed as voluntary
Page 164 of 377

submission or an implied acquiescence to the unreasonable search. The instant case is


similar to People v. Encinada,26 where this Court held:

[T]he Republic's counsel avers that appellant voluntarily handed the chairs containing
the package of marijuana to the arresting officer and thus effectively waived his right
against the warrantless search. This he gleaned from Bolonia's testimony.

Q: After Roel Encinada alighted from the motor tricycle, what happened next?

A: I requested to him to see his chairs that he carried.

Q: Are you referring to the two plastic chairs?

A: Yes, sir.

Q: By the way, when Roel Encinada agreed to allow you to examine the two chairs that
he carried, what did you do next?

A: I examined the chairs and I noticed that something inside in between the two chairs.

We are not convinced. While in principle we agree that consent will validate an otherwise
illegal search, we believe that appellant — based on the transcript quoted above — did
not voluntarily consent to Bolonia's search of his belongings. Appellant's silence should
not be lightly taken as consent to such search. The implied acquiescence to the search,
if there was any, could not have been more than mere passive conformity given under
intimidating or coercive circumstances and is thus considered no consent at all within
the purview of the constitutional guarantee. Furthermore, considering that the search
was conducted irregularly, i.e., without a warrant, we cannot appreciate consent based
merely on the presumption of regularity of the performance of duty." (Emphasis
supplied)

Thus, accused-appellant's lack of objection to the search is not tantamount to a waiver


of her constitutional rights or a voluntary submission to the warrantless search. As this
Court held in People v. Barros:27
Page 165 of 377

. . . [T]he accused is not to be presumed to have waived the unlawful search conducted
on the occasion of his warrantless arrest "simply because he failed to object" —

. . . To constitute a waiver, it must appear first that the right exists; secondly, that the
person involved had knowledge, actual or constructive, of the existence of such right;
and lastly, that said person had an actual intention to relinquish the right (Pasion Vda.
de Garcia v. Locsin, 65 Phil. 698). The fact that the accused failed to object to the entry
into his house does not amount to a permission to make a search therein (Magoncia v.
Palacio, 80 Phil. 770). As pointed out by Justice Laurel in the case of Pasion Vda. de
Garcia v. Locsin (supra):

xxx xxx xxx

. . . As the constitutional guaranty is not dependent upon any affirmative act of the
citizen, the courts do not place the citizen in the position of either contesting an officer's
authority by force, or waiving his constitutional rights; but instead they hold that a
peaceful submission to a search or seizure is not a consent or an invitation thereto, but
is merely a demonstration of regard for the supremacy of the law. (Citation omitted).

We apply the rule that: "courts indulge every reasonable presumption against waiver of
fundamental constitutional rights and that we do not presume acquiescence in the loss
of fundamental rights."28 (Emphasis supplied)

To repeat, to constitute a waiver, there should be an actual intention to relinquish the


right. As clearly illustrated in People v. Omaweng,29 where prosecution witness Joseph
Layong testified thus:

PROSECUTOR AYOCHOK:

Q — When you and David Fomocod saw the travelling bag, what did you do?

A — When we saw that traveling bag, we asked the driver if we could see the contents.

Q — And what did or what was the reply of the driver, if there was any?
Page 166 of 377

A — He said "you can see the contents but those are only clothings" (sic).

Q — When he said that, what did you do?

A — We asked him if we could open and see it.

Q — When you said that, what did he tell you?

A — He said "you can see it".

Q — And when he said "you can see and open it," what did you do?

A — When I went inside and opened the bag, I saw that it was not clothings (sic) that
was contained in the bag.

Q — And when you saw that it was not clothings (sic), what did you do?

A — When I saw that the contents were not clothes, I took some of the contents and
showed it to my companion Fomocod and when Fomocod smelled it, he said it was
marijuana. (Emphasis supplied)

In the above-mentioned case, accused was not subjected to any search which may be
stigmatized as a violation of his Constitutional right against unreasonable searches and
seizures. If one had been made, this Court would be the first to condemn it "as the
protection of the citizen and the maintenance of his constitutional rights is one of the
highest duties and privileges of the Court." He willingly gave prior consent to the search
and voluntarily agreed to have it conducted on his vehicle and traveling bag, which is
not the case with Aruta.

In an attempt to further justify the warrantless search, the Solicitor General next argues
that the police officers would have encountered difficulty in securing a search warrant
as it could be secured only if accused-appellant's name was known, the vehicle identified
and the date of its arrival certain, as in the Aminnudin case where the arresting officers
had forty-eight hours within which to act.
Page 167 of 377

This argument is untenable.

Article IV, Section 3 of the Constitution provides:

. . . [N]o search warrant or warrant of arrest shall issue except upon probable cause to
be determined by the judge, or such other responsible officer as may be authorized by
law, after examination under oath or affirmation of the complainant and the witnesses
he may produce, and particularly describing the place to be searched and the persons
or things to be seized. (Emphasis supplied)

Search warrants to be valid must particularly describe the place to be searched and the
persons or things to be seized. The purpose of this rule is to limit the things to be seized
to those and only those, particularly described in the warrant so as to leave the officers
of the law with no discretion regarding what articles they shall seize to the end that
unreasonable searches and seizures may not be made.30

Had the NARCOM agents only applied for a search warrant, they could have secured
one without too much difficulty, contrary to the assertions of the Solicitor General. The
person intended to be searched has been particularized and the thing to be seized
specified. The time was also sufficiently ascertained to be in the afternoon of December
14, 1988. "Aling Rosa" turned out to be accused-appellant and the thing to be seized
was marijuana. The vehicle was identified to be a Victory Liner bus. In fact, the NARCOM
agents purposely positioned themselves near the spot where Victory Liner buses
normally unload their passengers. Assuming that the NARCOM agents failed to
particularize the vehicle, this would not in any way hinder them from securing a search
warrant. The above particulars would have already sufficed. In any case, this Court has
held that the police should particularly describe the place to be searched and the person
or things to be seized, wherever and whenever it is feasible.31 (Emphasis supplied)

While it may be argued that by entering a plea during arraignment and by actively
participating in the trial, accused-appellant may be deemed to have waived objections
to the illegality of the warrantless search and to the inadmissibility of the evidence
obtained thereby, the same may not apply in the instant case for the following reasons:

1. The waiver would only apply to objections pertaining to the illegality of the arrest as
her plea of "not guilty" and participation in the trial are indications of her voluntary
submission to the court's jurisdiction.32 The plea and active participation in the trial
Page 168 of 377

would not cure the illegality of the search and transform the inadmissible evidence into
objects of proof. The waiver simply does not extend this far.

2. Granting that evidence obtained through a warrantless search becomes admissible


upon failure to object thereto during the trial of the case, records show that accused-
appellant filed a Demurrer to Evidence and objected and opposed the prosecution's
Formal Offer of Evidence.

It is apropos to quote the case of People v. Barros,33 which stated:

It might be supposed that the non-admissibility of evidence secured through an invalid


warrantless arrest or a warrantless search and seizure may be waived by an accused
person. The a priori argument is that the invalidity of an unjustified warrantless arrest,
or an arrest effected with a defective warrant of arrest may be waived by applying for
and posting of bail for provisional liberty, so as to estop an accused from questioning
the legality or constitutionality of his detention or the failure to accord him a preliminary
investigation. We do not believe, however, that waiver of the latter necessarily
constitutes, or carries with it, waiver of the former — an argument that the Solicitor
General appears to be making impliedly. Waiver of the non-admissibility of the "fruits"
of an invalid warrantless arrest and of a warrantless search and seizure is not casually
to be presumed, if the constitutional right against unlawful searches and seizures is to
retain its vitality for the protection of our people. In the case at bar, defense counsel had
expressly objected on constitutional grounds to the admission of the carton box and the
four (4) kilos of marijuana when these were formally offered in evidence by the
prosecution. We consider that appellant's objection to the admission of such evidence
was made clearly and seasonably and that, under the circumstances, no intent to waive
his rights under the premises can be reasonably inferred from his conduct before or
during the trial. (Emphasis supplied).

In fine, there was really no excuse for the NARCOM agents not to procure a search
warrant considering that they had more than twenty-four hours to do so. Obviously,
this is again an instance of seizure of the "fruit of the poisonous tree," hence illegal and
inadmissible subsequently in evidence.

The exclusion of such evidence is the only practical means of enforcing the
constitutional injunction against unreasonable searches and seizure. The non-
exclusionary rule is contrary to the letter and spirit of the prohibition against
unreasonable searches and seizures.34
Page 169 of 377

While conceding that the officer making the unlawful search and seizure may be held
criminally and civilly liable, the Stonehill case observed that most jurisdictions have
realized that the exclusionary rule is "the only practical means of enforcing the
constitutional injunction" against abuse. This approach is based on the justification
made by Judge Learned Hand that "only in case the prosecution which itself controls
the seizing officials, knows that it cannot profit by their wrong, will the wrong be
repressed."35

Unreasonable searches and seizures are the menace against which the constitutional
guarantees afford full protection. While the power to search and seize may at times be
necessary to the public welfare, still it may be exercised and the law enforced without
transgressing the constitutional rights of the citizens, for the enforcement of no statute
is of sufficient importance to justify indifference to the basic principles of government.36

Those who are supposed to enforce the law are not justified in disregarding the rights
of the individual in the name of order. Order is too high a price to pay for the loss of
liberty. As Justice Holmes declared: "I think it is less evil that some criminals escape
than that the government should play an ignoble part." It is simply not allowed in free
society to violate a law to enforce another, especially if the law violated is the
Constitution itself.37

WHEREFORE, in view of the foregoing, the decision of the Regional Trial Court, Branch
73, Olongapo City, is hereby REVERSED and SET ASIDE. For lack of evidence to
establish her guilt beyond reasonable doubt, accused-appellant ROSA ARUTA Y
MENGUIN is hereby ACQUITTED and ordered RELEASED from confinement unless she
is being held for some other legal grounds. No costs.

SO ORDERED.

Narvasa, C.J., Kapunan and Purisima, JJ., concur.


Page 170 of 377

G.R. No. L-69803 January 30, 1987

CYNTHIA D. NOLASCO, MILA AGUILAR-ROQUE and WILLIE C. TOLENTINO,


Petitioners, v. HON. ERNANI CRUZ PAÑO, Executive Judge, Regional Trial
Court of Quezon City; HON. ANTONIO P. SANTOS, Presiding Judge, Branch
XLII, Metropolitan Trial Court of Quezon City; HON. SERGIO F. APOSTOL,
City Fiscal, Quezon City; HON. JUAN PONCE ENRILE, LT. GEN. FIDEL
RAMOS and COL. JESUS ALTUNA, Respondents.

MELENCIO-HERRERA, J.:

For resolution are petitioners’ and public respondents’ respective Motions for Partial
Reconsideration of this Court’s Decision of October 8, 1985, which decreed that:

"WHEREFORE, while Search Warrant No. 80-84 issued on August 6, 1984 by


respondent Executive Judge Ernani Cruz Paño is hereby annulled and set aside, and
the Temporary Restraining Order enjoining respondents from introducing evidence
obtained pursuant to the Search Warrant in the Subversive Documents Case hereby
made permanent, the personalities seized may be retained by the Constabulary Security
Group for possible introduction as evidence in Criminal Case No. SMC-1-1, pending
before Special Military Commission No. 1, without prejudice to petitioner Mila Aguilar-
Roque objecting to their relevance and asking said Commission to return to her any and
all irrelevant documents and articles." (Rollo, p. 154; 139 SCRA 165)

In their Motion for Partial Reconsideration, public respondents maintain that the
subject Search Warrant meets the standards for validity and that it should be
considered in the context of the criminal offense of Rebellion for which the Warrant was
issued, the documents to establish which are less susceptible of particularization since
the offense does not involve an isolated act or transaction.

In their own Motion for Partial Reconsideration, petitioners assail that portion of the
Decision holding that, in so far as petitioner Mila Aguilar-Roque is concerned, the search
made in her premises was incident to her arrest and could be made without a search
warrant. Petitioners submit that a warrantless search can be justified only if it is an
incident to a lawful arrest and that since Mila Aguilar was not lawfully arrested a search
without warrant could not be made.

On April 10, 1986, we required the parties to MOVE in the premises considering the
supervening events, including the change of administration that have transpired, and
Page 171 of 377

pursuant to the provisions of Section 18 of Rule 3 in so far as the public respondents


are concerned (which requires the successor official to state whether or not he maintains
the action and position taken by his predecessor-in-office).

In their Compliance, petitioners maintain that the arrest of petitioners and the search
of their premises thereafter are both illegal and that the personalities seized should be
ordered returned to their owners.

The Solicitor General, on behalf of public respondents, "in deference to the dissenting
opinion of then Supreme Court Justice (now Chief Justice) Claudio Teehankee," now
offer no further objection to a declaration that the subject search is illegal and to the
return of the seized items to the petitioners. Respondents state, however, that they
cannot agree to having the arrest of petitioners declared illegal.

The pertinent portion of the dissenting opinion referred to reads:

". . . The questioned search warrant has correctly been declared null and void in the
Court’s decision as a general warrant issued in gross violation of the constitutional
mandate that ‘the right of the people to be secure on their persons, houses, papers and
effects against unreasonable searches and seizures of whatever nature and for any
purpose shall not be violated’ (Bill of Rights, sec. 3). The Bill of Rights orders the absolute
exclusion of all illegally obtained evidence: ‘Any evidence obtained in violation of this . .
. section shall be inadmissible for any purpose in any proceeding’ (Sec. 4[21). This
constitutional mandate expressly adopting the exclusionary rule has proved by
historical experience to be the only practical means of enforcing the constitutional
injunction against unreasonable searches and seizures by outlawing all evidence
illegally seized and thereby removing the incentive on the part of state and police officers
to disregard such basic rights. What the plain language of the Constitution mandates is
beyond the power of the courts to change or modify.

"All the articles thus seized fall under the exclusionary rule totally and unqualifiedly
and cannot be used against any of the three petitioners, as held by the majority in the
recent case of Galman v. Pamaran (G.R. Nos. 71208-09, August 30, 1985) . . ."

ACCORDINGLY, considering the respective positions now taken by the parties,


petitioners’ Motion for Partial Reconsideration of this Court’s Decision of October 8,
1985 is GRANTED, and the dispositive portion thereof is hereby revised to read as
follows:
Page 172 of 377

WHEREFORE, Search Warrant No. 80-84 issued on August 6, 1984 by respondent


Executive Judge Ernani Cruz Paño is hereby annulled and set aside, and the Temporary
Restraining Order enjoining respondents from introducing evidence obtained pursuant
to the Search Warrant in the Subversive Documents Case hereby made permanent. The
personalities seized by virtue of the illegal Search Warrant are hereby ordered returned
to petitioners.

SO ORDERED.

Yap, Fernan, Narvasa, Alampay, Gutierrez, Jr., Cruz, Paras, Feliciano, Gancayco,
Padilla and Bidin, JJ., concur.

Separate Opinions

TEEHANKEE, C.J., concurring:

I felicitate my colleagues for granting petitioners’ motion for reconsideration and now
totally applying the exclusionary rule by declaring that the search and seizure of the
personalities at petitioner Mila Aguilar Roque’s dwelling at Mayon Street, Quezon City
was illegal and could not be deemed as incident to her arrest earlier on board a public
vehicle on the road away from and outside of her dwelling. Solicitor General Sedfrey A.
Ordoñez’ stand in support hereof signifies one more great step in fulfillment of the pledge
of the present government of granting full recognition and restoration of the civil and
political liberties of the people and rejecting the oppressive and repressive measures of
the past authoritarian regime.

The original majority decision citing sec. 12, Rule 126 of the Rules of Court 1 had held
that said Rule states "a general rule that, as an incident of an arrest, the place or
premises where the arrest was made can also be searched without a search warrant. In
this latter case, ‘the extent and reasonableness of the search must be decided on its
own facts and circumstances, and it has been stated that, in the application of general
rules, there is some confusion in the decisions as to what constitutes the extent of the
place or premises which may be searched.’ ‘What must be considered is the balancing
of the individual’s right to privacy and the public’s interest in the prevention of crime
and the apprehension of criminals.’"
Page 173 of 377

This pronouncement had the support of a majority of nine (9) Justices of the Court at
the time. Three (3) members had dissented, 2 while two (2) other members took no part
or reserved their vote. 3

As the petitioners stressed in their motion for reconsideration," (I)f the majority opinion
becomes settled law, the constitutional protection would become meaningless. The
military or police would no longer apply for search warrants. All that they would do is
procure a search (sic) 4 warrant or better still, a PDA, for the person whose house they
would want to search. Armed with a warrant of arrest or a PDA, the military or police
would simply wait for the person to reach his house, then arrest him. Even if the person
arrested does not resist and has in fact been taken away already from his house, under
the majority ruling, the arresting party would still have the right to search the house of
the arrestee and cart away all his things and use them as evidence against him in court.

"In such a situation, what then happens to that stringent constitutional requirement
that ‘no search warrant . . . shall issue except upon probable cause to be determined by
the judge, or such other responsible officers as may be authorized by law, after
examination under oath or affirmation of the complainant and the witnesses he may
produce, and particularly describing the place to be searched, and the things to be
seized’ and the constitutional injunction that ‘any evidence obtained in violation of this
. . . shall be inadmissible for any purpose in any proceeding’" 5

The better and established rule is a strict application of the exception provided in Rule
126, sec. 12 and that is to absolutely limit a warrantless search of a person who is
lawfully arrested to his or her person at the time of and incident to his or her arrest and
to "dangerous weapons or anything which may be used as proof of the commission of
the offense." Such warrantless search obviously cannot be made in a place other than
the place of arrest. 6

Applying the aforestated rule to this case, the undisputed fact is that petitioner Mila
Aquilar Roque was arrested at 11:30 a.m. aboard a public vehicle on the road (at Mayon
and P. Margal Sts.). The pronouncement by the majority at that time, that as an incident
to her arrest, her dwelling at 239-B Mayon Street could be searched even without a
warrant for evidence of the charges of rebellion filed against her was contrary to the
constitutional proscription, as defined by law and jurisprudence. It was tantamount to
sanctioning an untenable violation, if not nullification, of the basic constitutional right
and guarantee against unreasonable searches and seizures.

With the Court now unanimously upholding the exclusionary rule, in toto, the
constitutional mandate is given full force and effect. This constitutional mandate
Page 174 of 377

expressly adopting the exclusionary rule has proved by historical experience to be the
practical means of enforcing the constitutional injunction against unreasonable
searches and seizures by outlawing all evidence illegally seized and thereby removing
the incentive on the part of the military and police officers to disregard such basic rights.
This is of special public importance and serves as a shield in the remote provinces and
rural areas to the people who have no access to courts for prompt and immediate relief
from violations of their constitutional rights against unreasonable searches and
seizures.

In this case, the arresting CSG group of the military themselves knew that they needed
a search warrant but they obtained the void general warrant in question. Necessarily,
the seizure of documents and personal effects with such a void warrant could not be
justified "as an incident of an arrest" outside petitioner’s dwelling and the Constitution
bars their admissibility as evidence and ordains their return to petitioners.
Page 175 of 377

G.R. No. L-27360 February 28, 1968

HON. RICARDO G. PAPA, as Chief of Police of Manila; HON. JUAN PONCE


ENRILE, as Commissioner of Customs; PEDRO PACIS, as Collector of
Customs of the Port of Manila; and MARTIN ALAGAO, as Patrolman of the
Manila Police Department, petitioners, vs. REMEDIOS MAGO and HILARION
U. JARENCIO, as Presiding Judge of Branch 23, Court of First Instance of
Manila, respondents.

Office of the Solicitor General for petitioners.


Juan T. David for respondents.

ZALDIVAR, J.:

This is an original action for prohibition and certiorari, with preliminary


injunction filed by Ricardo Papa, Chief of Police of Manila; Juan once Enrile,
Commissioner of Customs; Pedro Pacis, Collector of Customs of the Port of Manila; and
Martin Alagao, a patrolman of the Manila Police Department, against Remedios Mago
and Hon. Hilarion Jarencio, Presiding Judge of Branch 23 of the Court of First Instance
of Manila, praying for the annulment of the order issued by respondent Judge in Civil
Case No. 67496 of the Court of First Instance of Manila under date of March 7, 1967,
which authorized the release under bond of certain goods which were seized and held
by petitioners in connection with the enforcement of the Tariff and Customs Code, but
which were claimed by respondent Remedios Mago, and to prohibit respondent Judge
from further proceeding in any manner whatsoever in said Civil Case No. 67496.
Pending the determination of this case this Court issued a writ of preliminary injunction
restraining the respondent Judge from executing, enforcing and/or implementing the
questioned order in Civil Case No. 67496 and from proceeding with said case.

Petitioner Martin Alagao, head of the counter-intelligence unit of the Manila Police
Department, acting upon a reliable information received on November 3, 1966 to the
effect that a certain shipment of personal effects, allegedly misdeclared and
undervalued, would be released the following day from the customs zone of the port of
Manila and loaded on two trucks, and upon orders of petitioner Ricardo Papa, Chief of
Police of Manila and a duly deputized agent of the Bureau of Customs, conducted
surveillance at gate No. 1 of the customs zone. When the trucks left gate No. 1 at about
4:30 in the afternoon of November 4, 1966, elements of the counter-intelligence unit
went after the trucks and intercepted them at the Agrifina Circle, Ermita, Manila. The
load of the two trucks consisting of nine bales of goods, and the two trucks, were seized
on instructions of the Chief of Police. Upon investigation, a person claimed ownership
of the goods and showed to the policemen a "Statement and Receipts of Duties Collected
Page 176 of 377

in Informal Entry No. 147-5501", issued by the Bureau of Customs in the name of a
certain Bienvenido Naguit.

Claiming to have been prejudiced by the seizure and detention of the two trucks
and their cargo, Remedios Mago and Valentin B. Lanopa filed with the Court of First
Instance of Manila a petition "for mandamus with restraining order or preliminary
injunction, docketed as Civil Case No. 67496, alleging, among others, that Remedios
Mago was the owner of the goods seized, having purchased them from the Sta. Monica
Grocery in San Fernando, Pampanga; that she hired the trucks owned by Valentin
Lanopa to transport, the goods from said place to her residence at 1657 Laon Laan St.,
Sampaloc, Manila; that the goods were seized by members of the Manila Police
Department without search warrant issued by a competent court; that anila Chief of
Police Ricardo Papa denied the request of counsel for Remedios Mago that the bales be
not opened and the goods contained therein be not examined; that then Customs
Commissioner Jacinto Gavino had illegally assigned appraisers to examine the goods
because the goods were no longer under the control and supervision of the
Commissioner of Customs; that the goods, even assuming them to have been
misdeclared and, undervalued, were not subject to seizure under Section 2531 of the
Tariff and Customs Code because Remedios Mago had bought them from another person
without knowledge that they were imported illegally; that the bales had not yet been
opened, although Chief of Police Papa had arranged with the Commissioner of Customs
regarding the disposition of the goods, and that unless restrained their constitutional
rights would be violated and they would truly suffer irreparable injury. Hence, Remedios
Mago and Valentin Lanopa prayed for the issuance of a restraining order, ex parte,
enjoining the above-named police and customs authorities, or their agents, from
opening the bales and examining the goods, and a writ of mandamus for the return of
the goods and the trucks, as well as a judgment for actual, moral and exemplary
damages in their favor.

On November 10, 1966, respondent Judge Hilarion Jarencio issued an order ex


parte restraining the respondents in Civil Case No. 67496 — now petitioners in the
instant case before this Court — from opening the nine bales in question, and at the
same time set the hearing of the petition for preliminary injunction on November 16,
1966. However, when the restraining order was received by herein petitioners, some
bales had already been opened by the examiners of the Bureau of Customs in the
presence of officials of the Manila Police Department, an assistant city fiscal and a
representative of herein respondent Remedios Mago.

Under date of November 15, 1966, Remedios Mago filed an amended petition in
Civil Case No. 67496, including as party defendants Collector of Customs Pedro Pacis
of the Port of Manila and Lt. Martin Alagao of the Manila Police Department. Herein
petitioners (defendants below) filed, on November 24, 1966, their "Answer with
Opposition to the Issuance of a Writ of Preliminary Injunction", denying the alleged
Page 177 of 377

illegality of the seizure and detention of the goods and the trucks and of their other
actuations, and alleging special and affirmative defenses, to wit: that the Court of First
Instance of Manila had no jurisdiction to try the case; that the case fell within the
exclusive jurisdiction of the Court of Tax Appeals; that, assuming that the court had
jurisdiction over the case, the petition stated no cause of action in view of the failure of
Remedios Mago to exhaust the administrative remedies provided for in the Tariff and
Customs Code; that the Bureau of Customs had not lost jurisdiction over the goods
because the full duties and charges thereon had not been paid; that the members of the
Manila Police Department had the power to make the seizure; that the seizure was not
unreasonable; and the persons deputized under Section 2203 (c) of the Tariff and
Customs Code could effect search, seizures and arrests in inland places in connection
with the enforcement of the said Code. In opposing the issuance of the writ of
preliminary injunction, herein petitioners averred in the court below that the writ could
not be granted for the reason that Remedios Mago was not entitled to the main reliefs
she prayed for; that the release of the goods, which were subject to seizure proceedings
under the Tariff and Customs Code, would deprive the Bureau of Customs of the
authority to forfeit them; and that Remedios Mago and Valentin Lanopa would not suffer
irreparable injury. Herein petitioners prayed the court below for the lifting of the
restraining order, for the denial of the issuance of the writ of preliminary injunction,
and for the dismissal of the case.

At the hearing on December 9, 1966, the lower Court, with the conformity of the
parties, ordered that an inventory of the goods be made by its clerk of court in the
presence of the representatives of the claimant of the goods, the Bureau of Customs,
and the Anti-Smuggling Center of the Manila Police Department. On December 13,
1966, the above-named persons filed a "Compliance" itemizing the contents of the nine
bales.

Herein respondent Remedios Mago, on December 23, 1966, filed an ex parte


motion to release the goods, alleging that since the inventory of the goods seized did not
show any article of prohibited importation, the same should be released as per
agreement of the patties upon her posting of the appropriate bond that may be
determined by the court. Herein petitioners filed their opposition to the motion, alleging
that the court had no jurisdiction to order the release of the goods in view of the fact
that the court had no jurisdiction over the case, and that most of the goods, as shown
in the inventory, were not declared and were, therefore, subject to forfeiture. A
supplemental opposition was filed by herein petitioners on January 19, 1967, alleging
that on January 12, 1967 seizure proceedings against the goods had been instituted by
the Collector of Customs of the Port of Manila, and the determination of all questions
affecting the disposal of property proceeded against in seizure and forfeiture proceedings
should thereby be left to the Collector of Customs. On January 30, 1967, herein
petitioners filed a manifestation that the estimated duties, taxes and other charges due
on the goods amounted to P95,772.00. On February 2, 1967, herein respondent
Page 178 of 377

Remedios Mago filed an urgent manifestation and reiteration of the motion for the
release under bond of the goods.

On March 7, 1967, the respondent Judge issued an order releasing the goods to
herein respondent Remedios Mago upon her filing of a bond in the amount of
P40,000.00, and on March 13, 1967, said respondent filed the corresponding bond.

On March 13, 1967, herein petitioner Ricardo Papa, on his own behalf, filed a
motion for reconsideration of the order of the court releasing the goods under bond,
upon the ground that the Manila Police Department had been directed by the Collector
of Customs of the Port of Manila to hold the goods pending termination of the seizure
proceedings.

Without waiting for the court's action on the motion for reconsideration, and
alleging that they had no plain, speedy and adequate remedy in the ordinary course of
law, herein petitioners filed the present action for prohibition and certiorari with
preliminary injunction before this Court. In their petition petitioners alleged, among
others, that the respondent Judge acted without jurisdiction in ordering the release to
respondent Remedios Mago of the disputed goods, for the following reasons: (1) the
Court of First Instance of Manila, presided by respondent Judge, had no jurisdiction
over the case; (2) respondent Remedios Mago had no cause of action in Civil Case No.
67496 of the Court of First Instance of Manila due to her failure to exhaust all
administrative remedies before invoking judicial intervention; (3) the Government was
not estopped by the negligent and/or illegal acts of its agent in not collecting the correct
taxes; and (4) the bond fixed by respondent Judge for the release of the goods was
grossly insufficient.

In due time, the respondents filed their answer to the petition for prohibition and
certiorari in this case. In their answer, respondents alleged, among others: (1) that it
was within the jurisdiction of the lower court presided by respondent Judge to hear and
decide Civil Case No. 67496 and to issue the questioned order of March 7, 1967, because
said Civil Case No. 67496 was instituted long before seizure, and identification
proceedings against the nine bales of goods in question were instituted by the Collector
of Customs; (2) that petitioners could no longer go after the goods in question after the
corresponding duties and taxes had been paid and said goods had left the customs
premises and were no longer within the control of the Bureau of Customs; (3) that
respondent Remedios Mago was purchaser in good faith of the goods in question so that
those goods can not be the subject of seizure and forfeiture proceedings; (4) that the
seizure of the goods was affected by members of the Manila Police Department at a place
outside control of jurisdiction of the Bureau of Customs and affected without any search
warrant or a warrant of seizure and detention; (5) that the warrant of seizure and
Page 179 of 377

detention subsequently issued by the Collector of Customs is illegal and


unconstitutional, it not being issued by a judge; (6) that the seizing officers have no
authority to seize the goods in question because they are not articles of prohibited
importation; (7) that petitioners are estopped to institute the present action because
they had agreed before the respondent Judge that they would not interpose any
objection to the release of the goods under bond to answer for whatever duties and taxes
the said goods may still be liable; and (8) that the bond for the release of the goods was
sufficient.

The principal issue in the instant case is whether or not, the respondent Judge
had acted with jurisdiction in issuing the order of March 7, 1967 releasing the goods in
question.

The Bureau of Customs has the duties, powers and jurisdiction, among others,
(1) to assess and collect all lawful revenues from imported articles, and all other dues,
fees, charges, fines and penalties, accruing under the tariff and customs laws; (2) to
prevent and suppress smuggling and other frauds upon the customs; and (3) to enforce
tariff and customs laws. 1 The goods in question were imported from Hongkong, as
shown in the "Statement and Receipts of Duties Collected on Informal Entry". 2 As long
as the importation has not been terminated the imported goods remain under the
jurisdiction of the Bureau of customs. Importation is deemed terminated only upon the
payment of the duties, taxes and other charges upon the articles, or secured to be paid,
at the port of entry and the legal permit for withdrawal shall have been granted. 3 The
payment of the duties, taxes, fees and other charges must be in full. 4

The record shows, by comparing the articles and duties stated in the aforesaid
"Statement and Receipts of Duties Collected on Informal Entry" with the manifestation
of the Office of the Solicitor General 5 wherein it is stated that the estimated duties,
taxes and other charges on the goods subject of this case amounted to P95,772.00 as
evidenced by the report of the appraiser of the Bureau of Customs, that the duties, taxes
and other charges had not been paid in full. Furthermore, a comparison of the goods on
which duties had been assessed, as shown in the "Statement and Receipts of Duties
Collected on Informal Entry" and the "compliance" itemizing the articles found in the
bales upon examination and inventory, 6 shows that the quantity of the goods was
underdeclared, presumably to avoid the payment of duties thereon. For example, Annex
B (the statement and receipts of duties collected) states that there were 40 pieces of
ladies' sweaters, whereas Annex H (the inventory contained in the "compliance") states
that in bale No. 1 alone there were 42 dozens and 1 piece of ladies' sweaters of assorted
colors; in Annex B, only 100 pieces of watch bands were assessed, but in Annex H, there
were in bale No. 2, 209 dozens and 5 pieces of men's metal watch bands (white) and 120
dozens of men's metal watch band (gold color), and in bale No. 7, 320 dozens of men's
metal watch bands (gold color); in Annex B, 20 dozens only of men's handkerchief were
declared, but in Annex H it appears that there were 224 dozens of said goods in bale
Page 180 of 377

No. 2, 120 dozens in bale No. 6, 380 dozens in bale No. 7, 220 dozens in bale No. 8, and
another 200 dozens in bale No. 9. The articles contained in the nine bales in question,
were, therefore, subject to forfeiture under Section 2530, pars. e and m, (1), (3), (4), and
(5) of the Tariff and Customs Code. And this Court has held that merchandise, the
importation of which is effected contrary to law, is subject to forfeiture, 7 and that goods
released contrary to law are subject to seizure and forfeiture. 8

Even if it be granted, arguendo, that after the goods in question had been brought
out of the customs area the Bureau of Customs had lost jurisdiction over the same,
nevertheless, when said goods were intercepted at the Agrifina Circle on November 4,
1966 by members of the Manila Police Department, acting under directions and orders
of their Chief, Ricardo C. Papa, who had been formally deputized by the Commissioner
of Customs, 9 the Bureau of Customs had regained jurisdiction and custody of the
goods. Section 1206 of the Tariff and Customs Code imposes upon the Collector of
Customs the duty to hold possession of all imported articles upon which duties, taxes,
and other charges have not been paid or secured to be paid, and to dispose of the same
according to law. The goods in question, therefore, were under the custody and at the
disposal of the Bureau of Customs at the time the petition for mandamus, docketed as
Civil Case No. 67496, was filed in the Court of First Instance of Manila on November 9,
1966. The Court of First Instance of Manila, therefore, could not exercise jurisdiction
over said goods even if the warrant of seizure and detention of the goods for the purposes
of the seizure and forfeiture proceedings had not yet been issued by the Collector of
Customs.

The ruling in the case of "Alberto de Joya, et al. v. Hon. Gregorio Lantin, et al.,"
G.R. No. L-24037, decided by this Court on April 27, 1967, is squarely applicable to the
instant case. In the De Joya case, it appears that Francindy Commercial of Manila
bought from Ernerose Commercial of Cebu City 90 bales of assorted textiles and rags,
valued at P117,731.00, which had been imported and entered thru the port of Cebu.
Ernerose Commercial shipped the goods to Manila on board an inter-island vessel.
When the goods where about to leave the customs premises in Manila, on October 6,
1964, the customs authorities held them for further verification, and upon examination
the goods were found to be different from the declaration in the cargo manifest of the
carrying vessel. Francindy Commercial subsequently demanded from the customs
authorities the release of the goods, asserting that it is a purchaser in good faith of those
goods; that a local purchaser was involved so the Bureau of Customs had no right to
examine the goods; and that the goods came from a coastwise port. On October 26,
1964, Francindy Commercial filed in the Court of First Instance of Manila a petition for
mandamus against the Commissioner of Customs and the Collector of Customs of the
port of Manila to compel said customs authorities to release the goods.

Francindy Commercial alleged in its petition for mandamus that the Bureau of
Customs had no jurisdiction over the goods because the same were not imported to the
Page 181 of 377

port of Manila; that it was not liable for duties and taxes because the transaction was
not an original importation; that the goods were not in the hands of the importer nor
subject to importer's control, nor were the goods imported contrary to law with its
(Francindy Commercial's) knowledge; and that the importation had been terminated.
On November 12, 1964, the Collector of Customs of Manila issued a warrant of seizure
and identification against the goods. On December 3, 1964, the Commissioner of
Customs and the Collector of Customs, as respondents in the mandamus case, filed a
motion to dismiss the petition on the grounds of lack of jurisdiction, lack of cause of
action, and in view of the pending seizure and forfeiture proceedings. The Court of First
Instance held resolution on the motion to dismiss in abeyance pending decision on the
merits. On December 14, 1964, the Court of First Instance of Manila issued a preventive
and mandatory injunction, on prayer by Francindy Commercial, upon a bond of
P20,000.00. The Commissioner of Customs and the Collector of Customs sought the
lifting of the preliminary and mandatory injunction, and the resolution of their motion
to dismiss. The Court of First Instance of Manila, however, on January 12, 1965, ordered
them to comply with the preliminary and mandatory injunction, upon the filing by
Francindy Commercial of an additional bond of P50,000.00. Said customs authorities
thereupon filed with this Court, on January 14, 1965, a petition for certiorari and
prohibition with preliminary injunction. In resolving the question raised in that case,
this Court held:

This petition raises two related issues: first, has the Customs bureau jurisdiction
to seize the goods and institute forfeiture proceedings against them? and (2) has the
Court of First Instance jurisdiction to entertain the petition for mandamus to compel
the Customs authorities to release the goods?

Francindy Commercial contends that since the petition in the Court of first
Instance was filed (on October 26, 1964) ahead of the issuance of the Customs warrant
of seizure and forfeiture (on November 12, 1964),the Customs bureau should yield the
jurisdiction of the said court.

The record shows, however, that the goods in question were actually seized on
October 6, 1964, i.e., before Francindy Commercial sued in court. The purpose of the
seizure by the Customs bureau was to verify whether or not Custom duties and taxes
were paid for their importation. Hence, on December 23, 1964, Customs released 22
bales thereof, for the same were found to have been released regularly from the Cebu
Port (Petition Annex "L"). As to goods imported illegally or released irregularly from
Customs custody, these are subject to seizure under Section 2530 m. of the Tariff and
Customs Code (RA 1957).
Page 182 of 377

The Bureau of Customs has jurisdiction and power, among others to collect
revenues from imported articles, fines and penalties and suppress smuggling and other
frauds on customs; and to enforce tariff and customs laws (Sec. 602, Republic Act 1957).

The goods in question are imported articles entered at the Port of Cebu. Should
they be found to have been released irregularly from Customs custody in Cebu City,
they are subject to seizure and forfeiture, the proceedings for which comes within the
jurisdiction of the Bureau of Customs pursuant to Republic Act 1937.

Said proceeding should be followed; the owner of the goods may set up defenses
therein (Pacis v. Averia, L-22526, Nov. 20, 1966.) From the decision of the Commissioner
of Customs appeal lies to the Court of Tax Appeals, as provided in Sec. 2402 of Republic
Act 1937 and Sec. 11 of Republic Act, 1125. To permit recourse to the Court of First
Instance in cases of seizure of imported goods would in effect render ineffective the
power of the Customs authorities under the Tariff and Customs Code and deprive the
Court of Tax Appeals of one of its exclusive appellate jurisdictions. As this Court has
ruled in Pacis v. Averia, supra, Republic Acts 1937 and 1125 vest jurisdiction over
seizure and forfeiture proceedings exclusively upon the Bureau of Customs and the
Court of Tax Appeals. Such law being special in nature, while the Judiciary Act defining
the jurisdiction of Courts of First Instance is a general legislation, not to mention that
the former are later enactments, the Court of First Instance should yield to the
jurisdiction of the Customs authorities.

It is the settled rule, therefore, that the Bureau of Customs acquires exclusive
jurisdiction over imported goods, for the purposes of enforcement of the customs laws,
from the moment the goods are actually in its possession or control, even if no warrant
of seizure or detention had previously been issued by the Collector of Customs in
connection with seizure and forfeiture proceedings. In the present case, the Bureau of
Customs actually seized the goods in question on November 4, 1966, and so from that
date the Bureau of Customs acquired jurisdiction over the goods for the purposes of the
enforcement of the tariff and customs laws, to the exclusion of the regular courts. Much
less then would the Court of First Instance of Manila have jurisdiction over the goods in
question after the Collector of Customs had issued the warrant of seizure and detention
on January 12, 1967. 10 And so, it cannot be said, as respondents contend, that the
issuance of said warrant was only an attempt to divest the respondent Judge of
jurisdiction over the subject matter of the case. The court presided by respondent Judge
did not acquire jurisdiction over the goods in question when the petition for mandamus
was filed before it, and so there was no need of divesting it of jurisdiction. Not having
acquired jurisdiction over the goods, it follows that the Court of First Instance of Manila
had no jurisdiction to issue the questioned order of March 7, 1967 releasing said goods.
Page 183 of 377

Respondents also aver that petitioner Martin Alagao, an officer of the Manila
Police Department, could not seize the goods in question without a search warrant. This
contention cannot be sustained. The Chief of the Manila Police Department, Ricardo G.
Papa, having been deputized in writing by the Commissioner of Customs, could, for the
purposes of the enforcement of the customs and tariff laws, effect searches, seizures,
and arrests, 11 and it was his duty to make seizure, among others, of any cargo, articles
or other movable property when the same may be subject to forfeiture or liable for any
fine imposed under customs and tariff laws. 12 He could lawfully open and examine any
box, trunk, envelope or other container wherever found when he had reasonable cause
to suspect the presence therein of dutiable articles introduced into the Philippines
contrary to law; and likewise to stop, search and examine any vehicle, beast or person
reasonably suspected of holding or conveying such article as aforesaid. 13 It cannot be
doubted, therefore, that petitioner Ricardo G. Papa, Chief of Police of Manila, could
lawfully effect the search and seizure of the goods in question. The Tariff and Customs
Code authorizes him to demand assistance of any police officer to effect said search and
seizure, and the latter has the legal duty to render said assistance. 14 This was what
happened precisely in the case of Lt. Martin Alagao who, with his unit, made the search
and seizure of the two trucks loaded with the nine bales of goods in question at the
Agrifina Circle. He was given authority by the Chief of Police to make the interception of
the cargo. 15

Petitioner Martin Alagao and his companion policemen had authority to effect the
seizure without any search warrant issued by a competent court. The Tariff and
Customs Code does not require said warrant in the instant case. The Code authorizes
persons having police authority under Section 2203 of the Tariff and Customs Code to
enter, pass through or search any land, inclosure, warehouse, store or building, not
being a dwelling house; and also to inspect, search and examine any vessel or aircraft
and any trunk, package, or envelope or any person on board, or to stop and search and
examine any vehicle, beast or person suspected of holding or conveying any dutiable or
prohibited article introduced into the Philippines contrary to law, without mentioning
the need of a search warrant in said cases. 16 But in the search of a dwelling house,
the Code provides that said "dwelling house may be entered and searched only upon
warrant issued by a judge or justice of the peace. . . ." 17 It is our considered view,
therefor, that except in the case of the search of a dwelling house, persons exercising
police authority under the customs law may effect search and seizure without a search
warrant in the enforcement of customs laws.

Our conclusion finds support in the case of Carroll v. United States, 39 A.L.R.,
790, 799, wherein the court, considering a legal provision similar to Section 2211 of the
Philippine Tariff and Customs Code, said as follows:

Thus contemporaneously with the adoption of the 4th Amendment, we find in the
first Congress, and in the following second and fourth Congresses, a difference made as
Page 184 of 377

to the necessity for a search warrant between goods subject to forfeiture, when
concealed in a dwelling house of similar place, and like goods in course of transportation
and concealed in a movable vessel, where readily they could be put out of reach of a
search warrant. . . .

Again, by the 2d section of the Act of March 3, 1815 (3 Stat. at L.231, 232, chap.
94), it was made lawful for customs officers not only to board and search vessels within
their own and adjoining districts, but also to stop, search and examine any vehicle,
beast or person on which or whom they should suspect there was merchandise which
was subject to duty, or had been introduced into the United States in any manner
contrary to law, whether by the person in charge of the vehicle or beast or otherwise,
and if they should find any goods, wares, or merchandise thereon, which they had
probably cause to believe had been so unlawfully brought into the country, to seize and
secure the same, and the vehicle or beast as well, for trial and forfeiture. This Act was
renewed April 27, 1816 (3 Sta. at L. 315, chap. 100), for a year and expired. The Act of
February 28, 1865, revived § 2 of the Act of 1815, above described, chap. 67, 13 Stat.
at L. 441. The substance of this section was re-enacted in the 3d section of the Act of
July 18, 1866, chap. 201, 14 Stat. at L. 178, and was thereafter embodied in the Revised
Statutes as § 3061, Comp. Stat. § 5763, 2 Fed. Stat. Anno. 2d ed. p. 1161. Neither §
3061 nor any of its earlier counterparts has ever been attacked as unconstitutional.
Indeed, that section was referred to and treated as operative by this court in Von
Cotzhausen v. Nazro, 107 U.S. 215, 219, 27 L. ed. 540, 541, 2 Sup. Ct. Rep. 503. . . .

In the instant case, we note that petitioner Martin Alagao and his companion
policemen did not have to make any search before they seized the two trucks and their
cargo. In their original petition, and amended petition, in the court below Remedios
Mago and Valentin Lanopa did not even allege that there was a search. 18 All that they
complained of was,

That while the trucks were on their way, they were intercepted without any search
warrant near the Agrifina Circle and taken to the Manila Police Department, where they
were detained.

But even if there was a search, there is still authority to the effect that no search
warrant would be needed under the circumstances obtaining in the instant case. Thus,
it has been held that:

The guaranty of freedom from unreasonable searches and seizures is construed


as recognizing a necessary difference between a search of a dwelling house or other
structure in respect of which a search warrant may readily be obtained and a search of
Page 185 of 377

a ship, motorboat, wagon, or automobile for contraband goods, where it is not


practicable to secure a warrant because the vehicle can be quickly moved out of the
locality or jurisdiction in which the warrant must be sought. (47 Am. Jur., pp. 513-514,
citing Carroll v. United States, 267 U.S. 132, 69 L. ed., 543, 45 S. Ct., 280, 39 A.L.R.,
790; People v. Case, 320 Mich., 379, 190 N.W., 389, 27 A.L.R., 686.)

In the case of People v. Case (320 Mich., 379, 190 N.W., 389, 27 A.L.R., 686), the
question raised by defendant's counsel was whether an automobile truck or an
automobile could be searched without search warrant or other process and the goods
therein seized used afterwards as evidence in a trial for violation of the prohibition laws
of the State. Same counsel contended the negative, urging the constitutional provision
forbidding unreasonable searches and seizures. The Court said:

. . . Neither our state nor the Federal Constitution directly prohibits search and
seizure without a warrant, as is sometimes asserted. Only "unreasonable" search and
seizure is forbidden. . . .

. . . The question whether a seizure or a search is unreasonable in the language


of the Constitution is a judicial and not a legislative question; but in determining
whether a seizure is or is not unreasonable, all of the circumstances under which it is
made must be looked to.

The automobile is a swift and powerful vehicle of recent development, which has
multiplied by quantity production and taken possession of our highways in battalions
until the slower, animal-drawn vehicles, with their easily noted individuality, are rare.
Constructed as covered vehicles to standard form in immense quantities, and with a
capacity for speed rivaling express trains, they furnish for successful commission of
crime a disguising means of silent approach and swift escape unknown in the history
of the world before their advent. The question of their police control and reasonable
search on highways or other public places is a serious question far deeper and broader
than their use in so-called "bootleging" or "rum running," which is itself is no small
matter. While a possession in the sense of private ownership, they are but a vehicle
constructed for travel and transportation on highways. Their active use is not in homes
or on private premises, the privacy of which the law especially guards from search and
seizure without process. The baffling extent to which they are successfully utilized to
facilitate commission of crime of all degrees, from those against morality, chastity, and
decency, to robbery, rape, burglary, and murder, is a matter of common knowledge.
Upon that problem a condition, and not a theory, confronts proper administration of
our criminal laws. Whether search of and seizure from an automobile upon a highway
or other public place without a search warrant is unreasonable is in its final analysis to
Page 186 of 377

be determined as a judicial question in view of all the circumstances under which it is


made.

Having declared that the seizure by the members of the Manila Police Department
of the goods in question was in accordance with law and by that seizure the Bureau of
Customs had acquired jurisdiction over the goods for the purpose of the enforcement of
the customs and tariff laws, to the exclusion of the Court of First Instance of Manila,
We have thus resolved the principal and decisive issue in the present case. We do not
consider it necessary, for the purposes of this decision, to discuss the incidental issues
raised by the parties in their pleadings.

WHEREFORE, judgment is hereby rendered, as follows:

(a) Granting the writ of certiorari and prohibition prayed for by petitioners;

(b) Declaring null and void, for having been issued without jurisdiction, the order
of respondent Judge Hilarion U. Jarencio, dated March 7, 1967, in Civil Code No. 67496
of the Court of First Instance of Manila;

(c) Declaring permanent the preliminary injunction issued by this Court on March
31, 1967 restraining respondent Judge from executing, enforcing and/or implementing
his order of March 7, 1967 in Civil Case No. 67496 of the Court of First Instance of
Manila, and from proceeding in any manner in said case;

(d) Ordering the dismissal of Civil Case No. 67496 of the Court of First Instance
of Manila; and1äwphï1.ñët

(e) Ordering the private respondent, Remedios Mago, to pay the costs.

It is so ordered.

Concepcion, C.J., Reyes, J.B.L., Dizon, Makalintal, Bengzon, J.P., Sanchez, Castro,
Angeles and Fernando, JJ., concur.1äwphï1.ñët
Page 187 of 377

G.R. No. L-74869 July 6, 1988

PEOPLE OF THE PHILIPPINES, Plaintiff-Appellee, v. IDEL AMINNUDIN y


AHNI, Defendant-Appellant.

The Solicitor General, for Plaintiff-Appellee.


Herminio T. Llariza counsel de-officio,, for Defendant-Appellant.

SYLLABUS
1. CONSTITUTIONAL LAW; BILL OF RIGHTS; RIGHTS AGAINST UNREASONABLE
SEARCHES AND SEIZURES; WARRANTLESS ARREST AND SEIZURE BASED ON AN
INFORMER’S TIP, AT A TIME WHEN ACCUSED WAS NOT COMMITTING A CRIME,
ILLEGAL; EVIDENCE OBTAINED, INADMISSIBLE. — Where it is not disputed that the
PC officers had no warrant when they arrested Aminnudin while he was descending the
gangplank of the M/V Wilcon 9 and seized the bag he was carrying, and that their only
justification was the tip they had earlier received from a reliable and regular informer
who reported to them that Aminnudin was arriving in Iloilo by boat with marijuana, the
search was not an incident of a lawful arrest because there was no warrant of arrest
and warrantless arrest did not come under the exceptions allowed by the Rules of Court.
Hence, the warrantless search was also illegal and the evidence obtained was
inadmissible.

2. ID.; ID.; ID.; ID.; NO URGENCY COULD BE INVOKED IN PRESENT CASE TO


DISPENSE WITH OBTENTION OF ARREST AND SEARCH WARRANT. — The present
case presented no such urgency. From the conflicting declarations of the PC witnesses,
it is clear that they had at least two days within which they could have obtained a
warrant to arrest and search Aminnudin who was coming to Iloilo on the M/V Wilcon
9. His name was known. The vehicle was identified. The date of its arrival was certain.
And from the information they had received, they could have persuaded a judge that
there was probable cause, indeed, to justify the issuance of a warrant. Yet they did
nothing. No effort was made to comply with the law. The Bill of Rights was ignored
altogether because the PC lieutenant who was the head of the arresting team, had
determined on his own authority that a "search warrant was not necessary."

3. ID.; ID.; ID.; ID.; ACCUSED IN CASE AT BAR WAS NOT COMMITTING A CRIME
WHEN HE WAS ARRESTED. — In the case at bar, the accused-appellant was not, at
the moment of his arrest, committing a crime nor was it shown that he was about to do
so or that he had just done so. What he was doing was descending the gangplank of the
M/V Wilcon 9 and there was no outward indication that called for his arrest. To all
appearances, he was like any of the other passengers innocently disembarking from the
vessel. It was only when the informer pointed to him as the carrier of the marijuana that
Page 188 of 377

he suddenly became suspect and so subject to apprehension. It was the furtive finger
that triggered his arrest. The identification by the informer was the probable cause as
determined by the officers (and not a judge) that authorized them to pounce upon
Aminnudin and immediately arrest him.

AQUINO, J., Dissenting:

1. CONSTITUTIONAL LAW; BILL OF RIGHTS; RIGHT AGAINST UNREASONABLE


SEARCHES AND SEIZURES; ARREST AT TIME OF COMMISSION OF CRIME IS
LAWFUL; SEARCH LIKEWISE LAWFUL.— I hold that the accused was caught in
flagrante, for he was carrying marijuana leaves in his bag at the moment of his arrest.
He was not "innocently disembarking from the vessel." The unauthorized transportation
of marijuana (Indian hemp), which is a prohibited drug, is a crime. (Sec. 4, Rep. Act No.
6425). Since he was committing a crime, his arrest could be lawfully effected without a
warrant (Sec. 6-a, Rule 113, Rules of Court), and the search of his bag (which yielded
the marijuana leaves) without a search warrant was also lawful (Sec. 12, Rule 126, Rules
of Court).

CRUZ, J.:

The accused-appellant claimed his business was selling watches but he was nonetheless
arrested, tried and found guilty of illegally transporting marijuana. The trial court,
disbelieving him, held it was high time to put him away and sentenced him to life
imprisonment plus a fine of P20,000.00. 1

Idel Aminnudin was arrested on June 25, 1984, shortly after disembarking from the
M/V Wilcon 9 at about 8:30 in the evening, in Iloilo City. The PC officers who were in
fact waiting for him simply accosted him, inspected his bag and finding what looked
liked marijuana leaves took him to their headquarters for investigation. The two bundles
of suspect articles were confiscated from him and later taken to the NBI laboratory for
examination. When they were verified as marijuana leaves, an information for violation
of the Dangerous Drugs Act was filed against him. 2 Later, the information was amended
to include Farida Ali y Hassen, who had also been arrested with him that same evening
and likewise investigated. 3 Both were arraigned and pleaded not guilty. 4
Subsequently, the fiscal filed a motion to dismiss the charge against Ali on the basis of
a sworn statement of the arresting officers absolving her after a "thorough investigation."
5 The motion was granted, and trial proceeded only against the accused-appellant, who
was eventually convicted. 6
Page 189 of 377

According to the prosecution, the PC officers had earlier received a tip from one of their
informers that the accused-appellant was on board a vessel bound for Iloilo City and
was carrying marijuana. 7 He was identified by name. 8 Acting on this tip, they waited
for him in the evening of June 25, 1984, and approached him as he descended from the
gangplank after the informer had pointed to him. 9 They detained him and inspected
the bag he was carrying. It was found to contain three kilos of what were later analyzed
as marijuana leaves by an NBI forensic examiner, 10 who testified that she conducted
microscopic, chemical and chromatographic tests on them. On the basis of this finding,
the corresponding charge was then filed against Aminnudin.

In his defense, Aminnudin disclaimed the marijuana, averring that all he had in his bag
was his clothing consisting of a jacket, two shirts and two pairs of pants. 11 He alleged
that he was arbitrarily arrested and immediately handcuffed. His bag was confiscated
without a search warrant. At the PC headquarters, he was manhandled to force him to
admit he was carrying the marijuana, the investigator hitting him with a piece of wood
in the chest and arms even as he parried the blows while he was still handcuffed. 12 He
insisted he did not even know what marijuana looked like and that his business was
selling watches and sometimes cigarettes. 13 He also argued that the marijuana he was
alleged to have been carrying was not properly identified and could have been any of
several bundles kept in the stock room of the PC headquarters. 14

The trial court was unconvinced, noting from its own examination of the accused that
he claimed to have come to Iloilo City to sell watches but carried only two watches at
the time, traveling from Jolo for that purpose and spending P107.00 for fare, not to
mention his other expenses. 15 Aminnudin testified that he kept the two watches in a
secret pocket below his belt but, strangely, they were not discovered when he was bodily
searched by the arresting officers nor were they damaged as a result of his manhandling.
16 He also said he sold one of the watches for P400.00 and gave away the other,
although the watches belonged not to him but to his cousin, 17 to a friend whose full
name he said did not even know. 18 The trial court also rejected his allegations of
maltreatment, observing that he had not sufficiently proved the injuries sustained by
him. 19

There is no justification to reverse these factual findings, considering that it was the
trial judge who had immediate access to the testimony of the witnesses and had the
opportunity to weigh their credibility on the stand. Nuances of tone or voice, meaningful
pauses and hesitation, flush of face and dart of eyes, which may reveal the truth or
expose the lie, are not described in the impersonal record. But the trial judge sees all of
this, discovering for himself the truant fact amidst the falsities.
Page 190 of 377

The only exception we may make in this case is the trial court’s conclusion that the
accused-appellant was not really beaten up because he did not complain about it later
nor did he submit to a medical examination. That is hardly fair or realistic. It is possible
Aminnudin never had that opportunity as he was at that time under detention by the
PC authorities and in fact has never been set free since he was arrested in 1984 and up
to the present. No bail has been allowed for his release.

There is one point that deserves closer examination, however, and it is Aminnudin’s
claim that he was arrested and searched without warrant, making the marijuana
allegedly found in his possession inadmissible in evidence against him under the Bill of
Rights. The decision did not even discuss this point. For his part, the Solicitor General
dismissed this after an all-too-short argument that the arrest of Aminnudin was valid
because it came under Rule 113, Section 6(b) of the Rules of Court on warrantless
arrests. This made the search also valid as incidental to a lawful arrest.

It is not disputed, and in fact it is admitted by the PC officers who testified for the
prosecution, that they had no warrant when they arrested Aminnudin and seized the
bag he was carrying. Their only justification was the tip they had earlier received from
a reliable and regular informer who reported to them that Aminnudin was arriving in
Iloilo by boat with marijuana. Their testimony varies as to the time they received the tip,
one saying it was two days before the arrest, 20 another two weeks 21 and a third "weeks
before June 25." 22 On this matter, we may prefer the declaration of the chief of the
arresting team, Lt. Cipriano Querol, Jr., who testified as follow:

"Q You mentioned an intelligence report, you mean with respect to the coming of Idel
Aminnudin on June 25, 1984?

"A Yes, sir.

"Q When did you receive this intelligence report?

"A Two days before June 25, 1984 and it was supported by reliable sources.

"Q Were you informed of the coming of the Wilcon 9 and the possible trafficking of
marijuana leaves on that date?
Page 191 of 377

"A Yes, sir, two days before June 25,1984 when we received this information from that
particular informer, prior to June 25, 1984 we have already reports of the particular
operation which was being participated by Idel Aminnudin.

"Q You said you received an intelligence report two days before June 25,1984 with
respect to the coming of Wilcon 9?

"A Yes, sir.

"Q Did you receive any other report aside from this intelligence report?

"A Well, I have received also other reports but not pertaining to the coming of Wilcon 9.
For instance, report of illegal gambling operation.

"COURT:

"Q Previous to that particular information which you said two days before June 25,
1984, did you also receive any report regarding the activities of Idel Aminnudin?

"A Previous to June 25, 1984 we received reports on the activities of Idel Aminnudin.

"Q What were those activities?

"A Purely marijuana trafficking.

"Q From whom did you get that information?

"A It came to my hand which was written in a required sheet of information, maybe for
security reason and we cannot identify the person.

"Q But you received it from your regular informer?


Page 192 of 377

"A Yes, sir.

"ATTY. LLARIZA:

"Q Previous to June 25, 1984, you were more or less sure that Idel Aminnudin is coming
with drugs?

"A Marijuana, sir.

"Q And this information respecting Idel Aminnudin’s coming to Iloilo with marijuana
was received by you many days before you received the intelligence report in writing?

"A Not a report of the particular coming of Aminnudin but his activities.

"Q You only knew that he was coming on June 25, 1984 two days before?

"A Yes, sir.

"Q You mean that before June 23, 1984 you did not know that Aminnudin was coming?

"A Before June 23, 1984, I, in my capacity, did not know that he was coming but on
June 23, 1984 that was the time when I received the information that he was coming.
Regarding the reports on his activities, we have reports that he was already
consummated the act of selling and shipping marijuana stuff.

"COURT:

"Q And as a result of that report, you put him under surveillance?

"A Yes, sir.


Page 193 of 377

"Q In the intelligence report, only the name of Idel Aminnudin was mentioned?

"A Yes, sir.

"Q Are you sure of that?

"A On the 23rd he will be coming with the woman.

"Q So that even before you received the official report on June 23, 1984, you had already
gathered information to the effect that Idel Aminnudin was coming to Iloilo on June
25,1984?

"A Only on the 23rd of June.

"Q You did not try to secure a search warrant for the seizure or search of the subject
mentioned in your intelligence report?

"A No, more.

"Q Why not?

"A Because we were very very sure that our operation will yield positive result.

"Q Is that your procedure that whenever it will yield positive result you do not need a
search warrant anymore?

"A Search warrant is not necessary." 23

That last answer is a cavalier pronouncement, especially as it comes from a mere


lieutenant of the PC. The Supreme Court cannot countenance such a statement. This is
still a government of laws and not of men.
Page 194 of 377

The mandate of the Bill of Rights is clear:

"Sec. 2. The right of the people to be secure in their persons, houses, papers, and effects
against unreasonable searches and seizures of whatever nature and for any purpose
shall be inviolable, and no search warrant or warrant of arrest shall issue except upon
probable cause to be determined personally by the judge after examination under oath
or affirmation of the complainant and the witnesses he may produce, and particularly
describing the place to be searched and the persons or things to be seized."

In the case at bar, there was no warrant of arrest or search warrant issued by a judge
after personal determination by him of the existence of probable cause. Contrary to the
averments of the government, the accused-appellant was not caught in flagrante nor
was a crime about to be committed or had just been committed to justify the warrantless
arrest allowed under Rule 113 of the Rules of Court. Even expediency could not be
invoked to dispense with the obtention of the warrant as in the case of Roldan v. Arca,
24 for example. Here it was held that vessels and aircraft are subject to warrantless
searches and seizures for violation of the customs law because these vehicles may be
quickly moved out of the locality or jurisdiction before the warrant can be secured.

The present case presented no such urgency. From the conflicting declarations of the
PC witnesses, it is clear that they had at least two days within which they could have
obtained a warrant to arrest and search Aminnudin who was coming Iloilo on the M/V
Wilcon 9. His name was known. The vehicle was identified. The date of its arrival was
certain. And from the information they had received, they could have persuaded a judge
that there was probable cause, indeed, to justify the issuance of a warrant. Yet they did
nothing. No effort was made to comply with the law. The Bill of Rights was ignored
altogether because the PC lieutenant who was the head of the arresting team, had
determined on his own authority that "search warrant was not necessary."

In the many cases where this Court has sustained the warrantless arrest of violators of
the Dangerous Drugs Act, it has always been shown that they were caught red-handed,
as result of what are popularly called "buy-bust" operations of the narcotics agents. 25
Rule 113 was clearly applicable because at the precise time of arrest the accused was
in the act of selling the prohibited drug.

In the case at bar, the accused-appellant was not, at the moment of his arrest,
committing a crime nor was it shown that he was about to do so or that he had just
done so. What he was doing was descending the gangplank of the M/V Wilcon 9 and
there was no outward indication that called for his arrest. To all appearances, he was
like any of the other passengers innocently disembarking from the vessel. It was only
Page 195 of 377

when the informer pointed to him as the carrier of the marijuana that the suddenly
became suspect and so subject to apprehension. It was the furtive finger that triggered
his arrest. The identification by the informer was the probable cause as determined by
the officers (and not a judge) that authorized them to pounce upon Aminnudin and
immediately arrest him.

Now that we have succeeded in restoring democracy in our country after fourteen years
of the despised dictatorship, when any one could be picked up at will, detained without
charges and punished without trial, we will have only ourselves to blame if that kind of
arbitrariness is allowed to return, to once more flaunt its disdain of the Constitution
and the individual liberties its Bill of Rights guarantees.

While this is not to say that the accused-appellant is innocent, for indeed his very own
words suggest that he is lying, that fact alone does not justify a finding that he is guilty.
The constitutional presumption is that he is innocent, and he will be so declared even
if his defense is weak as long as the prosecution is not strong enough to convict him.

Without the evidence of the marijuana allegedly seized from Aminnudin, the case of the
prosecution must fall. That evidence cannot be admitted, and should never have been
considered by the trial court for the simple fact is that the marijuana was seized illegally.
It is the fruit of the poisonous tree, to use Justice Holmes’ felicitous phrase. The search
was not an incident of a lawful arrest because there was no warrant of arrest and the
warrantless arrest did not come under the exceptions allowed by the Rules of Court.
Hence, the warrantless search was also illegal and the evidence obtained thereby was
inadmissible.

The Court strongly supports the campaign of the government against drug addiction
and commends the efforts of our law-enforcement officers against those who would
inflict this malediction upon our people, especially the susceptible youth. But as
demanding as this campaign may be, it cannot be more so than the compulsions of the
Bill of Rights for the protection of the liberty of every individual in the realm, including
the basest of criminals. The Constitution covers with the mantle of its protection the
innocent and the guilty alike against any manner of high-handedness from the
authorities, however praiseworthy their intentions.

Those who are supposed to enforce the law are not justified in disregarding the rights
of the individual in the name of order. Order is too high a price for the loss of liberty. As
Justice Holmes, again, said, "I think it a less evil that some criminal should escape than
that the government should play an ignoble part." It is simply not allowed in the free
Page 196 of 377

society to violate a law to enforce another, especially if the law violated is the
Constitution itself.

We find that with the exclusion of the illegally seized marijuana as evidence against the
accused-appellant, his guilt has not been proved beyond reasonable doubt and he must
therefore be discharged on the presumption that he is innocent.

ACCORDINGLY, the decision of the trial court is REVERSE: and the accused-appellant
is ACQUITTED. It is so ordered.

Narvasa, Gancayco and Medialdea JJ. concur.

Separate Opinions
GRIÑO-AQUINO, J., dissenting:

I respectfully dissent. I hold that the accused was caught in flagrante for he was carrying
marijuana leaves in his bag at the moment of his arrest. He was not "innocently
disembarking from the vessel." The unauthorized transportation of marijuana (Indian
hemp), which is a prohibited drug, is a crime, (Sec. 4, Rep. Act No. 6425). Since he was
committing a crime his arrest could be lawfully effected without a warrant (Sec. 6-a,
Rule 113, Rules of Court), and the search of his bag (which yielded the marijuana leaves)
without a search warrant was also lawful (Sec. 12, Rule 126, Rules of Court). I vote to
affirm the judgment of the trial court finding him guilty of illegally transporting
marijuana.
Page 197 of 377

G.R. No. 91107 June 19, 1991

THE PEOPLE OF THE PHILIPPINES, Plaintiff-Appellee, v. MIKAEL


MALMSTEDT, * defendant-appellant.

PADILLA, J.:

In an information dated 15 June 1989, Accused-appellant Mikael Malmstedt


(hereinafter referred to as the accused) was charged before the Regional Trial Court
(RTC) of La Trinidad, Benguet, Branch 10, in Criminal Case No. 89-CR-0663, for
violation of Section 4, Art. II of Republic Act 6425, as amended, otherwise known as the
Dangerous Drugs Act of 1972, as amended. The factual background of the case is as
follows:

Accused Mikael Malmstedt, a Swedish national, entered the Philippines for the third
time in December 1988 as a tourist. He had visited the country sometime in 1982 and
1985.

In the evening of 7 May 1989, Accused left for Baguio City. Upon his arrival thereat in
the morning of the following day, he took a bus to Sagada and stayed in that place for
two (2) days.

At around 7:00 o’clock in the morning of 11 May 1989, Accused went to the Nangonogan
bus stop in Sagada to catch the first available trip to Baguio City. From Baguio City,
Accused planned to take a late afternoon trip to Angeles City, then proceed to Manila to
catch his flight out of the country, scheduled on 13 May 1989. From Sagada, Accused
took a Skyline bus with body number 8005 and Plate number AVC 902. 1

At about 8:00 o’clock in the morning of that same day (11 May 1989), Captain Alen
Vasco, the Commanding Officer of the First Regional Command (NARCOM) stationed at
Camp Dangwa, ordered his men to set up a temporary checkpoint at Kilometer 14, Acop,
Tublay, Mountain Province, for the purpose of checking all vehicles coming from the
Cordillera Region. The order to establish a checkpoint in the said area was prompted by
persistent reports that vehicles coming from Sagada were transporting marijuana and
other prohibited drugs. Moreover, information was received by the Commanding Officer
of NARCOM, that same morning, that a Caun coming from Sagada had in his possession
prohibited drugs. 2
Page 198 of 377

The group composed of seven (7) NARCOM officers, in coordination with Tublay Police
Station, set up a checkpoint at the designated area at about 10:00 o’clock in the morning
and inspected all vehicles coming from the Cordillera Region.

At about 1:30 o’clock in the afternoon, the bus where accused was riding was stopped.
Sgt. Fider and CIC Galutan boarded the bus and announced that they were members of
the NARCOM and that they would conduct an inspection. The two (2) NARCOM officers
started their inspection from the front going towards the rear of the bus. Accused who
was the sole foreigner riding the bus was seated at the rear thereof.

During the inspection, CIC Galutan noticed a bulge on accused’s waist. Suspecting the
bulge on accused’s waist to be a gun, the officer asked for accused’s passport and other
identification papers. When accused failed to comply, the officer required him to bring
out whatever it was that was bulging on his waist. The bulging object turned out to be
a pouch bag and when accused opened the same bag, as ordered, the officer noticed
four (4) suspicious-looking objects wrapped in brown packing tape, prompting the officer
to open one of the wrapped objects. The wrapped objects turned out to contain hashish,
a derivative of marijuana.

Thereafter, Accused was invited outside the bus for questioning. But before he alighted
from the bus, Accused stopped to get two (2) travelling bags from the luggage carrier.

Upon stepping out of the bus, the officers got the bags and opened them. A teddy bear
was found in each bag. Feeling the teddy bears, the officer noticed that there were bulges
inside the same which did not feel like foam stuffing. It was only after the officers had
opened the bags that accused finally presented his passport.

Accused was then brought to the headquarters of the NARCOM at Camp Dangwa, La
Trinidad, Benguet for further investigation. At the investigation room, the officers
opened the teddy bears and they were found to also contain hashish. Representative
samples were taken from the hashish found among the personal effects of accused and
the same were brought to the PC Crime Laboratory for chemical analysis.

In the chemistry report, it was established that the objects examined were hashish, a
prohibited drug which is a derivative of marijuana. Thus, an information was filed
against accused for violation of the Dangerous Drugs Act.
Page 199 of 377

During the arraignment, Accused entered a plea of "not guilty." For his defense, he
raised the issue of illegal search of his personal effects. He also claimed that the hashish
was planted by the NARCOM officers in his pouch bag and that the two (2) travelling
bags were not owned by him, but were merely entrusted to him by an Australian couple
whom he met in Sagada. He further claimed that the Australian couple intended to take
the same bus with him but because there were no more seats available in said bus, they
decided to take the next ride and asked accused to take charge of the bags, and that
they would meet each other at the Dangwa Station.

Likewise, Accused alleged that when the NARCOM officers demanded for his passport
and other identification papers, he handed to one of the officers his pouch bag which
was hanging on his neck containing, among others, his passport, return ticket to
Sweden and other papers. The officer in turn handed it to his companion who brought
the bag outside the bus. When said officer came back, he charged the accused that there
was hashish in the bag. He was told to get off the bus and his picture was taken with
the pouch bag placed around his neck. The trial court did not give credence to accused’s
defense.chanrobles.com:

The claim of the accused that the hashish was planted by the NARCOM officers, was
belied by his failure to raise such defense at the earliest opportunity. When accused was
investigated at the Provincial Fiscal’s Office, he did not inform the Fiscal or his lawyer
that the hashish was planted by the NARCOM officers in his bag. It was only two (2)
months after said investigation when he told his lawyer about said claim, denying
ownership of the two (2) travelling bags as well as having hashish in his pouch bag.

In a decision dated 12 October 1989, the trial court found accused guilty beyond
reasonable doubt for violation of the Dangerous Drugs Act, specifically Section 4, Art. II
of RA 6425, as amended. 3 The dispositive portion of the decision reads as follows:

"WHEREFORE, finding the guilt of the accused Mikael Malmstedt established beyond
reasonable doubt, this Court finds him GUILTY of violation of Section 4, Article II of
Republic Act 6425, as amended, and hereby sentences him to suffer the penalty of life
imprisonment and to pay a fine of Twenty Thousand Pesos (P20,000.00), with subsidiary
imprisonment in case of insolvency and to pay the costs.

Let the hashish subject of this case be turned over to the First Narcotics Regional Unit
at Camp Bado; Dangwa, La Trinidad, Benguet for proper disposition under Section 20,
Article IV of Republic Act 425, as amended.
Page 200 of 377

SO ORDERED." 4

Seeking the reversal of the decision of the trial court finding him guilty of the crime
charged, Accused argues that the search of his personal effects was illegal because it
was made without a search warrant and, therefore, the prohibited drugs which were
discovered during the illegal search are not admissible as evidence against him.

The Constitution guarantees the right of the people to be secure in their persons,
houses, papers and effects against unreasonable searches and seizures. 5 However,
where the search is made pursuant to a lawful arrest, there is no need to obtain a search
warrant. A lawful arrest without a warrant may be made by a peace officer or a private
person under the following circumstances. 6

"SEC. 5. Arrest without warrant; when lawful. — A peace officer or a private person may,
without a warrant, arrest a person:

(a) When, in his presence, the person to be arrested has committed, is actually
committing, or is attempting to commit an offense;

(b) When an offense has in fact just been committed, and he has personal knowledge of
facts indicating that the person to be arrested has committed it; and

(c) When the person to be arrested is a prisoner who has escaped from a penal
establishment or place where he is serving final judgment or temporarily confined while
his case is pending, or has escaped while being transferred from one confinement to
another.

In cases falling under paragraphs (a) and (b) hereof, the person arrested without a
warrant shall be forthwith delivered to the nearest police station or jail, and he shall be
proceeded against in accordance with Rule 112, Section 7. (6a, 17a)."

Accused was searched and arrested while transporting prohibited drugs (hashish). A
crime was actually being committed by the accused and he was caught in flagrante
delicto. Thus, the search made upon his personal effects falls squarely under paragraph
(1) of the foregoing provisions of law, which allow a warrantless search incident to a
lawful arrest. 7
Page 201 of 377

While it is true that the NARCOM officers were not armed with a search warrant when
the search was made over the personal effects of accused, however, under the
circumstances of the case, there was sufficient probable cause for said officers to believe
that accused was then and there committing a crime.chanrobles law library

Probable cause has been defined as such facts and circumstances which could lead a
reasonable, discreet and prudent man to believe that an offense has been committed,
and that the objects sought in connection with the offense are in the place sought to be
searched. 8 The required probable cause that will justify a warrantless search and
seizure is not determined by any fixed formula but is resolved according to the facts of
each case. 9

Warrantless search of the personal effects of an accused has been declared by this Court
as valid, because of existence of probable cause, where the smell of marijuana emanated
from a plastic bag owned by the accused, 10 or where the accused was acting
suspiciously, 11 and attempted to flee. 12

Aside from the persistent reports received by the NARCOM that vehicles coming from
Sagada were transporting marijuana and other prohibited drugs, their Commanding
Officer also received information that a Caun coming from Sagada on that particular
day had prohibited drugs in his possession. Said information was received by the
Commanding Officer of NARCOM the very same morning that accused came down by
bus from Sagada on his way to Baguio City.

When NARCOM received the information, a few hours before the apprehension of herein
accused, that a Caun travelling from Sagada to Baguio City was carrying with him
prohibited drugs, there was no time to obtain a search warrant. In the Tangliben case,
13 the police authorities conducted a surveillance at the Victory Liner Terminal located
at Bgy. San Nicolas, San Fernando Pampanga, against persons engaged in the traffic of
dangerous drugs, based on information supplied by some informers. Accused Tangliben
who was acting suspiciously and pointed out by an informer was apprehended and
searched by the police authorities. It was held that when faced with on-the spot
information, the police officers had to act quickly and there was no time to secure a
search warrant.

It must be observed that, at first, the NARCOM officers merely conducted a routine
check of the bus (where accused was riding) and the passengers therein, and no
extensive search was initially made. It was only when one of the officers noticed a bulge
on the waist of accused, during the course of the inspection, that accused was required
Page 202 of 377

to present his passport. The failure of accused to present his identification papers, when
ordered to do so, only managed to arouse the suspicion of the officer that accused was
trying to hide his identity. For is it not a regular norm for an innocent man, who has
nothing to hide from the authorities, to readily present his identification papers when
required to do so?

The receipt of information by NARCOM that a Caun coming from Sagada had prohibited
drugs in his possession, plus the suspicious failure of the accused to produce his
passport, taken together as a whole, led the NARCOM officers to reasonably believe that
the accused was trying to hide something illegal from the authorities. From these
circumstances arose a probable cause which justified the warrantless search that was
made on the personal effects of the accused. In other words, the acts of the NARCOM
officers in requiring the accused to open his pouch bag and in opening one of the
wrapped objects found inside said bag (which was discovered to contain hashish) as well
as the two (2) travelling bags containing two (2) teddy bears with hashish stuffed inside
them, were prompted by accused’s own attempt to hide his identity by refusing to
present his passport, and by the information received by the NARCOM that a Caun
coming from Sagada had prohibited drugs in his possession. To deprive the NARCOM
agents of the ability and facility to act accordingly, including, to search even without
warrant, in the light of such circumstances, would be to sanction impotence and
ineffectiveness in law enforcement, to the detriment of society.

WHEREFORE, premises considered, the appealed judgment of conviction by the trial


court is hereby AFFIRMED. Costs against the Accused-Appellant.

SO ORDERED.

Melencio-Herrera, Paras, Feliciano, Bidin, Griño-Aquino, Medialdea, Regalado and


Davide, Jr., JJ., concur.

Sarmiento, J., is on leave.

Separate Opinions
NARVASA, J., concurring and dissenting:

The ancient tradition that a man’s home is his castle, safe from intrusion even by the
king, has not only found its niche in all our charters, from 1935 to the present; it has
Page 203 of 377

also received unvarying recognition and acceptance in our case law. 1 The present
Constitution 2 declares that —

"The right of the people to be secure in their persons, houses, papers, and effects against
unreasonable searches and seizures of whatever nature and for any purpose, shall be
inviolable, and no search warrant or warrant of arrest shall issue except upon probable
cause to be determined personally by the judge after examination under oath or
affirmation of the complainant and the witnesses he may produce, and particularly
describing the place to be searched, and the persons or things to be seized."

It further ordains that any evidence obtained in violation of said right, among others,
"shall be inadmissible for any purpose in any proceeding." 3

The rule is that no person may be subjected by the police or other government authority
to a search of his body, or his personal effects or belongings, or his residence except by
virtue of a search warrant or on the occasion of a legitimate arrest. 4 An arrest is
legitimate, of course, if effected by virtue of a warrant of arrest. Even without a warrant,
an arrest may also be lawfully made by a peace officer or a private person: 5

(a) when, in his presence, the person to be arrested has committed, is actually
committing, or is attempting to commit an offense;

(b) When an offense has in fact just been committed, and he has personal knowledge of
facts indicating that the person to be arrested has committed it; and

(c) When the person to be arrested is a prisoner who has escaped from a penal
establishment or place where he is serving final judgment or temporarily confined while
his case is pending, or has escaped while being transferred from one confinement to
another.

In cases falling under paragraphs (a) and (b) hereof, the person arrested without a
warrant shall be forthwith delivered to the nearest police station or jail, and he shall be
proceeded against in accordance with Rule 112, Section 7."

In any of these instances of a lawful arrest, the person arrested "may be searched for
dangerous weapons or anything which may be used as proof of the commission of an
offense, without a search warrant." 6 And it has been held that the search may extend
Page 204 of 377

to the area "within his immediate control," i.e., the area from which said person arrested
might gain possession of a weapon or destructible evidence. 7

Apart from "search incidental to an arrest," a warrantless search has also been held to
be proper in cases of "search of a moving vehicle," 8 and "seizure of evidence in plain
view." 9 This was the pronouncement in Manipon, Jr. v. Sandiganbayan, 143 SCRA 267,
276, which drew attention to Moreno v. Ago Chi; 10 Alvero v. Dizon, 11 Papa v. Mago,
12 and an American precedent, Harris v. U.S. 13

If, on the other, a person is searched without a warrant, or under circumstances other
than those justifying an arrest without warrant in accordance with law, supra, merely
on suspicion that he is engaged in some felonious enterprise, and in order to discover if
he has indeed committed a crime, it is not only the arrest which is illegal but also, the
search on the occasion thereof, as being "the fruit of the poisonous tree." 14 In that
event, any evidence taken, even if confirmatory of the initial suspicion, is inadmissible
"for any purpose in any proceeding." 15 But the right against an unreasonable search
and seizure may be waived by the person arrested, provided he knew of such right and
knowingly decided not to invoke it. 16

There is unanimity among the members of the Court upon the continuing validity of
these established principles. However, the Court is divided as regards the ultimate
conclusions which may properly be derived from the proven facts and consequently, the
manner in which the principles just cited should apply thereto.

The proofs of the prosecution and those of the defense are diametrically at odds. What
is certain, however, is that the soldiers had no warrant of arrest when they conducted
a search of Malmstedt’s person and the things in his possession at the time. Indeed, the
Court a quo acknowledged that the soldiers could "not be expected to be armed with a
warrant or arrest nor a search warrant everytime they establish a temporary checkpoint
. . . (and) no judge would issue them one considering that searching questions have to
be asked before a warrant could be issued." Equally plain is that prior to the search, a
warrantless arrest of Malmstedt could not validly have been in accordance with the
norms of the law. For Malmstedt had not committed, nor was he actually committing or
attempting to commit a crime, in the soldiers’ presence, nor did said soldiers have
personal and competent knowledge that Malmstedt had in fact just committed a crime.
All they had was a suspicion that Malmstedt might have some prohibited drug on him
or in his bags; all they had was, in the words of the Trial Court, "the hope of intercepting
any dangerous drug being transported," or, as the Office of the Solicitor General asserts,
"information that most of the buses coming . . . (from the Cordillera) were transporting
marijuana and other prohibited drugs."
Page 205 of 377

This case, is remarkably similar to Peo. v. Aminnudin, decided on July 6, 1988 also by
the First Division. 17 There, Aminnudin was arrested without a warrant by PC officers
as he was disembarking from an inter-island vessel. The officers were waiting for him
because he was, according to an informer’s report, then transporting marijuana. The
search of Aminnudin’s bag confirmed the informer’s report; the bag indeed contained
marijuana. The Court nevertheless held that since the PC officers had failed to procure
a search warrant although they had sufficient time (two days) to do so and therefore,
the case presented no such urgency as to justify a warrantless search, the search of
Aminnudin’s person and bag, the seizure of the marijuana and his subsequent arrest
were illegal; and the marijuana was inadmissible in evidence in the criminal action
subsequently instituted against Aminnudin for violating the Dangerous Drugs Act.

There are, on the other hand, other cases adjudicated by this Court in which apparently
different conclusions were reached. It is needful to devote a few words to them so that
the relevant constitutional and legal propositions are not misunderstood.

In People v. Claudio (decision promulgated on April 15, 988), 18 the accused boarded a
"Victory Liner" passenger bus going to Olongapo from Baguio City. She placed the plastic
bag she was carrying at the back of the seat then occupied by Obiña, an INP member
"on Detached Service with the Anti-Narcotics Unit." This avowedly aroused Obiña’s
suspicion, and at the first opportunity, and without Claudio’s knowledge, he
surreptitiously looked into the plastic bag and noted that it contained camote tops as
well as a package, and that there emanated from the package the smell of marijuana
with which he had become familiar on account of his work. So when the bus stopped at
Sta. Rita, and Claudio alighted, Obiña accosted her, showed her his ID, identified
himself as a policeman, and announced his intention to search her bag which he said
contained marijuana because of the distinctive odor detected by him. Ignoring her plea
— "Please go with me, let us settle this at home" — he brought her to the police
headquarters, where examination of the package in Claudio’s bag confirmed his
suspicion that it indeed contained marijuana. The Court held the warrantless arrest
under the circumstances to be lawful, the search justified, and the evidence thus
discovered admissible in evidence against the accused.

In People v. Tangliben (decision promulgated on April 6, 1990), 19 two police officers


and a barangay tanod were conducting a "surveillance mission" at the Victory Liner
Terminal at San Nicolas, San Fernando, Pampanga, "aimed not only against persons
who may commit misdemeanors . . . (there) but also on persons who may be engaging
in the traffic of dangerous drugs based on information supplied by informers; . . . they
noticed a person carrying a red travelling bag . . . who was acting suspiciously;" they
asked him to open the bag; the person did so only after they identified themselves as
peace officers; found in the bag were marijuana leaves wrapped in plastic weighing one
kilogram, more or less; the person was then taken to the police headquarters at San
Fernando, Pampanga, where he was investigated; and an information was thereafter
Page 206 of 377

filed against that person, Tangliben, charging him with a violation of the Dangerous
Drugs Act of 1972 (RA 6425), as amended. Upon these facts it was ruled, citing Claudio,
supra, that there was a valid warrantless arrest and a proper warrantless search
incident thereto.

The facts in Tangliben were pronounced to be different from those in People v.


Aminnudin, supra. "In contrast" to Aminnudin where the Court perceived no urgency
as to preclude the application for and obtention of a search warrant, it was declared
that the Tangliben case —

". . . presented urgency. . . . (The evidence revealed) that there was an informer who
pointed to the accused-appellant as carrying marijuana. . . . Faced with such on-the-
spot information, the police officers had to act quickly. There was not enough time to
secure a search warrant. . . . To require search warrants during on-the-spot
apprehensions of drug pushers, illegal possessors of firearms, jueteng collectors,
smugglers of contraband goods, robber, etc. would make it extremely difficult, if not
impossible to contain the crimes with which these persons are associated."

In Tangliben, therefore, there was in the Court’s view sufficient evidence on hand to
enable the PC officers to secure a search warrant, had there been time. But because
there was actually no time to get the warrant, and there were "on-the-spot" indications
that Tangliben was then actually committing a crime, the search of his person and his
effects was considered valid.

Two other decisions presented substantially similar circumstances: Posadas v . C.A., Et


Al., decided on August 2, 1990, 20 and People v. Moises Maspil, Jr., Et Al., decided on
August 20, 1990. 21

In the first case, Posadas was seen to be acting suspiciously by two members of the INP,
Davao Metrodiscom, and when he was accosted by the two, who identified themselves
as police officers, he suddenly fled. He was pursued, overtaken and, notwithstanding
his resistance, placed in custody. The buri bag Posadas was then carrying was found to
contain a revolver, for which he could produce no license or authority to possess, four
rounds of live ammunition, and a tear gas grenade. He was prosecuted for illegal
possession of firearms and ammunition and convicted after trial. This Court affirmed
Posadas’ conviction, holding that there was, in the premises, probable cause for a search
without warrant, i.e., the appellant was acting suspiciously and attempted to flee with
the buri bag he had with him at the time. The Court cited with approval the ruling of
the U.S. Federal Supreme Court in John W. Terry v. State of Ohio, 22 a 1968 case,
which the Solicitor General had invoked to justify the search.
Page 207 of 377

In the case of Maspil, Et Al., a checkpoint was set up by elements of the First Narcotics
Regional Unit of the Narcotics Command at Sayangan, Atok, Benguet, to monitor,
inspect and scrutinize vehicles on the highway going towards Baguio City. This was
done because of a confidential report by informers that Maspil and another person,
Bagking, would be transporting a large quantity of marijuana to Baguio City. In fact,
the informers were with the policemen manning the checkpoint. As expected, at about
2 o’clock in the early morning of November 1, 1986, a jeepney approached the
checkpoint, driven by Maspil, with Bagking as passenger. The officers stopped the
vehicle and saw that on it were loaded 2 plastic sacks, a jute sack, and 3 big round tin
cans. When opened, the sacks and cans were seen to contain what appeared to be
marijuana leaves. The policemen thereupon placed Maspil and Bagking under arrest,
and confiscated the leaves which, upon scientific examination, were verified to be
marijuana leaves. The Court upheld the validity of the search thus conducted, as being
incidental to a lawful warrantless arrest, 23 and declared that, as in Tangliben, supra,
Maspil and Bagking had been caught in flagrante delicto transporting prohibited drugs
at the time of their arrest. Again, the Court took occasion to distinguish the case from
Aminnudin 24 in which, as aforestated, it appeared that the police officers were aware
of Aminnudin’s identity, his projected criminal enterprise and the vessel on which he
would be arriving, and, equally as importantly, had sufficient time and opportunity to
obtain a search warrant. In the case of Maspil and Bagking, the Court found that the
officers concerned had no exact description of the vehicle the former would be using to
transport marijuana, and no inkling of the definite time of the suspects’ arrival, and
pointed out that a jeepney on the road is not the same as a passenger boat on the high
seas whose route and time of arrival are more or less certain, and which ordinarily
cannot deviate from or otherwise alter its course, or select another destination.25

The most recent decision treating of warrantless search and seizure appears to be People
v. Lo Ho Wing; et al, G.R. No. 88017, decided on January 21, 1991 (per Gancayco, J.).
In that case, an undercover or "deep penetration" agent, Tia, managed somehow to gain
acceptance into a group of suspected drug smugglers, which included Peter Lo and Lim
Ching Huat. Tia accompanied Peter Lo, to Guangzhou, China, where he saw him and
other person empty the contents of six (6) tins of tea and replace them with white
powder. On their return to Manila with the cans of substituted "tea," they were met at
the airport by Lim. As they were leaving the airport in separate vehicles, they were
intercepted by officers and operatives of the Narcotics Command (NARCOM), who had
earlier been tipped off by Tia, and placed under arrest. As search of the luggage brought
in by Tia and Peter Lo, loaded on the group’s vehicles, quickly disclosed the six (6) tin
cans containing fifty-six (56) bags of white crystalline powder which, upon analysis, was
identified as metamphetamine. Tia, Lo and Lim were indicted for violation of the
Dangerous Drugs Act of 1972. Tia was discharged as state witness. Lo and Lim were
subsequently convicted and sentenced to life imprisonment. One of the questions raised
by them in this Court on appeal was whether the warrantless search of their vehicles
and personal effects was legal. The Court, citing Manipon, Jr. v. Sandiganbayan, 143
Page 208 of 377

SCRA 267 (1986), 26 held legal the search of the appellants’ moving vehicles and the
seizure therefrom of the dangerous drug, considering that there was intelligence
information, including clandestine reports by a planted spy actually participating in the
activity, that the appellants were bringing prohibited drugs into the country; that the
requirement of obtaining a search warrant "borders on the impossible in the case of
smuggling effected by the use of a moving vehicle that can transport contraband from
one place to another with impunity," and "it is not practicable to secure a warrant
because the vehicle can be quickly moved out of the locality or jurisdiction in which the
warrant must be sought." 27

In all five cases, Claudio, Tangliben, Posadas, Maspil, and Lo Ho Wing, facts existed
which were found by the Court as justifying warantless arrests. In Claudio, the arresting
officer had secretly ascertained that the woman he was arresting was in fact in
possession of marijuana; he had personally seen that her bag contained not only
vegetables but also a package emitting the odor of marijuana. In Tangliben, the person
arrested and searched was acting suspiciously, and had been positively pointed to as
carrying marijuana. And in both cases, the accused were about to board passenger
buses, making it urgent for the police officers concerned to take quick and decisive
action. In Posadas, the person arrested and searched was acting suspiciously, too, and
when accosted had attempted to flee from the police officers. And in Maspil and Lo Ho
Wing, there was definite information of the precise identity of the persons engaged in
transporting prohibited drugs at a particular time and place.

Now, as regards the precise issue at hand, whether or not the facts in the case at bar
make out a legitimate instance of a warrantless search and seizure, there is, as earlier
pointed out, a regrettable divergence of views among the members of the Court.

Contrary to the conclusion reached by the majority, I believe that the appellant should
be absolved on reasonable doubt. There was in this case no confidential report from, or
positive identification by an informer; no attempt to flee; no bag or package emitting tell-
tale odors; no other reasonably persuasive indications that Malmstedt was at the time
in process of perpetrating the offense for which he was subsequently prosecuted. Hence,
when the soldiers searched Malmstedt’s pouch and the bags in his possession, they
were simply "fishing" for evidence. It matters not that the search disclosed that the bags
contained prohibited substances, confirming their initial information and suspicion. The
search was not made by virtue of a warrant or as an incident of a lawful warrantless
arrest, i.e., under circumstances sufficient to engender a reasonable belief that some
crime was being or about to be committed, or had just been committed. There was no
intelligent and intentional waiver of the right against unreasonable searches and
seizure. The search was therefore illegal, since the law requires that there first be a
lawful arrest of an individual before a search of his body and his belongings may licitly
be made. The process cannot be reversed, i.e., a search be first undertaken, and then
an arrest effected, on the strength of the evidence yielded by the search. An arrest made
Page 209 of 377

in that case would be unlawful, and the search undertaken as an incident of such an
unlawful arrest, also unlawful.

The fact that when investigated at the headquarters of the Narcotic Command at Camp
Dangwa, La Trinidad, Malmstedt had, it is said, willingly admitted that there were was
hashish inside the "teddy bears" in the luggage found in his possession — an admission
subsequently confirmed by laboratory examination — does not help the cause of the
prosecution one bit. Nothing in the record even remotely suggests that Malmstedt was
accorded the rights guaranteed by the Constitution to all persons under custodial
investigation. 28 He was not informed, prior to being interrogated, that he had the "right
to remain silent and to have competent and independent counsel preferably of his own
choice," and that if he could not afford the services of counsel, he would be provided
with one; not does it appear at all that he waived those rights "in writing and in the
presence of counsel." The soldiers and the police officers simply went ahead with the
investigation of Malmstedt, without counsel. The admissions elicited from Malmstedt
under these circumstances, as the Constitution clearly states, are "inadmissible in
evidence against him. 29

The prohibited drugs supposedly discovered in Malmstedt’s bags, having been taken in
violation of the constitutional right against unreasonable searches and seizures, are
inadmissible against him "for any purpose in any proceeding." Also pronounced as
incompetent evidence against him are the admissions supposedly made by him without
his first being accorded the constitutional rights of persons under custodial
investigation. Without such object evidence and admissions, nothing remains of the
case against Malmstedt.

It may be conceded that, as the Trial Court points out, the evidence presented by
Malmstedt in his defense is feeble, unworthy of credence. This is beside the point; for
conformably to the familiar axiom, the State must rely on the strength of its evidence
and not on the weakness of the defense. The unfortunate fact is that although the
existence of the hashish is an objective physical reality that cannot but be conceded,
there is in law no evidence to demonstrate with any degree of persuasion, much less
beyond reasonable doubt, that Malmstedt was engaged in a criminal activity. This is the
paradox created by the disregard of the applicable constitutional safeguards. The
tangible benefit is that the hashish in question has been correctly confiscated and thus
effectively withdrawn from private use.

What is here said should not by any means be taken as a disapproval or a disparagement
of the efforts of the police and military authorities to deter and detect offenses, whether
they be possession of and traffic in prohibited drugs, or some other. Those efforts
obviously merit the support and commendation of the Courts and indeed of every
Page 210 of 377

responsible citizen. But those efforts must take account of the basic rights granted by
the Constitution and the law to persons who may fall under suspicion of engaging in
criminal acts. Disregard of those rights may not be justified by the objective of ferreting
out and punishing crime, no matter how eminently desirable attainment of that objective
might be. Disregard of those rights, as this Court has earlier stressed, may result in the
escape of the guilty, and all because the "constable has blundered," rendering the
evidence inadmissible even if truthful or otherwise credible. 30

I therefore vote to reverse the Trial Court’s judgment of October 12, 1989 and to acquit
the appellant on reasonable doubt.

Fernan, C.J., Gutierrez and Gancayco, JJ., dissents.

CRUZ, J., dissenting:


I join Mr. Justice Andres R. Narvasa in his dissent, which I believe represents the correct
application to the facts of this case of the provisions of the Bill of Rights and the Rules
of Court on searches and seizures. It is consistent with my ponencia in People v.
Amminudin, 163 SCRA 402, and also with Alih v. Castro, 151 SCRA 279, the latter
being a unanimous decision of the Court en banc, and my dissents in Umil v. Ramos
(on warrantless arrests, 187 SCRA 311, Valmonte v. De Villa (on checkpoints), 178,
SCRA 211, 185 SCRA 665, and Guazon v. De Villa (on "zonas"), 181 SCRA 623.

I write this separate opinion merely to remark on an observation made during the
deliberation on this case that some members of the Court seem to be coddling criminals
instead of extending its protection to society, which deserves our higher concern. The
inference is that because of our wrong priorities, criminals are being imprudently let
free, to violate our laws again; and it is all our fault.

Believing myself to be among those alluded to, I will say without apology that I do not
consider a person a criminal, until he is convicted by final judgment after a fair trial by
a competent and impartial court. Until then, the Constitution bids us to presume him
innocent. He may seem boorish or speak crudely or sport tattoos or dress weirdly or
otherwise fall short of our own standards of propriety and decorum. None of these makes
him a criminal although he may look like a criminal.

It is so easy to condemn a person on the basis of his appearance but it is also so wrong.
Page 211 of 377

On the question before us, it seems to be the inclination of some judges to wink at an
illegal search and seizure as long as the suspect has been actually found in possession
of a prohibited article. That fact will retroactively validate the violation of the Bill of
Rights for after all, as they would rationalize, the suspect is a criminal. What matters to
them is the fact of illegal possession, not the fact of illegal search and seizure.

This kind of thinking takes us back to the intolerant days of Moncado v. People’s Court,
80 Phil. 1, which was discredited in Stonehill v. Diokno, 20 SCRA 383, even before it
was definitely rejected by an express provision in the 1973 Constitution. That provision,
which has been retained in the present Constitution, again explicitly declares that any
evidence illegally obtained "shall be inadmissible for any purpose in any proceeding."

The fruit of the poisonous tree should not be allowed to poison our system of criminal
justice. In the case at bar, the search was made at a checkpoint established for the
preposterous reason that the route was being used by marijuana dealers and on an
individual who had something bulging at his waist that excited the soldier’s suspicion.
Was that probable cause? The ponencia notes that the military had advance information
that a Caun was coming from the Sagada with prohibited drugs in his possession. This
is what the military says now, after the fact, to justify the warrantless search. It is so
easy to make such a claim, and I am surprised that the majority should readily accept
it.

The conclusion that there was probable cause may have been influenced by the
subsequent discovery that the accused was carrying a prohibited drug. This is supposed
to justify the soldier’s suspicion. In other words, it was the fact of illegal possession that
retroactively established the probable cause that validated the illegal search and seizure.
It was the fruit of the poisonous tree that washed clean the tree itself.

In Olmstead v. U.S., 277 U.S. 438, Justice Holmes said sixty-four years ago:

. . . It is desirable that criminals should be detected, and to that end that all available
evidence should be used. It is also desirable that the government should not itself foster
and pay for other crimes, when they are the means by which the evidence is to be
obtained. If it pays its officers for having got evidence by crime, I do not see why it may
not as well pay them for getting it in the same way, and I can attach no importance to
protestations of disapproval if it knowingly accepts and pays and announces that in the
future it will pay for the fruits. We have to choose, and for my part I think it a less evil
that some criminals should escape than that the government should play an ignoble
part.
Page 212 of 377

If by deterring the government from playing "an ignoble part," I am "coddling criminals,"
I welcome the accusation and take pride in it. I would rather err in favor of the accused
who is impaled with outlawed evidence than exalt order at the price of liberty.

Fernan, C.J. and Gutierrez, J., dissent.


Page 213 of 377

G.R. No. 101124 May 17, 1993

PEOPLE OF THE PHILIPPINES, Plaintiff-Appellee, v. CARMELINA TABAR y


CARMILOTES and ROMMEL ARRIESGADO y TABAR, Accused. CARMELINA
TABAR y CARMILOTES, Accused-Appellant.

SYLLABUS

1. CRIMINAL LAW; DANGEROUS DRUG ACT (R.A. 6425); ILLEGAL SALE OF


PROHIBITED DRUG; TO BE LIABLE THEREOF, PROOF OF SALE IS REQUIRED; CASE
AT BAR. — After a careful perusal of the records and evaluation of the evidence, this
Court is inclined to agree with the appellant that she should not be convicted under
Section 4, Article II of R.A. No. 6425. We rule, however, that she is liable under Section
8, Article II of the said Act. Her conviction by the trial court under Section 4 is primarily
based on its conclusion that the appellant "actually employed her nephew Rommel
Arriesgado to sell marijuana from her store and that she had been in that illicit business
for quite sometime now." This conclusion is based on the trial court’s sweeping
statement that "talks in the community where the accused lives is rife with accusations
that she is indeed engaged with members of the family, in the sale and distribution of
prohibited drugs such as marijuana." We find no evidence on record to sustain this
charge. It may thus be said that such a conclusion is not based on established facts but
on "talks in the community." If indeed such was the fact, it would not have been difficult
for the prosecution to provide the court with overwhelming evidence. Yet, it presented
only Pat. Trangia who, rather unfortunately, did not even testify or volunteer information
that the main target of the buy-bust operation was the appellant. He did not also
disclose in his testimony that the appellant was among the reported "pushers" in Punta
Princesa, Cebu City. Nevertheless, the prosecutor who conducted the direct-
examination of Trangia did not ask further as to the identity of the pushers such that it
was not proven that the appellant was one of them. The Solicitor General, however,
maintains that there was conspiracy, established by circumstantial evidence, between
accused Rommel Arriesgado who was caught in flagrante selling three (3) sticks of
handrolled marijuana to the informant and accepting the marked money. We are not
persuaded since the evidence for the prosecution does not show that (a) the appellant
was in the mind of the members of the team when they planned the buy-bust operation
and when they carried out such plan, (b) the three (3) sticks of handrolled marijuana
came from the appellant, and (c) the appellant used Rommel as her agent to sell the
three (3) sticks to the informant. Moreover, if indeed the prosecution truly believed that
such conspiracy existed, it should not have willingly given its conformity to Rommel’s
plea to the lesser offense of illegal possession of prohibited drugs under Section 8, Article
II of R.A. No. 6425, as amended. Having been caught in flagrante for selling marijuana,
it was not difficult to prove Rommel’s culpability under Section 4, Article II of the Act.
Yet it readily consented to his offer to plead guilty to the said lesser offense. It was,
however, established beyond any shadow of doubt and, therefore, with moral certainty,
that the appellant kept in her possession handrolled sticks of marijuana placed in empty
Page 214 of 377

Hope, Philip Morris and Mark cigarette packs. She does not have any authority to
possess them. She may have acquired them with the intention to sell them for profit;
but without proof of sale, she cannot be held liable under Section 4, Article II of the
Dangerous Drugs Act.

2. ID.; ID.; POSSESSION OF PROHIBITED DRUGS; IMPOSABLE PENALTY. — For such


possession, her liability is covered by Section 8 of the said Article which penalizes
possession or use of prohibited drugs. Appellant may specifically be penalized under the
aforesaid last paragraph of Section 8, Article II of the Act. Applying the Indeterminate
Sentence Law, the penalty of eight (8) years as Minimum to twelve (12) years as
Maximum and a fine of P10,000.00 may then be imposed upon her.

3. REMEDIAL LAW; CRIMINAL PROCEDURE; SEARCH AND SEIZURE; SEARCH


WITHOUT WARRANT OF PERSON ARRESTED; VALID IN CASE AT BAR. — The evidence
for the prosecution discloses that the appellant placed the packs of marijuana sticks
under the rolled pair of pants which she was then carrying at the time she hurriedly left
her shanty after noticing the arrest of Rommel. When she was asked to spread it out,
which she voluntarily did, the the package containing the packs of marijuana sticks
were thus exposed in plain view to the members of the team. A crime was thus
committed in the presence of the policemen. Pursuant to Section 5, Rule 113 and
Section 12, Rule 126 of the Revised Rules of Court, she could lawfully be arrested and
searched for anything which may be used as proof of the commission of an offense
without the corresponding arrest and search warrants. Her own counsel on cross-
examination of prosecution witness Josephus Trangia further obtained a re-affirmation
of these facts.

4. CONSTITUTIONAL LAW; BILL OF RIGHTS; RIGHT AGAINST UNREASONABLE


SEARCH AND SEIZURE; DEEMED WAIVED WHEN ACCUSED VOLUNTARILY SUBMITS
HIMSELF THERETO. — Even assuming ex gratia argumenti that the search and seizure
were without a warrant, the appellant had effectively waived her constitutional right
relative thereto by voluntarily submitting to the search and seizure. In People v.
Malasugui, (63 Phil. 221, 226 [1936]) this Court ruled: "When one voluntarily submits
to a search and consents to have it made of his person or premises, he is precluded from
later complaining thereof (Cooley, Constitutional Limitations, 8th ed., vol. I, page 631).
The right to be secure from unreasonable search may, like every right, be waived and
such waiver may be made either expressly or impliedly." The exclusionary ruled relied
upon by the appellant does not provide her a safe refuge.
Page 215 of 377

DAVIDE, JR., J.:

Carmelina Tabar y Carmilotes and her nephew, Rommel Arriesgado y Tabar, of Tres de
Abril, Punta Princesa, Cebu City, were charged with the violation of Section 4, Article II
of R.A. No. 6425, as amended, in an Information filed by the Office of the City Fiscal of
Cebu City with the Regional Trial Court of Cebu City on 9 February 1989, the accusatory
portion of which reads as follows:

"That on or about the 8th day of February 1989, at about 3:00 PM, in the City of Cebu,
Philippines, and within the jurisdiction of this Honorable Court, the said accused,
conniving and confederating together and mutually helping each other, with deliberate
intent, did then and there sell and deliver, without authority of law, Three (3) sticks of
marijuana cigarettes, a (sic) prohibited drugs, to a person who posted himself as a buyer,
in Viol. of Sec. 4, Art. 11, of RA 6425, as amended, otherwise known as the Dangerous
Act of 1972." 1

The case was docketed as Criminal Case No. CBU-14863 and after it was raffled off to
Branch 15 of the said court, the accused were forthwith arraigned. Carmelina entered
a plea of not guilty while Rommel, then seventeen (17) years of age, with the conformity
of the prosecution, entered a plea of guilty to the lesser offense of possession of
marijuana under Section 8, Article II of R.A. No. 6425, as amended. 2 As a consequence
of his plea, the trial court handed down on 24 April 1989 an Order which reads in part
as follows:

"Therefore this court being satisfied that the accused herein is the same Ramil Tabar
described in Annex 1 (Certificate of Birth) of the said accused, and it appearing that he
is still a minor (17 years), he is entitled to a suspended sentence of the penalty for
possession of marijuana which is a jail term of six (6) years and one (1) day to twelve
(12) years and a fine of Six Thousand (P6,000.00) pesos (Sec. 8 RA 6425 as amended of
B.P. 179, March 2, 1982).

WHEREFORE, the accused Rommel Tabar y Arriesgado is hereby discharged on


probation (Sec. 32 of RA 6425 as amended by B.P. 179) and committed to the custody
of the Department of Social Welfare and Development, Cebu Regional Office (No. 7) until
he reaches the age of majority, or otherwise finally discharged upon orders of this court
pursuant to P.D. 603 and B.P. 179, but to be placed under the Supervision of the
Dangerous Drugs Board, the alleged crime being drug related, and for a period of one
(1) year from date hereof.
Page 216 of 377

The Regional Director of the DSWD is hereby ordered to conduct and submit a case
study of the accused minor to this court, within sixty days and to report on his conduct
once every four months, to this court." 3

Thereafter, trial proceeded as against Carmelina alone. The prosecution presented Pfc.
Josephus Trangia and Myrna P. Arreola, a forensic analyst of the PC Crime Laboratory
Service, as its witnesses. The testimony of Pfc. Raul Tumakay was ordered stricken out
since he could not be cross-examined. The defense had only Carmelina as its witness.

On 22 December 1990, the trial court promulgated its decision, dated 17 December
1990, 4 finding Carmelina "guilty, beyond reasonable doubt, for (sic) violation of Section
4, Article II RA 6425, otherwise known as the Dangerous Drugs Act of 1972 as amended
by PD 1675" and sentencing her to "Reclusion Perpetua and to pay a fine of P20,000.00
for the act of selling and distributing marijuana." 5

The conviction is premised on the following findings of fact:

"From the evidence which consists of the testimony of Pfc. Josephus Trangia, the court
gathered that at about 3:00 P.M. of February 8, 1989, he was with Pfc. Romeo Cortes
and Gualberto Gabales on a buy-bust operation for marijuana after receipt of
information about marijuana pushers in Punta Princesa, Cebu City and that they had
their informant go ahead of them after giving the P5.00 bill for him to purchase
marijuana.

He continued saying that their informant stood in front of a shanty while they posted
themselves at a distance of about 50 meters from the place where their informant was
standing. And that they saw a young boy approached their informant and handed
cigarettes to him who in turn handed the marked money to the young boy. Then, their
informant gave them the pre-arranged signal of scratching his head with his right hand;
that after the signal, he and his companions immediately approached the young boy
and the informant introduced them as police officers. This young boy was about 16-17
years old, by the name of Rommel Arriesgado y Tabar. He had earlier pleaded guilty to
the lesser offense of mere possession of marijuana and was, in fact, already convicted
by this court. Upon being shown a P5.00 bill with the initials written thereon as: GDG-
89 and bearing SL L F 637396, he identified the same bill as the one given to their
informant and marked as Exh. "A" for the prosecution. He explained the initials GDG
which stands for Gualberto G. Gabales, his team member. He further declared that after
the pre-arranged signal from their informant, they immediately proceeded to the scene
and were given three sticks of marijuana by their informant after buying the same from
the boy, Rommel Arriesgado and that they proceeded to confiscate the P5.00 bill from
Page 217 of 377

the boy. At this juncture, he claimed that he observed that after the transaction, the boy
went inside the shanty and the moment he got out, he handed the three sticks of
marijuana to the informant. In fact, he claimed that after proceeding to the shanty, they
met Carmelina Tabar, Accused herein, and that Carmelina Tabar was holding a white
pants from where they found other marijuana sticks in cigarette packs which they
confiscated. That they brought Carmelina Tabar to Fuente Police Station for
investigation. He claimed that there were 75 sticks of marijuana in the Hope Cigarette
pack; 22 sticks of marijuana cigarettes in the Philip Morris pack and 99 sticks of
marijuana in the Mark cigarette pack. He said that they also confiscated the pants, but
only the marijuana sticks were submitted for testing to the PC Crime Laboratory. That
pursuant to this requested analysis, Lieut. Fortunato Quijon of the Police issued a
Certification of Field Test, Exh. "B" which showed that the three sticks of handrolled
cigarettes marked Rommel-89 were positive for marijuana. Shown the three packs of
cigarettes distinctly marked as Hope, Mark and Philip Morris, he identified the same as
the ones confiscated from Carmelina Tabar. So did he identify the cream-colored pants
he said they confiscated from Carmelina Tabar and which according to him was used to
wrap marijuana sticks inside the pack. He finally told the court that this team was
composed of Gualberto D. Gabales, Romeo Cortes, Pfc. Tumakay and himself.

On cross-examination, this witness affirmed that when he asked the boy, Rommel where
he got the marijuana sticks, he was told he got it from his aunt, the accused herein. He
further told the court that they arrested Rommel Tabar first and that they arrested
Carmelina Tabar later. When the young boy went inside, they presumed the marijuana
came from inside the shanty and that when the accused Carmelina Tabar went out,
suspicious-looking and pale and afraid to face them, they told her to stop from going
left towards the houses and asked her to open the pants which revealed the three
cigarette packs containing marijuana. The witness candidly admitted they had no
search warrant at the time they effected the arrest and confiscation.

From the testimonies of Mrs. Myrna Areola, Police Lieutenant, Forensic Analyst of the
PC Crime Laboratory, it was established that the specimens submitted to her were
positive of marijuana. She then identified Exh. "C", as her Chemistry Report C-038-89.
She also identified her signature, Exh. "C-4" thereon and her findings `All are positive
of Marijuana’, Exh. "C-3" ; the specimens submitted as Exh. "C-2" and the name of the
subject, Carmelina Tabar as Exh. "C-1." She confirmed her findings on all handrolled
cigarettes in all cigarette packs; Hope, Exh. "E-1" to "E-75; Philip Morris pack as Exh.
"F-1" to "F-22" and Mark cigarette pack, Exh. "G-1" to "G-99." She informed the court
that the specimens were submitted to her by Pfc Gabales on February 11, 1989, at about
10:50 A.M., and that she herself typed her report, Exh. "C" ; that she examined the
sticks one by one and handrolled them again. She also claimed that the police did not
ask for a copy of her report and that this is the first time it is presented in court. She
informed the court that she placed the specimens in her evidence store room, with keys,
she herself kept." 6
Page 218 of 377

The trial court discredited the bare denials of Carmelina and unfavorably considered
against her an admission that she had been arrested before by the CANU for possession
of marijuana, was charged for the violation of Section 8, Article II of R.A. No. 6425 in
Criminal Case No. CBU-8573, was convicted therein, but is now on probation. 7 It
further considered against her an allegedly very damaging admission, thus:

"She made a very damaging admission to the court when the Presiding Judge asked her
whether it is not true that she kept on crying because she was caught again and she
said, "Yes" and at which juncture she admitted to the court that she was serving
probation for the same offense." 8

It then concluded that:

" [A]ccused actually employed her nephew, Rommel Arriesgado to sell marijuana from
her store and that she has been in that illicit business for quite sometime now. The
evidence notwithstanding, talks in the community where the accused lives is rife with
accusations (sic) that she is indeed engaged with members of the family, in the sale and
distribution of prohibited drugs such as marijuana. Between the positive testimony of
the arresting officers, who appear to be more credible than the accused’s worthless and
untrustworthy denials, the court gives credence to the evidence of the prosecution." 9

Unable to accept the verdict, Carmelina filed her notice of appeal 10 manifesting therein
that she is appealing from the decision to the Court of Appeals. In its Order of 27
February 1991, 11 the trial court gave due course to the appeal and directed the clerk
of court "to submit all the records, evidences (sic) and transcripts of this proceeding to
the Hon. Court of Appeals, for proper disposition." Considering that the penalty imposed
is reclusion perpetua, the Court of Appeals transmitted to this Court the records of the
case on 12 August 1991. 12 In the Resolution of 11 September 1991, this Court
accepted the appeal.

In her Appellant’s Brief, 13 Carmelina, hereinafter referred to as the Appellant, imputes


upon the trial court the commission of the following errors in the appealed decision:

"I
Page 219 of 377

. . . IN CONVICTING ACCUSED-APPELLANTS (sic) OF VIOLATION OF SECTION 4,


ARTICLE II, REPUBLIC ACT 6425 AS AMENDED WHEN THE EVIDENCE DOES NOT
WARRANT IT.

II

. . . IN ADMITTING EVIDENCE SEIZED WITHOUT ANY SEARCH WARRANT." 14

As to the first assigned error, the appellant claims that the prosecution presented no
evidence that she sold marijuana and since there exists no convincing, positive and
conclusive proof of conspiracy between her and her co-accused, Rommel Arriesgado,
she cannot be held liable for violation of Section 4, Article Ii of R.A. No. 6425, as
amended.

In support of the second assigned error, the appellant maintains that the marijuana
cigarettes seized from her are inadmissible in evidence because they were obtained in
violation of the constitutional guarantee against unreasonable search and seizure.

After a careful perusal of the records and evaluation of the evidence, this Court is
inclined to agree with the appellant that she should not be convicted under Section 4,
Article II of R.A. No. 6425. We rule, however, that she is liable under Section 8, Article
II of the said Act. Her conviction by the trial court under Section 4 is primarily based on
its conclusion that the appellant "actually employed her nephew Rommel Arriesgado to
sell marijuana from her store and that she had been in that illicit business for quite
sometime now." This conclusion is based on the trial court’s sweeping statement that
"talks in the community where the accused lives is rife with accusations that she is
indeed engaged with members of the family, in the sale and distribution of prohibited
drugs such as marijuana." We find no evidence on record to sustain this charge. It may
thus be said that such a conclusion is not based on established facts but on "talks in
the community." If indeed such was the fact, it would not have been difficult for the
prosecution to provide the court with overwhelming evidence. Yet, it presented only Pat.
Trangia who, rather unfortunately, did not even testify or volunteer information that the
main target of the buy-bust operation was the appellant. He did not also disclose in his
testimony that the appellant was among the reported "pushers" in Punta Princesa, Cebu
City. It may be recalled that the buy-bust operation on 8 February 1989 was conducted
because, as he alleged:
Page 220 of 377

"A Before that time we have already received information from the community of Punta
Princesa regarding marijuana pushers in that place." 15

Nevertheless, the prosecutor who conducted the direct-examination of Trangia did not
ask further as to the identity of the pushers such that it was not proven that the
appellant was one of them. The Solicitor General, however, maintains that there was
conspiracy, established by circumstantial evidence, between accused Rommel
Arriesgado who was caught in flagrante selling three (3) sticks of handrolled marijuana
to the informant and accepting the marked money. We are not persuaded since the
evidence for the prosecution does not show that (a) the appellant was in the mind of the
members of the team when they planned the buy-bust operation and when they carried
out such plan, (b) the three (3) sticks of handrolled marijuana came from the appellant,
and (c) the appellant used Rommel as her agent to sell the three (3) sticks to the
informant. Moreover, if indeed the prosecution truly believed that such conspiracy
existed, it should not have willingly given its conformity to Rommel’s plea to the lesser
offense of illegal possession of prohibited drugs under Section 8, Article II of R.A. No.
6425, as amended. Having been caught in flagrante for selling marijuana, it was not
difficult to prove Rommel’s culpability under Section 4, Article II of the Act. Yet it readily
consented to his offer to plead guilty to the said lesser offense.

It was, however, established beyond any shadow of doubt and, therefore, with moral
certainty, that the appellant kept in her possession handrolled sticks of marijuana
placed in empty Hope, Philip Morris and Mark cigarette packs. 16 She does not have
any authority to possess them. She may have acquired them with the intention to sell
them for profit; but without proof of sale, she cannot be held liable under Section 4,
Article II of the Dangerous Drugs Act. For such possession, her liability is covered by
Section 8 of the said Article which penalizes possession or use of prohibited drugs. The
last paragraph thereof reads:

x x x

"The penalty of imprisonment ranging from six years and one day to twelve years and a
fine ranging from six thousand to twelve thousand pesos shall be imposed upon any
person who, unless authorized by law, shall posses or use Indian hemp."

Indian hemp is otherwise known as Marijuana. 17


Page 221 of 377

Appellant, therefore, may specifically be penalized under the aforesaid last paragraph of
Section 8, Article II of the Act. Applying the Indeterminate Sentence Law, 18 the penalty
of eight (8) years as Minimum to twelve (12) years as Maximum and a fine of P10,000.00
may then be imposed upon her.

The second assigned error is without merit. The evidence for the prosecution discloses
that the appellant placed the packs of marijuana sticks under the rolled pair of pants
which she was then carrying at the time she hurriedly left her shanty after noticing the
arrest of Rommel. When she was asked to spread it out, which she voluntarily did, the
package containing the packs of marijuana sticks were thus exposed in plain view to
the members of the team. A crime was thus committed in the presence of the policemen.
Pursuant to Section 5, Rule 113 and Section 12, Rule 126 of the Revised Rules of Court,
she could lawfully be arrested and searched for anything which may be used as proof of
the commission of an offense without the corresponding arrest and search warrants.
Her own counsel on cross-examination of prosecution witness Josephus Trangia further
obtained a re-affirmation of these facts, thus:

"Q You mean to say that when you saw Carmelina Tabar allegedly went (sic) out of the
shanty you only saw the white long pants and not the cigarettes?

A Only the pants.

Q Did you say that in order to find out what are the contents of the pants, you asked
her to open the pants. Isn’t it?

A Yes.

Q Now, after she opened it, what did you see?

A Three (3) packs of marijuana cigarettes.

Q Who among you in your team approached Carmelina Tabar?

A It was PFC Raul Tumakay." 19


Page 222 of 377

Even assuming ex gratia argumenti that the search and seizure were without a warrant,
the appellant had effectively waived her constitutional right relative thereto by
voluntarily submitting to the search and seizure. In People v. Malasugui, 20 this Court
ruled:

"When one voluntarily submits to a search and consents to have it made of his person
or premises, he is precluded from later complaining thereof (Cooley, Constitutional
Limitations, 8th ed., vol. I, page 631). The right to be secure from unreasonable search
may, like every right, be waived and such waiver may be made either expressly or
impliedly."

The exclusionary ruled relied upon by the appellant does not provide her a safe
refuge.chanrobles law library

Before We close this case, a final observation for the guidance of trial judges must be
made.

For the violation of Section 4, Article II of R.A. No. 6425, as amended, the trial court
imposed the penalty of reclusion perpetua. The penalty provided for therein is "life
imprisonment to death and a fine ranging from twenty thousand to thirty thousand
pesos." In view of Section 19(1), Article III of the 1987 Constitution which prohibits the
imposition of the death penalty, the maximum penalty then imposable thereunder
would only be life imprisonment. Life imprisonment, however, is not synonymous with
reclusion perpetua. We have reiterated this time and again 21 and admonished judges
to employ the proper legal terminology in the imposition of imprisonment penalties
because of their different accompanying legal accessories and effects. 22

IN THE LIGHT OF THE FOREGOING, judgment is hereby rendered modifying the


challenged Decision of Branch 15 of the Regional Trial Court of Cebu in Criminal Case
NO. CBU-14863 dated 17 December 1990 and, as modified, finding appellant
CARMELINA TABAR y CARMILOTES guilty beyond reasonable doubt of illegal
possession of marijuana under Section 8, Article II of R.A. No. 6425, otherwise known
as the Dangerous Drugs Act of 1972, as amended, and, applying the Indeterminate
Sentence Law, she is sentenced to suffer imprisonment of eight (8) years as minimum
to twelve (12) years as maximum and to pay a fine of Ten Thousand Pesos (P10,000.00).

Costs against the Appellant. SO ORDERED. Feliciano, Bidin, Romero and Melo, JJ.,
concur.
Page 223 of 377

G.R. No. 170180 November 23, 2007

ARSENIO VERGARA VALDEZ, Petitioner, vs.PEOPLE OF THE PHILIPPINES,


Respondent.

TINGA, J.:

The sacred right against an arrest, search or seizure without valid warrant is not only
ancient. It is also zealously safeguarded. The Constitution guarantees the right of the
people to be secure in their persons, houses, papers and effects against unreasonable
searches and seizures.1 Any evidence obtained in violation of said right shall be
inadmissible for any purpose in any proceeding. Indeed, while the power to search and
seize may at times be necessary to the public welfare, still it must be exercised and the
law implemented without contravening the constitutional rights of the citizens, for the
enforcement of no statute is of sufficient importance to justify indifference to the basic
principles of government.2

On appeal is the Decision3 of the Court of Appeals dated 28 July 2005, affirming the
Judgment4 of the Regional Trial Court (RTC), Branch 31, Agoo, La Union dated 31
March 2004 finding petitioner Arsenio Vergara Valdez guilty beyond reasonable doubt
of violating Section 11 of Republic Act No. 9165 (R.A. No. 9165)5 and sentencing him to
suffer the penalty of imprisonment ranging from eight (8) years and one (1) day of prision
mayor medium as minimum to fifteen (15) years of reclusion temporal medium as
maximum and ordering him to pay a fine of ₱350,000.00.6

I.

On 26 June 2003, petitioner was charged with violation of Section 11, par. 2(2) of R.A.
No. 9165 in an Information7 which reads:

That on or about the 17th day of March 2003, in the Municipality of Aringay, Province
of La Union, Philippines and within the jurisdiction of this Honorable Court, the above-
named accused, did then and there willfully, unlawfully and feloniously have in his
possession, control and custody dried marijuana leaves wrapped in a cellophane and
newspaper page, weighing more or less twenty-five (25) grams, without first securing
the necessary permit, license or prescription from the proper government agency.

CONTRARY TO LAW.8
Page 224 of 377

On arraignment, petitioner pleaded not guilty. Thereafter, trial on the merits ensued
with the prosecution presenting the three (3) barangay tanods of San Benito Norte,
Aringay, La Union namely, Rogelio Bautista (Bautista), Nestor Aratas (Aratas) and
Eduardo Ordoño (Ordoño), who arrested petitioner.

Bautista testified that at around 8:00 to 8:30 p.m. of 17 March 2003, he was conducting
the routine patrol along the National Highway in Barangay San Benito Norte, Aringay,
La Union together with Aratas and Ordoño when they noticed petitioner, lugging a bag,
alight from a mini-bus. The tanods observed that petitioner, who appeared suspicious
to them, seemed to be looking for something. They thus approached him but the latter
purportedly attempted to run away. They chased him, put him under arrest and
thereafter brought him to the house of Barangay Captain Orencio Mercado (Mercado)
where he, as averred by Bautista, was ordered by Mercado to open his bag. Petitioner’s
bag allegedly contained a pair of denim pants, eighteen pieces of eggplant and dried
marijuana leaves wrapped in newspaper and cellophane. It was then that petitioner was
taken to the police station for further investigation.9

Aratas and Ordoño corroborated Bautista’s testimony on most material points. On


cross-examination, however, Aratas admitted that he himself brought out the contents
of petitioner’s bag before petitioner was taken to the house of Mercado.10 Nonetheless,
he claimed that at Mercado’s house, it was petitioner himself who brought out the
contents of his bag upon orders from Mercado. For his part, Ordoño testified that it was
he who was ordered by Mercado to open petitioner’s bag and that it was then that they
saw the purported contents thereof.11

The prosecution likewise presented Police Inspector Valeriano Laya II (Laya), the forensic
chemist who conducted the examination of the marijuana allegedly confiscated from
petitioner. Laya maintained that the specimen submitted to him for analysis, a sachet
of the substance weighing 23.10 grams and contained in a plastic bag, tested positive
of marijuana. He disclosed on cross-examination, however, that he had knowledge
neither of how the marijuana was taken from petitioner nor of how the said substance
reached the police officers. Moreover, he could not identify whose marking was on the
inside of the cellophane wrapping the marijuana leaves.12

The charges were denied by petitioner. As the defense’s sole witness, he testified that at
around 8:30 p.m. on 17 March 2003, he arrived in Aringay from his place in Curro-oy,
Santol, La Union. After alighting from the bus, petitioner claimed that he went to the
house of a friend to drink water and then proceeded to walk to his brother’s house. As
he was walking, prosecution witness Ordoño, a cousin of his brother’s wife, allegedly
approached him and asked where he was going. Petitioner replied that he was going to
Page 225 of 377

his brother’s house. Ordoño then purportedly requested to see the contents of his bag
and appellant acceded. It was at this point that Bautista and Aratas joined them. After
inspecting all the contents of his bag, petitioner testified that he was restrained by the
tanod and taken to the house of Mercado. It was Aratas who carried the bag until they
reached their destination.13

Petitioner maintained that at Mercado’s house, his bag was opened by the tanod and
Mercado himself. They took out an item wrapped in newspaper, which later turned out
to be marijuana leaves. Petitioner denied ownership thereof. He claimed to have been
threatened with imprisonment by his arrestors if he did not give the prohibited drugs to
someone from the east in order for them to apprehend such person. As petitioner
declined, he was brought to the police station and charged with the instant offense.
Although petitioner divulged that it was he who opened and took out the contents of his
bag at his friend’s house, he averred that it was one of the tanod who did so at Mercado’s
house and that it was only there that they saw the marijuana for the first time.14

e. replied that he was going to his brother'en proceeded to walk to his brother'w

Finding that the prosecution had proven petitioner’s guilt beyond reasonable doubt, the
RTC rendered judgment against him and sentenced him to suffer indeterminate
imprisonment ranging from eight (8) years and one (1) day of prision mayor medium as
minimum to fifteen (15) years of reclusion temporal medium as maximum and ordered
him to pay a fine of ₱350,000.00.15

Aggrieved, petitioner appealed the decision of the RTC to the Court of Appeals.1âwphi1
On 28 July 2005, the appellate court affirmed the challenged decision. The Court of
Appeals, finding no cogent reason to overturn the presumption of regularity in favor of
the barangay tanod in the absence of evidence of ill-motive on their part, agreed with
the trial court that there was probable cause to arrest petitioner. It observed further:

That the prosecution failed to establish the chain of custody of the seized marijuana is
of no moment. Such circumstance finds prominence only when the existence of the
seized prohibited drugs is denied. In this case, accused-appellant himself testified that
the marijuana wrapped in a newspaper was taken from his bag. The corpus delicti of
the crime, i.e.[,] the existence of the marijuana and his possession thereof, was amply
proven by accused-appellant Valdez’s own testimony.16

In this appeal, petitioner prays for his acquittal and asserts that his guilt of the crime
charged had not been proven beyond reasonable doubt. He argues, albeit for the first
Page 226 of 377

time on appeal, that the warrantless arrest effected against him by the barangay tanod
was unlawful and that the warrantless search of his bag that followed was likewise
contrary to law. Consequently, he maintains, the marijuana leaves purportedly seized
from him are inadmissible in evidence for being the fruit of a poisonous tree.

Well-settled is the rule that the findings of the trial court on the credibility of witnesses
and their testimonies are accorded great respect and weight, in the absence of any clear
showing that some facts and circumstances of weight or substance which could have
affected the result of the case have been overlooked, misunderstood or misapplied.17

After meticulous examination of the records and evidence on hand, however, the Court
finds and so holds that a reversal of the decision a quo under review is in order.

II.

At the outset, we observe that nowhere in the records can we find any objection by
petitioner to the irregularity of his arrest before his arraignment. Considering this and
his active participation in the trial of the case, jurisprudence dictates that petitioner is
deemed to have submitted to the jurisdiction of the trial court, thereby curing any defect
in his arrest. The legality of an arrest affects only the jurisdiction of the court over his
person.18 Petitioner’s warrantless arrest therefore cannot, in itself, be the basis of his
acquittal.

However, to determine the admissibility of the seized drugs in evidence, it is


indispensable to ascertain whether or not the search which yielded the alleged
contraband was lawful. The search, conducted as it was without a warrant, is justified
only if it were incidental to a lawful arrest.19 Evaluating the evidence on record in its
totality, as earlier intimated, the reasonable conclusion is that the arrest of petitioner
without a warrant is not lawful as well.

Petitioner maintains, in a nutshell, that after he was approached by the tanod and asked
to show the contents of his bag, he was simply herded without explanation and taken
to the house of the barangay captain. On their way there, it was Aratas who carried his
bag. He denies ownership over the contraband allegedly found in his bag and asserts
that he saw it for the first time at the barangay captain’s house.
Page 227 of 377

Even casting aside petitioner’s version and basing the resolution of this case on the
general thrust of the prosecution evidence, the unlawfulness of petitioner’s arrest stands
out just the same.

Section 5, Rule 113 of the Rules on Criminal Procedure provides the only occasions on
which a person may be arrested without a warrant, to wit:

Section 5. Arrest without warrant; when lawful.—A peace officer or a private person
may, without a warrant, arrest a person:

(a) When, in his presence, the person to be arrested has committed, is actually
committing, or is attempting to commit an offense;

(b) When an offense has just been committed and he has probable cause to believe based
on personal knowledge of facts or circumstances that the person to be arrested has
committed it; and

(c) When the person to be arrested is a prisoner who has escaped from a penal
establishment or place where he is serving final judgment or temporarily confined while
his case is pending, or has escaped while being transferred from one confinement to
another.

xxx

It is obvious that based on the testimonies of the arresting barangay tanod, not one of
these circumstances was obtaining at the time petitioner was arrested. By their own
admission, petitioner was not committing an offense at the time he alighted from the
bus, nor did he appear to be then committing an offense.20 The tanod did not have
probable cause either to justify petitioner’s warrantless arrest.

For the exception in Section 5(a), Rule 113 to operate, this Court has ruled that two (2)
elements must be present: (1) the person to be arrested must execute an overt act
indicating that he has just committed, is actually committing, or is attempting to commit
a crime; and (2) such overt act is done in the presence or within the view of the arresting
officer.21 Here, petitioner’s act of looking around after getting off the bus was but
natural as he was finding his way to his destination. That he purportedly attempted to
run away as the tanod approached him is irrelevant and cannot by itself be construed
Page 228 of 377

as adequate to charge the tanod with personal knowledge that petitioner had just
engaged in, was actually engaging in or was attempting to engage in criminal activity.
More importantly, petitioner testified that he did not run away but in fact spoke with
the barangay tanod when they approached him.

Even taking the prosecution’s version generally as the truth, in line with our assumption
from the start, the conclusion will not be any different. It is not unreasonable to expect
that petitioner, walking the street at night, after being closely observed and then later
tailed by three unknown persons, would attempt to flee at their approach. Flight per se
is not synonymous with guilt and must not always be attributed to one’s consciousness
of guilt.22 Of persuasion was the Michigan Supreme Court when it ruled in People v.
Shabaz23 that "[f]light alone is not a reliable indicator of guilt without other
circumstances because flight alone is inherently ambiguous." Alone, and under the
circumstances of this case, petitioner’s flight lends itself just as easily to an innocent
explanation as it does to a nefarious one.

Moreover, as we pointed out in People v. Tudtud,24 "[t]he phrase ‘in his presence’
therein, connot[es] penal knowledge on the part of the arresting officer. The right of the
accused to be secure against any unreasonable searches on and seizure of his own body
and any deprivation of his liberty being a most basic and fundamental one, the statute
or rule that allows exception to the requirement of a warrant of arrest is strictly
construed. Its application cannot be extended beyond the cases specifically provided by
law."25

Indeed, the supposed acts of petitioner, even assuming that they appeared dubious,
cannot be viewed as sufficient to incite suspicion of criminal activity enough to validate
his warrantless arrest.26 If at all, the search most permissible for the tanod to conduct
under the prevailing backdrop of the case was a stop-and-frisk to allay any suspicion
they have been harboring based on petitioner’s behavior. However, a stop-and-frisk
situation, following Terry v. Ohio,27 must precede a warrantless arrest, be limited to
the person’s outer clothing, and should be grounded upon a genuine reason, in light of
the police officer’s experience and surrounding conditions, to warrant the belief that the
person detained has weapons concealed about him.28

Accordingly, petitioner’s waiver of his right to question his arrest notwithstanding, the
marijuana leaves allegedly taken during the search cannot be admitted in evidence
against him as they were seized during a warrantless search which was not lawful.29
As we pronounced in People v. Bacla-an —
Page 229 of 377

A waiver of an illegal warrantless arrest does not also mean a waiver of the
inadmissibility of evidence seized during an illegal warrantless arrest. The following
searches and seizures are deemed permissible by jurisprudence: (1) search of moving
vehicles (2) seizure in plain view (3) customs searches (4) waiver or consent searches (5)
stop and frisk situations (Terry Search) and (6) search incidental to a lawful arrest. The
last includes a valid warrantless search and seizure pursuant to an equally valid
warrantless arrest, for, while as a rule, an arrest is considered legitimate if effected with
a valid warrant of arrest, the Rules of Court recognize permissible warrantless arrests,
to wit: (1) arrests in flagrante delicto, (2) arrests effected in hot pursuit, and, (3) arrests
of escaped prisoners.30

When petitioner was arrested without a warrant, he was neither caught in flagrante
delicto committing a crime nor was the arrest effected in hot pursuit. Verily, it cannot
therefore be reasonably argued that the warrantless search conducted on petitioner was
incidental to a lawful arrest.

In its Comment, the Office of the Solicitor General posits that apart from the warrantless
search being incidental to his lawful arrest, petitioner had consented to the search. We
are not convinced. As we explained in Caballes v. Court of Appeals31 —

Doubtless, the constitutional immunity against unreasonable searches and seizures is


a personal right which may be waived. The consent must be voluntary in order to
validate an otherwise illegal detention and search, i.e., the consent is unequivocal,
specific, and intelligently given, uncontaminated by any duress or coercion. Hence,
consent to a search is not to be lightly inferred, but must be shown by clear and
convincing evidence. The question whether a consent to a search was in fact voluntary
is a question of fact to be determined from the totality of all the circumstances. Relevant
to this determination are the following characteristics of the person giving consent and
the environment in which consent is given: (1) the age of the defendant; (2) whether he
was in a public or secluded location; (3) whether he objected to the search or passively
looked on; (4) the education and intelligence of the defendant; (5) the presence of
coercive police procedures; (6) the defendant's belief that no incriminating evidence will
be found; (7) the nature of the police questioning; (8) the environment in which the
questioning took place; and (9) the possibly vulnerable subjective state of the person
consenting. It is the State which has the burden of proving, by clear and positive
testimony, that the necessary consent was obtained and that it was freely and
voluntarily given.32

In the case at bar, following the theory of the prosecution— albeit based on conflicting
testimonies on when petitioner’s bag was actually opened, it is apparent that petitioner
was already under the coercive control of the public officials who had custody of him
Page 230 of 377

when the search of his bag was demanded. Moreover, the prosecution failed to prove
any specific statement as to how the consent was asked and how it was given, nor the
specific words spoken by petitioner indicating his alleged "consent." Even granting that
petitioner admitted to opening his bag when Ordoño asked to see its contents, his
implied acquiescence, if at all, could not have been more than mere passive conformity
given under coercive or intimidating circumstances and hence, is considered no consent
at all within the contemplation of the constitutional guarantee.33 As a result,
petitioner’s lack of objection to the search and seizure is not tantamount to a waiver of
his constitutional right or a voluntary submission to the warrantless search and
seizure.34

III.

Notably, the inadmissibility in evidence of the seized marijuana leaves for being the fruit
of an unlawful search is not the lone cause that militates against the case of the
prosecution. We likewise find that it has failed to convincingly establish the identity of
the marijuana leaves purportedly taken from petitioner’s bag.

In all prosecutions for violation of the Dangerous Drugs Act, the following elements must
concur: (1) proof that the transaction took place; and (2) presentation in court of the
corpus delicti or the illicit drug as evidence.35 The existence of dangerous drugs is a
condition sine qua non for conviction for the illegal sale of dangerous drugs, it being the
very corpus delicti of the crime.36

In a line of cases, we have ruled as fatal to the prosecution’s case its failure to prove
that the specimen submitted for laboratory examination was the same one allegedly
seized from the accused.37 There can be no crime of illegal possession of a prohibited
drug when nagging doubts persist on whether the item confiscated was the same
specimen examined and established to be the prohibited drug.38 As we discussed in
People v. Orteza39 , where we deemed the prosecution to have failed in establishing all
the elements necessary for conviction of appellant for illegal sale of shabu –

First, there appears nothing in the record showing that police officers complied with the
proper procedure in the custody of seized drugs as specified in People v. Lim, i.e., any
apprehending team having initial control of said drugs and/or paraphernalia should,
immediately after seizure or confiscation, have the same physically inventoried and
photographed in the presence of the accused, if there be any, and or his representative,
who shall be required to sign the copies of the inventory and be given a copy thereof.
The failure of the agents to comply with the requirement raises doubt whether what was
submitted for laboratory examination and presented in court was actually recovered
Page 231 of 377

from appellant. It negates the presumption that official duties have been regularly
performed by the police officers.

In People v. Laxa, where the buy-bust team failed to mark the confiscated marijuana
immediately after the apprehension of the accused, the Court held that the deviation
from the standard procedure in anti-narcotics operations produced doubts as to the
origins of the marijuana. Consequently, the Court concluded that the prosecution failed
to establish the identity of the corpus delicti.

The Court made a similar ruling in People v. Kimura, where the Narcom operatives failed
to place markings on the seized marijuana at the time the accused was arrested and to
observe the procedure and take custody of the drug.

More recently, in Zarraga v. People, the Court held that the material inconsistencies
with regard to when and where the markings on the shabu were made and the lack of
inventory on the seized drugs created reasonable doubt as to the identity of the corpus
delicti. The Court thus acquitted the accused due to the prosecution’s failure to
indubitably show the identity of the shabu.

In the case at bar, after the arrest of petitioner by the barangay tanod, the records only
show that he was taken to the house of the barangay captain and thereafter to the police
station. The Joint Affidavit40 executed by the tanod merely states that they confiscated
the marijuana leaves which they brought to the police station together with petitioner.
Likewise, the Receipt41 issued by the Aringay Police Station merely acknowledged
receipt of the suspected drugs supposedly confiscated from petitioner.

Not only did the three tanod contradict each other on the matter of when petitioner’s
bag was opened, they also gave conflicting testimony on who actually opened the same.
The prosecution, despite these material inconsistencies, neglected to explain the
discrepancies. Even more damning to its cause was the admission by Laya, the forensic
chemist, that he did not know how the specimen was taken from petitioner, how it
reached the police authorities or whose marking was on the cellophane wrapping of the
marijuana. The non-presentation, without justifiable reason, of the police officers who
conducted the inquest proceedings and marked the seized drugs, if such was the case,
is fatal to the case. Plainly, the prosecution neglected to establish the crucial link in the
chain of custody of the seized marijuana leaves from the time they were first allegedly
discovered until they were brought for examination by Laya.
Page 232 of 377

The Court of Appeals found as irrelevant the failure of the prosecution to establish the
chain of custody over the seized marijuana as such "[f]inds prominence only when the
existence of the seized prohibited drug is denied."42 We cannot agree.

To buttress its ratiocination, the appellate court narrowed on petitioner’s testimony that
the marijuana was taken from his bag, without taking the statement in full context.43
Contrary to the Court of Appeals’ findings, although petitioner testified that the
marijuana was taken from his bag, he consistently denied ownership thereof.44
Furthermore, it defies logic to require a denial of ownership of the seized drugs before
the principle of chain of custody comes into play.

The onus of proving culpability in criminal indictment falls upon the State. In
conjunction with this, law enforcers and public officers alike have the corollary duty to
preserve the chain of custody over the seized drugs. The chain of evidence is constructed
by proper exhibit handling, storage, labeling and recording, and must exist from the
time the evidence is found until the time it is offered in evidence. Each person who takes
possession of the specimen is duty-bound to detail how it was cared for, safeguarded
and preserved while in his or her control to prevent alteration or replacement while in
custody. This guarantee of the integrity of the evidence to be used against an accused
goes to the very heart of his fundamental rights.

The presumption of regularity in the performance of official duty invoked by the


prosecution and relied upon by the courts a quo cannot by itself overcome the
presumption of innocence nor constitute proof of guilt beyond reasonable doubt.45
Among the constitutional rights enjoyed by an accused, the most primordial yet often
disregarded is the presumption of innocence. This elementary principle accords every
accused the right to be presumed innocent until the contrary is proven beyond
reasonable doubt. Thus, the burden of proving the guilt of the accused rests upon the
prosecution.

Concededly, the evidence of the defense is weak and uncorroborated. Nevertheless, this
"[c]annot be used to advance the cause of the prosecution as its evidence must stand or
fall on its own weight and cannot be allowed to draw strength from the weakness of the
defense."46 Moreover, where the circumstances are shown to yield two or more
inferences, one inconsistent with the presumption of innocence and the other
compatible with the finding of guilt, the court must acquit the accused for the reason
that the evidence does not satisfy the test of moral certainty and is inadequate to
support a judgment of conviction.47
Page 233 of 377

Drug addiction has been invariably denounced as "an especially vicious crime,"48 and
"one of the most pernicious evils that has ever crept into our society,"49 for those who
become addicted to it "not only slide into the ranks of the living dead, what is worse,
they become a grave menace to the safety of law-abiding members of society,"50 whereas
"peddlers of drugs are actually agents of destruction."51 Indeed, the havoc created by
the ruinous effects of prohibited drugs on the moral fiber of society cannot be
underscored enough. However, in the rightfully vigorous campaign of the government to
eradicate the hazards of drug use and drug trafficking, it cannot be permitted to run
roughshod over an accused’s right to be presumed innocent until proven to the contrary
and neither can it shirk from its corollary obligation to establish such guilt beyond
reasonable doubt.

In this case, the totality of the evidence presented utterly fails to overcome the
presumption of innocence which petitioner enjoys. The failure of the prosecution to
prove all the elements of the offense beyond reasonable doubt must perforce result in
petitioner’s exoneration from criminal liability.

IV.

A final word. We find it fitting to take this occasion to remind the courts to exercise the
highest degree of diligence and prudence in deliberating upon the guilt of accused
persons brought before them, especially in light of the fundamental rights at stake. Here,
we note that the courts a quo neglected to give more serious consideration to certain
material issues in the determination of the merits of the case. We are not oblivious to
the fact that in some instances, law enforcers resort to the practice of planting evidence
to extract information or even harass civilians. Accordingly, courts are duty-bound to
be "[e]xtra vigilant in trying drug cases lest an innocent person be made to suffer the
unusually severe penalties for drug offenses."52 In the same vein, let this serve as an
admonition to police officers and public officials alike to perform their mandated duties
with commitment to the highest degree of diligence, righteousness and respect for the
law.

WHEREFORE, the assailed Decision is REVERSED and SET ASIDE. Petitioner Arsenio
Vergara Valdez is ACQUITTED on reasonable doubt. The Director of the Bureau of
Corrections is directed to cause the immediate release of petitioner, unless the latter is
being lawfully held for another cause; and to inform the Court of the date of his release,
or the reasons for his continued confinement, within ten (10) days from notice. No costs.

SO ORDERED.
Page 234 of 377

G.R. No. L-95630 June 18, 1992

SPOUSES LEOPOLDO and MA. LUISA VEROY, petitioners, vs. THE HON.
WILLIAM L. LAYAGUE, Presiding Judge, Branch XIV, Regional Trial Court
at Davao City; and BRIG. GEN. PANTALEON DUMLAO, Commanding
General, PC-Criminal Investigation Service, respondents.

PARAS, J.:

This was originally a petition for certiorari, mandamus and prohibition under Rule 65
of the Rules of Court: certiorari, to review the Order of the respondent Judge dated
October 2, 1990 denying herein petitioner's Motion for Hospital Confinement;
mandamus, to compel respondent Judge to resolve petitioners' long pending motion for
bail; and prohibition, to enjoin further proceedings on the ground that the legal basis
therefore is unconstitutional for being violative of the due process and equal protection
clauses of the Constitution.

The facts of this case are as follows:

Petitioners are husband and wife who owned and formerly resided at No. 13 Isidro St.,
Skyline Village. Catalunan Grande, Davao City. When petitioner Leopoldo Veroy was
promoted to the position of Assistant Administrator of the Social Security System
sometime in June, 1988, he and his family transferred to 130 K-8th St., East Kamias,
Quezon City, where they are presently residing. The care and upkeep of their residence
in Davao City was left to two (2) houseboys, Jimmy Favia and Eric Burgos, who had
their assigned quarters at a portion of the premises. The Veroys would occasionally send
money to Edna Soguilon for the salary of the said houseboys and other expenses for the
upkeep of their house. While the Veroys had the keys to the interior of the house, only
the key to the kitchen, where the circuit breakers were located, was entrusted to Edna
Soguilon to give her access in case of an emergency. Hence, since 1988, the key to the
master's bedroom as well as the keys to the children's rooms were retained by herein
Petitioners so that neither Edna Soguilon nor the caretakers could enter the house.

On April 12, 1990, Capt. Reynaldo Obrero of the Talomo Patrol Station, PC/INP, acting
upon a directive issued by Metrodiscom Commander Col. Franco Calida, raided the
house of herein petitioners in Davao City on information that the said residence was
being used as a safehouse of rebel soldiers. They were able to enter the yard with the
help of the caretakers but did not enter the house since the owner was not present and
they did not have a search warrant. Petitioner Ma. Luisa was contacted by telephone in
her Quezon City residence by Capt. Obrero to ask permission to search the house in
Davao City as it was reportedly being used as a hideout and recruitment center of rebel
Page 235 of 377

soldiers. Petitioner Ma. Luisa Veroy responded that she is flying to Davao City to witness
the search but relented if the search would not be conducted in the presence of Major
Ernesto Macasaet, an officer of the PC/INP, Davao City and a long time family friend of
the Veroys. The authority given by Ma. Luisa Veroy was relayed by Capt. Obrero to Major
Macasaet who answered that Ma. Luisa Veroy has called him twice by telephone on the
matter and that the permission was given on the condition that the search be conducted
in his presence.

The following day, Capt. Obrero and Major Macasaet met at the house of herein
petitioners in Skyline Village to conduct the search pursuant to the authority granted
by petitioner Ma. Luisa Veroy. The caretakers facilitated their entry into the yard, and
using the key entrusted to Edna Soguilon, they were able to gain entrance into the
kitchen. However, a locksmith by the name of George Badiang had to be employed to
open the padlock of the door leading to the children's room. Capt. Obrero and Major
Macasaet then entered the children's room and conducted the search. Capt. Obrero
recovered a .45 cal. handgun with a magazine containing seven (7) live bullets in a black
clutch bag inside an unlocked drawer. Three (3) half-full jute sacks containing printed
materials of RAM-SFP (samples of which were attached as Annexes "H" and "H-1" of the
petition) (Rollo, pp. 49-55) were also found in the children's room. A search of the
children's recreation and study area revealed a big travelling bag containing assorted
polo shirts, men's brief, two (2) pieces polo barong and short sleeve striped gray polo.
sweat shirt, two (2) pairs men's socks, a towel made in U.S.A., one blanket, a small
black bag, Gandhi brand, containing a book entitled "Islamic Revolution Future Path of
the Nation", a road map of the Philippines, a telescope, a plastic bag containing assorted
medicines and religious pamphlets was found in the master's bedroom. Sgt. Leo
Justalero was instructed by Capt. Obrero to make an inventory and receipt of the
articles seized, in the house (Annex "F" of the Petition, Rollo, p. 48). Said receipt was
signed by Eric Burgos, one of the caretakers, and George Badiang, the locksmith, as
witnesses. Sgt. Justalero turned over the articles to Sgt. Rodolfo Urbano at the police
station.

The case was referred for preliminary investigation to Quezon City Assistant Prosecutor
Rodolfo Ponferrada who was designated Acting Provincial Prosecutor for Davao City by
the Department of Justice through Department Order No. 88 dated May 16, 1990. In a
resolution dated August 6, 1990, Fiscal Ponferrada recommended the filing of an
information against herein petitioners for Violation of Presidential Decree No. 1866
(Illegal Possession of Firearms and Ammunitions in Furtherance of Rebellion) (Annex
"L" of the Petition, Rollo, p. 71). Hence, on August 8, 1990. an Information for the said
offense was filed by the Office of the City Prosecutor of Davao City before the Regional
Trial Court, 11th Judicial Region, Davao City, docketed as Criminal Case No. 20595-90
and entitled "People of the Philippines v. Atty. Leopoldo Veroy and Mrs. Maria Luisa
Veroy" (Annex "K" of the Petition, Rollo, p. 70). No bail was recommended by the
prosecution.
Page 236 of 377

The aforementioned resolution dated August 6, 1990 of Fiscal Ponferrada was received
by the petitioners on August 13, 1990. On the same day, the latter filed a Motion for
Bail before herein respondent Judge Layague which was denied on August 17, 1990 for
being premature since at that time, petitioners had not yet been arrested. Despite the
fact that the warrants for their arrest have not yet been served on them, herein
petitioners voluntarily surrendered themselves to Brig. Gen. Pantaleon Dumlao, PC-CIS
Chief, since it was the CIS that initiated the complaint. However, the latter refused to
receive them on the ground that his office has not yet received copies of their warrants
of arrest.

In the meantime, on August 15, 1990, herein petitioners were admitted to the St. Luke's
Hospital for various ailments brought about or aggravated by the stress and anxiety
caused by the filing of the criminal complaint. On August 17, 1990, Brig. Gen. Dumlao
granted their request that they be allowed to be confined at the hospital and placed
under guard thereat.

In an Indorsement dated August 20, 1990, the CIS through Capt. Benjamin de los
Santos, made its return to the trial court informing the latter of the voluntary surrender
of herein petitioners and the fact that they were under hospital confinement. Herein
Petitioner reiterated their Motion for Bail. In an Order dated August 24, 1990 (Annex
"M" of the Petition, Rollo, p. 74), the hearing for the Motion for Ball was set for August
31, 1990 to enable the prosecution to present evidence it opposition to said motion. The
prosecution filed its written opposition (Annex "N" of the Petition, Rollo, p. 75) on August
28, 1990, arguing that the evidence of petitioners' guilt was strong and thereafter
presented its evidence.

On September 21, 1990, respondent Judge required the CIS to produce the bodies of
herein petitioners on October 1, 1990 for arraignment (Annex "O" of the Petition, Rollo,
p. 76). Upon their arraignment, herein Petitioners entered a plea of not guilty and filed
an "Urgent Motion for Hospital Confinement" (Annex "OO" of the Petition Rollo, p. 77)
which was denied by the court in its Order dated October 2, 1990 (Annex "P" of the
Petition, Rollo, p. 80). It likewise ordered their commitment at the Davao City
Rehabilitation Center, Ma-a, Davao City pending trial on the merits. Herein petitioners
argued orally a motion for reconsideration which was opposed by the prosecution. At
the conclusion thereof, the court a quo issued a second order annex "Q" of the Petition,
Rollo, p. 83) denying then motion for reconsideration and as to the alternative prayer to
reopen the motion for hospital confinement, set the continuance thereof to October 17,
1990. It was further ordered that the petitioners shall remain under the custody of the
PC-CIS pending resolution of the case.
Page 237 of 377

Meanwhile, petitioners were returned to the St. Luke's Hospital where their physical
condition remained erratic. On or about October 18, 1990, herein petitioners were
informed that Brig. Gen. Dumlao had issued a directive for their transfer from the St.
Luke's Hospital to Camp Crame on the basis of the October 2, 1990 Order (Annex "Q"
of the Petition, Rollo, p. 83). Petitioners made representations that the tenor of the court
order warranted maintenance of the status quo, i.e., they were to continue their hospital
confinement. However, Brig, Gen. Dumlao informed them that unless otherwise
restrained by the court, they would proceed with their transfer pursuant to the order of
the trial court.

Hence, this petition on October 25, 1990 this Court issued a Temporary Restraining
Order, effective immediately and continuing until further orders from this Court,
ordering: (a) respondent Hon. William L. Layague to refrain from further proceeding with
petitioners' "Motion for Hospital Confinement" in Criminal Case No. 20595-90 entitled
"People of the Philippines v. Leopoldo Veroy and Ma. Luisa Veroy"; and (b) respondent
Brig. Gen. Pantaleon Dumlao to refrain from transferring petitioners from the St. Luke's
Hospital (Rollo, pp. 84-A to 84-C).

On November 2, 1990, respondent Judge issued an order denying petitioners' Motion


for Bail (Annex "A" of the Second Supplemental Petition, Rollo, p. 133). Petitioners filed
a Supplemental Petition on November 7, 1990 (Rollo, P. 105) and a Second
Supplemental Petition on November 16, 1990 (Rollo, p. 120) which sought to review the
order of the trial court dated November 2, 1990 denying their petition for bail.

Acting on the Supplemental Petition filed by Petitioners and taking into consideration
several factors such as: a) that the possibility that they will flee or evade the processes
of the court is fairly remote; b) their poor medical condition; and c) the matters in their
Second Supplemental Petition especially since the prosecution's evidence refers to
constructive possession of the disputed firearms in Davao City through the two (2)
caretakers while petitioners lived in Manila since 1988, this Court, on November 20,
1990, granted petitioners' provisional liberty and set the bail bond at P20,000.00 each
(Rollo, p. 141). Petitioners posted a cash bond in the said amount on November 23, 1990
(Rollo, pp. 143-145).

The petition was given due course on July 16, 1991 (Rollo, p. 211). Respondents adopted
their Comment dated December 28, 1990 (Rollo, pp. 182-191) as their Memorandum
while, petitioners filed their Memorandum on September 9, 1991 (Rollo, pp. 218-269).

As submitted by the respondents, and accepted by petitioners, the petition for


mandamus to compel respondent Judge to resolve petitioners' Motion for Bail, and the
Page 238 of 377

petition for certiorari to review the order of respondent judge initially denying their
Motion for Hospital Confinement, were rendered moot and academic by the resolutions
of this Court dated November 20, 1990 and October 25, 1990, respectively. What
remains to be resolved is the petition for prohibition where petitioners raised the
following issues:

1. Presidential Decree No. 1866, or at least the third paragraph of Section 1 thereof, is
unconstitutional for being violative of the due process and equal protection clauses of
the Constitution;

2. Presidential Decree No. 1866 has been repealed by Republic Act No. 6968;

3. Assuming the validity of Presidential Decree No. 1866 the respondent judge gravely
abused his discretion in admitting in evidence certain articles which were clearly
inadmissible for being violative of the prohibition against unreasonable searches and
seizures.

The issue of constitutionality of Presidential Decree No. 1866 has been laid to rest in
the case of Misolas v. Panga, G.R. No. 83341, January 30, 1990 (181 SCRA 648), where
this Court held that the declaration of unconstitutionality of the third paragraph of
Section 1 of Presidential Decree No. 1866 is wanting in legal basis since it is neither a
bill of attainder nor does it provide a possibility of a double jeopardy.

Likewise, petitioners' contention that Republic Act 6968 has repealed Presidential
Decree No. 1866 is bereft of merit. It is a cardinal rule of statutory construction that
where the words and phrases of a statute are not obscure or ambiguous. its meaning
and the intention of the legislature must be determined from the language employed,
and where there is no ambiguity in the words, there is no room for construction
(Provincial Board of Cebu v. Presiding Judge of Cebu, CFI, Br. IV, G.R. No. 34695, March
7, 1989 [171 SCRA 1]). A perusal of the aforementioned laws would reveal that the
legislature provided for two (2) distinct offenses: (1) illegal possession of firearms under
Presidential Decree No. 1866; and (2) rebellion, coup d' etat, sedition and disloyalty
under Republic Act 6968; evidently involving different subjects which were not clearly
shown to have eliminated the others.

But petitioners contend that Section 1 of Presidential Decree No. 1866 is couched in
general or vague terms. The terms "deal in", "acquire", "dispose" or "possess" are capable
of various interpretations such that there is no definiteness as to whether or not the
definition includes "constructive possession" or how the concept of constructive
Page 239 of 377

possession should be applied. Petitioners were not found in actual possession of the
firearm and ammunitions. They were in Quezon City while the prohibited articles were
found in Davao City. Yet they were being charged under Presidential Decree No. 1866
upon the sole circumstance that the house wherein the items were found belongs to
them (Memorandum for Petitioners, Rollo, pp. 242-244).

Otherwise stated, other than their ownership of the house in Skyline Village, there was
no other evidence whatsoever that herein petitioners possessed or had in their control
the items seized (Ibid., pp. 248-250). Neither was it shown that they had the intention
to possess the Firearms or to further rebellion (Ibid., P. 252).

In a similar case, the revolver in question was found in appellant's store and the
question arouse whether he had possession or custody of it within the meaning of the
law.

This Court held that:

The animus possidendi must be proved in opium cases where the prohibited drug was
found on the premises of the accused and the same rule is applicable to the possession
of firearms. The appellant denied all knowledge of the existence of the revolver, and the
Government's principal witness stated that there were a number of employees in the
store. The only testimony which tends to show that the appellant had the possession or
custody of this revolver is the inference drawn from the fact that it was found in his
store, but we think that this inference is overcome by the positive testimony of the
appellant, when considered with the fact that there were a number of employees in the
store, who, of course, could have placed the revolver in the secret place where it was
found without the knowledge of the appellant. At least there is a very serious doubt
whether he knew of the existence of this revolver. In such case the doubt must be
resolved in favor of the appellant. (U.S. v. Jose and Tan Bo., 34 Phil. 724 [1916])

But more importantly, petitioners question the admissibility in evidence of the articles
seized in violation of their constitutional right against unreasonable search and seizure.

Petitioners aver that while they concede that Capt. Obrero had permission from Ma.
Luisa Veroy to break open the door of their residence, it was merely for the purpose of
ascertaining thereat the presence of the alleged "rebel" soldiers. The permission did not
include any authority to conduct a room to room search once inside the house. The
items taken were, therefore, products of an illegal search, violative of their constitutional
rights As such, they are inadmissible in evidence against them.
Page 240 of 377

The Constitution guarantees the right of the people to be secure in their persons,
houses, papers and effects against unreasonable searches and seizures (Article III,
Section 2 of the 1987 Constitution). However, the rule that searches and seizures must
be supported by a valid warrant is not an absolute one. Among the recognized exceptions
thereto are: (1) a search incidental to an arrest; (2) a search of a moving vehicle; and (3)
seizure of evidence in plain view (People v. Lo Ho Wing, G.R. No. 88017, January 21,
1991 [193 SCRA 122]).

None of these exceptions pertains to the case at bar. The reason for searching the house
of herein petitioners is that it was reportedly being used as a hideout and recruitment
center for rebel soldiers. While Capt. Obrero was able to enter the compound, he did not
enter the house because he did not have a search warrant and the owners were not
present. This shows that he himself recognized the need for a search warrant, hence,
he did not persist in entering the house but rather contacted the Veroys to seek
permission to enter the same. Permission was indeed granted by Ma. Luisa Veroy to
enter the house but only to ascertain the presence of rebel soldiers. Under the
circumstances it is undeniable that the police officers had ample time to procure a
search warrant but did not.

In a number of cases decided by this Court, (Guazon v. De Villa, supra.; People v.


Aminnudin, G.R. No. L-74869, July 6, 1988 [163 SCRA 402]; Alih v. Castro, G.R. No. L-
69401, June 23, 1987 [151 SCRA 279]), warrantless searches were declared illegal
because the officials conducting the search had every opportunity to secure a search
Warrant. The objects seized, being products of illegal searches, were inadmissible in
evidence in the criminal actions subsequently instituted against the accused-appellants
(People v. Cendana, G.R. No. 84715, October 17, 1990 [190 SCRA 538]).

Undeniably, the offense of illegal possession of firearms is malum prohibitum but it does
not follow that the subject thereof is necessarily illegal per se. Motive is immaterial in
mala prohibita but the subjects of this kind of offense may not be summarily seized
simply because they are prohibited. A search warrant is still necessary. Hence, the rule
having been violated and no exception being applicable, the articles seized were
confiscated illegally and are therefore protected by the exclusionary principle. They
cannot be used as evidence against the petitioners in the criminal action against them
for illegal possession of firearms. (Roan v. Gonzales, 145 SCRA 689-690 [1986]). Besides,
assuming that there was indeed a search warrant, still in mala prohibita, while there is
no need of criminal intent, there must be knowledge that the same existed. Without the
knowledge or voluntariness there is no crime.
Page 241 of 377

PREMISES CONSIDERED, the petition as granted and the criminal case against the
petitioners for illegal possession of firearms is DISMISSED.

SO ORDERED.
Narvasa, C.J., Gutierrez, Jr., Cruz, Feliciano, Padilla, Bidin, Griño-Aquino, Medialdea,
Regalado, Davide, Jr., Romero and Bellosillo, JJ., concur.
Nocon, J., is on leave.
Page 242 of 377

G.R. Nos. 102009-10 July 6, 1994

PEOPLE OF THE PHILIPPINES, Plaintiff-Appellee, v. ROLANDO DE GRACIA,


CHITO HENSON and JOHN DOES, Accused. ROLANDO DE GRACIA, Accused-
Appellant.

REGALADO, J.:

The incidents involved in this case took place at the height of the coup d’etat staged in
December, 1989 by ultra-rightist elements headed by the Reform the Armed Forces
Movement-Soldiers of the Filipino People (RAM-SFP) against the Government. At that
time, various government establishments and military camps in Metro Manila were
being bombarded by the rightist group with their "tora-tora" planes. At around midnight
of November 30, 1989, the 4th Marine Battalion of the Philippine Marines occupied
Villamor Air Base, while the Scout Rangers took over the Headquarters of the Philippine
Army, the Army Operations Center, and Channel 4, the government television station.
Also, some elements of the Philippine Army coming from Fort Magsaysay occupied the
Greenhills Shopping Center in San Juan, Metro Manila. 1

Accused-appellant Rolando de Gracia was charged in two separate informations for


illegal possession of ammunition and explosives in furtherance of rebellion, and for
attempted homicide, docketed as Criminal Cases Nos. Q-90-11755 and Q-90-11756,
respectively, which were tried jointly by the Regional Trial Court of Quezon City, Branch
103.chanrobles.com : virtual law library

In Criminal Case No. Q-90-11755, Rolando de Gracia, Chito Henson and several John
Does whose true names and identities have not as yet been ascertained, were charged
with the crime of illegal possession of ammunition and explosives in the furtherance of
rebellion, penalized under Section 1, paragraph 3, of Presidential Decree No. 1866,
allegedly committed as follows:

"That on or about the 5th day of DECEMBER, 1989, in QUEZON CITY, METRO MANILA,
PHILIPPINES, and within the jurisdiction of this Honorable Court, the above-named
accused, conspiring and confederating together and mutually helping one another, and
without authority of law, did then and there willfully, unlawfully, feloniously and
knowingly have in their possession, custody and control, the following to wit:

Five (5) bundles of C-4 or dynamites


Page 243 of 377

Six (6) cartoons of M-16 ammunition at 20 each

One hundred (100) bottles of MOLOTOV bombs.

without first securing the necessary license and/or permit to possess the same from the
proper authorities, and armed with said dynamites, ammunition and explosives and
pursuant to their conspiracy heretofore agreed upon by them and prompted by common
designs, come to an agreement and decision to commit the crime of rebellion, by then
and there participating therein and publicly taking arms against the duly constituted
authorities, for the purpose of overthrowing the Government of the Republic of the
Philippines, disrupting and jeopardizing its activities and removing from its allegiance
the territory of the Philippines or parts thereof." 2

In Criminal Case No. Q-90-11756, Rolando de Gracia, Chito Henson, Lamberto Bicus,
Rodolfo Tor and several John Does were charged with attempted homicide allegedly
committed on December 1, 1989 in Quezon city upon the person of Crispin Sagario who
was shot and hit on the right thigh.

Appellant was convicted for illegal possession of firearms in furtherance of rebellion, but
was acquitted of attempted homicide.

During the arraignment, appellant pleaded not guilty to both charges. However, he
admitted that he is not authorized to posses any firearms, ammunition and/or
explosive. 3 The parties likewise stipulated that there was a rebellion during the period
from November 30 up to December 9, 1989. 4

The records show that in the early morning of December 1, 1989, Maj. Efren Soria of
the Intelligence Division, National Capital Region Defense Command, was on board a
brown Toyota car conducting a surveillance of the Eurocar Sales Office located at
Epifanio de los Santos Avenue in Quezon City, together with his team composed of Sgt.
Crispin Sagario, M/Sgt. Ramon Briones, S/Sgt. Henry Aquino, one S/Sgt. Simon and a
Sgt. Ramos. The surveillance, which actually started on the night of November 30, 1989
at around 10:00 P.M., was conducted pursuant to an intelligence report received by the
division that said establishment was being occupied by elements of the RAM-SFP as a
communication command post.

Sgt. Crispin Sagario, the driver of the car, parked the vehicle around ten to fifteen meters
away from the Eurocar building near P. Tuazon Street, S/Sgt. Henry Aquino had earlier
Page 244 of 377

alighted from the car to conduct his surveillance on foot. A crowd was then gathered
near the Eurocar office watching the on-going bombardment near Camp Aguinaldo.
After a while a group of five men disengaged themselves from the crowd and walked
towards the car of the surveillance team. At that moment, Maj. Soria, who was then
seated in front, saw the approaching group and immediately ordered Sgt. Sagario to
start the car and leave the area. As they passed by the group, then only six meters away,
the latter pointed to them, drew their guns and fired at the team, which attack resulted
in the wounding of Sgt. Sagario on the right thigh. Nobody in the surveillance team was
able to retaliate because they sought cover inside the car and they were afraid that
civilians or bystanders might be caught in the cross-fire.

As a consequence, at around 6:30 A.M. of December 5, 1989, searching them composed


of F/Lt. Virgilio Babao as team leader, M/Sgt. Lacdao, Sgt. Magallion, Sgt. Patricio
Pacatang, and elements of the 16th Infantry Battalion under one Col. delos Santos
raided the Eurocar Sales Office. They were able to find and confiscate six cartons of M-
16 ammunition, five bundles of C-4 dynamites, M-shells of different calibers, and
"molotov" bombs inside one of the rooms belonging to a certain Col. Matillano which is
located at the right portion of the building. St. Oscar Obenia, the first one to enter the
Eurocar building, saw appellant De Gracia inside the office of Col. Matillano, holding a
C-4 and suspiciously peeping through a door. De Gracia was the only person then
present through a door. De Gracia was the only person then present inside the room. A
uniform with the nametag of Col. Matillano was also found. As a result of the raid, the
team arrested appellant, as well as Soprieso Verbo and Roberto Jimena who were
janitors at the Eurocar building. They were then made to sign an inventory, written in
Tagalog, of the explosives and ammunition confiscated by the raiding team. No search
warrant was secured by the raiding team. No search warrant was secured by the raiding
team because, according to them, at that time there was so much disorder considering
that the nearby Camp Aguinaldo was being mopped up by the rebel forces and there
was simultaneous firing within the vicinity of the Eurocar office, aside from the fact that
the courts were consequently closed. The group was able to confirm later that the owner
of Eurocar office is a certain Mr. Gutierrez and that appellant is supposedly a "boy"
therein.

Appellant Rolando de Gracia gave another version of the incident. First, he claims that
on November 30, 1989, he was in Antipolo to help in the birthday party of Col. Matillano.
He denies that he was at the Eurocar Sales Office on December 1, 1989. Second, he
contends that when the raiding team arrived at the Eurocar Sales Office on December
5, 1989, he was inside his house, a small nipa hut which is adjacent to the building.
According to him, he was tasked to guard the office of Col. Matillano which is located at
the right side of the building. He denies, however, that he was inside the room of Col.
Matillano when the raiding team barged in and that he had explosives in his possession.
He testified that when the military raided the office, he was ordered to get out of his
house and made to lie on the ground face down, together with "Obet" and "Dong" who
were janitors of the building. He avers that he does not know anything about the
Page 245 of 377

explosives and insists that when they were asked to stand up, the explosives were
already there.

Appellants stated that he visited Col. Matillano in 1987 at the stockade of the Philippine
Constabulary-Integrated National Police (PC-INP), and that he knew Matillano was
detained because of the latter’s involvement in the 1987 coup d’etat. In July, 1989,
appellant again went to see Matillano because he had no job. Col. Matillano then told
him that he could stay in the PC-INP stockade and do the marketing for them. From
that time until his arrest at the Eurocar office, appellant worked for Matillano.

De Gracia believes that the prosecution witnesses were moved to testify against him
because "bata raw ako ni Col. Matillano eh may atraso daw sa kanila si Col. Matillano
kaya sabi nila ito na lang bata niya ang ipitin natin."

On February 22, 1991, the trial court rendered judgment 5 acquitting appellant Rolando
de Gracia of attempted homicide, but found him guilty beyond reasonable doubt of the
offense of illegal possession of firearms in furtherance of rebellion and sentenced him to
serve the penalty of reclusion perpetua. Moreover, it made a recommendation that"
(i)nasmuch as Rolando de Gracia appears to be merely executing or obeying orders and
pursuant to the spirit contained in the 2nd paragraph of Art. 135, R.P.C., the court
recommends that Rolando de Gracia be extended executive clemency after serving a jail
term of five (5) years of good behavior."

That judgment of conviction is now challenged before us in this appeal.

Appellant principally contends that he cannot be held guilty of illegal possession of


firearms for the reason that he did not have either physical or constructive possession
thereof considering that he had no intent to possess the same; he is neither the owner
nor a tenant of the building where the ammunition and explosives were found; he was
merely employed by Col. Matillano as an errand boy; he was guarding the explosives for
and in behalf of Col. Matillano; and he did not have actual possession of the explosives.
He claims that intent to possess, which is necessary before one can be convicted under
Presidential Decree No. 1866, was not present in the case at
bar.chanroblesvirtualawlibrary

Presidential Decree No. 1866 provides as follows:


Page 246 of 377

"SECTION 1. Unlawful Manufacture, Sale, Acquisition, Disposition or Possession of


Firearms or Ammunition or Instruments Used or intended to be used in the
Manufacture of Firearms or Ammunition. — The Penalty of reclusion temporal in its
maximum period to reclusion perpetua shall be imposed upon any person who shall
unlawfully manufacture, deal in, acquire, dispose, or possess any firearms, part of
firearms, ammunition or machinery, took or instrument used or intended to be used in
the manufacture of any firearm or ammunition.

If homicide or murder is committed with the use of an unlicensed firearm, the penalty
of death shall be imposed.

If the violation of this Section is in furtherance of, or incident to, or in connection with
the crimes of rebellion, insurrection or supervision, the penalty of death shall be
imposed."

Presidential Decree No. 1866 was passed because of an upsurge of crimes vitally
affecting public order and safety due to the proliferation of illegally possessed and
manufactured firearms, ammunition and explosives, and which criminal acts have
resulted in loss of human lives, damage to property and destruction of valuable
resources of the country. The series of coup d’etats unleashed in the country during the
first few years of the transitional government under then President Corazon P. Aquino
attest to the ever-growing importance of laws such as Presidential Decree No. 1866
which seek to nip in the bud and preempt the commission of any act or acts which tend
to disturb public peace and order.

I. The first issue to be resolved is whether or not intent to possess is an essential element
of the offense punishable under Presidential Decree No. 1866 and, if so, whether
appellant De Gracia did intend to illegally possess firearms and ammunition.

The rule is that ownership is not an essential element of illegal possession of firearms
and ammunition. what the law requires is merely possession which includes not only
actual physical possession but also constructive possession or the subjection of the
thing to one’s control and management. 6 This has to be so if the manifest intent of the
law is to be effective. The same evils, the same perils to public security, which the law
penalizes exist whether the unlicensed holder of a prohibited weapon be its owner or a
borrower. To accomplish the object of this law the proprietary concept of the possession
can have no bearing whatsoever. 7
Page 247 of 377

But is the mere fact of physical or constructive possession sufficient to convict a person
for unlawful possession of firearms or must there be an intent to possess to constitute
a violation of the law? This query assumes significance since the offense of illegal
possession of firearms is a malum prohibitum punished by a special law, 8 in which
case good faith and absence of criminal intent are not valid defenses. 9

When the crime is punished by a special law, as a rule, intent to commit the crime is
not necessary. It is sufficient that the offender has the intent to perpetrate the act
prohibited by the special law. Intent to commit the crime and intent to perpetrate the
act prohibited by the special law. Intent to commit the crime and intent to perpetrate
the act must be distinguished. A person may not have consciously intended to commit
a crime; but he did intend to commit an act, and that act is, by the very nature of things,
the crime itself. In the first (intent to commit the crime), there must be criminal intent;
in the second (intent to perpetrate the act) it is enough that the prohibited act is done
freely and consciously. 10

In the present case, a distinction should be made between criminal intent and intent to
possess. While mere possession, without criminal intent, is sufficient to convict a person
for illegal possession of a firearm, it must still be shown that there was animus
possidendi or an intent to possess on the part of the accused. 11 Such intent to possess
is, however, without regard to any other criminal or felonious intent which the accused
may have harbored in possessing the firearm. Criminal intent here refers to the intention
of the accused to commit an offense with the use of an unlicensed firearm. This is not
important in convicting a person under Presidential Decree No. 1866. Hence, in order
that one may be found guilty of a violation of the decree, it is sufficient that the accused
had no authority or licensed to possess a firearm, and that he intended to possess the
same, even if such possession was in good faith and without criminal
intent.chanrobles.com:

Concomitantly, a temporary, incidental, casual, or harmless possession or control of a


firearm cannot be considered a violation of a statute prohibiting the possession of this
kind of weapon, 12 such as Presidential Decree No. 1866. Thus, although there is
physical or constructive possession, for as long as the animus possidendi is absent,
there is no offense committed.

Coming now to the case before us, there is no doubt in our minds that appellant De
Gracia is indeed guilty of having intentionally possessed several firearms, explosives and
ammunition without the requisite license or authority therefor. Prosecution witness Sgt.
Oscar Abenia categorically testified that he was the first one to enter the Eurocar Sales
Office when the military operatives raided the same, and he saw De Gracia standing in
the room and holding the several explosives marked in evidence as Exhibits D to D-4.
Page 248 of 377

13 At first, appellant denied any knowledge about the explosives. Then, he alternatively
contended that his act of guarding the explosives for and in behalf of Col. Matillano does
not constitute for and in behalf of Col. Matillano does not constitute illegal possession
thereof because there was no intent on his part to possess the same, since he was merely
employed as an errand by Col. Matillano. His pretension of impersonal or indifferent
material possession does not and cannot inspire credence.

Animus possidendi is a state of mind which may be determined on a case to case basis,
taking into consideration the prior and coetaneous acts of the accused and the
surrounding circumstances. What exists in the realm of thought is often disclosed in
the range of action. It is not controverted that appellant De Gracia is a former soldier,
having served with the Philippine Constabulary prior to his separation from the service
for going on absence without leave (AWOL). 14 We do not hesitate, therefore, to believe
and conclude that he is familiar with and knowledgeable about the dynamites, "molotov"
bombs, and various kinds of ammunition which were confiscated by the military from
his possession. As a former soldier, it would be absurd for him not to know anything
about the dangerous uses and power of these weapons. A fortiori, he cannot feign
ignorance on the import of having in his possession such a large quantity of explosives
and ammunition. Furthermore, the place where the explosives were found is not a
military camp or office, nor one where such items can ordinarily but lawfully be stored,
as in a gun store, a arsenal or armory. Even an ordinarily prudent man would be put
on guard and be suspicious if he finds articles of this nature in a place intended to carry
out the business of selling cars and which has nothing to do at all, directly or indirectly,
with the trade of firearms and ammunition.

On the basis of the foregoing disquisition, it is apparent, and we sold hold, that appellant
De Gracia actually intended to possess the articles confiscated from his person.

II. The next question that may be asked is whether or not there was a valid search and
seizure in this case. While the matter has not been squarely put in issue, we deem it
our bounden duty, in light of advertence thereto by the parties, to delve into the legality
of the warrantless search conducted by the raiding them, considering the gravity of the
offense for which herein appellant stands to be convicted and the penalty sought to be
imposed.

It is admitted that the military operatives who raided the Eurocar Sales Office were not
armed with a search warrant at that time. 15 The raid was actually precipitated by
intelligence reports that said office was being used as headquarters by the RAM. 16 Prior
to the raid, there was a surveillance conducted on the premises wherein the surveillance
team was fired at by a group of men coming from the Eurocar building. When the
military operatives raided the place, the occupants thereof refused to open the door
Page 249 of 377

despite the requests for them to do so, thereby compelling the former to break into the
office. 17 The Eurocar Sales Office is obviously not a gun store and it is definitely not
an armory or arsenal which are the usual depositories for explosives and ammunition.
It is primarily and solely engaged in the sale of automobiles. The presence of an unusual
quantity of high-powered firearms and explosives could not be justifiably or even
colorably explained. In addition, there was general chaos and disorder at that time
because of simultaneous and intense firing within the vicinity of the office and in the
nearby Camp Aguinaldo which was under attack by rebel forces. 18 The courts in the
surrounding areas were obviously closed and, for that matter, the building and houses
therein were deserted.

Under the foregoing circumstances, it is out considered opinion that the instant case
falls under one of the exceptions to the prohibition against a warrantless search. In the
first place, the military operatives, taking into account the facts obtaining in this case,
had reasonable ground to believe that a crime was being committed. There was
consequently more than sufficient probable cause to warrant their action. Furthermore,
under the situation then prevailing, the raiding team had no opportunity to apply for
and secure a search warrant from the courts. The trial judge himself manifested that on
December 5, 1989 when the raid was conducted, his court was closed. 19 Under such
urgency and exigency of the moment, a search warrant could lawfully be dispensed with.

The view that we here take is in consonance with our doctrinal ruling which was amply
explained in People v. Malmsted 20 and bears reiteration:

"While it is true that the NARCOM officers were not alarmed with a search was made
over the personal effects of accused, however, under the circumstances of the case, there
was sufficient probable cause for said officers to believe that accused was then and there
committing a crime.

"Probable cause has been defined as such facts and circumstances which would lead a
reasonable, discreet and prudent man to believe that an offense has been committed,
and that the objects sought in connection with the offense are in the place sought to be
searched. The required probable cause that will justify a warrantless search and seizure
is not determined by any fixed formula but is resolved according to the facts of each
case.

"Warrantless search of the personal effects of an accused has been declared by this
Court as valid, because of existence of probable cause, where the smell of marijuana
emanated from a plastic bag owned by the accused, or where the accused was acting
suspiciously, and attempted to flee.
Page 250 of 377

"Aside from the persistent reports received by the NARCOM that vehicles coming from
Sagada were transporting marijuana and other prohibited drugs, their Commanding
Officer also received information that a Caucasian coming from Sagada on that
particular day had prohibited drugs in his possession. Said information was received by
the Commanding Officer of NARCOM the very same morning that accused came down
by bus from Sagada on his way to Baguio City.

"When NARCOM received the information, a few hours before the apprehension of herein
accused, that a Caucasian travelling from Sagada to Baguio City was carrying with him
prohibited drugs, there was no time to obtain a search warrant. In the Tangliben case,
the police authorities conducted a surveillance at the Victory Liner Terminal located at
Bgy. San Nicolas, San Fernando, Pampanga, against persons engaged in the traffic of
dangerous drugs, based on information supplied by some informers. Accused Tangliben
who was acting suspiciously and pointed out by an informer was apprehended and
searched by the police authorities. It was held that when faced with on-the-spot
information, the police officers had to act quickly and there was no time to secure a
search warrant.

"It must be observed that, at first, the NARCOM officers merely conducted a routine
check passengers therein, and no extensive search was initially made. It was only when
one of the officers noticed a bulge on the waist of accused, during the course of the
inspection, that accused was required to present his passport. The failure of accused to
present his identification papers, when ordered to do so, only managed to arouse the
suspicion of the officer that accused was trying to hide his identity. For is it not a regular
norm for an innocent man, who has nothing to hide from the authorities, to readily
present his identification papers when required to do so?

"The receipt of information by NARCOM that a Causasian coming from Sagada had
prohibited drugs in his possession, plus the suspicious failure of the accused to produce
his passport, taken together as a whole, led the NARCOM officers to reasonably believe
that the accused was trying to hide something illegal from the authorities. From these
circumstances arose a probable cause which justified the warrantless search that was
made on the personal effects of the accused. In other words, the acts of the NARCOM
officers in requiring the accused to open his pouch bag and in opening one of the
wrapped objects found inside said bag (which was discovered to contain hashish) as well
as the two (2) teddy bears with hashish stuffed inside the, were prompted by accused’s
own attempt to hide his identity by refusing to present his passport, and by the
information received by the NARCOM that a Caucasian coming from Sagada had
prohibited drugs in his possession. To deprive the NARCOM agents of of the ability and
facility to act accordingly, including, to search even without warrant, in the light of such
Page 251 of 377

circumstances, would be to sanction impotence and ineffectiveness in law enforcement,


to the detriment of society."

In addition, we find the principles enunciated in Umil, Et. Al. v. Ramos, Et Al., 21
applicable, by analogy, to the present case:

"The arrest of persons involved in the rebellion whether as its fighting armed elements,
or for committing non-violent acts but in furtherance of the rebellion, is more an act of
capturing them in the course of an armed conflict, to quell the rebellion, than for the
purpose of immediately prosecuting them in court for a statutory offense. The arrest,
therefore, need not follow the usual procedure in the prosecution of offenses which
requires the determination by a judge of the existence of probable cause before the
issuance of a judicial warrant of arrest and the granting of bail if the offense is bailable.
Obviously the absence of a judicial warrant is no legal impediment to arresting or
capturing persons committing overt acts of violence against government forces, or any
other milder acts but really in pursuance of the rebellious movement. The arrest or
capture is thus impelled by the exigencies of the situation that involves the very survival
of society and its government and duly constituted authorities. If killing and other acts
of violence against the rebels find justification in the exigencies of armed hostilities
which (are) of the essence of waging a rebellion or insurrection, most assuredly so in
case of invasion, merely seizing their persons and detaining them while any of these
contigencies continues cannot be less justified."

III. As earlier stated, it was stipulated and admitted by both parties that from November
30, 1989 up to and until December 9, 1989, there was a rebellion. Ergo, our next inquiry
is whether or not appellant’s possession of the firearms, explosives and ammunition
seized and recovered from him was for the purpose and in furtherance of rebellion.

The trial court found accused guilty of illegal possession of firearms in furtherance of
rebellion pursuant to paragraph 2 of Article 135 of the Revised Penal Code which states
that "any person merely participating or executing the command of others in a rebellion
shall suffer the penalty of prision mayor in its minimum period." The court below held
that appellant De Gracia, who has been servicing the personal needs of Co. Matillano
(whose active armed opposition against the Government, particularly at the Camelot
Hotel, was well known), is guilty of the act of guarding the explosives and "molotov"
bombs for and in behalf of the latter. We accept this finding of the lower court.

The above provision of the law was, however, erroneously and improperly used by the
court below as a basis in determining the degree of liability of appellant and the penalty
to be imposed on him. It must be made clear that appellant is charged with the qualified
Page 252 of 377

offense of illegal possession of firearms in furtherance of rebellion under Presidential


Decree No. 1866 which, in law, is distinct from the crime of rebellion punished under
Articles 134 and 135 of the Revised Penal Code. These are two separate statutes
penalizing different offenses with discrete penalties. The Revised Penal Code treats
rebellion as a crime apart from murder, homicide, arson, or other offenses, such as
illegal possession of firearms, that might conceivably be committed in the course of a
rebellion. Presidential Decree No. 1866 defines and punishes, as a specific offense, the
crime of illegal possession of firearms committed in the course or as part of a rebellion.
22

As a matter of fact, in one case involving the constitutionality of Section 1 of Presidential


Decree No. 1866, the Court has explained that said provision of the law will not be
invalidated by the mere fact that the same act is penalized under two different statutes
with different penalties, even if considered highly advantageous to the prosecution and
onerous to the accused. 23 It follows that, subject to the presence of the requisite
elements in each case, unlawful possession of an unlicensed firearm in furtherance of
rebellion may give rise to separate prosecutions for a violation of Section 1 of Presidential
Decree No. 1866, and also a violation of Articles 134 and 135 of the Revised Penal Code
on rebellion. Double jeopardy in this case cannot be invoked because the first is an
offense punished by a special law while the second is a felony punished by the Revised
Penal Code, 24 with variant elements.

It was a legal malapropism for the lower court to interject the aforestated provision of
the Revised Penal Code in this prosecution for a crime under a special law.
Consequently, there is no basis for its recommendation for executive clemency in favor
of appellant De Gracia after he shall have served a jail term of five years with good
behavior. In any event, this is a matter within the exclusive prerogative of the President
whose decision thereon should be insulated against any tenous importunity. Withal, we
are duly convinced that the firearms, explosives and ammunition confiscated from
appellant De Gracia were illegally possessed by him in furtherance of the rebellion then
admittedly existing at that time. In the words of the court a quo:

"2. the nature and quantity of the items — 5 bundles of C-4 dynamites, 6 cartons of M-
16 ammo and 100 bottles of molotov bombs indicate that the reports received by the
military that the Eurocar Sales Building was being used by the rebels was not without
basis. Those items are clearly not for one’s personal defense. They are for offensive
operations. De Gracia admitted that per instruction of Col. Matillano he went down to
Eurocar Sales Building from Antipolo to stay guard there.

"His manifestation of innocence of those items and what he had been guarding in that
office is not credible for: (a) he was a former military personnel; (b) at the birthday party
Page 253 of 377

of Col. Matillano on November 30, 1989 may soldiers and ex-soldiers were present which
self-evidently discloses that De Gracia, in the company of his boss, was till very much
at home and constantly in touch with soldiers and the armed rebellion of November 30,
1989 to December 8 or 9, 1989 was a military coup d’etat; (c) it appears that he is the
only person tasked with caretaking (sic) there in the Matillano office, which shows that
he is a highly trusted right-hand man of Col. Matillano; and (d) as heretofore discussed,
de Gracia was earlier seen with some who fired upon a car of the AFP intelligence
agents."25

Presidential Decree No. 1866 imposes the death penalty where the illegal possession
firearms and ammunition is committed in furtherance of rebellion. At the time the
offense charged in this case was committed under the governance of that law, the
imposition of the death penalty was prescribed by the Constitution. Consequently,
appellant De Gracia could only sentenced to serve the penalty of reclusion perpetua
which was correctly meted out by the trial court, albeit with an erroneous
recommendation in connection therewith.

WHEREFORE, the impugned judgment of the trial court is hereby AFFIRMED, but its
recommendation therein for executive clemency and the supposed basis thereof are
hereby DELETED, with cots against Accused-Appellant.

SO ORDERED.

Narvasa, C.J., Padilla, Regalado, Puno and Mendoza, JJ., concur.


Page 254 of 377

G.R. No. 89139 August 2, 1990

ROMEO POSADAS y ZAMORA, petitioner, vs. THE HONORABLE COURT OF


APPEALS and THE PEOPLE OF THE PHILIPPINES, respondents.

GANCAYCO, J.:

The validity of a warrantless search on the person of petitioner is put into issue in this
case.

On October 16, 1986 at about 10:00 o'clock in the morning Pat. Ursicio Ungab and Pat.
Umbra Umpar, both members of the Integrated National Police (INP) of the Davao
Metrodiscom assigned with the Intelligence Task Force, were conducting a surveillance
along Magallanes Street, Davao City. While they were within the premises of the Rizal
Memorial Colleges they spotted petitioner carrying a "buri" bag and they noticed him to
be acting suspiciously.

They approached the petitioner and identified themselves as members of the INP.
Petitioner attempted to flee but his attempt to get away was thwarted by the two
notwithstanding his resistance.

They then checked the "buri" bag of the petitioner where they found one (1) caliber .38
Smith & Wesson revolver with Serial No. 770196 1 two (2) rounds of live ammunition
for a .38 caliber gun 2 a smoke (tear gas) grenade,3 and two (2) live ammunitions for a
.22 caliber gun. 4 They brought the petitioner to the police station for further
investigation. In the course of the same, the petitioner was asked to show the necessary
license or authority to possess firearms and ammunitions found in his possession but
he failed to do so. He was then taken to the Davao Metrodiscom office and the prohibited
articles recovered from him were indorsed to M/Sgt. Didoy the officer then on duty. He
was prosecuted for illegal possession of firearms and ammunitions in the Regional Trial
Court of Davao City wherein after a plea of not guilty and trial on the merits a decision
was rendered on October 8, 1987 finding petitioner guilty of the offense charged as
follows:

WHEREFORE, in view of all the foregoing, this Court , finds the accused guilty beyond
reasonable doubt of the offense charged.

It appearing that the accuse d was below eighteen (18) years old at the time of the
commission of the offense (Art. 68, par. 2), he is hereby sentenced to an indeterminate
Page 255 of 377

penalty ranging from TEN (10) YEARS and ONE (1) DAY of prision mayor to TWELVE
(12) Years, FIVE (5) months and Eleven (11) days of Reclusion Temporal, and to pay the
costs.

The firearm, ammunitions and smoke grenade are forfeited in favor of the government
and the Branch Clerk of Court is hereby directed to turn over said items to the Chief,
Davao Metrodiscom, Davao City. 5

Not satisfied therewith the petitioner interposed an appeal to the Court of Appeals
wherein in due course a decision was rendered on February 23, 1989 affirming in toto
the appealed decision with costs against the petitioner. 6

Hence, the herein petition for review, the main thrust of which is that there being no
lawful arrest or search and seizure, the items which were confiscated from the
possession of the petitioner are inadmissible in evidence against him.

The Solicitor General, in justifying the warrantless search of the buri bag then carried
by the petitioner, argues that under Section 12, Rule 136 of the Rules of Court a person
lawfully arrested may be searched for dangerous weapons or anything used as proof of
a commission of an offense without a search warrant. It is further alleged that the arrest
without a warrant of the petitioner was lawful under the circumstances.

Section 5, Rule 113 of the 1985 Rules on Criminal Procedure provides as follows:

SEC. 5. Arrest without warrant; when lawful — A peace officer or a private person may,
without a warrant, arrest a person:

(a) When in his presence, the person to be arrested has committed is actually
committing, or is attempting to commit an offense;

(b) When an offense has in fact just been committed, and he has personal knowledge of
facts indicating that the person to be arrested has committed it; and

(c) When the person to be arrested is a prisoner who has escaped from a penal
establishment or place where he is serving final judgment or temporarily confined while
Page 256 of 377

his case is pending, or has escaped while being transferred from one confinement to
another.

In cases falling under paragraphs (a) and (b) hereof, the person arrested without a
warrant shall be forthwith delivered to the nearest police station or jail, and he shall be
proceeded against in accordance with Rule 112, Section 7. (6a, 17a)

From the foregoing provision of law it is clear that an arrest without a warrant may be
effected by a peace officer or private person, among others, when in his presence the
person to be arrested has committed, is actually committing, or is attempting to commit
an offense; or when an offense has in fact just been committed, and he has personal
knowledge of the facts indicating that the person arrested has committed it.

The Solicitor General argues that when the two policemen approached the petitioner, he
was actually committing or had just committed the offense of illegal possession of
firearms and ammunitions in the presence of the police officers and consequently the
search and seizure of the contraband was incidental to the lawful arrest in accordance
with Section 12, Rule 126 of the 1985 Rules on Criminal Procedure. We disagree.

At the time the peace officers in this case identified themselves and apprehended the
petitioner as he attempted to flee they did not know that he had committed, or was
actually committing the offense of illegal possession of firearms and ammunitions. They
just suspected that he was hiding something in the buri bag. They did now know what
its contents were. The said circumstances did not justify an arrest without a warrant.

However, there are many instances where a warrant and seizure can be effected without
necessarily being preceded by an arrest, foremost of which is the "stop and search"
without a search warrant at military or police checkpoints, the constitutionality or
validity of which has been upheld by this Court in Valmonte vs. de Villa, 7 as follows:

Petitioner Valmonte's general allegation to the effect that he had been stopped and
searched without a search warrant by the military manning the checkpoints, without
more, i.e., without stating the details of the incidents which amount to a violation of his
light against unlawful search and seizure, is not sufficient to enable the Court to
determine whether there was a violation of Valmonte's right against unlawful search
and seizure. Not all searches and seizures are prohibited. Those which are reasonable
are not forbidden. A reasonable search is not to be determined by any fixed formula but
is to be resolved according to the facts of each case.
Page 257 of 377

Where, for example, the officer merely draws aside the curtain of a vacant vehicle which
is parked on the public fair grounds, or simply looks into a vehicle or flashes a light
therein, these do not constitute unreasonable search.

The setting up of the questioned checkpoints in Valenzuela (and probably in other areas)
may be considered as a security measure to enable the NCRDC to pursue its mission of
establishing effective territorial defense and maintaining peace and order for the benefit
of the public. Checkpoints may also be regarded as measures to thwart plots to
destabilize the government in the interest of public security. In this connection, the
Court may take judicial notice of the shift to urban centers and their suburbs of the
insurgency movement, so clearly reflected in the increased killings in cities of police and
military men by NPA "sparrow units," not to mention the abundance of unlicensed
firearms and the alarming rise in lawlessness and violence in such urban centers, not
all of which are reported in media, most likely brought about by deteriorating economic
conditions — which all sum up to what one can rightly consider, at the very least, as
abnormal times. Between the inherent right of the state to protect its existence and
promote public welfare and an individual's right against a warrantless search which is
however reasonably conducted, the former should prevail.

True, the manning of checkpoints by the military is susceptible of abuse by the men in
uniform in the same manner that all governmental power is susceptible of abuse. But,
at the cost of occasional inconvenience, discomfort and even irritation to the citizen, the
checkpoints during these abnormal times, when conducted within reasonable limits,
are part of the price we pay for an orderly society and a peaceful community. (Emphasis
supplied).

Thus, as between a warrantless search and seizure conducted at military or police


checkpoints and the search thereat in the case at bar, there is no question that, indeed,
the latter is more reasonable considering that unlike in the former, it was effected on
the basis of a probable cause. The probable cause is that when the petitioner acted
suspiciously and attempted to flee with the buri bag there was a probable cause that he
was concealing something illegal in the bag and it was the right and duty of the police
officers to inspect the same.

It is too much indeed to require the police officers to search the bag in the possession
of the petitioner only after they shall have obtained a search warrant for the purpose.
Such an exercise may prove to be useless, futile and much too late.

In People vs. CFI of Rizal, 8 this Court held as follows:


Page 258 of 377

. . . In the ordinary cases where warrant is indispensably necessary, the mechanics


prescribed by the Constitution and reiterated in the Rules of Court must be followed
and satisfied. But We need not argue that there are exceptions. Thus in the
extraordinary events where warrant is not necessary to effect a valid search or seizure,
or when the latter cannot be performed except without warrant, what constitutes a
reasonable or unreasonable search or seizure becomes purely a judicial question,
determinable from the uniqueness of the circumstances involved, including the purpose
of the search or seizure, the presence or absence of probable cause, the manner in which
the search and seizure was made, the place or thing searched and the character of the
articles procured.

The Court reproduces with approval the following disquisition of the Solicitor General:

The assailed search and seizure may still be justified as akin to a "stop and frisk"
situation whose object is either to determine the identity of a suspicious individual or
to maintain the status quo momentarily while the police officer seeks to obtain more
information. This is illustrated in the case of Terry vs. Ohio, 392 U.S. 1 (1968). In this
case, two men repeatedly walked past a store window and returned to a spot where they
apparently conferred with a third man. This aroused the suspicion of a police officer. To
the experienced officer, the behaviour of the men indicated that they were sizing up the
store for an armed robbery. When the police officer approached the men and asked them
for their names, they mumbled a reply. Whereupon, the officer grabbed one of them,
spun him around and frisked him. Finding a concealed weapon in one, he did the same
to the other two and found another weapon. In the prosecution for the offense of carrying
a concealed weapon, the defense of illegal search and seizure was put up. The United
States Supreme Court held that "a police officer may in appropriate circumstances and
in an appropriate manner approach a person for the purpose of investigating possible
criminal behaviour even though there is no probable cause to make an arrest." In such
a situation, it is reasonable for an officer rather than simply to shrug his shoulder and
allow a crime to occur, to stop a suspicious individual briefly in order to determine his
identity or maintain the status quo while obtaining more information. . . .

Clearly, the search in the case at bar can be sustained under the exceptions heretofore
discussed, and hence, the constitutional guarantee against unreasonable searches and
seizures has not been violated. 9

WHEREFORE, the petition is DENIED with costs against petitioner.

SO ORDERED.
Narvasa (Chairman), Cruz, Griño-Aquino and Medialdea, JJ., concur.
Page 259 of 377

G.R. No. 87059 June 22, 1992

THE PEOPLE OF THE PHILIPPINES, Plaintiff-Appellee, v. ROGELIO


MENGOTE Y TEJAS, Accused-Appellant.

SYLLABUS

1. CONSTITUTIONAL LAW; BILL OF RIGHTS; RIGHT AGAINST ILLEGAL SEARCH AND


SEIZURE; EVIDENCE OBTAINED IN VIOLATION THEREOF; EFFECT; CASE AT BAR.
— It is submitted in the Appellant’s Brief that the revolver should not have been
admitted in evidence because of its illegal seizure, no warrant therefor having been
previously obtained. Neither could it have been seized as an incident of a lawful arrest
because the arrest of Mengote was itself unlawful, having been also effected without a
warrant. The defense also contends that the testimony regarding the alleged robbery in
Danganan’s house was irrelevant and should also have been disregarded by the trial
court. There is no question that evidence obtained as a result of an illegal search or
seizure is inadmissible in any proceeding for any purpose. That is the absolute
prohibition of Article III, Section 3(2), of the Constitution. This is the celebrated
exclusionary rule based on the justification given by Judge Learned Hand that "only in
case the prosecution which itself controls the seizing officials, knows that it cannot profit
by their wrong will the wrong be repressed."

2. REMEDIAL LAW; CRIMINAL PROCEDURE; ARREST WITHOUT WARRANT; WHEN


LAWFUL; REQUISITES; NOT ESTABLISHED IN CASE AT BAR. — The Solicitor General,
while conceding the rule, maintains that it is not applicable in the case at bar. His
reason is that the arrest and search of Mengote and the seizure of the revolver from him
were lawful under Rule 113, Section 5, of the Rules of Court. We have carefully examined
the wording of this rule and cannot see how we can agree with the prosecution. Par. (c)
of Section 5 is obviously inapplicable as Mengote was not an escapee from a penal
institution when he was arrested. We therefore confine ourselves to determining the
lawfulness of his arrest under either Par. (a) or Par. (b) of this section. Par. (a) requires
that the person be arrested (1) after he has committed or while he is actually committing
or is at least attempting to commit an offense, (2) in the presence of the arresting officer.
These requirements have not been established in the case at bar. At the time of the
arrest in question, the accused-appellant was merely "looking from side to side" and
"holding his abdomen," according to the arresting officers themselves. There was
apparently no offense that had just been committed or was being actually committed or
at least being attempted by Mengote in their presence. Par. (b) is no less applicable
because its no less stringent requirements have also not been satisfied. The prosecution
has not shown that at the time of Mengote’s arrest an offense had in fact just been
committed and that the arresting officers had personal knowledge of facts indicating
that Mengote had committed it. All they had was hearsay information from the telephone
caller, and about a crime that had yet to be committed.
Page 260 of 377

CRUZ, J.:

Accused-appellant Rogelio Mengote was convicted of illegal possession of firearms on


the strength mainly of the stolen pistol found on his person at the moment of his
warrantless arrest. In this appeal, he pleads that the weapon was not admissible as
evidence against him because it had been illegally seized and was therefore the fruit of
the poisonous tree. The Government disagrees. It insists that the revolver was validly
received in evidence by the trial judge because its seizure was incidental to an arrest
that was doubtless lawful even if admittedly without warrant.

The incident occurred shortly before noon of August 8, 1987, after the Western Police
District received a telephone call from an informer that there were three suspicious-
looking persons at the corner of Juan Luna and North Bay Boulevard in Tondo, Manila.
A surveillance team of plainclothesmen was forthwith dispatched to the place. As later
narrated at the trial by Patrolmen Rolando Mercado and Alberto Juan, 1 they there saw
two men "looking from side to side," one of whom was holding his abdomen. They
approached these persons and identified themselves as policemen, whereupon the two
tried to run away but were unable to escape because the other lawmen had surrounded
them. The suspects were then searched. One of them, who turned out to be the accused-
appellant, was found with a .38 caliber Smith and Wesson revolver with six live bullets
in the chamber. His companion, later identified as Nicanor Morellos, had a fan knife
secreted in his front right pantspocket. The weapons were taken from them. Mengote
and Morellos were then turned over to police headquarters for investigation by the
Intelligence Division.

On August 11, 1987, the following information was filed against the accused-appellant
before the Regional Trial Court of Manila:

The undersigned accuses ROGELIO MENGOTE y TEJAS of a violation of Presidential


Decree No. 1866, committed as follows:

That on or about August 8, 1987, in the City of Manila, Philippines, the said accused
did then and there wilfully, unlawfully and knowingly have in his possession and under
his custody and control a firearm, to wit:

one (1) cal. 38 "S & W" bearing


Page 261 of 377

Serial No. 8720-T.

Without first having secured the necessary license or permit therefor from the proper
authorities.

Besides the police officers, one other witness presented by the prosecution was
Rigoberto Danganan, who identified the subject weapon as among the articles stolen
from him during the robbery in his house in Malabon on June 13, 1987. He pointed to
Mengote as one of the robbers. He had duly reported the robbery to the police, indicating
the articles stolen from him, including the revolver. 2 For his part, Mengote made no
effort to prove that he owned the firearm or that he was licensed to possess it and
claimed instead that the weapon had been "planted" on him at the time of his arrest. 3

The gun, together with the live bullets and its holster, were offered as Exhibits A, B and
C and admitted over the objection of the defense. As previously stated, the weapon was
the principal evidence that led to Mengote’s conviction for violation of P.D. 1866. He was
sentenced to reclusion perpetua. 4

It is submitted in the Appellant’s Brief that the revolver should not have been admitted
in evidence because of its illegal seizure, no warrant therefor having been previously
obtained. Neither could it have been seized as an incident of a lawful arrest because the
arrest of Mengote was itself unlawful, having been also effected without a warrant. The
defense also contends that the testimony regarding the alleged robbery in Danganan’s
house was irrelevant and should also have been disregarded by the trial court.

The following are the pertinent provision of the Bill of Rights:

Sec. 2. The right of the people to be secure in their persons, houses, papers, and effects
against unreasonable searches and seizures of whatever nature and for any purpose
shall be inviolable, and no search warrant or warrant of arrest shall issue except upon
probable cause to be determined personally by the judge after examination under oath
or affirmation of the complainant and the witnesses he may produce, and particularly
describing the place to be searched and the persons or things to be seized.

Sec. 3 (1). The privacy of communication and correspondence shall be inviolable except
upon lawful order of the court, or when public safety or order requires otherwise as
prescribed by law.
Page 262 of 377

(2) Any evidence obtained in violation of this or the preceding section shall be
inadmissible for any purpose in any proceeding.

There is no question that evidence obtained as a result of an illegal search or seizure is


inadmissible in any proceeding for any purpose. That is the absolute prohibition of
Article III, Section 3(2), of the Constitution. This is the celebrated exclusionary rule
based on the justification given by Judge Learned Hand that "only in case the
prosecution, which itself controls the seizing officials, knows that it cannot profit by
their wrong will the wrong be repressed."

The Solicitor General, while conceding the rule, maintains that it is not applicable in
the case at bar. His reason is that the arrest and search of Mengote and the seizure of
the revolver from him were lawful under Rule 113, Section 5, of the Rules of Court
reading as follows:

Sec. 5. Arrest without warrant; when lawful. — A peace officer or private person may
without a warrant, arrest a person:chanrobles virtual lawlibrary

(a) When, in his presence, the person to be arrested has committed, is actually
committing, or is attempting to commit an offense;

(b) When an offense has in fact just been committed, and he has personal knowledge of
facts indicating that the person to be arrested has committed it; and

(c) When the person to be arrested is a prisoner who has escaped from a penal
establishment or place where he is serving final judgment or temporarily confined while
his case is pending, or has escaped while being transferred from one confinement to
another.

In cases falling under paragraphs (a) and (b) hereof, the person arrested without a
warrant shall be forthwith delivered to the nearest police station or jail, and he shall be
proceeded against in accordance with Rule 112, Section 7.

We have carefully examined the wording of this rule and cannot see how we can agree
with the prosecution.
Page 263 of 377

Par. (c) of Section 5 is obviously inapplicable as Mengote was not an escapee from a
penal institution when he was arrested. We therefore confine ourselves to determining
the lawfulness of his arrest under either Par. (a) or Par. (b) of this section.

Par. (a) requires that the person be arrested (1) after he has committed or while he is
actually committing or is at least attempting to commit an offense, (2) in the presence
of the arresting officer.

These requirements have not been established in the case at bar. At the time of the
arrest in question, the accused-appellant was merely "looking from side to side" and
"holding his abdomen," according to the arresting officers themselves. There was
apparently no offense that had just been committed or was being actually committed or
at least being attempted by Mengote in their presence.

The Solicitor General submits that the actual existence of an offense was not necessary
as long as Mengote’s acts "created a reasonable suspicion on the part of the arresting
officers and induced in them the belief that an offense had been committed and that the
accused-appellant had committed it." The question is, What offense? What offense could
possibly have been suggested by a person "looking from side to side" and "holding his
abdomen" and in a place not exactly forsaken?

These are certainly not sinister acts. And the setting of the arrest made them less so, if
at all. It might have been different if Mengote had been apprehended at an ungodly hour
and in a place where he had no reason to be, like a darkened alley at 3 o’clock in the
morning. But he was arrested at 11:30 in the morning and in a crowded street shortly
after alighting from a passenger jeep with his companion. He was not skulking in the
shadows but walking in the clear light of day. There was nothing clandestine about his
being on that street at that busy hour in the blaze of the noonday sun.

On the other hand, there could have been a number of reasons, all of them innocent,
why his eyes were darting from side to side and he was holding his abdomen. If they
excited suspicion in the minds of the arresting officers, as the prosecution suggests, it
has nevertheless not been shown what their suspicion was all about. In fact, the
policemen themselves testified that they were dispatched to that place only because of
the telephone call from the informer that there were "suspicious-looking" persons in that
vicinity who were about to commit a robbery at North Bay Boulevard. The caller did not
explain why he thought the men looked suspicious nor did he elaborate on the
impending crime.
Page 264 of 377

In the recent case of People v. Malmstedt, 5 the Court sustained the warrantless arrest
of the accused because there was a bulge in his waist that excited the suspicion of the
arresting officer and, upon inspection, turned out to be a pouch containing hashish. In
People v. Claudio, 6 the accused boarded a bus and placed the buri bag she was carrying
behind the seat of the arresting officer while she herself sat in the seat before him. His
suspicion aroused, he surreptitiously examined the bag, which he found to contain
marijuana. He then and there made the warrantless arrest and seizure that we
subsequently upheld on the ground that probable cause had been sufficiently
established.

The case before us is different because there was nothing to support the arresting
officers’ suspicion other than Mengote’s darting eyes and his hand on his abdomen. By
no stretch of the imagination could it have been inferred from these acts that an offense
had just been committed, or was actually being committed, or was at least being
attempted in their presence.

This case is similar to People v. Aminnudin, 7 where the Court held that the warrantless
arrest of the accused was unconstitutional. This was effected while he was coming down
a vessel, to all appearances no less innocent than the other disembarking passengers.
He had not committed nor was he actually committing or attempting to commit an
offense in the presence of the arresting officers. He was not even acting suspiciously. In
short, there was no probable cause that, as the prosecution incorrectly suggested,
dispensed with the constitutional requirement of a warrant.

Par. (b) is no less applicable because its no less stringent requirements have also not
been satisfied. The prosecution has not shown that at the time of Mengote’s arrest an
offense had in fact just been committed and that the arresting officers had personal
knowledge of facts indicating that Mengote had committed it. All they had was hearsay
information from the telephone caller, and about a crime that had yet to be committed.

The truth is that they did not know then what offense, if at all, had been committed and
neither were they aware of the participation therein of the Accused-Appellant. It was
only later, after Danganan had appeared at the police headquarters, that they learned
of the robbery in his house and of Mengote’s supposed involvement therein. 8 As for the
illegal possession or the firearm found on Mengote’s person, the policemen discovered
this only after he had been searched and the investigation conducted later revealed that
he was not its owners nor was he licensed to possess it.
Page 265 of 377

Before these events, the peace officers had no knowledge even of Mengote’ identity, let
alone the fact (or suspicion) that he was unlawfully carrying a firearm or that he was
involved in the robbery of Danganan’s house.

In the landmark case of People v. Burgos, 9 this Court declared:

Under Section 6(a) of Rule 113, the officer arresting a person who has just committed,
is committing, or is about to commit an offense must have personal knowledge of the
fact. The offense must also be committed in is presence or within his view. (Sayo v. Chief
of Police, 80 Phil. 859). (Emphasis supplied)

x x x

In arrests without a warrant under Section 6(b), however, it is not enough that there is
reasonable ground to believe that the person to be arrested has committed a crime. A
crime must in fact or actually have been committed first. That a crime has actually been
committed is an essential precondition. It is not enough to suspect that a crime may
have been committed. The fact of the commission of the offense must be undisputed.
The test of reasonable ground applies only to the identity of the perpetrator. (Emphasis
supplied)

This doctrine was affirmed in Alih v. Castro, 10 thus:

If the arrest was made under Rule 113, Section 5, of the Rules of Court in connection
with a crime about to be committed, being committed, or just committed, what was that
crime? There is no allegation in the record of such a justification. Parenthetically, it may
be observed that under the Revised Rule 113, Section 5(b), the officer making the arrest
must have personal knowledge of the ground therefor as stressed in the recent case of
People v. Burgos. (Emphasis supplied)

It would be a sad day, indeed, if any person could be summarily arrested and searched
just because he is holding his abdomen, even if it be possibly because of a stomach-
ache, or if a peace officer-could clamp handcuffs on any person with a shifty look on
suspicion that he may have committed a criminal act or is actually committing or
attempting it. This simply cannot be done in a free society. This is not a police state
Page 266 of 377

where order is exalted over liberty or, worse, personal malice on the part of the arresting
officer may be justified in the name of security.

There is no need to discuss the other issues raised by the accused-appellant as the
ruling we here make is sufficient to sustain his exoneration. Without the evidence of the
firearm taken from him at the time of his illegal arrest, the prosecution has lost its most
important exhibit and must therefore fail. The testimonial evidence against Mengote
(which is based on the said firearm) is not sufficient to prove his guilt beyond reasonable
doubt of the crime imputed to him.

We commend Atty. Violeta Calvo-Drilon for her able and spirited defense of the accused-
appellant not only in the brief but also in the reply brief, which she did not have to file
but did so just the same to stress the constitutional rights of her client. The fact that
she was acting only as a counsel de oficio with no expectation of material reward makes
her representation even more commendable.

The Court feels that if the peace officers had been more mindful of the provisions of the
Bill of Rights, the prosecution of the accused-appellant might have succeeded. As it
happened, they allowed their over-zealousness to get the better of them, resulting in
their disregard of the requirements of a valid search and seizure that rendered
inadmissible the vital evidence they had invalidly seized.

This should be a lesson to other peace officers. Their impulsiveness may be the very
cause of the acquittal of persons who deserve to be convicted, escaping the clutches of
the law because, ironically enough, it has not been observed by those who are supposed
to enforce it.

WHEREFORE, the appealed decision is REVERSED and SET ASIDE. The accused-
appellant is ACQUITTED and ordered released immediately unless he is validly detained
for other offenses. No costs.

SO ORDERED.

Griño-Aquino, Medialdea and Bellosillo, JJ., concur.


Page 267 of 377

G.R. No. L-68955 September 4, 1986

PEOPLE OF THE PHILIPPINES, plaintiff-appellee, vs. RUBEN BURGOS y


TITO, defendant-appellant.

GUTIERREZ, JR., J.:

This is an appeal from the decision of the Regional Trial Court of Davao del Sur, 11 th
Judicial Region, Digos, Davao del Sur convicting defendant- appellant Ruben Burgos y
Tito of The crime of Illegal Possession of Firearms in Furtherance of Subversion. The
dispositive portion of the decision reads:

WHEREFORE, finding the guilt of accused Ruben Burgos sufficiently established


beyond reasonable doubt, of the offense charges , pursuant to Presidential Decree No.
9, in relation to General Order No. 6, dated September 22, 1972, and General Order No.
7, dated September 23, 1972, in relation further to Presidential Decree No. 885, and
considering that the firearm subject of this case was not used in the circumstances as
embraced in paragraph I thereof, applying the provision of indeterminate sentence law,
accused Ruben Burgos is hereby sentenced to suffer an imprisonment of twenty (20)
years of reclusion temporal maximum, as minimum penalty, to reclusion perpetua, as
maximum penalty, pursuant to sub-paragraph B, of Presidential Decree No. 9, as
aforementioned, with accessory penalties, as provided for by law.

As a result of this judgment, the subject firearm involved in this case (Homemade
revolver, caliber .38, Smith and Wesson, with Serial No. 8.69221) is hereby ordered
confiscated in favor of the government, to be disposed of in accordance with law.
Likewise, the subversive documents, leaflets and/or propaganda seized are ordered
disposed of in accordance with law.

The information charged the defendant-appellant with the crime of illegal possession of
firearm in furtherance of subversion in an information which reads as follows:

That in the afternoon of May 13, 1982 and thereabout at Tiguman, Digos, Davao del
Sur, Philippines, within the jurisdiction of this Court, the above- named accused with
intent to possess and without the necessary license, permit or authority issued by the
proper government agencies, did then and there wilfully, unlawfully and feloniously
keep, possess, carry and have in his possession, control and custody one (1) homemade
revolver, caliber .38, make Smith and Wesson, with Serial No. 8.69221, which firearm
was issued to and used by the accused at Tiguman, Digos, Davao del Sur, his area of
operations by one Alias Commander Pol for the New People's Army (NPA), a subversive
Page 268 of 377

organization organized for the purpose of overthrowing the Government of the Republic
of the Philippines through lawless and violent means, of which the accused had
knowledge, and which firearm was used by the accused in the performance of his
subversive tasks such as the recruitment of New Members to the NPA and collection of
contributions from the members.

CONTRARY TO LAW.

The evidence for the prosecution is summarized in the decision of the lower court as
follows:

xxx xxx xxx

. . . Through the testimony of Pat. Pepito Bioco, and Sgt. Romeo Taroy, it appears that
by virtue of an intelligent information obtained by the Constabulary and INP units,
stationed at Digos, Davao del Sur, on May 12, 1982, one Cesar Masamlok personally
and voluntarily surre0ndered to the authorities at about 9:00 o'clock A.M. at Digos,
Davao del Sur Constabulary Headquarters, stating that he was forcibly recruited by
accused Ruben Burgos as member of the NPA, threatening him with the use of firearm
against his life, if he refused.

Along with his recruitment, accused was asked to contribute one (1) chopa of rice and
one peso (P1.00) per month, as his contribution to the NPA TSN, page 5, Hearing-
October 14, 1982).

Immediately, upon receipt of said information, a joint team of PC-INP units, composed
of fifteen (15) members, headed by Captain Melchesideck Bargio, (PC), on the following
day, May 13, 1982, was dispatched at Tiguman; Davao del Sur, to arrest accused Ruben
Burgos. The team left the headquarter at 1:30 P.M., and arrived at Tiguman, at more or
less 2:00 o'clock PM where through the help of Pedro Burgos, brother of accused, the
team was able to locate accused, who was plowing his field. (TSN, pages 6-7, Hearing-
October 14, 1982).

Right in the house of accused, the latter was caned by the team and Pat. Bioco asked
accused about his firearm, as reported by Cesar Masamlok. At first accused denied
possession of said firearm but later, upon question profounded by Sgt. Alejandro
Buncalan with the wife of the accused, the latter pointed to a place below their house
where a gun was buried in the ground. (TSN, page 8, Hearing-October 14, 1982).
Page 269 of 377

Pat. Bioco then verified the place pointed by accused's wife and dug the grounds, after
which he recovered the firearm, Caliber .38 revolver, marked as Exhibit "A" for the
prosecution.

After the recovery of the firearm, accused likewise pointed to the team, subversive
documents which he allegedly kept in a stock pile of qqqcogon at a distance of three (3)
meters apart from his house. Then Sgt. Taroy accordingly verified beneath said cogon
grass and likewise recovered documents consisting of notebook colored maroon with
spiral bound, Exhibit "B" for the prosecution; a pamphlet consisting of eight (8) leaves,
including the front and back covers entitled Ang Bayan, Pahayagan ng Partido
Komunista ng Pilipinas, Pinapatnubayan ng Marxismo, Leninismo Kaisipang Mao
qqqZedong dated December 31, 1980, marked as Exhibit "C", and another pamphlet
Asdang Pamantalaang Masa sa Habagatang Mindanao, March and April 1981 issue,
consisting of ten (10) pages, marked as Exhibit "D" for the prosecution.

Accused, when confronted with the firearm Exhibit "A", after its recovery, readily
admitted the same as issued to him by Nestor Jimenez, otherwise known as a certain
Alias Pedipol, allegedly team leader of the sparrow unit of New People's Army,
responsible in the liquidation of target personalities, opposed to NPA Ideological
movement, an example was the killing of the late Mayor Llanos and Barangay Captain
of Tienda Aplaya Digos, Davao del Sur. (TSN, pages 1-16, Hearing-October 14,1982).

To prove accused's subversive activities, Cesar Masamlok, a former NPA convert was
presented, who declared that on March 7, 1972, in his former residence at Tiguman
Digos, Davao del Sur, accused Ruben Burgos, accompanied by his companions
Landrino Burgos, Oscar Gomez and Antonio Burgos, went to his house at about 5:00
o'clock P.M. and called him downstair. Thereupon, accused told Masamlok, their
purpose was to ask rice and one (1) peso from him, as his contribution to their
companions, the NPA of which he is now a member. (TSN, pages 70, 71, 72, Hearing-
January 4, 1983).

Accused and his companions told Masamlok, he has to join their group otherwise, he
and his family will be killed. He was also warned not to reveal anything with the
government authorities. Because of the threat to his life and family, Cesar Masamlok
joined the group. Accused then told him, he should attend a seminar scheduled on April
19, 1982. Along with this invitation, accused pulled gut from his waistline a .38 caliber
revolver which Masamlok really saw, being only about two (2) meters away from
accused, which make him easily Identified said firearm, as that marked as Exhibit "A"
for the prosecution. (TSN, pages 72, 73, and 74, Hearing-January 4, 1983).
Page 270 of 377

On April 19, 1982, as previously invited, Masamlok, accompanied by his father,


Matuguil Masamlok, Isabel Ilan and Ayok Ides went to the house of accused and
attended the seminar, Those present in the seminar were: accused Ruben Burgos,
Antonio Burgos, Oscar Gomez, Landrino Burgos, alias Pedipol and one alias Jamper.

The first speaker was accused Ruben Burgos, who said very distinctly that he is an NPA
together with his companions, to assure the unity of the civilian. That he encouraged
the group to overthrow the government, emphasizing that those who attended the
seminar were already members of the NPA, and if they reveal to the authorities, they
will be killed.

Accused, while talking, showed to the audience pamphlets and documents, then finally
shouted, the NPA will be victorious. Masamlok likewise Identified the pamphlets as
those marked as Exh. exhibits "B", "C", and "D" for the prosecution. (TSN, pages 75, 76
and 77, Hearing-January 4, 1983)

Other speakers in said meeting were Pedipol, Jamper and Oscar Gomez, who likewise
expounded their own opinions about the NPA. It was also announced in said seminar
that a certain Tonio Burgos, will be responsible for the collection of the contribution
from the members. (TSN, pages 78-79, Hearing- January 4, 1983)

On May 12, 1982, however, Cesar Masamlok surrendered to Captain Bargio of the
Provincial Headquarters of the Philippine Constabulary, Digos, Davao del Sur.

Assistant Provincial Fiscal Panfilo Lovitos was presented t prove that on May 19, 1982,
he administered the subscription of th extra-judicial confession of accused Ruben
Burgos, marked as Exhibit "E " for the prosecution, consisting of five (5) pages.

Appearing voluntarily in said office, for the subscription of his confession, Fiscal Lovitos,
realizing that accused was not represented by counsel, requested the services of Atty.
Anyog, whose office is adjacent to the Fiscal's Office, to assist accused in the
subscription of his extra-judicial statement.

Atty. Anyog assisted accused in the reading of his confession from English to Visayan
language, resulting to the deletion of question No. 19 of the document, by an inserted
certification of Atty. Anyog and signature of accused, indicating his having understood,
the allegations of his extra-judicial statement.
Page 271 of 377

Fiscal Lovitos, before accused signed his statement, explained to him his constitutional
rights to remain silent, right to counsel and right to answer any question propounded
or not.

With the aid of Atty. Anyog, accused signed his confession in the presence of Atty. Anyog
and Fiscal Lovitos, without the presence of military authorities, who escorted the
accused, but were sent outside the cubicle of Fiscal Lovitos while waiting for the
accused. (TSN, pages 36-40, nearing November 15, 1982)

Finally, in order to prove illegal possession by accused of the subject firearm, Sgt.
Epifanio Comabig in-charge of firearms and explosives, NCO Headquarter, Philippine
Constabulary, Digos, Davao del Sur, was presented and testified, that among the lists
of firearm holders in Davao del Sur, nothing was listed in the name of accused Ruben
Burgos, neither was his name included among the lists of persons who applied for the
licensing of the firearm under Presidential Decree No. 1745.

After the above-testimony the prosecution formally closed its case and offered its
exhibits, which were all admitted in evidence, despite objection interposed by counsel
for accused, which was accordingly overruled.

On the other hand, the defendant-appellant's version of the case against him is stated
in the decision as follows:

From his farm, the military personnel, whom he said he cannot recognize, brought him
to the PC Barracks at Digos, Davao del Sur, and arrived there at about 3:00 o'clock, on
the same date. At about 8:00 o'clock P.M., in the evening, he was investigated by
soldiers, whom he cannot Identify because they were wearing a civilian attire. (TSN, page
14 1, Hearing-June 15, 1983)

The investigation was conducted in the PC barracks, where he was detained with respect
to the subject firearm, which the investigator, wished him to admit but accused denied
its ownership. Because of his refusal accused was mauled, hitting him on the left and
right side of his body which rendered him unconscious. Accused in an atmosphere of
tersed solemnity, crying and with emotional attachment, described in detail how he was
tortured and the ordeals he was subjected.
Page 272 of 377

He said, after recovery of his consciousness, he was again confronted with subject
firearm, Exhibit "A", for him to admit and when he repeatedly refused to accept as his
own firearm, he was subjected to further prolong (sic) torture and physical agony.
Accused said, his eyes were covered with wet black cloth with pungent effect on his eyes.
He was undressed, with only blindfold, pungent water poured in his body and over his
private parts, making his entire body, particularly his penis and testicle, terribly
irritating with pungent pain.

All along, he was investigated to obtain his admission, The process of beating, mauling,
pain and/or ordeal was repeatedly done in similar cycle, from May 13 and 14, 1982.
intercepted only whenever he fell unconscious and again repeated after recovery of his
senses,

Finally on May 15, 1982, after undergoing the same torture and physical ordeal he was
seriously warned, if he will still adamantly refuse to accept ownership of the subject
firearm, he will be salvaged, and no longer able to bear any further the pain and agony,
accused admitted ownership of subject firearm.

After his admission, the mauling and torture stopped, but accused was made to sign
his affidavit marked as Exhibit "E" for the prosecution, consisting of five (5) pages,
including the certification of the administering officer, (TSN, pages 141-148, Hearing-
June 15, 1983)

In addition to how he described the torture inflicted on him, accused, by way of


explanation and commentary in details, and going one by one, the allegations and/or
contents of his alleged extrajudicial statement, attributed his answers to those
questions involuntarily made only because of fear, threat and intimidation of his person
and family, as a result of unbearable excruciating pain he was subjected by an
investigator, who, unfortunately he cannot Identify and was able to obtain his admission
of the subject firearm, by force and violence exerted over his person.

To support denial of accused of being involved in any subversive activities, and also to
support his denial to the truth of his alleged extra-judicial confession, particularly
questions Nos. 35, 38, 41, 42, 43, 44, 45, 46 and 47, along with qqqs answers to those
questions, involving Honorata Arellano ahas Inday Arellano, said Honorata Arellano
appeared and declared categorically, that the above-questions embraced in the numbers
allegedly stated in the extrajudicial confession of accused, involving her to such NPA
personalities, as Jamper, Pol, Anthony, etc., were not true because on the date referred
on April 28, 1982, none of the persons mentioned came to her house for treatment,
Page 273 of 377

neither did she meet the accused nor able to talk with him. (TSN, pages 118- 121,
Hearing-May 18, 1983)

She, however, admitted being familiar with one Oscar Gomez, and that she was
personally charged with subversion in the Office of the Provincial Commander,
Philippine Constabulary, Digos, Davao del Sur, but said charge was dismissed without
reaching the Court. She likewise stated that her son, Rogelio Arellano, was likewise
charged for subversion filed in the Municipal Trial Court of Digos, Davao del Sur, but
was likewise dismissed for lack of sufficient evidence to sustain his conviction. (TSN,
pages 121-122, in relation to her cross-examination, Hearing-May 18, 1983)

To support accused's denial of the charge against him, Barangay Captain of Tiguman,
Digos, Davao del Sur, Salvador qqqGalaraga was presented, who declared, he was not
personally aware of any subversive activities of accused, being his neighbor and member
of his barrio. On the contrary, he can personally attest to his good character and
reputation, as a law abiding citizen of his barrio, being a carpenter and farmer thereat.
(TSl pages 128-129, Hearing-May 18, 1983)

He however, admitted in cross-examination, that there were a lot of arrests made by the
authorities in his barrio involving subversive activities but they were released and were
not formally charged in Court because they publicly took their oath of allegiance with
the government. (TSN, pages 133-134, in relation to page 136, Hearing-May 18, 1983)

Finally, to support accused's denial of the subject firearm, his wife, Urbana Burgos, was
presented and who testified that the subject firearm was left in their house by Cesar
Masamlok and one Pedipol on May 10, 1982. It was night time, when the two left the
gun, alleging that it was not in order, and that they will leave it behind, temporarily for
them to claim it later. They were the ones who buried it. She said, her husband, the
accused, was not in their house at that time and that she did not inform him about said
firearm neither did she report the matter to the authorities, for fear of the life of her
husband. (TSN, page 24, November 22, 1983)

On cross-examination, she said, even if Masamlok during the recovery of the firearm,
was wearing a mask, she can still Identify him. (TSN, page 6, Hearing-November 22,
1983)

After the above-testimony, accused through counsel formally rested his case in support
of accused's through counsel manifestation for the demurrer to evidence of the
prosecution, or in the alternative for violation merely of simple illegal possession of
Page 274 of 377

firearm, 'under the Revised Administrative Code, as amended by Republic Act No. 4,
reflected in the manifestation of counsel for accused. (TSN, pages 113-114, Hearing-
May 18, 1983)

Accused-appellant Ruben Burgos now raises the following assignments of error, to wit:

I THE TRIAL COURT ERRED IN HOLDING THAT (SIC) THE ARREST OF ACCUSED-
APPELLANT WITHOUT VALID WARRANT TO BE LAWFUL.

II THE TRIAL COURT ERRED IN HOLDING THE SEARCH IN THE HOUSE OF


ACCUSED-APPELLANT FOR FIREARM WITHOUT VALID WARRANT TO BE LAWFUL.

III THE TRIAL COURT ERRED IN HOLDING ACCUSED-APPELLANT GUILTY BEYOND


REASONABLE DOUBT FOR VIOLATION OF P.D. No. 9 IN RELATION TO GENERAL
ORDERS NOS. 6 AND 7

Was the arrest of Ruben Burgos lawful? Were the search of his house and the
subsequent confiscation of a firearm and documents allegedly found therein conducted
in a lawful and valid manner? Does the evidence sustaining the crime charged meet the
test of proving guilt beyond reasonable doubt?

The records of the case disclose that when the police authorities went to the house of
Ruben Burgos for the purpose of arresting him upon information given by Cesar
Masamlok that the accused allegedly recruited him to join the New People's Army (NPA),
they did not have any warrant of arrest or search warrant with them (TSN, p. 25, October
14, 1982; and TSN, p. 61, November 15, 1982).

Article IV, Section 3 of the Constitution provides:

The right of the people to be secure in their persons, houses, papers, and effects against
unreasonable searches and seizures of whatever nature and for any purpose shall not
be violated, and no search warrant or warrant of arrest shall issue except upon probable
cause to be determined by the judge, or such other responsible officer as may be
authorized by law, after examination under oath or affirmation of the complainant and
the witnesses he may produce, and particularly describing the place to be searched, and
the persons or things to be seized.
Page 275 of 377

The constitutional provision is a safeguard against wanton and unreasonable invasion


of the privacy and liberty of a citizen as to his person, papers and effects. This Court
explained in Villanueva vs. Querubin (48 SCRA 345) why this right is so important:

It is deference to one's personality that lies at the core of this right, but it could be also
looked upon as a recognition of a constitutionally protected area, primarily one's home,
but not necessarily thereto confined. (Cf. Hoffa v. United States, 385 US 293 [19661)
What is sought to be guarded is a man's prerogative to choose who is allowed entry to
his residence. In that haven of refuge, his individuality can assert itself not only in the
choice of who shall be welcome but likewise in the kind of objects he wants around him.
There the state, however powerful, does not as such have access except under the
circumstances above noted, for in the traditional formulation, his house, however
humble, is his castle. Thus is outlawed any unwarranted intrusion by government,
which is called upon to refrain from any invasion of his dwelling and to respect the
privacies of his life, (Cf. Schmerber v. California, 384 US 757 [1966], Brennan, J. and
Boyd v. United States, 116 US 616, 630 [1886]). In the same vein, Landynski in his
authoritative work (Search and Seizure and the Supreme Court [1966], could fitly
characterize this constitutional right as the embodiment of a 'spiritual concept: the belief
that to value the privacy of home and person and to afford its constitutional protection
against the long reach of government is no legs than to value human dignity, and that
his privacy must not be disturbed except in case of overriding social need, and then only
under stringent procedural safeguards.' (Ibid, p. 47).

The trial court justified the arrest of the accused-appelant without any warrant as falling
under one of the instances when arrests may be validly made without a warrant. Rule
113, Section 6 * of the Rules of Court, provides the exceptions as follows:

a) When the person to be arrested has committed, is actually committing, or is about to


commit an offense in his presence;

b) When an offense has in fact been committed, and he has reasonable ground to believe
that the person to be arrested has committed it;

c) When the person to be arrested is a prisoner who has escaped from a penal
establishment or place where he is serving final judgment or temporarily confined while
his case is pending or has escaped while being transferred from one confinement to
another.
Page 276 of 377

The Court stated that even if there was no warrant for the arrest of Burgos, the fact that
"the authorities received an urgent report of accused's involvement in subversive
activities from a reliable source (report of Cesar Masamlok) the circumstances of his
arrest, even without judicial warrant, is lawfully within the ambit of Section 6-A of Rule
113 of the Rules of Court and applicable jurisprudence on the matter."

If the arrest is valid, the consequent search and seizure of the firearm and the alleged
subversive documents would become an incident to a lawful arrest as provided by Rule
126, Section 12, which states:

A person charged with an offense may be searched for dangerous weapons or anything
which may be used as proof of the commission of the offense.

The conclusions reached by the trial court are erroneous.

Under Section 6(a) of Rule 113, the officer arresting a person who has just committed,
is committing, or is about to commit an offense must have personal knowledge of that
fact. The offense must also be committed in his presence or within his view. (Sayo v.
Chief of Police, 80 Phil. 859).

There is no such personal knowledge in this case. Whatever knowledge was possessed
by the arresting officers, it came in its entirety from the information furnished by Cesar
Masamlok. The location of the firearm was given by the appellant's wife.

At the time of the appellant's arrest, he was not in actual possession of any firearm or
subversive document. Neither was he committing any act which could be described as
subversive. He was, in fact, plowing his field at the time of the arrest.

The right of a person to be secure against any unreasonable seizure of his body and any
deprivation of his liberty is a most basic and fundamental one. The statute or rule which
allows exceptions to the requirement of warrants of arrest is strictly construed. Any
exception must clearly fall within the situations when securing a warrant would be
absurd or is manifestly unnecessary as provided by the Rule. We cannot liberally
construe the rule on arrests without warrant or extend its application beyond the cases
specifically provided by law. To do so would infringe upon personal liberty and set back
a basic right so often violated and so deserving of full protection.
Page 277 of 377

The Solicitor General is of the persuasion that the arrest may still be considered lawful
under Section 6(b) using the test of reasonableness. He submits that. the information
given by Cesar Masamlok was sufficient to induce a reasonable ground that a crime has
been committed and that the accused is probably guilty thereof.

In arrests without a warrant under Section 6(b), however, it is not enough that there is
reasonable ground to believe that the person to be arrested has committed a crime. A
crime must in fact or actually have been committed first. That a crime has actually been
committed is an essential precondition. It is not enough to suspect that a crime may
have been committed. The fact of the commission of the offense must be undisputed.
The test of reasonable ground applies only to the identity of the perpetrator.

In this case, the accused was arrested on the sole basis of Masamlok's verbal report.
Masamlok led the authorities to suspect that the accused had committed a crime. They
were still fishing for evidence of a crime not yet ascertained. The subsequent recovery of
the subject firearm on the basis of information from the lips of a frightened wife cannot
make the arrest lawful, If an arrest without warrant is unlawful at the moment it is
made, generally nothing that happened or is discovered afterwards can make it lawful.
The fruit of a poisoned tree is necessarily also tainted.

More important, we find no compelling reason for the haste with which the arresting
officers sought to arrest the accused. We fail to see why they failed to first go through
the process of obtaining a warrant of arrest, if indeed they had reasonable ground to
believe that the accused had truly committed a crime. There is no showing that there
was a real apprehension that the accused was on the verge of flight or escape. Likewise,
there is no showing that the whereabouts of the accused were unknown,

The basis for the action taken by the arresting officer was the verbal report made by
Masamlok who was not required to subscribe his allegations under oath. There was no
compulsion for him to state truthfully his charges under pain of criminal prosecution.
(TSN, p. 24, October 14, 1982). Consequently, the need to go through the process of
securing a search warrant and a warrant of arrest becomes even more clear. The arrest
of the accused while he was plowing his field is illegal. The arrest being unlawful, the
search and seizure which transpired afterwards could not likewise be deemed legal as
being mere incidents to a valid arrest.

Neither can it be presumed that there was a waiver, or that consent was given by the
accused to be searched simply because he failed to object. To constitute a waiver, it
must appear first that the right exists; secondly, that the person involved had
knowledge, actual or constructive, of the existence of such a right; and lastly, that said
Page 278 of 377

person had an actual intention to relinquish the right (Pasion Vda. de Garcia v. Locsin,
65 Phil. 689). The fact that the accused failed to object to the entry into his house does
not amount to a permission to make a search therein (Magoncia v. Palacio, 80 Phil. 770).
As pointed out by Justice Laurel in the case of Pasion Vda. de Garcia V. Locsin (supra)

xxx xxx xxx

. . . As the constitutional guaranty is not dependent upon any affirmative act of the
citizen, the courts do not place the citizen in the position of either contesting an officer's
authority by force, or waiving his constitutional rights; but instead they hold that a
peaceful submission to a search or seizure is not a consent or an invitation thereto, but
is merely a demonstration of regard for the supremacy of the law. (56 C.J., pp. 1180,
1181).

We apply the rule that: "courts indulge every reasonable presumption against waiver of
fundamental constitutional rights and that we do not presume acquiescence in the loss
of fundamental rights." (Johnson v. Zerbst 304 U.S. 458).

That the accused-appellant was not apprised of any of his constitutional rights at the
time of his arrest is evident from the records:

A CALAMBA:

Q When you went to the area to arrest Ruben Burgos, you were not armed with an arrest
warrant?

A None Sir.

Q Neither were you armed with a search warrant?

A No Sir.

Q As a matter of fact, Burgos was not present in his house when you went there?

A But he was twenty meters away from his house.


Page 279 of 377

Q Ruben Burgos was then plowing his field?

A Yes Sir.

Q When you called for Ruben Burgos you interviewed him?

A Yes Sir.

Q And that you told him that Masamlok implicated him?

A No Sir.

Q What did you tell him?

A That we received information that you have a firearm, you surrender that firearm, first
he denied but when Sgt. Buncalan interviewed his wife, his wife told him that it is
buried, I dug the firearm which was wrapped with a cellophane.

Q In your interview of Burgos you did not remind him of his rights under the constitution
considering that he was purposely under arrest?

A I did not.

Q As a matter of fact, he denied that he has ever a gun?

A Yes Sir.

Q As a matter of fact, the gun was not in his possession?

A It was buried down in his horse.


Page 280 of 377

Q As a matter of fact, Burgos did not point to where it was buried?

A Yes Sir.

(TSN, pp. 25-26, Hearing-October 14, 1982)

Considering that the questioned firearm and the alleged subversive documents were
obtained in violation of the accused's constitutional rights against unreasonable
searches and seizures, it follows that they are inadmissible as evidence.

There is another aspect of this case.

In proving ownership of the questioned firearm and alleged subversive documents, the
prosecution presented the two arresting officers who testified that the accused readily
admitted ownership of the gun after qqqs wife pointed to the place where it was buried.
The officers stated that it was the accused himself who voluntarily pointed to the place
where the alleged subversive documents were hidden.

Assuming this to be true, it should be recalled that the accused was never informed of
his constitutional rights at the time of his arrest. So that when the accused allegedly
admitted ownership of the gun and pointed to the location of the subversive documents
after questioning, the admissions were obtained in violation of the constitutional right
against self-incrimination under Sec. 20 of Art. IV of the Bill of Rights winch provides:

No person shall be compelled to be a witness against himself. Any person under


investigation for the commission of an offense shall have the right to remain silent and
to counsel, and to be informed of such right.. . .

The Constitution itself mandates that any evidence obtained in violation of this right is
inadmissible in evidence. Consequently, the testimonies of the arresting officers as to
the admissions made by the appellant cannot be used against him.

The trial court validly rejected the extra-judicial confession of the accused as
inadmissible in evidence. The court stated that the appellant's having been exhaustively
Page 281 of 377

subjected to physical terror, violence, and third degree measures may not have been
supported by reliable evidence but the failure to present the investigator who conducted
the investigation gives rise to the "provocative presumption" that indeed torture and
physical violence may have been committed as stated.

The accused-appellant was not accorded his constitutional right to be assisted by


counsel during the custodial interrogation. The lower court correctly pointed out that
the securing of counsel, Atty. Anyog, to help the accused when he subscribed under
oath to his statement at the Fiscal's Office was too late. It could have no palliative effect.
It cannot cure the absence of counsel at the time of the custodial investigation when the
extrajudicial statement was being taken.

With the extra-judicial confession, the firearm, and the alleged subversive documents
inadmissible in evidence against the accused-appellant, the only remaining proof to
sustain the charge of Illegal Possession of Firearm in Furtherance of Subversion is the
testimony of Cesar Masamlok.

We find the testimony of Masamlok inadequate to convict Burgos beyond reasonable


doubt. It is true that the trial court found Masamlok's testimony credible and
convincing. However, we are not necessarily bound by the credibility which the trial
court attaches to a particular witness. As stated in People vs.. Cabrera (100 SCRA 424):

xxx xxx xxx

. . .Time and again we have stated that when it comes to question of credibility the
findings of the trial court are entitled to great respect upon appeal for the obvious reason
th+at it was able to observe the demeanor, actuations and deportment of the witnesses
during the trial. But we have also said that this rule is not absolute for otherwise there
would be no reversals of convictions upon appeal. We must reject the findings of the
trial court where the record discloses circumstances of weight and substance which
were not properly appreciated by the trial court.

The situation under which Cesar Masamlok testified is analogous to that found in People
vs. Capadocia (17 SCRA 98 1):

. . . The case against appellant is built on Ternura's testimony, and the issue hinges on
how much credence can be accorded to him. The first consideration is that said
testimony stands uncorroborated. Ternura was the only witness who testified on the
mimeographing incident. . . .
Page 282 of 377

xxx xxx xxx


. . .He was a confessed Huk under detention at the time. He knew his fate depended
upon how much he cooperated with the authorities, who were then engaged in a
vigorous anti-dissident campaign. As in the case of Rodrigo de Jesus, whose testimony
We discounted for the same reason, that of Ternura cannot be considered as proceeding
from a totally unbiased source. . . .

In the instant case, Masamlok's testimony was totally uncorroborated. Considering that
Masamlok surrendered to the military certainly his fate depended on how eagerly he
cooperated with the authorities. Otherwise, he would also be charged with subversion.
The trade-off appears to be his membership in the Civil Home Defense Force. (TSN, p.
83, January 4, 1983). Masamlok may be considered as an interested witness. It can not
be said that his testimony is free from the opportunity and temptation to be exaggerated
and even fabricated for it was intended to secure his freedom.

Despite the fact that there were other persons present during the alleged NPA seminar
of April 19, 1982 i.e., Masamlok's father ,Matuguil Masamlok, Isabel Ilan and Ayok Ides
(TSN, p. 74, January 4, 1983) who could have corroborated Cesar Masamlok's testimony
that the accused used the gun in furtherance of subversive activities or actually engaged
in subversive acts, the prosecution never presented any other witness.

This Court is, therefore, constrained to rule that the evidence presented by the
prosecution is insufficient to prove the guilt of the accused beyond reasonable doubt.

As held in the case of People vs. Baia (34 SCRA 347):

It is evident that once again, reliance can be placed on People v. Dramayo (42 SCRA 59),
where after stressing that accusation is not, according to the fundamental law,
synonymous with guilt, it was made clear: 'Only if the judge below and the appellate
tribunal could arrive at a conclusion that the crime had been committed precisely by
the person on trial under such an exacting test should the sentence be one of conviction.
It is thus required that every circumstance favoring his innocence be duly taken into
account. The proof against him must survive the test of reason; the strongest suspicion
must not be permitted to sway judgment. The conscience must be satisfied that on the
defendant could be laid the responsibility for the offense charged; that not only did he
perpetrate the act but that it amounted to a crime. What is required then is moral
certainty.' (Ibid, 64. Cf. People v. Alvarez, 55 SCRA 81; People v. Joven, 64 SCRA 126;
People vs. Ramirez, 69 SCRA 144; People vs. Godov 72 SCRA 69; People v. Lopez, 74
Page 283 of 377

SCRA 205; People v. Poblador, 76 SCRA 634; People v. Quiazon, 78 SCRA 513; People
v. Nazareno, 80 SCRA 484; People vs. Gabilan 115 SCRA 1; People v. Gabiana, 117
SCRA 260; and People vs. Ibanga 124 SCRA 697).

We are aware of the serious problems faced by the military in Davao del Sur where there
appears to be a well-organized plan to overthrow the Government through armed
struggle and replace it with an alien system based on a foreign ideology. The open
defiance against duly constituted authorities has resulted in unfortunate levels of
violence and human suffering publicized all over the country and abroad. Even as we
reiterate the need for all freedom loving citizens to assist the military authorities in their
legitimate efforts to maintain peace and national security, we must also remember the
dictum in Morales vs. Enrile (1 21 SCRA 538, 569) when this Court stated:

While the government should continue to repel the communists, the subversives, the
rebels, and the lawless with an the means at its command, it should always be
remembered that whatever action is taken must always be within the framework of our
Constitution and our laws.

Violations of human rights do not help in overcoming a rebellion. A cavalier attitude


towards constitutional liberties and protections will only fan the increase of subversive
activities instead of containing and suppressing them.

WHEREFORE, the judgment of conviction rendered by the trial court is REVERSED and
SET ASIDE. The accused-appellant is hereby ACQUITTED, on grounds of reasonable
doubt, of the crime with which he has been charged.

The subject firearm involved in this case (homemade revolver, caliber .38, Smith and
Wesson, with Serial No. 8.69221) and the alleged subversive documents are ordered
disposed of in accordance with law.

Cost de oficio.
SO ORDERED.
Feria (Chairman), Fernan, Alampay and Paras, JJ., concur.
Page 284 of 377

G.R. No. 83260 April 18, 1990

PEOPLE OF THE PHILIPPINES, plaintiff-appellee, vs. JUAN DE LA CRUZ y


GONZALES and REYNALDO BELTRAN y ANIBAN, accused-appellants.

The Solicitor General for plaintiff-appellee.


Augusta J. Salas for accused-appellants.

REGALADO, J.:

Accused-appellant Juan de la Cruz y Gonzales and his co-accused Reynaldo Beltran y


Aniban were charged in Criminal Case No. 87-54417 of the Regional Trial Court of
Manila with violation of Section 4, Art. II, in relation to Section 21, Article IV of Republic
Act No. 6425, as amended, in an information which reads:

That on or about May 4, 1987, in the City of Manila, Philippines, the said accused,
conspiring and confederating together and mutually helping each other, not being
authorized by law to sell, deliver, give away to another or distribute any prohibited drug,
did then and there wilfully, unlawfully, and knowingly sell, deliver or give away to and
other the following:

1. One (1) cigarette foil wrapper containing marijuana;

2. Two (2) cigarette foil wrapper (sic) containing marijuana which are prohibited drugs.

Contrary to law.1

The accused, who were assisted by a counsel de oficio, pleaded not guilty when
arraigned on May 26, 1987. On August 18, 1987, trial on the merits started, with the
prosecution thereafter presenting as its witnesses P/Pfc. Adolfo Arcoy, P/Capt. Luena
Layador, T/Sgt. Jaime Raposas, Sgt. Vicente Jimenez, and S/Sgt. Armando Isidro. On
its part, the defense presented both accused, Lolita Mendoza and Maribeth Manapat as
its witnesses.

The court a quo, in a comparative evaluation of evidence, painstakingly summarized the


clashing factual versions of the prosecution and defense, as follows:
Page 285 of 377

. . . On its part, the prosecution alleged that after receiving a confidential report from
Arnel, their informant, a "buy-bust" operation was conducted by the 13th Narcotics
Regional Unit through a team composed of T/Sgt. Jaime Raposas as Team Leader,
S/Sgt. Rodelito Oblice, Sgt. Dante Yang, Sgt. Vicente Jimenez, P/Pfc. Adolfo Arcoy as
poseur-buyer and Pat. Deogracias Gorgonia at Maliclic St., Tondo, Manila at around
2:30 o'clock in the afternoon of May 4, 1987 to catch the pusher/s. P/Pfc. Adolfo Arcoy
acted as the poseur-buyer with Arnel as his companion to buy marijuana worth P10.00
from the two accused, Juan de la Cruz and Reynaldo Beltran. At the scene, it was Juan
de la Cruz whom Arcoy first negotiated (with) on the purchase and when Arcoy told De
la Cruz that he was buying P10.00 worth of marijuana, De la Cruz instructed Reynaldo
Beltran to give one aluminum foil of marijuana which Beltran got from his pants' pocket
and delivered it to Arcoy. After ascertaining that the foil of suspected marijuana was
really marijuana, Arcoy gave the prearranged signal to his teammates by scratching his
head and his teammates who were strategically positioned in the vicinity, converged at
the place, identified themselves as NARCOM agents and effected the arrest of De la Cruz
and Beltran. The P10.00 marked bill (Exhibit C-1) used by Arcoy was found in the
possession of Juan de la Cruz together with two aluminum foils and containing
marijuana (Exhibits "B-2" and "B-3").

Traversing this version is that of the defense which, in brief, consists of a denial to (sic)
the prosecution's theory and the claim that accused Juan de la Cruz, who was then
suffering from loose bowel movement, was all the time in bed at their place at 3034
Maliclic St., Tondo, Manila; that he never left their place throughout that day of May 4,
1987; that he never had a visitor on that day and that he was never engaged in the sale
of marijuana. The NARCOM agents raided his place without search warrant or without
first securing his previous permission. One searched thoroughly his place, the second
acted as a guard posted at the door of De la Cruz' place and the third agent was a mere
observer. His place was ransacked and he was even bodily searched. As regards accused
Reynaldo Beltran, he was arrested by the same group (prior to the arrest of Juan de la
Cruz) while he was playing "pool" at Aling Ely's place along Maliclic St. that afternoon
and that without much ado, he was taken because he was fingered by one Arnel to be
engaged in selling marijuana. Both accused were brought to a parked vehicle of the
raiding team, From there, they were taken to NARCOM headquarters for investigation
where for the first time they came to know that they were being charged of selling
marijuana. 2

Finding the version of the prosecution more worthy of credit, the court a quo rendered
its decision3 on March 15, 1988, the decretal portion of which states:

WHEREFORE, in the light of the foregoing consideration, the Court finds the accused,
JUAN DE LA CRUZ y GONZALES and REYNALDO BELTRAN y ANIBAN, guilty beyond
Page 286 of 377

reasonable doubt of the Violation of Section 4, Article II, in relation to Section 21, Article
IV, both of Republic Act No. 6425, otherwise known as Dangerous Drugs Act of 1972,
as further amended by Presidential Decree No. 1675 and as charged in the Information,
and, accordingly, hereby sentences each of them to suffer the penalty of reclusion
perpetua, with the accessory penalties provided by law; to pay a fine of TWENTY
THOUSAND (P20,000.00) PESOS, Philippine currency, without subsidiary
imprisonment in case of insolvency, and each to pay one-half of the costs.

The three (3) aluminum foils containing marijuana (Exhibits "B-2" to "B-4") placed in an
empty Marlboro pack (Exhibit "B-1") are hereby ordered confiscated and forfeited in
favor of the government and once this Decision shall become final and executory, the
same shall be turned over to the Dangerous Drugs Board through the Director, National
Bureau of Investigation, Manila, for proper disposition while the P10.00 bill (Exhibit "C-
1") bearing Serial No. F-215962 shall be returned to T/Sgt. Jaime Raposas.

Furnish copy of this Decision to the Honorable Supreme Court through the Honorable
Court Administrator. 4

From this decision, accused Juan de la Cruz y Gonzales and co-accused Reynaldo
Beltran y Aniban interposed the instant appeal.

In a letter of the Warden, Manila City Jail, dated March 3, 1989, 5 the Court was
informed of the death of accused-appellant Juan de la Cruz y Gonzales on February 21,
1989. Counsel de oficio having thereafter submitted a certified true copy of the death
certificate of the accused 6 as directed by the Court, the criminal case against said
accused-appellant was dismissed in our resolution of September 25, 1989. 7

The present appellate proceeding is, therefore, limited only to appellant Reynaldo
Beltran y Aniban who now faults the trial court with the following assignment of errors:

1. The Buy-Bust Operation being done to enforce Republic Act 6425 is unconstitutional
and any evidence acquired under such method should not be admissible in court.

2. The Buy-Bust Operation should be declared illegal for it breeds corruption of police
and military officers through planting of evidence for purposes of extortion.
Page 287 of 377

3. The Court erred in giving probative value to the confiscated marijuana sticks despite
the fact that no civilian or other neutral person signed as a witness to its taking. If it
were true, there must be at least one civic-minded citizen who could easily be convinced
by the police to witness it.

4. The Court erred in considering the evidence, Exhibits "B-2," "B-3" and "B-4", as the
very ones confiscated.

If they were the very ones taken from the accused, the original receipt prepared at the
scene of the crime would not have been thrown away by the very agent who acted as the
buyer. Exhibit "E" should have been given no probative value for having been executed
by someone who did not actually confiscate the marijuana.

5. The Court erred in giving probative value to the Buy-Bust Operation when even the
alleged marked money utilized in the operation could not be identified by the leader,
T/Sgt. Jaime Raposas.

6. The Court erred in not giving value to the testimony of the two disinterested witnesses
for the defense, namely, Lolita Mendoza and Maribeth Manapat, whose testimony
corroborated substantially that of the accused.

7. The Court erred in concluding that there was no motive for the military to
manufacture evidence.1âwphi1 It is common knowledge that apprehensions of this kind
are made to fill up a quota of arrest in cases handled to comply with standard operating
procedure and efficiency reports. 8

We affirm the judgment of conviction.

Appellant assails, unconstitutional, the manner in which the so-called buy-bust


operation is conducted in order to enforce the Dangerous Drugs Act. He stigmatizes it
as no different from seizure of evidence from one's person or abode without a search
warrant. He argues that this procedure is pregnant with opportunities, and gives rise to
situations, for corrupting our law enforcers.

We are not unmindful of the fact that the common modus operandi of narcotic agents
in utilizing poseur-buyers does not always commend itself as the most reliable way to
go after violators of the Dangerous Drugs Act as it is susceptible of mistakes as well as
Page 288 of 377

harassment, extortion and abuse. 9 By the very nature of this anti-narcotics operation,
the possibility of abuse is great. 10

We are not, however, inclined to shackle the hands of narcotics agents whose task, as
it is, is already formidable and attended with great risk, lest their dedicated efforts for
the apprehension and successful prosecution of prohibited drug violators be unduly
hampered. The proliferation of drug addiction and trafficking has already reached an
alarming level and has spawned a network of incorrigible, cunning and dangerous
operations. Our experience has proven entrapment to be an effective means of
apprehending drug peddlers as exemplified by this case.

The Solicitor General explains that a buy-bust operation is the method employed by
peace officers to trap and catch a malefactor in flagrante delicto. It is essentially a form
of entrapment since the peace officer neither instigates nor induces the accused to
commit a crime. 11 Entrapment is the employment of such ways and means for the
purpose of trapping or capturing a lawbreaker from whose mind the criminal intent
originated. Oftentimes, it is the only effective way of apprehending a criminal in the act
of the commission of the offense.12

While it is conceded that in a buy-bust operation, there is seizure of evidence from one's
person without a search warrant, needless to state a search warrant is not necessary,
the search being incident to a lawful arrest. 13 A peace officer may, without a warrant,
arrest a person when, in his presence, the person to be arrested has committed, is
actually committing or is attempting to commit an offense. 14 It is a matter of judicial
experience that in the arrest of violators of the Dangerous Drugs Act in a buy-bust
operation, the malefactors were invariably caught red-handed. 15 There being no
violation of the constitutional right against unreasonable search and seizure, the
confiscated articles are admissible in evidence.

Appellant castigates the prosecution for not having presented any civilian or other
neutral person who could attest that the foils of marijuana were indeed confiscated from
him. The absence of any civilian witness should not undermine the case for the
prosecution. The natural reaction of a civilian to inhibit himself from being a witness to
a crime is understandable. A criminal proceeding entails a lot of unavoidable
inconveniences, aside from the time involved in attendance as a witness in
investigations and hearings. Adding to this the inherent fear of reprisal, we have the
natural reticence and abhorrence of most people to get involved in a criminal case.

At any rate, the testimony of other witnesses in this case would only be cumulative or
corroborative as they would only be repeating the facts already amply testified to by the
Page 289 of 377

government witnesses. Credence should be accorded to the prosecution's evidence more


so as it consisted mainly of testimonies of policemen. Law enforcers are presumed to
have regularly performed their duty in the absence of proof to the contrary. 16

Appellant maintains that the court below should have rejected Exhibit E, which
evidences the receipt of marijuana from appellant and which was prepared by Sgt.
Vicente Jimenez, in the absence of the original receipt prepared at the scene of the crime
by P/Pfc. Arcoy who was the poseur-buyer.

We agree with the Solicitor General, since this is borne out by the records, that Exhibit
E is actually based on, as it is merely a clearer copy of, the receipt prepared at the scene
of the crime by P/Pfc. Arcoy. Since the draft receipt had to be prepared hurriedly at the
scene in order that the accused could be brought to the Narcotics Command, such draft
receipt was not clearly written, so Sgt. Vicente Jimenez mechanically transferred the
written entries of P/Pfc. Arcoy into a more legible copy. 17 Nonetheless, there is no
dispute that Sgt. Jimenez, a member of the team, had personal knowledge of the facts
set forth in both receipts, being an eyewitness to the events that had transpired.

The testimony of T/Sgt. Jaime Raposas, the team leader who gave P/Pfc. Arcoy the
money to pay for the marijuana, is challenged in that he failed to identify the marked
money utilized in the operation. Appellant insists that the marked money must be
recorded, if not photographed in order to be admissible as evidence. This is clutching at
evidentiary and argumental straws.

As found by the trial court, the money was in the possession of P/Pfc. Arcoy who had
been assigned as the poseur-buyer.1âwphi1 In the ensuing transaction, the foil of
marijuana was handed to Arcoy by appellant and then Arcoy gave the money to accused
Juan dela Cruz. 18

Suffice it to say that even if the money given to De la Cruz was not presented in court,
the same would not militate against the People's case. 19 In fact, there was even no need
to prove that the marked money was handed to the appellants in payment of the goods.
The crime could have been consummated by the mere delivery of the prohibited drugs.
What the law proscribes is not only the act of selling but also, albeit not limited to, the
act of delivering. In the latter case, the act of knowingly passing a dangerous drug to
another personally or otherwise, and by any means, with or without consideration,
consummates the offense. 20
Page 290 of 377

On the trial court's rejection of the testimony of the alleged two disinterested witnesses
for the defense, namely, Lolita Mendoza and Maribeth Manapat, we find no reason to
disturb its ruling. We reiterate the time-honored principle that on the issue of which
version to accept, the findings of the trial court on the credibility of witnesses are given
great weight and the highest degree of respect by the appellate court. Subject to
exceptions which do not obtain in the present case, the trial court is in a better position
to decide this question, having seen and heard the witnesses themselves and observed
their deportment and manner of testifying during the trial. 21

Appellant imputes insidious motives on the part of the military to manufacture evidence,
theorizing that a buy-bust operation is for the purpose either of extorting money or, in
line with alleged internal policies, complying with a quota of arrests. 22 These are bare
unsupported allegations. From the evidence of record, we find no reason why the
prosecution witness should fabricate their testimonies and implicate appellant in such
a serious crime. The defense has not established any cogent motive for the police officers
to falsely charge the accused with peddling marijuana. As found by the trial court, there
is not even a breath, much less an accusation by the defense, that the military and
police personnel involved were indeed engaged in such nefarious activities. 23

Finally, appellant reproaches the prosecution for not presenting the civilian informer as
a witness. 24 It is settled that the non-presentation of a certain witness by the
prosecution is not a sufficiently plausible defense. If the accused believes that the
testimony of said witness is important to his cause, he should avail thereof, even by
compulsory judicial process if necessary. Furthermore, the non-presentation of some
prosecution witnesses does not detract from the prosecution's case, since the number
of such witnesses who should be called to testify is addressed to the sound discretion
of the prosecuting officers. 25

WHEREFORE, the judgment of the Regional Trial Court of Manila in Criminal Case No.
87-54417, insofar as accused-appellant Reynaldo Beltran y Aniban is concerned, is
hereby AFFIRMED.

SO ORDERED.
Melencio-Herrera, Paras, Padilla and Sarmiento, JJ., concur.
Page 291 of 377

G.R. No. 125299 January 22, 1999

PEOPLE OF THE PHILIPPINES, plaintiff-appellee, vs. FLORENCIO DORIA y


BOLADO and VIOLETA GADDAO y CATAMA @ "NENETH," accused-
appellants.

PUNO, J.:

On December 7, 1995, accused-appellants Florencio Doria y Bolado and Violeta Gaddao


y Catama @ "Neneth" were charged with violation of Section 4, in relation to Section 21
of the Dangerous Drugs Act of 1972. 1 The information reads:

That on or about the 5th day of December, 1995 in the City of Mandaluyong, Philippines,
a place within the jurisdiction of this Honorable Court, the above-named accused,
conspiring, confederating and mutually helping and aiding one another and without
having been authorized by law, did, then and there willfully, unlawfully and feloniously
sell, administer, deliver and give away to another eleven (11) plastic bags of suspected
marijuana fruiting tops weighing 7,641.08 grams in violation of the above-cited law.

CONTRARY TO LAW.2

The prosecution contends the offense was committed as follows: In November 1995,
members of the North Metropolitan District, Philippine National Police (PNP) Narcotics
Command (Narcom), received information from two (2) civilian informants (CI) that one
"Jun" was engaged in illegal drug activities in Mandaluyong City. The Narcom agents
decided to entrap and arrest "Jun" in a buy-bust operation. As arranged by one of the
CI's, a meeting between the Narcom agents and "Jun" was scheduled on December 5,
1995 at E. Jacinto Street in Mandaluyong City.

On December 5, 1995, at 6:00 in the morning, the CI went to the PNP Headquarters at
EDSA, Kamuning, Quezon City to prepare for the buy-bust operation. The Narcom
agents formed Team Alpha composed of P/Insp. Nolasco Cortes as team leader and PO3
Celso Manlangit, SPO1 Edmund Badua and four (4) other policemen as members.
P/Insp. Cortes designated P03 Manlangit as the poseur-buyer and SPO1 Badua as his
back-up, and the rest of the team as perimeter security. Superintendent Pedro
Alcantara, Chief of the North Metropolitan District PNP Narcom, gave the team P2, 000.
00 to cover operational expenses. From this sum, PO3 Manlangit set aside P1,600.00 —
a one thousand peso bill and six (6) one hundred peso bills 3 — as money for the buy-
bust operation. The market price of one kilo of marijuana was then P1,600.00. P03
Page 292 of 377

Manlangit marked the bills with his initials and listed their serial numbers in the police
blotter.4 The team rode in two cars and headed for the target area.

At 7:20 of the same morning, "Jun" appeared and the CI introduced PO3 Manlangit as
interested in buying one (1) kilo of marijuana. P03 Manlangit handed "Jun" the marked
bills worth P1,600.00. "Jun" instructed P03 Manlangit to wait for him at the corner of
Shaw Boulevard and Jacinto Street while he got the marijuana from his associate.5 An
hour later, "Jun" appeared at the agreed place where P03 Manlangit, the CI and the rest
of the team were waiting. "Jun" took out from his bag an object wrapped in plastic and
gave it to P03 Manlangit. P03 Manlangit forthwith arrested "Jun" as SPO1 Badua rushed
to help in the arrest. They frisked "Jun" but did not find the marked bills on him. Upon
inquiry, "Jun" revealed that he left the money at the house of his associate named
"Neneth.6 "Jun" led the police team to "Neneth's" house nearby at Daang Bakal.

The team found the door of "Neneth's" house open and a woman inside. "Jun" identified
the woman as his associate.7 SPO1 Badua asked "Neneth" about the P1,600.00 as PO3
Manlangit looked over "Neneth's" house. Standing by the door, PO3 Manlangit noticed
a carton box under the dining table. He saw that one of the box's flaps was open and
inside the box was something wrapped in plastic. The plastic wrapper and its contents
appeared similar to the marijuana earlier "sold" to him by "Jun." His suspicion aroused,
PO3 Manlangit entered "Neneth's" house and took hold of the box. He peeked inside the
box and found that it contained ten (10) bricks of what appeared to be dried marijuana
leaves.

Simultaneous with the box's discovery, SPO1 Badua recovered the marked bills from
"Neneth."8 The policemen arrested "Neneth." They took "Neneth" and "Jun," together
with the box, its contents and the marked bills and turned them over to the investigator
at headquarters. It was only then that the police learned that "Jun" is Florencio Doria y
Bolado while "Neneth" is Violeta Gaddao y Catama. The one (1) brick of dried marijuana
leaves recovered from "Jun" plus the ten (10) bricks recovered from "Neneth's" house
were examined at the PNP Crime Laboratory.9 The bricks, eleven (11) in all, were found
to be dried marijuana fruiting tops of various weights totalling 7,641.08 grams. 10

The prosecution story was denied by accused-appellants Florencio Doria and Violeta
Gaddao. Florencio Doria, a 33-year old carpenter, testified that on December 5, 1995,
at 7:00 in the morning, he was at the gate of his house reading a tabloid newspaper.
Two men appeared and asked him if he knew a certain "Totoy." There were many
"Totoys" in their area and as the men questioning him were strangers, accused-appellant
denied knowing any "Totoy." The men took accused-appellant inside his house and
accused him of being a pusher in their community. When accused-appellant denied the
charge, the men led him to their car outside and ordered him to point out the house of
Page 293 of 377

"Totoy." For five (5) minutes, accused-appellant stayed in the car. Thereafter, he gave in
and took them to "Totoy's" house.

Doria knocked on the door of "Totoy's" house but no one answered. One of the men,
later identified as P03 Manlangit, pushed open the door and he and his companions
entered and looked around the house for about three minutes. Accused-appellant Doria
was left standing at the door. The policemen came out of the house and they saw Violeta
Gaddao carrying water from the well. He asked Violeta where "Totoy" was but she replied
he was not there. Curious onlookers and kibitzers were, by that time, surrounding them.
When Violeta entered her house, three men were already inside. Accused-appellant
Doria, then still at the door, overheard one of the men say that they found a carton box.
Turning towards them, Doria saw box on top of the table. The box was open and had
something inside. P03 Manlangit ordered him and Violeta to go outside the house and
board the car. They were brought to police headquarters where they were investigated.

Accused-appellant Doria further declared that his co-accused, Violeta Gaddao, is the
wife of his acquaintance, Totoy Gaddao. He said that he and Totoy Gaddao sometimes
drank together at the neighborhood store. This closeness, however, did not extend to
Violeta, Totoy's wife.11

Accused-appellant Violeta Gaddao, a 35-year old rice vendor, claimed that on December
5, 1995, she was at her house at Daang Bakal, Mandaluyong City where she lived with
her husband and five (5) children, namely, Arvy, aged 10, Arjay, aged 8, the twins
Raymond and Raynan, aged 5, and Jason, aged 3. That day, accused-appellant woke
up at 5:30 in the morning and bought pan de sal for her children's breakfast. Her
husband, Totoy, a housepainter, had left for Pangasinan five days earlier. She woke her
children and bathed them. Her eldest son, Arvy, left for school at 6:45 A.M. Ten minutes
later, she carried her youngest son, Jayson, and accompanied Arjay to school. She left
the twins at home leaving the door open. After seeing Arjay off, she and Jayson remained
standing in front of the school soaking in the sun for about thirty minutes. Then they
headed for home. Along the way, they passed the artesian well to fetch water. She was
pumping water when a man clad in short pants and denim jacket suddenly appeared
and grabbed her left wrist. The man pulled her and took her to her house. She found
out later that the man was P03 Manlangit.

Inside her house were her co-accused Doria and three (3) other persons. They asked her
about a box on top of the table. This was the first time she saw the box. The box was
closed and tied with a piece of green straw. The men opened the box and showed her its
contents. She said she did not know anything about the box and its contents.
Page 294 of 377

Accused-appellant Violeta Gaddao confirmed that her co-accused Florencio Doria was
a friend of her husband, and that her husband never returned to their house after he
left for Pangasinan. She denied the charge against her and Doria and the allegation that
marked bills were found in her person.12

After trial, the Regional Trial Court, Branch 156, Pasig City convicted the accused-
appellants. The trial court found the existence of an "organized/syndicated crime group"
and sentenced both accused-appellants to death and pay a fine of P500,000.00 each.
The dispositive portion of the decision reads as follows:

WHEREFORE, the guilt of accused, FLORENCIO DORIA y BOLADO @ "Jun" and


VIOLETA GADDAO y CATAMA @ "Neneth" having been established beyond reasonable
doubt, they are both
CONVICTED of the present charge against them.

According to the amendatory provisions of Sec. 13 of Republic Act No. 7659 which cover
violations of Sec. 4 of Republic Act No. 6425 and which was exhaustively discussed in
People v. Simon, 234 SCRA 555, the penalty imposable in this case is reclusion perpetua
to death and a fine ranging from five hundred thousand pesos to ten million pesos.
Taking into consideration, however, the provisions of Sec. 23, also of Republic Act No.
7659 which explicitly state that:

The maximum penalty shall be imposed if the offense was committed by any person who
belongs to an organized/syndicated crime group.

An organized/syndicated crime group means a group of two or more persons


collaborating, confederating or mutually helping one another for purposes of gain in the
commission of any crime.

the Court is hereby constrained to sentence (hereby sentences) said FLORENCIO DORIA
y BOLADO @ "Jun" and VIOLETA GADDAO y CATAMA @ "Neneth" to DEATH and to pay
a fine of Five Hundred Thousand Pesos (P500,000.00) each without subsidiary
imprisonment in case of insolvency and to pay the costs.

The confiscated marijuana bricks (7,641.08 grams) shall be turned over to the
Dangerous Drugs Board, NBI for destruction in accordance with law.
Page 295 of 377

Let a Commitment Order be issued for the transfer of accused DORIA from the
Mandaluyong City Jail to the New Bilibid Prisons, Muntinlupa City and also for accused
GADDAO for her transfer to the Correctional Institute for Women, Mandaluyong City.

Let the entire records of this case be forwarded immediately to the Supreme Court for
mandatory review.

SO ORDERED. 13

Before this Court, accused-appellant Doria assigns two errors, thus:

THE COURT A QUO GRAVELY ERRED IN GIVING WEIGHT TO THE TESTIMONY OF


THE WITNESSES FOR THE PROSECUTION WHEN THEIR TESTIMONIES WERE SHOT
WITH DISCREPANCIES, INCONSISTENCIES AND THAT THE CORPUS DELICTI OF THE
MARIJUANA ALLEGEDLY TAKEN FROM APPELLANT WAS NOT POSITIVELY
IDENTIFIED BY THE POSEUR-BUYER.

II

THE COURT A QUO GRAVELY ERRED IN ADMITTING AS EVIDENCE THE MARIJUANA


FRUITINGS FOUND INSIDE THE CARTON BOX AS THESE WERE OBTAINED
THROUGH A WARRANTLESS SEARCH AND DOES NOT COME WITHIN THE PLAIN
VIEW DOCTRINE. 14

Accused-appellant Violeta Gaddao contends:

THE LOWER COURT ERRED IN FINDING APPELLANT GUILTY DESPITE THE


INCREDIBILITY OF THE POLICE VERSION OF THE MANNER THE ALLEGED BUY-
BUST WAS CONDUCTED.
Page 296 of 377

II

THE PNP OFFICERS' VERSIONS AS TO WHERE THE BUY-BUST MONEY CAME FROM
ARE INCONSISTENT WITH ONE ANOTHER AND ALSO REEKS WITH INCREDIBILITY.

III

THE LOWER COURT ERRED IN FINDING APPELLANT GUILTY AND SENTENCING HER
TO DEATH DESPITE THE MANIFESTLY IRRECONCILABLE INCONSISTENCIES IN THE
VERSIONS OF THE POLICE AS TO HOW AND BY WHOM THE ALLEGED BUY-BUST
MONEY WAS RECOVERED FROM HER, WHICH IN CONSEQUENCE RESULTS IN THE
EVIDENCE, OF RETRIEVAL FROM HER OF THE SAME, NEBULOUS, AT BEST, NIL, AT
WORST.

IV

THE LOWER COURT ERRED IN UPHOLDING THE VALIDITY OF THE WARRANTLESS


SEARCH LEADING TO THE SEIZURE OF THE MARIJUANA ALLEGEDLY FOUND
INSIDE THE HOUSE OF ACCUSED-APPELLANT. 15

The assigned errors involve two principal issues: (1) the validity of the buy-bust
operation in the apprehension of accused-appellant Doria; and (2) the validity of the
warrantless arrest of accused-appellant Gaddao, the search of her person and house,
and the admissibility of the pieces of evidence obtained therefrom.

Accused-appellants were caught by the police in a buy-bust operation. A buy-bust


operation is a form of entrapment employed by peace officers as an effective way of
apprehending a criminal in the act of the commission of an offense.16 Entrapment has
received judicial sanction when undertaken with due regard to constitutional and legal
safeguards.17

Entrapment was unknown in common law. It is a judicially created twentieth-century


American doctrine that evolved from the increasing use of informers and undercover
agents in the detection of crimes, particularly liquor and narcotics offenses.18
Entrapment sprouted from the doctrine of estoppel and the public interest in the
formulation and application of decent standards in the enforcement of criminal law.19
It also took off from a spontaneous moral revulsion against using the powers of
Page 297 of 377

government to beguile innocent but ductile persons into lapses that they might
otherwise resist.20

In the American jurisdiction, the term "entrapment" has a generally negative meaning
because it is understood as the inducement of one to commit a crime not contemplated
by him, for the mere purpose of instituting a criminal prosecution against him.21 The
classic definition of entrapment is that articulated by Justice Roberts in Sorrells v.
United States,22 the first Supreme Court decision to acknowledge the concept:
"Entrapment is the conception and planning of an offense by an officer, and his
procurement of its commission by one who would not have perpetrated it except for the
trickery, percuasion or fraud of the officers."23 It consists of two (2) elements: (a) acts
of percuasion, trickery, or fraud carried out by law enforcement officers or the agents to
induce a defendant to commit a crime; and (b) the origin of the criminal design in the
minds of the government officials rather than that of the innocent defendant, such that
the crime is the product of the creative activity of the law enforcement officer.24

It is recognized that in every arrest, there is a certain amount of entrapment used to


outwit the persons violating or about to violate the law. Not every deception is forbidden.
The type of entrapment the law forbids is the inducing of another to violate the law, the
"seduction" of an otherwise innocent person into a criminal career.25 Where the
criminal intent originates criminal in the mind of the entrapping person and the accused
is lured into the commission of the offense charged in order to prosecute him, there is
entrapment and no conviction may be had.26 Where, however, the criminal intent
originates in the mind of the accused and the criminal offense is completed, the fact
that a person acting as a decoy for the state, or public officials furnished the accused
an opportunity for commission of the offense, or that the accused is aided in the
commission of the crime in order to secure the evidence necessary to prosecute him,
there is no entrapment and the accused must be convicted.27 The law tolerates the use
of decoys and other artifices to catch a criminal.

Entrapment is recognized as a valid defense28 that can be raised by an accused and


partakes of the nature of a confession and avoidance.29 It is a positive defense. Initially,
an accused has the burden of providing sufficient evidence that the government induced
him to commit the offense. Once established, the burden shifts to the governmet to show
otherwise.30 When entrapment is raised as a defense, American federal courts and a
majority of state courts use the "subjective" or "origin of intent" test laid down in Sorrells
v. United States 31 to determine whether entrapment actually occurred. The focus of
the inquiry is on the accused's predisposition to commit the offense charged, his state
of mind and inclination before his initial exposure to government agents. 32 All relevant
facts such as the accused's mental and character traits, his past offenses, activities, his
eagerness in committing the crime, his reputation, etc., are considered to assess his
state of mind before the crime.33 The predisposition test emphasizes the accused's
propensity to commit the offense rather than the officer's misconduct 34 and reflects an
Page 298 of 377

attempt to draw a line between a "trap for the unwary innocent and the trap for the
unwary criminal."35 If the accused was found to have been ready and willing to commit
the offense at any favorable opportunity, the entrapment defense will fail even if a police
agent used an unduly persuasive inducement.36 Some states, however, have adopted
the "objective" test. 37 This test was first authoritatively laid down in the case of
Grossman v. State 38 rendered by the Supreme Court of Alaska. Several other states
have subsequently adopted the test by judicial pronouncement or legislation. Here, the
court considers the nature of the police activity involved and the propriety of police
conduct.39 The inquiry is focused on the inducements used by government agents, on
police conduct, not on the accused and his predisposition to commit the crime. For the
goal of the defense is to deter unlawful police conduct.40 The test of entrapment is
whether the conduct of the law enforcement agent was likely to induce a normally law-
abiding person, other than one who is ready and willing, to commit the offense;41 for
purposes of this test, it is presumed that a law-abiding person would normally resist
the temptation to commit a crime that is presented by the simple opportunity to act
unlawfully. 42 Official conduct that merely offers such an opportunity is permissible,
but overbearing conduct, such as badgering, cajoling or importuning,43 or appeals to
sentiments such as pity, sympathy, friendship or pleas of desperate illness, are not.44
Proponents of this test believe that courts must refuse to convict an entrapped accused
not because his conduct falls outside the legal norm but rather because, even if his guilt
has been established, the methods employed on behalf of the government to bring about
the crime "cannot be countenanced." To some extent, this reflects the notion that the
courts should not become tainted by condoning law enforcement improprieties.45
Hence, the transactions leading up to the offense, the interaction between the accused
and law enforcement officer and the accused's response to the officer's inducements,
the gravity of the crime, and the difficulty of detecting instances of its commission are
considered in judging what the effect of the officer's conduct would on a normal
person.46

Both the "subjective" and "objective" approaches have been criticized and objected to. It
is claimed that the "subjective" test creates an "anything goes" rule, i.e, if the court
determines that an accused was predisposed to commit the crime charged, no level of
police deceit, badgering or other unsavory practices will be deemed impermissible.47
Delving into the accused's character and predisposition obscures the more important
task of judging police behavior and prejudices the accused more generally. It ignores
the possibility that no matter what his past crimes and general disposition were, the
accused might not have committed the particular crime unless confronted with
inordinate inducements.48 On the other extreme, the purely "objective" test eliminates
entirely the need for considering a particular accused's predisposition. His
predisposition, at least if known by the police, may have an important bearing upon the
question of whether the conduct of the police and and their agents was proper. 49 The
undisputed fact that the accused was a dangerous and chronic offender or that he was
a shrewd and active member of a criminal syndicate at the time of his arrest is relegated
to irrelevancy.50
Page 299 of 377

Objections to the two tests gave birth to hybrid approaches to entrapment. Some states
in the United States now combine both the "subjective" and "objective"51 In Cruz v.
State,52 the Florida Supreme Court declared that the permissibility of police conduct
must first be determined. If this objective test is satisfied, then the analysis turns to
whether the accused was predisposed to commit the crime.53 In Baca v. State,54 the
New Mexico Supreme Court modified the state's entrapment analysis by holding that "a
criminal defendant may successfully assert a defense of entrapment, either by showing
lack of predisposition to commit the crime for which he is charged, or, that the police
exceeded the standards of proper investigation.55 The hybrid approaches combine and
apply the "objective" and "subjective" tests alternatively or concurrently.

As early as 1910, this Court has examined the conduct of law enforcers while
apprehending the accused caught in flagrante delicto. In United States v. Phelps,56 we
acquitted the accused from the offense of smoking opium after finding that the
government employee, a BIR personnel, actually induced him to commit the crime in
order to prosecute him. Smith, the BIR agent, testified that Phelps' apprehension came
after he overheard Phelps in a saloon say that he liked smoking opium on some
occasions. Smith's testimony was disregarded. We accorded significance to the fact that
it was Smith who went to the accused three times to convince him to look for an opium
den where both of them could smoke this drug.57 The conduct of the BIR agent was
condemned as "most reprehensible."58 In People v. Abella,59 we acquitted the accused
of the crime of selling explosives after examining the testimony of the apprehending
police officer who pretended to be a merchant. The police officer offered "a tempting
price, . . . a very high one" causing the accused to sell the explosives. We found that
there was inducement, "direct, persistent and effective" by the police officer and that
outside of his testimony, there was no evidence sufficient to convict the accused.60 In
People v. Lua Chu and Uy Se Tieng, 61 we convicted the accused after finding that there
was no inducement on the part of the law enforcement officer. We stated that the
Customs secret serviceman smoothed the way for the introduction of opium from
Hongkong to Cebu after the accused had already planned its importation and ordered
said drug. We ruled that the apprehending officer did not induce the accused to import
opium but merely entrapped him by pretending to have an understanding with the
Collector of Customs of Cebu to better assure the seizure of the prohibited drug and the
arrest of the surreptitious importers.62

It was also in the same case of People v. Lua Chu and Uy Se Tieng 63 we first laid down
the distinction between entrapment vis-a-vis instigation or inducement. Quoting 16
Corpus Juris,64 we held:

ENTRAPMENT AND INSTIGATION. — While it has been said that the practice of
entrapping persons into crime for the purpose of instituting criminal prosecutions is to
Page 300 of 377

be deplored, and while instigation, as distinguished from mere entrapment, has often
been condemned and has sometimes been held to prevent the act from being criminal
or punishable, the general rule is that it is no defense to the perpetrator of a crime that
facilities for its commission were purposely placed in his way, or that the criminal act
was done at the 'decoy solicitation' of persons seeking to expose the criminal, or that
detectives feigning complicity in the act were present and apparently assisting in its
commission. Especially is this true in that class of cases where the offense is one of a
kind habitually committed, and the solicitation merely furnishes evidence of a course of
conduct. Mere deception by the detective will not shield defendant, if the offense was
committed by him, free from the influence or instigation of the detective. The fact that
an agent of an owner acts as a supposed confederate of a thief is no defense to the latter
in a prosecution for larceny, provided the original design was formed independently of
such agent; and where a person approached by the thief as his confederate notifies the
owner or the public authorities, and, being authorised by them to do so, assists the thief
in carrying out the plan, the larceny is nevertheless committed. It is generally held that
it is no defense to a prosecution for an illegal sale of liquor that the purchase was made
by a "spotter," detective, or hired informer; but there are cases holding the contrary. 65

The distinction above-quoted was reiterated in two (2) decisions of the Court of Appeals.
In People v. Galicia,66 the appellate court declared that "there is a wide difference
between entrapment and instigation." The instigator practically induces the would-be
accused into the commission of the offense and himself becomes a co-principal. In
entrapment, ways and means are resorted to by the peace officer for the purpose of
trapping and capturing the lawbreaker in the execution of his criminal plan.67 In People
v. Tan Tiong,68 the Court of Appeals further declared that "entrapment is no bar to the
prosecution and conviction of the lawbreaker.69

The pronouncement of the Court of Appeals in People v. Galicia was affirmed by this
Court in People v. Tiu Ua.70 Entrapment, we further held, is not contrary to public
policy. It is instigation that is deemed contrary to public policy and illegal.71

It can thus be seen that the concept of entrapment in the American jurisdiction is similar
to instigation or inducement in Philippine jurisprudence. Entrapment in the Philippines
is not a defense available to the accused. It is instigation that is a defense and is
considered an absolutory cause.72 To determine whether there is a entrapment or
instigation, our courts have mainly examined the conduct of the apprehending officers,
not the predisposition of the accused to commit the crime. The "objective" test first
applied in United States v. Phelps has been followed in a series of similar cases.73
Nevertheless, adopting the "objective" approach has not precluded us from likewise
applying the "subjective" test. In People v. Boholst,74 we applied both tests by examining
the conduct of the police officers in a buy-bust operation and admitting evidence of the
accused's membership with the notorious and dreaded Sigue-Sigue Sputnik Gang. We
also considered accused's previous his convictions of other crimes 75 and held that his
Page 301 of 377

opprobrious past and membership with the dreaded gang strengthened the state's
evidence against him. Conversely, the evidence that the accused did not sell or smoke
marijuana and did not have any criminal record was likewise admitted in People v. Yutuc
76 thereby sustaining his defense that led to his acquittal.

The distinction between entrapment and instigation has proven to be very material in
anti-narcotics operations. In recent years, it has become common practice for law
enforcement officers and agents to engage in buy-bust operations and other entrapment
procedures in apprehending drug offenders. Anti-narcotics laws, like anti-gambling laws
are regulatory statutes.77 They are rules of convenience designed to secure a more
orderly regulation of the affairs of society, and their violation gives rise to crimes mala
prohibita.78 They are not the traditional type of criminal law such as the law of murder,
rape, theft, arson, etc. that deal with crimes mala in se or those inherently wrongful and
immoral. 79 Laws defining crimes mala prohibita condemn behavior directed, not
against particular individuals, but against public order.80 Violation is deemed a wrong
against society as a whole and is generally unattended with any particular harm to a
definite person.81 These offenses are carried on in secret and the violators resort to
many devices and subterfuges to avoid detection. It is rare for any member of the public,
no matter how furiously he condemns acts mala prohibita, to be willing to assist in the
enforcement of the law. It is necessary, therefore, that government in detecting and
punishing violations of these laws, rely, not upon the voluntary action of aggrieved
individuals, but upon the diligence of its own officials. This means that the police must
be present at the time the offenses are committed either in an undercover capacity or
through informants, spies or stool pigeons.82

Though considered essential by the police in enforcing vice legislation, the confidential
informant system breeds abominable abuse. Frequently, a person who accepts payment
from the police in the apprehension of drug peddlers and gamblers also accept payment
from these persons who deceive the police. The informant himself maybe a drug addict,
pickpocket, pimp, or other petty criminal. For whatever noble purpose it serves, the
spectacle that government is secretly mated with the underworld and uses underworld
characters to help maintain law and order is not an inspiring one.83 Equally odious is
the bitter reality of dealing with unscrupulous, corrupt and exploitative law enforcers.
Like the informant, unscrupulous law enforcers' motivations are legion — harassment,
extortion, vengeance, blackmail, or a desire to report an accomplishment to their
superiors. This Court has taken judicial notice of this ugly reality in a number of cases84
where we observed that it is a common modus operandi of corrupt law enforcers to prey
on weak and hapless persons, particularly unsuspecting provincial hicks.85 The use of
shady underworld characters as informants, the relative ease with which illegal drugs
may be planted in the hands or property of trusting and ignorant persons, and the
imposed secrecy that inevitably shrouds all drug deals have compelled this Court to be
extra-vigilant in deciding drug cases.86 Criminal activity is such that stealth and
strategy, although necessary weapons in the arsenal of the police officer, become as
Page 302 of 377

objectionable police methods as the coerced confession and the unlawful search. As well
put by the Supreme Court of California in People v. Barraza,87

[E]ntrapment is a facet of a broader problem. Along with illegal search and seizures,
wiretapping, false arrest, illegal detention and the third degree, it is a type of lawless
enforcement. They all spring from common motivations. Each is a substitute for skillful
and scientific investigation. Each is condoned by the sinister sophism that the end,
when dealing with known criminals of the 'criminal class,' justifies the employment of
illegal means. 88

It is thus imperative that the presumption, juris tantum, of regularity in the


performance of official duty by law enforcement agents raised by the Solicitor General
be applied with studied restraint. This presumption should not by itself prevail over the
presumption of innocence and the constitutionally-protected rights of the individual.89
It is the duty of courts to preserve the purity of their own temple from the prostitution
of the criminal law through lawless enforcement.90 Courts should not allow themselves
to be used as an instrument of abuse and injustice lest an innocent person be made to
suffer the unusually severe penalties for drug offenses.91

We therefore stress that the "objective" test in buy-bust operations demands that the
details of the purported transaction must be clearly and adequately shown. This must
start from the initial contact between the poseur-buyer and the pusher, the offer to
purchase, the promise or payment of the consideration until the consummation of the
sale by the delivery of the illegal drug subject of the sale.92 The manner by which the
initial contact was made, whether or not through an informant, the offer to purchase
the drug, the payment of the "buy-bust" money, and the delivery of the illegal drug,
whether to the informant alone or the police officer, must be the subject of strict scrutiny
by courts to insure that law-abiding citizens are not unlawfully induced to commit an
offense. Criminals must be caught but not at all cost. At the same time, however,
examining the conduct of the police should not disable courts into ignoring the
accused's predisposition to commit the crime. If there is overwhelming evidence of
habitual delinquency, recidivism or plain criminal proclivity, then this must also be
considered. Courts should look at all factors to determine the predisposition of an
accused to commit an offense in so far as they are relevant to determine the validity of
the defense of inducement.1âwphi1.nêt

In the case at bar, the evidence shows that it was the confidential informant who initially
contacted accused-appellant Doria. At the pre-arranged meeting, the informant was
accompanied by PO3 Manlangit who posed as the buyer of marijuana. P03 Manlangit
handed the marked money to accused-appellant Doria as advance payment for one (1)
Page 303 of 377

kilo of marijuana. Accused-appellant Doria was apprehended when he later returned


and handed the brick of marijuana to P03 Manlangit.

PO3 Manlangit testified in a frank, spontaneous, straightforward and categorical


manner and his credibility was not crumpled on cross-examination by defense counsel.
Moreover, P03 Manlangit's testimony was corroborated on its material points by SPO1
Badua, his back-up security. The non-presentation of the confidential informant is not
fatal to the prosecution. Informants are usually not presented in court because of the
need to hide their identity and preserve their invaluable service to the police.93 It is
well-settled that except when the appellant vehemently denies selling prohibited drugs
and there are material inconsistencies in the testimonies of the arresting officers,94 or
there are reasons to believe that the arresting officers had motives to testify falsely
against the appellant,95 or that only the informant was the poseur-buyer who actually
witnessed the entire transaction,96 the testimony of the informant may be dispensed
with as it will merely be corroborative of the apprehending officers' eyewitness
testimonies.97 There is no need to present the informant in court where the sale was
actually witnessed and adequately proved by prosecution witnesses.98

The inconsistencies in P03 Manlangit's and SPO1 Badua's testimonies and the other
police officers' testimonies are minor and do not detract from the veracity and weight of
the prosecution evidence. The source of the money for the buy-bust operation is not a
critical fact in the case at bar. It is enough that the prosecution proved that money was
paid to accused-appellant Doria in consideration of which he sold and delivered the
marijuana.

Contrary to accused-appellant Doria's claim, the one kilo of marijuana "sold" by him to
PO3 Manlangit was actually identified by PO3 Manlangit himself before the trial court.
After appellants' apprehension, the Narcom agents placed this one (1) brick of marijuana
recovered from appellant Doria inside the carton box lumping it together with the ten
(10) bricks inside. This is why the carton box contained eleven (11) bricks of marijuana
when brought before the trial court. The one (1) brick recovered from appellant Doria
and each of the ten (10) bricks, however, were identified and marked in court. Thus:

ATTY. ARIAS, Counsel for Florencio Doria:

Mr. Police Officer, when you identified that box,. Tell the court, how were you able to
identify that box?
Page 304 of 377

A This is the box that I brought to the crime laboratory which contained the eleven
pieces of marijuana brick we confiscated from the suspect, sir.

Q Please open it and show those eleven bricks.

PROSECUTOR Witness bringing out from the said box. . .

ATTY. VALDEZ, Counsel for Violeta Gaddao:

Your Honor, I must protest the line of questioning considering the fact that we are now
dealing with eleven items when the question posed to the witness was what was handed
to him by Jun?

COURT So be it.

ATTY. ARIAS May we make it of record that the witness is pulling out them after item
from the box showed to him and brought in front of him.

COURT Noted.

Q Now tell the court, how did you know that those are the eleven bricks?

xxx xxx xxx

A I have markings on these eleven bricks, sir.

Q Point to the court, where are those markings?

A Here, sir, my signature, my initials with the date, sir.

PROSECUTOR Witness showed a white wrapper and pointing to CLM and the signature.
Page 305 of 377

Q Whose signature is that?

ATTY. VALDEZ Your Honor, may we just limit the inquiry to the basic question of the
fiscal as to what was handed to him by the accused Jun, your Honor?

PROSECUTOR Your Honor, there is already a ruling by this Honorable Court, your
Honor, despite reconsideration.

COURT Let the prosecution do its own thing and leave the appreciation of what it has
done to the court.

ATTY. VALDEZ We submit, your Honor.

A This brick is the one that was handed to me by the suspect Jun, sir.

COURT Why do you know that that is the thing? Are you sure that is not "tikoy?"

A Yes, your Honor.

Q What makes you so sure?

A I am sure that this is the one, your Honor. This is the Exhibit "A" which I marked
before I brought it to the PCCL, your Honor.

Q What are you sure of?

A I am sure that this is the brick that was given to me by one alias Jun, sir.

Q What makes you so sure?


Page 306 of 377

A Because I marked it with my own initials before giving it to the investigator and before
we brought it to the PCCL, your Honor.

xxx xxx xxx

PROSECUTOR May we request that a tag be placed on this white plastic bag and this
be marked as Exhibit "D?"

COURT Mark it as Exhibit "D."

Q To stress, who made the entries of this date, Exhibit "A" then the other letters and
figures on this plastic?

A This one, the signature, I made the signature, the date and the time and this Exhibit
"A."

Q How about this one?

A I don't know who made this marking, sir.

PROSECUTOR May it be of record that this was just entered this morning.

Q I am asking you about this "itim" and not the "asul."

A This CLM, the date and the time and the Exhibit "A," I was the one who made these
markings, sir.

PROSECUTOR May we place on record that the one that was enclosed. . .

ATTY. ARIAS Your Honor, there are also entries included in that enclosure where it
appears D-394-95 also Exhibit "A," etc. etc., that was not pointed to by the witness. I
want to make it of record that there are other entries included in the enclosure.
Page 307 of 377

COURT Noted. The court saw it.

Q Now, and this alleged brick of marijuana with a piece of paper, with a newspaper
wrapping with a piece of paper inside which reads: "D-394-95, Exhibit A, 970 grams
SSL" be marked as our Exhibit "D-2?"

COURT Tag it. Mark it.

Q This particular exhibit that you identified, the wrapper and the contents was given to
you by whom?

A It was given to me by suspect Jun, sir.

Q Whereat?

A At the corner of Boulevard and Jacinto St., sir.

Q How about the other items that you were able to recover?

xxx xxx xxx

A These other marijuana bricks, because during our follow-up, because according to
Jun the money which I gave him was in the hands of Neneth and so we proceeded to
the house of Neneth, sir.

xxx xxx xxx 99

The first brick identified by P03 Manlangit was the brick of marijuana "given to [him] by
suspect Jun" at the corner of Boulevard and Jacinto Streets. This brick, including the
newspaper and white plastic wrapping were marked as Exhibits "D," "D-l," and "D-2"
and described as weighing nine hundred seventy (970) grams. 100
Page 308 of 377

We also reject appellant's submission that the fact that P03 Manlangit and his team
waited for almost one hour for appellant Doria to give them the one kilo of marijuana
after he "paid" P1,600.00 strains credulity. Appellant cannot capitalize on the
circumstance that the money and the marijuana in the case at bar did not change hands
under the usual "kaliwaan" system. There is no rule of law which requires that in "buy-
bust" operations there must be a simultaneous exchange of the marked money and the
prohibited drug between the poseur- buyer and the pusher. 101 Again, the decisive fact
is that the poseur-buyer received the marijuana from the accused-appellant. 102

We also hold that the warrantless arrest of accused-appellant Doria is not unlawful.
Warrantless arrests are allowed in three instances as provided by Section 5 of Rule 113
of the 1985 Rules on Criminal Procedure, to wit:

Sec. 5. Arrest without warrant; when lawful. — A peace officer or a private person may,
without a warrant, arrest a person:

(a) When, in his presence, the person to be arrested has committed, is actually
committing, or is attempting to commit an offense;

(b) When an offense has in fact just been committed, and he has personal knowledge of
facts indicating that the person to be arrested has committed it; and

(c) When the person to be arrested is a prisoner who escaped from a penal establishment
or place where he is serving final judgment or temporarily confined while his case is
pending, or has escaped while being transferred from one confinement to another.

xxx xxx xxx 103

Under Section 5 (a), as above-quoted, a person may be arrested without a warrant if he


"has committed, is actually committing, or is attempting to commit an offense."
Appellant Doria was caught in the act of committing an offense. When an accused is
apprehended in flagrante delicto as a result of a buy-bust operation, the police are not
only authorized but duty-bound to arrest him even without a warrant. 104
Page 309 of 377

The warrantless arrest of appellant Gaddao, the search of her person and residence,
and the seizure of the box of marijuana and marked bills are different matters.

Our Constitution proscribes search and seizure without a judicial warrant and any
evidence obtained without such warrant is inadmissible for any purpose in any
proceeding. 105 The rule is, however, not absolute. Search and seizure may be made
without a warrant and the evidence obtained therefrom may be admissible in the
following instances: 106 (1) search incident to a lawful arrest;107 (2) search of a moving
motor vehicle; 108 (3) search in violation of customs laws; 109 (4) seizure of evidence in
plain view; 110 (5) when the accused himself waives his right against unreasonable
searches and seizures. 111

The prosecution admits that appellant Gaddao was arrested without a warrant of arrest
and the search and seizure of the box of marijuana and the marked bills were likewise
made without a search warrant. It is claimed, however, that the warrants were not
necessary because the arrest was made in "hot pursuit" and the search was an incident
to her lawful arrest.

To be lawful, the warrantless arrest of appellant Gaddao must fall under any of the three
(3) instances enumerated in Section 5 of Rule 113 of the 1985 Rules on Criminal
Procedure as aforequoted. The direct testimony of PO3 Manlangit, the arresting officer,
however shows otherwise:

ATTY. VALDEZ, Counsel for appellant Gaddao:

We submit at this juncture, your Honor, that there will be no basis for that question.

Q This particular exhibit that you identified, the wrapper and the contents was given to
you by whom?

A It was given to me by suspect Jun, sir.

Q Whereat?

A At the corner of Boulevard and Jacinto Street, sir.


Page 310 of 377

Q How about, the other items that you were able to recover?

ATTY. VALDEZ: We submit at this juncture, your Honor, that there will be no basis for
that question.

COURT There is. Answer.

A These other marijuana bricks, because during our follow-up, because according to
Jun the money which I gave him was in the hands of Neneth and so we proceeded to
the house of Neneth, sir.

Q Whereat?

A At Daang Bakal near the crime scene at Shaw Boulevard, sir.

Q And what happened upon arrival thereat?

A We saw alias Neneth inside the house and we asked him to give us the buy-bust
money, sir.

Q You mentioned "him?"

A Her, sir. We asked her to give us the money, the marked money which Jun gave her,
sir.

Q And what happened?

A At this instance, it was SPO1 Badua who can testify regarding this buy-bust money,
sir.

xxx xxx xxx 112


Page 311 of 377

SPO1 Badua testified on cross-examination that:

Q What was your intention in going to the house of Aling Neneth?

A To arrest her, sir.

Q But the fact is, Mr. Witness, when you reached the house of Aling Neneth, Aling
Neneth was there?

A Yes, sir.

Q As far as you can see, she was just inside her house?

A I saw her outside, sir.

Q She was fetching water as a matter of fact?

A She was 'sa bandang poso.'

Q Carrying a baby?

A No, sir.

Q At that particular time when you reached the house of Aling Neneth and saw her
outside the house, she was not committing any crime, she was just outside the house?

A No, sir.

Q She was not about to commit any crime because she was just outside the house doing
her daily chores. Am I correct?
Page 312 of 377

A I just saw her outside, sir.

Q And at that point in time you already wanted to arrest her. That is correct, is it not?

A Yes, sir.

Q Now, if any memory of your testimony is correct, according to you SPO1 Manlangit
approached her?

A PO3 Manlangit, sir.

Q You did not approach her because P03 Manlangit approached her?

A Yes, sir.

Q During all the time that this confrontation, arrest or whatever by SPO3 Manlangit was
taking place, you were just in the side lines?

A I was just watching, sir.

Q So you were just an on-looker to what Manlangit was doing, because precisely
according to you your role in this buy-bust operation was as a back-up?

A Yes, sir.

Q Who got the alleged marijuana from inside the house of Mrs. Neneth?

A P03 Manlangit, sir.

Q Manlangit got the marijuana?


Page 313 of 377

A Yes, sir.

Q And the money from Aling Neneth?

A I don't know, sir.

Q You did not even know who got the money from Aling Neneth?

PROSECUTOR:

There is no basis for this question, your Honor. Money, there 's no testimony on that.

ATTY. VALDEZ:

I was asking him precisely.

PROSECUTOR:

No basis.

COURT:

Sustained.

Q Alright. I will ask you a question and I expect an honest answer. According to the
records, the amount of P1,600.00 was recovered from the person of Aling Neneth. That's
right?

A Yes, sir, the buy-bust money.


Page 314 of 377

Q What you are now saying for certain and for the record is the fact that you were not
the one who retrieved the money from Aling Neneth, it was Manlangit maybe?

A I saw it, sir.

Q It was Manlangit who got the money from Aling Neneth?

A The buy-bust money was recovered from the house of Aling Neneth, sir.

Q It was taken from the house of Aling Neneth, not from the person of Aling Neneth. Is
that what you are trying to tell the Court?

A No, sir.

ATTY. VALDEZ:

I am through with this witness, your Honor. 113

Accused-appellant Gaddao was not caught red-handed during the buy-bust operation
to give ground for her arrest under Section 5 (a) of Rule 113. She was not committing
any crime. Contrary to the finding of the trial court, there was no occasion at all for
appellant Gaddao to flee from the policemen to justify her arrest in "hot pursuit."114 In
fact, she was going about her daily chores when the policemen pounced on her.

Neither could the arrest of appellant Gaddao be justified under the second instance of
Rule 113. "Personal knowledge" of facts in arrests without warrant under Section 5 (b)
of Rule 113 must be based upon "probable cause" which means an "actual belief or
reasonable grounds of suspicion."115 The grounds of suspicion are reasonable when,
in the absence of actual belief of the arresting officers, the suspicion that the person to
be arrested is probably guilty of committing the offense, is based an actual facts, i.e.,
supported by circumstances sufficiently strong in themselves to create the probable
cause of guilt of the person to be arrested.116 A reasonable suspicion therefore must
be founded on probable cause, coupled with good faith on the part of the peace officers
making the arrest.117
Page 315 of 377

Accused-appellant Gaddao was arrested solely on the basis of the alleged identification
made by her co-accused. PO3 Manlangit, however, declared in his direct examination
that appellant Doria named his co-accused in response to his (PO3 Manlangit's) query
as to where the marked money was.118 Appellant Doria did not point to appellant
Gaddao as his associate in the drug business, but as the person with whom he left the
marked bills. This identification does not necessarily lead to the conclusion that
appellant Gaddao conspired with her co-accused in pushing drugs. Appellant Doria may
have left the money in her house,119 with or without her knowledge, with or without
any conspiracy. Save for accused-appellant Doria 's word, the Narcom agents had no
reasonable grounds to believe that she was engaged in drug pushing. If there is no
showing that the person who effected the warrantless arrest had, in his own right,
knowledge of facts implicating the person arrested to the perpetration of a criminal
offense, the arrest is legally objectionable.120

Since the warrantless arrest of accused-appellant Gaddao was illegal, it follows that the
search of her person and home and the subsequent seizure of the marked bills and
marijuana cannot be deemed legal as an incident to her arrest. This brings us to the
question of whether the trial court correctly found that the box of marijuana was in
plain view, making its warrantless seizure valid.

Objects falling in plain view of an officer who has a right to be in the position to have
that view are subject to seizure even without a search warrant and maybe introduced
in evidence.121 The "plain view" doctrine applies when the following requisites concur:
(a) the law enforcement officer in search of the evidence has a prior justification for an
intrusion or is in a position from which he can view a particular area; (b) the discovery
of the evidence in plain view is inadvertent; (c) it is immediately apparent to the officer
that the item he observes may be evidence of a crime, contraband or otherwise subject
to seizure.122 The law enforcement officer must lawfully make an initial intrusion or
properly be in a position from which he can particularly view the area.123 In the course
of such lawful intrusion, he came inadvertently across a piece of evidence incriminating
the accused.124 The object must be open to eye and
hand125 and its discovery inadvertent.126

It is clear that an object is in plain view if the object itself is plainly exposed to sight.
The difficulty arises when the object is inside a closed container. Where the object seized
was inside a closed package, the object itself is not in plain view and therefore cannot
be seized without a warrant. However, if the package proclaims its contents, whether by
its distinctive configuration, its transparency, or if its contents are obvious to an
observer, then the contents are in plain view and may be seized.127 In other words, if
the package is such that an experienced observer could infer from its appearance that
Page 316 of 377

it contains the prohibited article, then the article is deemed in plain view.128 It must
be immediately apparent to the police that the items that they observe may be evidence
of a crime, contraband or otherwise subject to seizure.129

PO3 Manlangit, the Narcom agent who found the box, testified on cross-examination as
follows:

ATTY. VALDEZ:

So here we are. When you and Badua arrived, Aling Neneth was inside the house?

A Yes, sir.

Q Badua demanded from Aling Neneth the buy-bust money?

A Yes, sir.

Q At that particular instance, you saw the carton?

A Yes, sir.

Q This carton, according to you was under a table?

A Yes, sir, dining table.

Q I noticed that this carton has a cover?

A Yes, sir.

Q I ask you were the flaps of the cover raised or closed?


Page 317 of 377

A It was open, sir. Not like that.

COURT

Go down there. Show to the court.

INTERPRETER

Witness went down the witness stand and approached a carton box.

A Like this, sir.

PROSECUTOR

Can we describe it?

ATTY. VALDEZ

Yes.

PROSECUTOR

One flap is inside and the other flap is standing and with the contents visible.

COURT

Noted.

Q At this juncture, you went inside the house?


Page 318 of 377

A Yes, sir.

Q And got hold of this carton?

A Yes, sir.

Q Did you mention anything to Aling Neneth?

A I asked her, what's this. . .

Q No, no. no. did you mention anything to Aling Neneth before getting the carton?

A I think it was Badua who accosted Aling Neneth regarding the buy-bust money and
he asked "Sa iyo galing ang marijuanang ito, nasaan ang buy-bust money namin?" sir.

Q Making reference to the marijuana that was given by alias Jun?

A Yes, sir.

Q When you proceeded to take hold of this carton, Aling Neneth was not yet frisked, is
it not [sic]?

A I just don't know if she was frisked already by Badua, sir.

Q Who got hold of this?

A I was the one, sir.

Q You were the one who got this?


Page 319 of 377

A Yes, sir.

Q At that particular point in time, you did not know if the alleged buy-bust money was
already retrieved by Badua?

A Yes, sir.

Q You went inside the house?

A Yes, sir.

Q You did not have any search warrant?

A Yes, sir.

Q In fact, there was nothing yet as far as you were concerned to validate the fact that
Mrs. Gadao was in possession of the buy-bust money because according to you, you did
not know whether Badua already retrieved the buy-bust money from her?

A Yes, sir.

Q How far was this from the door?

A Two and a half meters from the door, sir. It was in plain view.

Q Under the table according to you?

A Yes, sir, dining table.

Q Somewhere here?
Page 320 of 377

A It's far, sir.

PROSECUTOR

May we request the witness to place it, where he saw it?

A Here, sir.

Q What you see is a carton?

A Yes, sir, with plastic.

Q Marked "Snow Time Ice Pop?

A Yes, sir.

Q With a piece of plastic visible on top of the carton?

A Yes, sir.

Q That is all that you saw?

A Yes, sir.

PROSECUTOR

For the record, your Honor. . .


Page 321 of 377

Q You were only able to verify according to you . . .

PRESECUTOR

Panero, wait. Because I am objecting to the words a piece of plastic. By reading it . . .

ATTY. VALDEZ

That's a piece of plastic.

PROSECUTOR

By reading it, it will connote . . . this is not a piece of plastic.

ATTY. VALDEZ

What is that? What can you say, Fiscal? I'm asking you?

PROSECUTOR

With due respect, what I am saying is, let's place the size of the plastic. A piece of plastic
may be big or a small one, for record purposes.

COURT

Leave that to the court.

PROSECUTOR

Leave that to the court.


Page 322 of 377

Q The only reason according to you, you were able to . . . Look at this, no even Superman
. . . I withdraw that. Not even a man with very kin [sic] eyes can tell the contents here.
And according to the Court, it could be "tikoy," is it not [sic]?

A Yes, sir.

Q Siopao?

A Yes, sir.

Q Canned goods?

A Yes, sir.

Q It could be ice cream because it says Snow Pop, Ice Pop?

A I presumed it was also marijuana because it may . . .

Q I am not asking you what your presumptions are. I'm asking you what it could
possibly be.

A It's the same plastic, sir.

ATTY. VALDEZ

I'm not even asking you that question so why are you voluntarily saying the information.
Let the prosecutor do that for you.

COURT
Page 323 of 377

Continue. Next question.

xxx xxx xxx 130

P03 Manlangit and the police team were at appellant Gaddao's house because they were
led there by appellant Doria. The Narcom agents testified that they had no information
on appellant Gaddao until appellant Doria name her and led them to her.131 Standing
by the door of appellant Gaddao's house, P03 Manlangit had a view of the interior of
said house. Two and a half meters away was the dining table and underneath it was a
carton box. The box was partially open and revealed something wrapped in plastic.

In his direct examination, PO3 Manlangit said that he was sure that the contents of the
box were marijuana because he himself checked and marked the said contents.132 On
cross-examination, however, he admitted that he merely presumed the contents to be
marijuana because it had the same plastic wrapping as the "buy-bust marijuana." A
close scrutiny of the records reveals that the plastic wrapper was not colorless and
transparent as to clearly manifest its contents to a viewer. Each of the ten (10) bricks of
marijuana in the box was individually wrapped in old newspaper and placed inside
plastic bags — white, pink or blue in color.133 PO3 Manlangit himself admitted on
cross-examination that the contents of the box could be items other than marijuana. He
did not know exactly what the box contained that he had to ask appellant Gaddao about
its contents.134 It was not immediately apparent to PO3 Manlangit that the content of
the box was marijuana. The marijuana was not in plain view and its seizure without the
requisite search warrant was in violation of the law and the Constitution.135 It was fruit
of the poisonous tree and should have been excluded and never considered by the trial
court.136

The fact that the box containing about six (6) kilos of marijuana137 was found in the
house of accused-appellant Gaddao does not justify a finding that she herself is guilty
of the crime charged.138 Apropos is our ruling in People v. Aminnudin,139 viz:

The Court strongly supports the campaign of the government against drug addiction
and commends the efforts of our law enforcement officers against those who would
inflict this malediction upon our people, especially the susceptible youth. But as
demanding as this campaign may be, it cannot be more so than the compulsions of the
Bill of Rights for the protection of the liberty of every individual in the realm, including
the basest of criminals. The Constitution covers with the mantle of its protection the
innocent and the guilty alike against any manner of high-handedness from the
authorities, however praiseworthy their intentions.
Page 324 of 377

Those who are supposed to enforce the law are not justified in disregarding the right of
the individual in the name of order. Order is too high a price for the loss of liberty. As
Justice Holmes, again, said, 'I think it a less evil that some criminals should escape
than that the government should play an ignoble part.' It is simply not allowed in the
free society to violate a law to enforce another, especially if the law violated is the
Constitution itself. 140

Section 4 of Republic Act No. 6425, the Dangerous Drugs Act of 1972, as amended by
Section 13 of Republic Act No. 7659 punishes the "sale, administration, delivery,
distribution and transportation of a prohibited drug" with the penalty of reclusion
perpetua to death and a fine ranging from P500,000.00 to P10 million, to wit:

Sec. 4. Sale, Administration, Delivery, Distribution and Transportation of Prohibited


Drugs. — The penalty of reclusion perpetua to death, and a fine ranging from five
hundred thousand pesos to ten million pesos shall be imposed upon any person who,
unless authorized by law, shall sell, administer, deliver, give away to another, distribute,
dispatch in transit or transport any prohibited drug, or shall act as a broker in any of
such transactions.

xxx xxx xxx

In every prosecution for illegal sale of dangerous drugs, what is material is the
submission of proof that the sale took place between the poseur-buyer and the seller
thereof and the presentation of the drug, i.e., the corpus delicti, as evidence in court.141
The prosecution has clearly established the fact that in consideration of P1,600.00
which he received, accused-appellant Doria sold and delivered nine hundred seventy
(970) grams of marijuana to PO3 Manlangit, the poseur-buyer. The prosecution,
however, has failed to prove that accused-appellant Gaddao conspired with accused-
appellant Doria in the sale of said drug. There being no mitigating or aggravating
circumstances, the lower penalty of reclusion perpetua must be imposed.142

IN VIEW WHEREOF, the decision of the Regional Trial Court, Branch 156, Pasig City
acting as a Special Court in Criminal Case No. 3307-D is reversed and modified as
follows:

1. Accused-appellant Florencio Doria y Bolado is sentenced to suffer the penalty of


reclusion perpetua and to pay a fine of five hundred thousand pesos (P500,000.00).
Page 325 of 377

2. Accused-appellant Violeta Gaddao y Catama is acquitted.

SO ORDERED.

Davide, Jr., C.J., Romero, Bellosillo, Melo, Vitug, Kapunan, Mendoza, Martinez,
Quisumbing, Purisima, Pardo, Buena and Gonzaga-Reyes, JJ., concur.

Panganiban, J., please see concurring opinion.

Separate Opinions

PANGANIBAN, J., concurring opinion;

I fully concur with the exhaustive and incisive ponencia of Mr. Justice Reynato S. Puno.
This Decision rightfully brings the Court back to well-settled doctrines on warrantless
arrests and searches, which have seemingly been modified through an obiter in People
v. Ruben Montilla.1 I just wish to outline some guidelines on when an arrest or a search
without a warrant is valid. Hopefully, they would be of help, especially to our law
enforcers who are often faced with actual situations that promptly call for their
application.

Valid Arrests

Without Warrants

Sec. 5 of Rule 113 of the Rules of Court lays down the basic rule on when an arrest
without a warrant is lawful. It states:

Sec. 5. Arrest without warrant; when lawful. — A peace officer or a private person may,
without a warrant, arrest a person:
Page 326 of 377

(a) When, in his presence, the person to be arrested has committed, is actually
committing, or is attempting to commit an offense;

(b) When an offense has in fact just been committed, and he has personal knowledge of
facts indicating that the person to be arrested has committed it; and

(c) When the person to be arrested is a prisoner who escaped from a penal establishment
or place where he is serving final judgment or temporarily confined while his case is
pending, or has escaped while being transferred from one confinement to another.

xxx xxx xxx

I shall focus my discussion on the first two rules, which have been most frequently
misapplied and misinterpreted, not only by law enforcers but some trial judges and
lawyers as well.

At the very outset, I wish to underscore that in both cases the arresting officer must
have personal knowledge of the fact of the commission of an offense. Under Section 5
(a), the officer himself is a witness to the crime; under Section 5 (b), he knows for a fact
that a crime has just been committed. Let me elaborate.

1. In Flagrante Delicto Arrests

Sec. 5 (a) is commonly referred to as the rule on in flagrante delicto arrests.2 The
accused is apprehended at the very moment he is committing or attempting to commit
or has just committed an offense in the presence of the arresting officer. There are two
elements that must concur: (1) the person to be arrested must execute an overt act
indicating that he has just committed, is actually committing, or is attempting to commit
a crime; and (2) such overt act is done in the presence or within the view of the arresting
officer.3

It is not sufficient that the suspect exhibits unusual or strange acts or simply appears
suspicious. Thus, in the recent en banc case of Malicat v. Court of Appeals,4 the Court,
through now Chief Justice Hilario G. Davide Jr., held that the fact that the appellant's
eyes were "moving very fast" and looking at every approaching person were not sufficient
to suspect him of "attempting to commit a crime," much less to justify his arrest and
subsequent search without a warrant. The Court said that "there was nothing in
Page 327 of 377

[Malacat's] behavior or conduct which could have reasonably elicited even mere
suspicion" that he was armed with a deadly weapon. In other words, there was no overt
physical act on the part of the suspect, positively indicating that he had just committed
a crime or was committing or attempting to commit one. There was, therefore, no valid
reason for the police officers to arrest or search him.

The same was true in People v. Mengote,5 where the arresting police tried to justify the
warrantless arrest of the appellant on the ground that he appeared suspicious. The
"suspicious" acts consisted of his darting eyes and the fact that his hand was over his
abdomen. The Court, rejecting such justification, stated: "By no stretch of the
imagination could it have been inferred from these acts that an offense had just been
committed, or was actually being committed, or was at least being attempted in their
presence.6

In other words, the behavior or conduct of the person to be arrested must be clearly
indicative of a criminal act. If there is no outward indication at all that calls for an arrest,
the suspect cannot be validly apprehended under this paragraph, notwithstanding a tip
from an informant that he would at the time be undertaking a felonious enterprise.

This doctrine found strength in People v. Aminnudin 7 and again in People v. Encinada.8
In both cases, the appellants were arrested while disembarking from a ship, on account
of a tip received from an informant that they were carrying prohibited drugs. The Court
invalidated their warrantless arrests, explaining that at the moment of their arrests, the
appellants were simply descending the gangplank, without manifesting any suspicious
behavior that would reasonably invite the attention of the police. To all appearances,
they were not committing a crime; nor was it shown that they were about to do so or
had just done so. There was, therefore, no valid reason for their arrests.

Adhering to (and having faith in) the above rules, I respectfully disagreed with the
distinguished Mr. Justice Florenz D. Regalado in People v. Montilla,9 when he upheld
the validity of the warrantless arrest of the appellant while the latter was merely
alighting from a passenger jeepney. I opined that Montilla could not have been perceived
as committing a crime while merely alighting from a jeepney carrying a traveling bag
and a carton. He did not exhibit any overt act or strange conduct that would reasonably
arouse in the minds of the police suspicion that he was embarking on a felonious
undertaking. There was no outward manifestation that he had just committed or was
committing or attempting to commit an offense. Mercifully, the statement of the Court
that Montilla's arrest was valid because he was caught in flagrante delicto was only an
obiter, for what finally nailed him down was his implied waiver of any objection to the
validity of his arrest.
Page 328 of 377

2. "Hot Pursuit" Arrests

Sec. 5 (b) is otherwise known as the rule on "hot pursuit" arrests.10 Here, two elements
must also concur prior to the arrest: (1) and "offense has in fact been committed," (2)
the arresting officer "has personal knowledge of facts indicating that the person to be
arrested . . . committed [the offense]." In effecting this type of arrest, "it is not enough
that there is reasonable ground to believe that the person to be arrested has committed
a crime. A crime must in fact or actually have been committed first. . . . The fact of the
commission of the offense must be undisputed.11

Thus, while the law enforcers may not actually witness the execution of acts constituting
the offense, they must have direct knowledge or view of the crime right after its
commission. They should know for a fact that a crime was committed. AND they must
also perceive acts exhibited by the person to be arrested, indicating that he perpetrated
the crime. Again, mere intelligence information that the suspect committed the crime
will not suffice. The arresting officers themselves must have personal knowledge of facts
showing that the suspect performed the criminal act. Personal knowledge means actual
belief or reasonable grounds of suspicion, based on actual facts, that the person to be
arrested is probably guilty of committing the crime.12

In several cases wherein third persons gave law enforcers information that certain
individuals or groups were engaged in some felonious activities, such relayed
information was not deemed equivalent to personal knowledge of the lawmen. In People
v. Burgos,13 a certain Masamlok informed police authorities that the appellant was
involved in subversive activities. Acting on the strength of such information and without
securing a judicial warrant, the police proceeded to appellant's house to arrest him.
There, they also allegedly recovered an unlicensed firearm and subversive materials.

The Court held that there was no personal knowledge on the part of the arresting
officers, since the information came in its entirety from Masamlok, a civilian. We pointed
out that at the time of his arrest, appellant was not in actual possession of any firearm
or subversive document; neither was he committing a subversive act.14 His warrantless
arrest, therefore, could not be allowed under any of the instances in Rule 113, Section
6 (now 5) of the Rules of Court.

Also in Encinada, the appellant was arrested without a warrant, on the justification that
the arresting officer "received an intelligence report that appellant who was carrying
marijuana would arrive the next morning aboard M/V Sweet Pearl." The Court
categorically stated that such "[r]aw intelligence information is not a sufficient ground
for a warrantless arrest."15 And since, at the time of his arrest, no act or fact
Page 329 of 377

demonstrating a felonious enterprise could be ascribed to appellant, there was no valid


justification for his arrest.

To be distinguished from the above cases are those involving continuing offenses for
which the culprit could be arrested any time in flagrante delicto. In Umil v. Ramos,16
there were strong objections to the warrantless arrest of a suspected member of the New
People's Army (NPA), while he was being treated for a gunshot wound in a hospital. He
alleged that there was no valid justification for his arrest without a warrant, because he
was not then committing any offense nor were there any indications that he had just
committed or was about to commit one; he was in fact confined in a hospital.

The Court held that subversion, for which he was arrested and subsequently charged,
was a continuing offense. For purposes of arrest, the Court said, the NPA member "did
not cease to be, or became less of a subversive, . . . simply because he was, at the time
of his arrest, confined in the . . . [hospital]." "Unlike other so-called 'common' offenses,
i.e., adultery, murder, arson, etc., which generally end upon their commission,
subversion and rebellion are anchored on an ideological base which compels the
repetition of the same acts of lawlessness and violence until the overriding object of
overthrowing organized government is attained.17

In the above instances where the arrests without warrants were held unlawful, so were
the searches conducted subsequent thereto. Thus, the items seized consequent to the
invalid search, though clearly prohibited by law (e.g. marijuana or unlicensed firearm),
were considered inadmissable as evidence against the person wrongfully arrested.
Important to bear in mind always is that any search conducted without a judicial
warrant must be prcceded by a lawful arrest, whether with or without a warrant duly
issued therefor.

To underscore the rationale behind these strict rules, I deem it quite apt to quote these
inspiring words from the precedent-setting case of People v. Burgos:18

The right of a person to be secure against any unreasonable seizure of his body and any
deprivation of his liberty is a most basic and fundamental one. The statute or rule which
allows exceptions to the requirement of warrants of arrest is strictly construed. Any
exception must clearly fall within the situations when securing a warrant would be
absurd or is manifestly unnecessary as provided by the Rule. We cannot liberally
construe the rule on arrests without warrant or extend its application beyond the cases
specifically provided by law. To do so would infringe upon personal liberty and set back
a basic right so often violated and so deserving of full protection.
Page 330 of 377

Valid Searches Without Warrant

The general rule is that a judicial warrant must first be duly obtained before search and
seizure may be conducted. The only allowable instances in which a search may be
conducted without a warrant are: (1) search incident to lawful arrest, (2) search
pursuant to the "plain view" doctrine, (3) search of moving vehicles, (4) searches
incidental to violation of customs laws, (5) search with consent, and (6) a "stop and
frisk.19

1. Search Incident to Lawful Arrest

Section 12 of Rule 126 provides that a lawfully arrested person may be searched without
a warrant for dangerous weapons or anything else that may be used as evidence of the
offense. Such incidental search is, however, limited to the person of the arrestee at the
time of the apprehension. The search cannot be extended to or made in a place other
than the place of the arrest.20

2. The "Plain View" Doctrine

The "plain view" doctrine applies when the following requisites concur: (1) the law
enforcement officer is in a position where he has a clear view of a particular area or has
prior justification for an intrusion; (2) said officer inadvertently comes across (or sees in
plainview) a piece of incriminating evidence; and (3) it is immediately apparent to such
officer that the item he sees may be evidence of a crime or a contraband or is otherwise
subject to seizure.21

3. Search of Moving Vehicles

The warrantless search of moving vehicles (including shipping vessels and aircraft) is
justified by practicability, viz.:22

The guaranty of freedom from unreasonable searches and seizures construed as


recognizing a necessary difference between a search of a dwelling house or other
structure in respect of which a search warrant may readily be obtained and a search of
a ship, motorboat, wagon, or automobile for contraband goods, where it is not
practicable to secure a warrant, because the vehicle can be quickly moved out of the
locality or jurisdiction in which the warrant must be sought.
Page 331 of 377

xxx xxx xxx

The automobile is a swift and powerful vehicle . . . Constructed as covered vehicles to


standard form in immense quantities, and with a capacity for speed rivaling express
trains they furnish for successful commission of crime a distinguishing means of silent
approach and swift escape unknown in the history of the world before their advent. The
question of their police control and reasonable search on highways or other public place
is a serious question far deeper and broader than their use in so-called 'bootlegging' or
'rum running,' which in itself is no small matter. While a possession in the sense of
private ownership, they are but a vehicle constructed for travel and transportation on
highways. Their active use is not in homes or on private premises, the privacy of which
the law especially guards from search and seizure without process. The baffling extent
to which they are successfully utilized to facilitate commission of crime of all degrees,
from those against morality, chastity, and decency to robbery, rape, burglary, and
murder, is a matter of common knowledge. Upon that problem, a condition, and not a
theory, confronts proper administration of our criminal laws. Whether search of and
seizure from an automobile upon a highway or other public place without a search
warrant is unreasonable is in its final analysis to be determined as a judicial question
in view of all the circumstances under which it is made.

4. Customs Searches

Under the Tariff and Customs Code, searches, seizures and arrests may be made even
without warrants, for purposes of enforcing customs and tariff laws. Without mention
of the need to priorly obtain a judicial warrant, the Code specifically allows police
authorities to "enter, pass through or search any land, enclosure, warehouse, store or
building, not being a dwelling house; and also to inspect, search and examine any vessel
or aircraft and any trunk, package, box or envelope or any person on board[;]or stop
and search and examine any vehicle, beast or person suspected of holding or conveying
any dutiable or prohibited article introduced into the Philippines contrary to law.23

5. Search With Consent

Waiver of any objection to the unresonableness or invalidity of a search is a recognized


exception to the rule against a warrantless search.24 The consent to the search,
however, must be express knowing and voluntary. A search based merely on implied
acquiescene is not valid, because such consent is not within the purview of the
constitutional gurantee, but only a passive conformity to the search given under
intimidating and coercive circumstances.25
Page 332 of 377

6. Stop and Frisk

The "stop and frisk" concept is of American origin, the most notable case thereon being
Terry v. Ohio.27 The idea is that a police officer may after properly introducing himself
and making initial inquiries, approach and restrain a person manifesting unusual and
suspicious conduct, in order to check, the latter's outer clothing for possibly concealed
weapons. The strict manner in which this notion should be applied has been laid down
as follows:28

. . . where a police officer observes unusual conduct which leads him reasonably to
conclude in the light of his experience that criminal activity may be afoot and that the
persons with whom he is dealing may be armed and presently dangerous, where in the
course of investigating this behaviour, he identifies himself as a policeman and makes
reasonable inquiries, and where nothing in the initial stages of the encounter serves to
dispel his reasonable fear for his own and others' safety, he is entitled for the protection
of himself and others in the area to conduct a carefully limited search of the outer
clothing of such persons in an attempt to discover weapons which might be used to
assault him.

As in the warrantless arrest of a person reasonably suspected of having just committed


a crime, mere suspicious behavior would not call for a "stop and frisk." There must be
a genuine reason, in accordance with the police officer's experience and the surrounding
conditions, to warrant the belief that the person to be held has weapons (or contraband)
concealed about him.29

A valid application of the doctrine was recognized in Posadas v. Court of Appeals 30 and
in Manalili v. Court of Appeals.31 In Manalili, the law enforcers who were members of
the Anti-Narcotics Unit of the Caloocan City Police, observed during their surveillance
that appellant had red eyes and was walking in a wobby manner along the city cemetery
which, according to police information, was a popular hangout of drug addicts. Based
on police experience, such suspicious behaviour was characteristic of persons who were
"high" on drugs. The Court held that past experience and the surrounding
circumstances gave the police sufficient reason to stop the suspect and to investigate if
he was really high on drugs. The marijuana that they found in the suspect's possession
was held to be admissible in evidence.

Before I end, I must reiterate that the above exceptions to the general rule on the
necessity of a judicial warrant for any arrest, search and seizure must all be strictly
Page 333 of 377

construed. Foremost in our minds must still be every person's prized and fundamental
right to liberty and security, a right protected and guaranteed by our Constitution.

WHEREFORE, I vote to ACQUIT Appellant Violeta Gaddao y Catama, as well as to


REDUCE the penalty of Appellant Florencio Doria y Bolado to reclusion perpetua and a
fine of P500,000.
Page 334 of 377

G.R. No. 81567 July 9, 1990

IN THE MATTER OF THE PETITION FOR HABEAS CORPUS OF ROBERTO


UMIL, ROLANDO DURAL and RENATO VILLANUEVA. MANOLITA O. UMIL,
and NICANOR P. DURAL, FELICITAS V. SESE, petitioners, vs. FIDEL V.
RAMOS, MAJ. GEN. RENATO DE VILLA, BRIG. GEN. RAMON MONTANO,
BRIG. GEN. ALEXANDER AGUIRRE, respondents.

G.R. Nos. 84581-82 July 9, 1990

AMELIA ROQUE and WILFREDO BUENAOBRA, petitioners, vs. GEN. RENATO DE


VILLA and GEN. RAMON MONTANO, respondents.

G.R. Nos. 84583-84 July 9, 1990

IN THE MATTER OF THE PETITION FOR HABEAS CORPUS OF ATTY. DOMINGO T.


ANONUEVO and RAMON CASIPLE. DOMINGO T. ANONUEVO and RAMON CASIPLE,
petitioners, vs. HON. FIDEL V. RAMOS, GEN. RENATO S. DE VILLA, COL. EVARISTO
CARINO, LT. COL. REX D. PIAD, T/SGT. CONRADO DE TORRES, S/SGT. ARNOLD
DURIAN, and Commanding Officer, PC-INP Detention Center, Camp Crame, Quezon
City, respondents.

G.R. No. 83162 July 9, 1990

IN THE MATTER OF THE APPLICATION FOR HABEAS CORPUS OF VICKY A. OCAYA


AND DANNY RIVERA. VIRGILIO A. OCAYA, petitioner, vs. BRIG. GEN. ALEXANDER
AGUIRRE, COL. HERCULES CATALUNA, COL. NESTOR MARIANO, respondents.

G.R. No. 85727 July 9, 1990

IN THE MATTER OF APPLICATION FOR HABEAS CORPUS OF: DEOGRACIAS


ESPIRITU, petitioner, vs. BRIG. GEN. ALFREDO S. LIM, COL. RICARDO REYES,
respondents.

G.R. No. 86332 July 9, 1990


IN THE MATTER OF THE PETITION FOR HABEAS CORPUS OF NARCISO B.
NAZARENO. ALFREDO NAZARENO, petitioner, vs. THE STATION COMMANDER OF
THE MUNTINGLUPA POLICE STATION, Muntinglupa, Metro Manila, P/SGT. JACINTO
MEDINA, P/SGT. ELADIO TAGLE, P/SGT. LEVI SOLEDAD, and P/SGT. MAURO
AROJADO, respondents.
Page 335 of 377

PER CURIAM:

The are eight (8) petitioners for habeas corpus filed before the Court, which have been
consolidated because of the similarity of issues raised, praying for the issuance of the
writ of habeas corpus, ordering the respective respondents to produce the bodies of the
persons named therein and to explain why they should not be set at liberty without
further delay.

In their respective Returns, the respondents uniformly assert that the privilege of the
writ of habeas corpus is not available to the petitioners as they have been legally arrested
and are detained by virtue of valid informations filed in court against them.

The petitioners counter that their detention is unlawful as their arrests were made
without warrant and, that no preliminary investigation was first conducted, so that the
informations filed against them are null and void.

The Court has carefully reviewed the contentions of the parties in their respective
pleadings, and it finds that the persons detained have not been illegally arrested nor
arbitrarily deprived of their constitutional right to liberty, and that the circumstances
attending these cases do not warrant their release on habeas corpus.

The arrest of a person without a warrant of arrest or previous complaint is recognized


in law. The occasions or instances when such an arrest may be effected are clearly
spelled out in Section 5, Rule 113 of the Rules of Court, as amended, which provides:

Sec. 5. Arrest without warrant; when lawful. — A peace officer or a private person may,
without a warrant, arrest a person:

(a) When, in his presence, the person to be arrested has committed, is actually
committing, or is attempting to commit an offense;

(b) When an offense has in fact just been committed, and he has personal knowledge of
facts indicating that the person to be arrested has committed it; and

(c) When the person to be arrested is a prisoner who has escaped from a penal
establishment or place where he is serving final judgment or temporarily confined while
Page 336 of 377

his case is pending, or has escaped while being transferred from one confinement to
another.

In cases falling under paragraphs (a) and (b) hereof, the person arrested without a
warrant shall be forthwith delivered to the nearest police station or jail, and he shall be
proceeded against in accordance with Rule 112, Section 7.

An arrest without a warrant of arrest, under Section 5 paragraphs (a) and (b) of Rule
113 of the Rules of Court, as amended, is justified when the person arrested is caught
in flagranti delicto, viz., in the act of committing an offense; or when an offense has just
been committed and the person making the arrest has personal knowledge of the facts
indicating that the person arrested has committed it. The rationale behind lawful
arrests, without warrant, was stated by this Court in the case of People vs. Kagui
Malasugui 1 thus:

To hold that no criminal can, in any case, be arrested and searched for the evidence and
tokens of his crime without a warrant, would be to leave society, to a large extent, at the
mercy of the shrewdest, the most expert, and the most depraved of criminals, facilitating
their escape in many instances.

The record of the instant cases would show that the persons in whose behalf these
petitions for habeas corpus have been filed, had freshly committed or were actually
committing an offense, when apprehended, so that their arrests without a warrant were
clearly justified, and that they are, further, detained by virtue of valid informations filed
against them in court.

A brief narration of the facts and events surrounding each of the eight (8) petitions is in
order.

In G.R. No. 81567 (Umil vs. Ramos), the record shows that, on 1 February 1988, the
Regional Intelligence Operations Unit of the Capital Command (RIOU-CAPCOM) received
confidential information about a member of the NPA Sparrow Unit (liquidation squad)
being treated for a gunshot wound at the St. Agnes Hospital in Roosevelt Avenue,
Quezon City. Upon verification, it was found that the wounded person, who was listed
in the hospital records as Ronnie Javelon, is actually Rolando Dural, a member of the
NPA liquidation squad, responsible for the killing of two (2) CAPCOM soldiers the day
Page 337 of 377

before, or on 31 January 1988, in Macanining Street, Bagong Barrio, Caloocan City. In


view of this verification, Rolando Dural was transferred to the Regional Medical Services
of the CAPCOM, for security reasons. While confined thereat, or on 4 February 1988,
Rolando Dural was positively identified by eyewitnesses as the gunman who went on
top of the hood of the CAPCOM mobile patrol car, and fired at the two (2) CAPCOM
soldiers seated inside the car identified as T/Sgt. Carlos Pabon and CIC Renato
Manligot.

As a consequence of this positive identification, Rolando Dural was referred to the


Caloocan City Fiscal who conducted an inquest and thereafter filed with the Regional
Trial Court of Caloocan City an information charging Rolando Dural alias Ronnie
Javelon with the crime of "Double Murder with Assault Upon Agents of Persons in
Authority." The case was docketed therein as Criminal Case No. C-30112 and no bail
was recommended. On 15 February 1988, the information was amended to include, as
defendant, Bernardo Itucal, Jr. who, at the filing of the original information, was still
unidentified.

Meanwhile, on 6 February 1988, a petition for habeas corpus was filed with this Court
on behalf of Roberto Umil, Rolando Dural, and Renato Villanueva. The Court issued the
writ of habeas corpus on 9 February 1988 and the respondents filed a Return of the
Writ on 12 February 1988. Thereafter, the parties were heard on 15 February 1988.

On 26 February 1988, however, Roberto Umil and Renato Villanueva posted bail before
the Regional Trial Court of Pasay City where charges for violation of the Anti-Subversion
Act had been filed against them, and they were accordingly released. The petition for
habeas corpus, insofar as Umil and Villanueva are concerned, is now moot and
academic and is accordingly dismissed, since the writ of habeas corpus does not lie in
favor of an accused in a criminal case who has been released on bail. 2

As to Rolando Dural, it clearly appears that he was not arrested while in the act of
shooting the two (2) CAPCOM soldiers aforementioned. Nor was he arrested just after
the commission of the said offense for his arrest came a day after the said shooting
incident. Seemingly, his arrest without warrant is unjustified.

However, Rolando Dural was arrested for being a member of the New Peoples Army
(NPA), an outlawed subversive organization. Subversion being a continuing offense, the
arrest of Rolando Dural without warrant is justified as it can be said that he was
committing an offense when arrested. The crimes of rebellion, subversion, conspiracy
or proposal to commit such crimes, and crimes or offenses committed in furtherance
Page 338 of 377

thereof or in connection therewith constitute direct assaults against the State and are
in the nature of continuing crimes. As stated by the Court in an earlier case:

From the facts as above-narrated, the claim of the petitioners that they were initially
arrested illegally is, therefore, without basis in law and in fact. The crimes of
insurrection or rebellion, subversion, conspiracy or proposal to commit such crimes,
and other crimes and offenses committed in the furtherance, on the occasion thereof,
or incident thereto, or in connection therewith under Presidential Proclamation No.
2045, are all in the nature of continuing offenses which set them apart from the common
offenses, aside from their essentially involving a massive conspiracy of nationwide
magnitude. Clearly then, the arrest of the herein detainees was well within the bounds
of the law and existing jurisprudence in our jurisdiction.

2. The arrest of persons involved in the rebellion whether as its fighting armed elements,
or for committing non-violent acts but in furtherance of the rebellion, is more an act of
capturing them in the course of an armed conflict, to quell the rebellion, than for the
purpose of immediately prosecuting them in court for a statutory offense. The arrest,
therefore, need not follow the usual procedure in the prosecution of offenses which
requires the determination by a judge of the existence of probable cause before the
issuance of a judicial warrant of arrest and the granting of bail if the offense is bailable.
Obviously, the absence of a judicial warrant is no legal impediment to arresting or
capturing persons committing overt acts of violence against government forces, or any
other milder acts but equally in pursuance of the rebellious movement. The arrest or
capture is thus impelled by the exigencies of the situation that involves the very survival
of society and its government and duly constituted authorities. If killing and other acts
of violence against the rebels find justification in the exigencies of armed hostilities
which is of the essence of waging a rebellion or insurrection, most assuredly so in case
of invasion, merely seizing their persons and detaining them while any of these
contingencies continues cannot be less justified. . . . 3

The record, moreover, shows that the criminal case filed against Rolando Dural and
Bernardo Itucal, Jr. for "Double Murder, etc." was tried in the court below and at the
conclusion thereof, or on 17 August 1988, Rolando Dural and Bernardo Itucal, Jr. were
found guilty of the charge and sentenced accordingly. Rolando Dural is now serving the
sentence imposed upon him by the trial court. Thus, the writ of habeas corpus is no
longer available to him. For, as held in the early case of U.S. vs. Wilson: 4

In this case, whatever may be said about the manner of his arrest, the fact remains that
the defendant was actually in court in the custody of the law on March 29, when a
complaint sufficient in form and substance was read to him. To this he pleaded not
guilty. The trial followed, in which, and in the judgment of guilty pronounced by the
Page 339 of 377

court, we find no error. Whether, if there were irregularities in bringing him personally
before the court, he could have been released on a writ of habeas corpus or now has a
civil action for damages against the person who arrested him we need not inquire. It is
enough to say that such irregularities are not sufficient to set aside a valid judgment
rendered upon a sufficient complaint and after a trial free from error.

II

In G.R. Nos. 84581-82 (Roque vs. De Villa), the arrest of Amelia Roque and Wilfredo
Buenaobra, without warrant, is also justified. When apprehended at the house of Renato
Constantino in Marikina Heights, Marikina, Metro Manila, Wilfredo Buenaobra admitted
that he was an NPA courier and he had with him letters to Renato Constantino and
other members of the rebel group. Amelia Roque, upon the other hand, was a member
of the National United Front Commission, in charge of finance, and admitted ownership
of subversive documents found in the house of her sister in Caloocan City. She was also
in possession of ammunition and a fragmentation grenade for which she had no permit
or authority to possess.

The record of these two (2) cases shows that on 27 June 1988, one Rogelio Ramos y
Ibanes, a member of the NPA, who had surrendered to the military authorities, told
military agents about the operations of the Communist Party of the Philippines (CPP)
and the New Peoples Army (NPA) in Metro Manila. He identified some of his former
comrades as "Ka Mong", a staff member of the Communications and Transportation
Bureau; "Ka Nelia", a staff member in charge of finance; "Ka Miller", an NPA courier from
Sorsogon and Lopez, Quezon; "Ka Ted", and "Ka Totoy". He also pointed to a certain
house occupied by Renato Constantino located in the Villaluz Compound, Molave St.,
Marikina Heights, Marikina, Metro Manila, which is used as a safehouse of the National
United Front Commission (NUFC) of the CPP-NPA.

In view of these revelations, the Constantino house was placed under military
surveillance and on 12 August 1988, pursuant to a search warrant issued by Judge
Eutropio Migrino of the Regional Trial Court of Pasig, a search of the house was
conducted at about 5:00 o'clock in the afternoon, by a combined team of the Criminal
Investigation Service, National Capital District (CIS-NCD) and the Constabulary Security
Group (CSG). In the course of the search, the following articles were found and taken
under proper receipt:

a) One (1) Colt M16A1 long rifle with defaced serial number;
Page 340 of 377

b) One (1) Cal. .380 ACT/9mm Model PPK/8 SN: 260577 & 2605778;

c) Two (2) fragmentation hand grenades;

d) Fifty-six (56) live ammunition for Cal. 5.56 mm;

e) Five (5) live ammunition for Cal. .380;

f) One (1) ICOM VHF FM Radio Transciever SN: 14903

g) One (1) Regulated power supply 220V AC;

h) One (1) Antennae (adjustable);

i) One (1) Speaker with cord ALEXAR;

j) Voluminous Subversive documents.

When confronted, Renato Constatino could not produce any permit or authority to
possess the firearms, ammunition, radio and other communications equipment. Hence,
he was brought to the CIS Headquarters for investigation. When questioned, he refused
to give a written statement, although he admitted that he was a staff member of the
executive committee of the NUFC and a ranking member of the International
Department of the Communist Party of the Philippines (CPP).

At about 8:00 o'clock in the evening of the same day (12 August 1988), Wilfredo
Buenaobra arrived at the house of Renato Constantino in the Villaluz Compound. When
accosted, he readily admitted to the military agents that he is a regular member of the
CPP/NPA and that he went to the place to deliver letters to "Ka Mong", referring to
Renato Constatino, and other members of the rebel group. On further questioning, he
also admitted that he is known as "Ka Miller" and that he was from Barangay San Pedro,
Lopez, Quezon. Among the items taken from him were the following:
Page 341 of 377

(1) Handwritten letter addressed to "Ka Bing & Co. from A & Co." dated August 11, 1988;

(2) Handwritten letter addressed to "ROD from VIC (Schell datre)" dated August 11,
1988;

(3) Handwritten letter addressed to "Suzie" from "Vic", dated August 11, 1988.

Also found Buenaobra's possession was a piece of paper containing a written but
jumbled telephone number of Florida M. Roque, sister of Amelia Roque alias "Ka Nelia",
at 69 Geronimo St., Caloocan City. Acting on the lead provided as to the whereabouts
of Amelia Roque, the military agents went to the given address the next day (13 August
1988). They arrived at the place at about 11:00 o'clock in the morning. After identifying
themselves as military agents and after seeking permission to search the place, which
was granted, the military agents conducted a search in the presence of the occupants
of the house and the barangay captain of the place, one Jesus D. Olba.

The military agents found the place to be another safehouse of the NUFC/CPP. They
found ledgers, journals, vouchers, bank deposit books, folders, computer diskettes, and
subversive documents as well as live ammunition for a .38 SPL Winchester, 11 rounds
of live ammunition for a cal. .45, 19 rounds of live ammunition for an M16 Rifle, and a
fragmentation grenade. As a result, Amelia Roque and the other occupants of the house
were brought to the PC-CIS Headquarters at Camp Crame, Quezon City, for
investigation. Amelia Roque admitted to the investigators that the voluminous
documents belonged to her and that the other occupants of the house had no knowledge
of them. As a result, the said other occupants of the house were released from custody.

On 15 August 1988, Amelia Roque was brought to the Caloocan City Fiscal for inquest
after which an information charging her with violation of PD 1866 was filed with the
Regional Trial Court of Caloocan City. The case is docketed therein as Criminal Case
No. C-1196. Another information for violation of the Anti-Subversion Act was filed
against Amelia Roque before the Metropolitan Trial Court of Caloocan City, which is
docketed therein as Criminal Case No. C-150458.

An information for violation of the Anti-Subversion Act was filed against Wilfredo
Buenaobra before the Metropolitan Trial Court of Marikina, Metro Manila. The case is
docketed therein as Criminal Case No. 23715. Bail was set at P4,000.00.
Page 342 of 377

On 24 August 1988, a petition for habeas corpus was filed before this Court on behalf
of Amelia Roque and Wilfredo Buenaobra. At the hearing of the case, however, Wilfredo
Buenaobra manifested his desire to stay in the PC-INP Stockade at Camp Crame,
Quezon City. According, the petition for habeas corpus filed on his behalf is now moot
and academic. Only the petition of Amelia Roque remains for resolution.

The contention of respondents that petitioners Roque and Buenaobra are officers and/or
members of the National United Front Commission (NUFC) of the CPP was not
controverted or traversed by said petitioners. The contention must be deemed admitted.
5 As officers and/or members of the NUFC-CPP, their arrest, without warrant, was
justified for the same reasons earlier stated vis-a-vis Rolando Dural. The arrest without
warrant of Roque was additionally justified as she was, at the time of apprehension, in
possession of ammunitions without license to possess them.

III

In G.R. Nos. 84583-84 (Anonuevo vs. Ramos), the arrest of Domingo Anonuevo and
Ramon Casiple, without warrant, is also justified under the rules. Both are admittedly
members of the standing committee of the NUFC and, when apprehended in the house
of Renato Constatino, they had a bag containing subversive materials, and both carried
firearms and ammunition for which they had no license to possess or carry.

The record of these two (2) cases shows that at about 7:30 o'clock in the evening of 13
August 1988, Domingo T. Anonuevo and Ramon Casiple arrived at the house of Renato
Constatino at Marikina Heights, Marikina, which was still under surveillance by military
agents. The military agents noticed bulging objects on their waist lines. When frisked,
the agents found them to be loaded guns. Anonuevo and Casiple were asked to show
their permit or license to possess or carry firearms and ammunition, but they could not
produce any. Hence, they were brought to PC Headquarters for investigation. Found in
their possession were the following articles:

a) Voluminous subversive documents

b) One (1) Cal. 7.65 MOD 83 2C Pistol SN: 001412 with one (1) magazine for Cal. 7.65
containing ten (10) live ammunition of same caliber;

c) One (1) Cal. 7.65 Pietro Barreta SN; A18868 last digit tampered with one (1) magazine
containing five (5) live ammunition of same caliber.
Page 343 of 377

At the PC Stockade, Domingo Anonuevo was identified as "Ka Ted", and Ramon Casiple
as "Ka Totoy" of the CPP, by their comrades who had previously surrendered to the
military.

On 15 August 1988, the record of the investigation and other documentary evidence
were forwarded to the Provincial Fiscal at Pasig, Metro Manila, who conducted an
inquest, after which Domingo Anonuevo and Ramon Casiple were charged with violation
of Presidential Decree No. 1866 before the Regional Trial Court of Pasig, Metro Manila.
The cases are docketed therein as Criminal Cases Nos. 74386 ad 74387, respectively.
No bail was recommended.

On 24 August 1988, a petition for habeas corpus was filed with this Court on behalf of
Domingo Anonuevo and Ramon Casiple, alleging that the said Anonuevo and Casiple
were unlawfully arrested without a warrant and that the informations filed against them
are null and void for having been filed without prior hearing and preliminary
investigation. On 30 August 1988, the Court issued the writ of habeas corpus, and after
the respondents had filed a Return of the Writ, the parties were heard.

The petitioners' (Anonuevo and Casiple) claim that they were unlawfully arrested
because there was no previous warrant of arrest, is without merit The record shows that
Domingo Anonuevo and Ramon Casiple were carrying unlicensed firearms and
ammunition in their person when they were apprehended.

There is also no merit in the contention that the informations filed against them are null
and void for want of a preliminary investigation. The filing of an information, without a
preliminary investigation having been first conducted, is sanctioned by the Rules. Sec.
7, Rule 112 of the Rules of Court, as amended, reads:

Sec. 7. When accused lawfully arrested without a warrant. — When a person is lawfully
arrested without a warrant for an offense cognizable by the Regional Trial Court the
complaint or information may be filed by the offended party, peace officer or fiscal
without a preliminary investigation having been first conducted, on the basis of the
affidavit of the offended party or arresting officer or person.

However, before the filing of such complaint or information, the person arrested may
ask for a preliminary investigation by a proper officer in accordance with this Rule, but
he must sign a waiver of the provisions of Article 125 of the Revised Penal Code, as
amended, with the assistance of a lawyer and in case of non-availability of a lawyer, a
Page 344 of 377

responsible person of his choice. Notwithstanding such waiver, he may apply for bail as
provided in the corresponding rule and the investigation must be terminated within
fifteen (15) days from its inception.

If the case has been filed in court without a preliminary investigation having been first
conducted, the accused may within five (5) days from the time he learns of the filing of
the information, ask for a preliminary investigation with the same right to adduced
evidence in his favor in the manner prescribed in this Rule.

The petitioners Domingo Anonuevo and Ramon Casiple, however, refused to sign a
waiver of the provisions of Article 125 of the Revised Penal Code, as amended. In the
informations filed against them, the prosecutor made identical certifications, as follows:

This is to certify that the accused has been charged in accordance with Sec. 7, Rule 112
of the 1985 Rules on Criminal Procedure, that no preliminary investigation was
conducted because the accused has not made and signed a waiver of the provisions of
Art. 125 of the Revised Penal Code, as amended; that based on the evidence presented,
there is reasonable ground to believe that the crime has been committed, and that the
accused is probably guilty thereof.

Nor did petitioners ask for a preliminary investigation after the informations had been
filed against them in court. Petitioners cannot now claim that they have been deprived
of their constitutional right to due process.

IV

In G.R. No. 83162 (Ocaya vs. Aguirre), the arrest without warrant, of Vicky Ocaya is
justified under the Rules, since she had with her unlicensed ammunition when she was
arrested. The record of this case shows that on 12 May 1988, agents of the PC
Intelligence and Investigation of the Rizal PC-INP Command, armed with a search
warrant issued by Judge Eutropio Migrino of the Regional Trial Court of Pasig, Metro
Manila, conducted a search of a house located at Block 19, Phase II, Marikina Green
Heights, Marikina, Metro Manila, believed to be occupied by Benito Tiamson, head of
the CPP-NPA. In the course of the search, Vicky Ocaya arrived in a car driven by Danny
Rivera. Subversive documents and several rounds of ammunition for a .45 cal. pistol
were found in the car of Vicky Ocaya. As a result, Vicky Ocaya and Danny Rivera were
brought to the PC Headquarters for investigation. When Vicky Ocaya could not produce
any permit or authorization to possess the ammunition, an information charging her
with violation of PD 1866 was filed with the Regional Trial Court of Pasig, Metro Manila.
Page 345 of 377

The case is docketed therein as Criminal Case No. 73447. Danny Rivera, on the other
hand, was released from custody.

On 17 May 1988, a petition for habeas corpus was filed, with this Court on behalf of
Vicky Ocaya and Danny Rivera. It was alleged therein that Vicky Ocaya was illegally
arrested and detained, and denied the right to a preliminary investigation.

It would appear, however, that Vicky Ocaya was arrested in flagranti delicto so that her
arrest without a warrant is justified. No preliminary investigation was conducted
because she was arrested without a warrant and she refused to waive the provisions of
Article 125 of the Revised Penal Code, pursuant to Sec. 7, Rule 112 of the Rule of Court,
as amended.

The petitioners Vicky Ocaya, Domingo Anonuevo, Ramon Casiple, and Amelia Roque
claim that the firearms, ammunition and subversive documents alleged to have been
found in their possession when they were arrested, did not belong to them, but were
"planted" by the military agents to justify their illegal arrest.

The petitioners, however, have not introduced any evidence to support their aforesaid
claim. On the other hand, no evil motive or ill-will on the part of the arresting officers
that would cause the said arresting officers in these cases to accuse the petitioners
falsely, has been shown. Besides, the arresting officers in these cases do not appear to
be seekers of glory and bounty hunters for, as counsel for the petitioners Anonuevo and
Casiple say, "there is absolutely nothing in the evidence submitted during the inquest
that petitioners are on the 'AFP Order of Battle with a reward of P150,000.00 each on
their heads.'" 6 On the other hand, as pointed out by the Solicitor General, the arrest of
the petitioners is not a product of a witch hunt or a fishing expedition, but the result of
an in-depth surveillance of NPA safehouses pointed to by no less than former comrades
of the petitioners in the rebel movement.

The Solicitor General, in his Consolidated Memorandum, aptly observes:

. . . . To reiterate, the focal point in the case of petitioners Roque, Buenaobra, Anonuevo
and Casiple, was the lawful search and seizure conducted by the military at the
residence of Renato Constantino at Villaluz Compound, Molave St., Marikina Heights,
Marikina, Metro Manila. The raid at Constantino's residence, was not a witch hunting
Page 346 of 377

or fishing expedition on the part of the military. It was a result of an in-depth military
surveillance coupled with the leads provided by former members of the underground
subversive organizations. That raid produced positive results. to date, nobody has
disputed the fact that the residence of Constantino when raided yielded communication
equipment, firearms and ammunitions, as well as subversive documents.

The military agents working on the information provided by Constantino that other
members of his group were coming to his place, reasonably conducted a "stake-out"
operation whereby some members of the raiding team were left behind the place. True
enough, barely two hours after the raid and Constantino's arrest, petitioner Buenaobra
arrived at Constantino's residence. He acted suspiciously and when frisked and
searched by the military authorities, found in his person were letters. They are no
ordinary letters, as even a cursory reading would show. Not only that, Buenaobra
admitted that he is a NPA courier and was there to deliver the letters to Constantino.

Subsequently, less than twenty four hours after the arrest of Constantino and
Buenaobra, petitioners Anonuevo and Casiple arrived at Constantino's place. Would it
be unreasonable for the military agents to believe that petitioners Anonuevo and Casiple
are among those expected to visit Constantino's residence considering that Constatino's
information was true, in that Buenaobra did come to that place? Was it unreasonable
under the circumstances, on the part of the military agents, not to frisk and search
anyone who should visit the residence of Constantino, such as petitioners Anonuevo
and Casiple? Must this Honorable Court yield to Anonuevo and Casiple's flimsy and
bare assertion that they went to visit Constantino, who was to leave for Saudi Arabia on
the day they were arrested thereat?

As to petitioner Roque, was it unreasonable for the military authorities to effect her
arrest without warrant considering that it was Buenaobra who provided the leads on
her identity? It cannot be denied that Buenaobra had connection with Roque. Because
the former has the phone number of the latter. Why the necessity of jumbling Roque's
telephone number as written on a piece of paper taken from Buenaobra's possession?
Petitioners Roque and Buenaobra have not offered any plausible reason so far.

In all the above incidents, respondents maintain that they acted reasonably, under the
time, place and circumstances of the events in question, especially considering that at
the time of petitioner's arrest, incriminatory evidence, i.e, firearms, ammunitions and/or
subversive documents were found in their possession.

Petitioners, when arrested, were neither taking their snacks nor innocently visiting a
camp, but were arrested in such time, place and circumstances, from which one can
Page 347 of 377

reasonably conclude tat they were up to a sinister plot, involving utmost secrecy and
comprehensive conspiracy.

IV

In. G.R. No. 85727 (Espiritu vs. Lim), the release on habeas corpus of the petitioner
Deogracias Espiritu, who is detained by virtue of an Information for Violation of Article
142 of the Revised Penal Code (Inciting to Sedition) filed with the Regional Trial Court
of Manila, is similarly not warranted.

The record of the case shows that the said petitioner is the General Secretary of the
Pinagkaisahang Samahan ng Tsuper at Operators Nationwide (PISTON), an association
of drivers and operators of public service vehicles in the Philippines, organized for their
mutual aid and protection.

Petitioner claims that at about 5:00 o'clock in the morning of 23 November 1988, while
he was sleeping in his home located at 363 Valencia St., Sta. Mesa, Manila, he was
awakened by his sister Maria Paz Lalic who told him that a group of persons wanted to
hire his jeepney. When he went down to talk to them, he was immediately put under
arrest. When he asked for the warrant of arrest, the men, headed by Col. Ricardo Reyes,
bodily lifted him and placed him in their owner-type jeepney. He demanded that his
sister, Maria Paz Lalic, be allowed to accompany him, but the men did not accede to his
request and hurriedly sped away.

He was brought to Police Station No. 8 of the Western Police District at Blumentritt,
Manila where he was interrogated and detained. Then, at about 9:00 o'clock of the same
morning, he was brought before the respondent Lim and, there and then, the said
respondent ordered his arrest and detention. He was thereafter brought to the General
Assignment Section, Investigation Division of the Western Police District under Police
Capt. Cresenciano A. Cabasal where he was detained, restrained and deprived of his
liberty. 7

The respondents claim however, that the detention of the petitioner is justified in view
of the Information filed against him before the Regional Trial Court of Manila, docketed
therein as Criminal Case No. 88-683-85, charging him with violation of Art. 142 of the
Revised Penal Code (Inciting to Sedition).
Page 348 of 377

The respondents also claim that the petitioner was lawfully arrested without a judicial
warrant of arrest since petitioner when arrested had in fact just committed an offense
in that in the afternoon of 22 November 1988, during a press conference at the National
Press Club.

Deogracias Espiritu through tri-media was heard urging all drivers and operators to go
on nationwide strike on November 23, 1988, to force the government to give into their
demands to lower the prices of spare parts, commodities, water and the immediate
release from detention of the president of the PISTON (Pinag-isang Samahan ng Tsuper
Operators Nationwide). Further, we heard Deogracias Espiritu taking the place of
PISTON president Medardo Roda and also announced the formation of the Alliance
Drivers Association to go on nationwide strike on November 23, 1988. 8

Policemen waited for petitioner outside the National Pres Club in order to investigate
him, but he gave the lawmen the slip. 9 He was next seen at about 5:00 o'clock that
afternoon at a gathering of drivers and symphatizers at the corner of Magsaysay Blvd.
and Valencia Street, Sta. Mesa, Manila where he was heard to say:

Bukas tuloy ang welga natin, sumagot na ang Cebu at Bicol na kasali sila, at hindi tayo
titigil hanggang hindi binibigay ng gobyerno ni Cory ang gusto nating pagbaba ng halaga
ng spare parts, bilihin at and pagpapalaya sa ating pinuno na si Ka Roda hanggang sa
magkagulo na. 10 (emphasis supplied)

The police finally caught up with the petitioner on 23 November 1988. He was invited
for questioning and brought to police headquarters after which an Information for
violation of Art. 142 of the Revised Penal Code was filed against him before the Regional
Trial Court of Manila. 11

Since the arrest of the petitioner without a warrant was in accordance with the
provisions of Rule 113, Sec. 5(b) of the Rules of Court and that the petitioner is detained
by virtue of a valid information filed with the competent court, he may not be released
on habeas corpus. He may, however be released upon posting bail as recommended.
However, we find the amount of the recommended bail (P60,000.00) excessive and we
reduce it to P10,000.00 only.

VII
Page 349 of 377

In G.R. No. 86332 (Nazareno vs. Station Commander), we also find no merit in the
submission of Narciso Nazareno that he was illegally arrested and is unlawfully
detained. The record of this case shows that at about 8:30 o'clock in the morning of 14
December 1988, one Romulo Bunye II was killed by a group of men near the corner of
T. Molina and Mendiola Streets in Alabang, Muntinglupa, Metro Manila. One of the
suspects in the killing was Ramil Regal who was arrested by the police on 28 December
1988. Upon questioning, Regal pointed to Narciso Nazareno as on of his companions in
the killing of the said Romulo Bunye II. In view thereof, the police officers, without
warrant, picked up Narciso Nazareno and brought him to the police headquarters for
questioning. Obviously, the evidence of petitioner's guilt is strong because on 3 January
1989, an information charging Narciso Nazareno, Ramil Regala, and two (2) others, with
the killing of Romulo Bunye II was filed with the Regional Trial Court of Makati, Metro
Manila. The case is docketed therein as Criminal Case No. 731.

On 7 January 1989, Narciso Nazareno filed a motion to post bail, but the motion was
denied by the trial court in an order dated 10 January 1989, even as the motion to post
bail, earlier filed by his co-accused, Manuel Laureaga, was granted by the same trial
court.

On 13 January 1989, a petition for habeas corpus was filed with this Court on behalf of
Narciso Nazareno and on 13 January 1989, the Court issued the writ of habeas corpus,
returnable to the Presiding Judge of the Regional Trial Court of Biñan, Laguna, Branch
24, ordering said court to hear the case on 30 January 1989 and thereafter resolve the
petition.

At the conclusion of the hearing, or on 1 February 1989, the Presiding Judge of the
Regional Trial Court of Biñan, Laguna issued a resolution denying the petition for
habeas corpus, it appearing that the said Narciso Nazareno is in the custody of the
respondents by reason of an information filed against him with the Regional Trial Court
of Makati, Metro Manila which had taken cognizance of said case and had, in fact,
denied the motion for bail filed by said Narciso Nazareno (presumably because of the
strength of the evidence against him).

The findings of the Presiding Judge of the Regional Trial Court of Biñan, Laguna are
based upon the facts and the law. Consequently, we will not disturb the same. Evidently,
the arrest of Nazareno was effected by the police without warrant pursuant to Sec. 5(b),
Rule 113, Rules of Court after he was positively implicated by his co-accused Ramil
Regala in the killing of Romulo Bunye
II; and after investigation by the police authorities. As held in People vs. Ancheta: 12
Page 350 of 377

The obligation of an agent of authority to make an arrest by reason of a crime, does not
presuppose as a necessary requisite for the fulfillment thereof, the indubitable existence
of a crime. For the detention to be perfectly legal, it is sufficient that the agent or person
in authority making the arrest has reasonably sufficient grounds to believe the existence
of an act having the characteristics of a crime and that the same grounds exist to believe
that the person sought to be detained participated therein.

VIII

It is to be noted that, in all the petitions here considered, criminal charges have been
filed in the proper courts against the petitioners. The rule is, that if a person alleged to
be restrained of his liberty is in the custody of an officer under process issued by a court
judge, and that the court or judge had jurisdiction to issue the process or make the
order, of if such person is charged before any court, the writ of habeas corpus will not
be allowed. Section 4, Rule 102, Rules of Court, as amended is quite explicit in providing
that:

Sec. 4. When writ is allowed or discharge authorized. — If it appears that the person
alleged to be restrained of his liberty is in the custody of an officer under process issued
by a court or judge or by virtue of a judgment or order of a court of record, and that the
court or judge had jurisdiction to issue the process, render the judgment, or make the
order, the writ shall not be allowed; or if the jurisdiction appears after the writ is allowed,
the person shall not be discharged by reason of any informality or defect in the process,
judgment, or order. Nor shall anything in this rule be held to authorize the discharge of
a person charged with a convicted of an offense in the Philippines or of a person suffering
imprisonment under lawful judgment. (emphasis supplied)

At this point, we refer to petitioner's plea for the Court of re-examine and, thereafter,
abandon its pronouncement in Ilagan vs. Enrile, 13 that a writ of habeas corpus is no
longer available after an information is filed against the person detained and a warrant
of arrest or an order of commitment, is issued by the court where said information has
been filed. 14 The petitioners claim that the said ruling, which was handed down during
the past dictatorial regime to enforce and strengthen said regime, has no place under
the present democratic dispensation and collides with the basic, fundamental, and
constitutional rights of the people. Petitioners point out that the said doctrine makes
possible the arrest and detention of innocent persons despite lack of evidence against
them, and, most often, it is only after a petition for habeas corpus is filed before the
court that the military authorities file the criminal information in the courts of law to be
able to hide behind the protective mantle of the said doctrine. This, petitioners assert,
stands as an obstacle to the freedom and liberty of the people and permits lawless and
arbitrary State action.
Page 351 of 377

We find, however, no compelling reason to abandon the said doctrine. It is based upon
express provision of the Rules of Court and the exigencies served by the law. The fears
expressed by the petitioners are not really unremediable. As the Court sees it, re-
examination or reappraisal, with a view to its abandonment, of the Ilagan case doctrine
is not the answer. The answer and the better practice would be, not to limit the function
of the habeas corpus to a mere inquiry as to whether or not the court which issued the
process, judgment or order of commitment or before whom the detained person is
charged, had jurisdiction or not to issue the process, judgment or order or to take
cognizance of the case, but rather, as the Court itself states in Morales, Jr. vs. Enrile,
15 "in all petitions for habeas corpus the court must inquire into every phase and aspect
of petitioner's detention-from the moment petition was taken into custody up to the
moment the court passes upon the merits of the petition;" and "only after such a
scrutiny can the court satisfy itself that the due process clause of our Constitution has
in fact been satisfied." This is exactly what the Court has done in the petitions at bar.
This is what should henceforth be done in all future cases of habeas corpus. In Short,
all cases involving deprivation of individual liberty should be promptly brought to the
courts for their immediate scrutiny and disposition.

WHEREFORE, the petitions are hereby DISMISSED, except that in G.R. No. 85727
(Espiritu vs. Lim), the bail bond for petitioner's provisional liberty is hereby ordered
reduced from P60,000.00 to P10,000.00. No costs.

SO ORDERED.

Fernan C.J., Narvasa, C.J., Melencio-Herrera, Gutierrez, Jr., Paras, Gancayco, Padilla,
Bidin, Griño-Aquino, Medialdea and Regalado, JJ., concur.

Separate Opinions

CRUZ, J., dissenting and concurring:


I dissent insofar as the ponencia affirms the ruling in Garcia-Padilla v. Enrile that
subversion is a continuing offense, to justify the arrest without warrant of any person
at any time as long as the authorities say he has been placed under surveillance on
suspicion of the offense. That is a dangerous doctrine. A person may be arrested when
he is doing the most innocent acts, as when he is only washing his hands, or taking his
supper, or even when he is sleeping, on the ground that he is committing the
"continuing" offense of subversion. Libertarians were appalled when that doctrine was
imposed during the Marcos regime. I am alarmed that even now this new Court is willing
Page 352 of 377

to sustain it. I strongly urge my colleagues to discard it altogether as one of the


disgraceful vestiges of the past dictatorship and uphold the rule guaranteeing the right
of the people against unreasonable searches and seizures. We can do no less if we are
really to reject the past oppression and commit ourselves to the true freedom. Even if it
be argued that the military should be given every support in our fight against
subversion, I maintain that that fight must be waged honorably, in accordance with the
Bill of Rights. I do not believe that in fighting the enemy we must adopt the ways of the
enemy, which are precisely what we are fighting against. I submit that our more
important motivation should be what are we fighting for.

Except for this reservation and appeal, I concur with the decision.

FELICIANO, J., concurring


I concur in the result reached in each of the eight (8) consolidated Petitions for Habeas
Corpus. At the same time, I have some reservations concerning certain statements made
by the Court in G.R. No. 81567 (Umil, et al. v. Ramos) (Part I of the Decision) and in
G.R. No. 85727 (Espiritu v. Lim) (Part VI of the Decision).

In G.R. No. 81567 (Umil, et al. v. Ramos), the per curiam opinion states categorically
that: "the crimes of rebellion, subversion, conspiracy or proposal to commit such crimes,
and crimes or offenses committed in furtherance thereof or in connection therewith
constitute direct assaults against the State and are in the nature of continuing crimes."
The majority here relies upon Garcia-Padilla v. Enrile (121 SCRA 472 [1983]). The
majority there made the same equally broad statement but without any visible effort to
examine the basis, scope and meaning of such a sweeping statement. Garcia-Padilla did
not even identify the specific offenses which it regarded as "in the nature of continuing
offenses which set them apart from the common offenses" (121 SCRA at 489). It appears
to me that in G.R. No. 85727 (Espiritu v. Lim) (Part VI of the Decision), the per curiam
opinion has in effect included the offense of "inciting to sedition" penalized under Article
142 of the Revised Penal Code as a "continuing offense" under the capacious blanket of
the majority opinion in Garcia-Padilla, at least for purposes of determining the legality
of the arrest without a warrant of petitioner Deogracias Espiritu.

I would respectfully recall to my learned colleagues in the Court that "inciting to


sedition" is defined in Article 142 of the Revised Penal Code in terms of speech 1 and
that consequently it is important constantly do distinguish between speech which is
protected by the constitutional guaranty of freedom of speech and of the press and
speech which may constitutionally be regarded as violative of Article 142 of the Revised
Penal Code. Precisely because speech which the police authorities might regard as
Page 353 of 377

seditious or as criminal inciting to sedition may well turn out to be only an exercise of
a constitutionally guaranteed freedom, I would submit that we must apply the concept
of "continuing offense" narrowly for purposes of application of Section 5(b), Rule 113 of
the Revised Rules of Court.

In my view, the very broad statement made about "continuing crimes" in G.R. No. 81567
(Umil, et al v. Ramos) constitutes dictum, considering that Rolando Dural and Bernardo
Itucal, Jr. had already been tried in the court below for "double murder, etc." and found
guilty of the offense charged, sentenced accordingly, and at least in the case of Rolando
Dural, service of the sentence imposed upon him by the trial court had already begun.

Similarly, in G.R. No. 85727 (Espiritu v. Lim) the statement that the arrest of petitioner
Espiritu without a warrant was in accordance with the provisions of Section 5(b), Rule
113 of the Revised Rules of Court does not appear strictly necessary, considering that
the petitioner had already been charged in a valid information filed with the competent
court, which court had presumably issued an order for his commitment, and
considering further that he is entitled to bail.

There is thus no obstacle, to my mind, to a careful examination of the doctrine of


"continuing crimes" as applied to such offenses as subversion and inciting to sedition
and possibly other offenses, in some future case where that issue is raised squarely and
is unavoidable.

Cortes, J., concurs.

SARMIENTO, J., dissenting:


I beg to differ from my brethren. I submit that habeas corpus lies in all eight cases.

G.R. No. 81567

The majority says that Rolando Dural's arrest without a warrant is lawful under the
Rules of Court, which reads:

Sec. 5. Arrest without warrant; when lawful. — A peace officer or a private person may,
without a warrant, arrest a person:
Page 354 of 377

(a) When, in his presence, the person to be arrested has committed, is actually
committing, or is attempting to commit an offense;

(b) When an offense has in fact just been committed, and he has personal knowledge of
facts indicating that the person to be arrested has committed it; and

(c) When the person to be arrested is a prisoner who has escaped from a penal
establishment or place where he is serving final judgment or temporarily confined while
his case is pending, or has escaped while being transferred from one confinement to
another.

In cases falling under paragraphs (a) and (b) hereof, the person arrested without a
warrant shall be forthwith delivered to the nearest police station or jail, and he shall be
proceeded against in accordance with Rule 112, Section 7. 1

"Rolando Dural," so states the majority, "was arrested for being a member of the New
People's Army (NPA), an outlawed subversive organization," 2 and that "[s]ubversion
being a continuing offense, the arrest of Rolando Dural without a warrant is justified as
it can be said that he was committing an offense when arrested." 3

As I said, I beg to differ.

First, Rolando Dural was charged with "Double Murder with Assault upon Agents of
Authority." 4 If he had been guilty of subversion — the offense for which he was
supposedly arrested via a warrantless arrest — subversion was the logical crime with
which he should have been charged.

The authorities could not have rightly arrested him for subversion on account of the
slay of the two CAPCOM soldiers, a possible basis for violation of the Anti-Subversion
Act, because as the majority points out, "he was not arrested while in the act of shooting
[them] . . . [n]or was he arrested just after the commission of the said offense for his
arrest came a day after the said shooting incident." 5

Second, I do not believe that a warrantless (or citizen's) arrest is possible in case of
subversion — in the absence of any overt act that would justify the authorities to act.
Page 355 of 377

"Subversion," as the term is known in law, means "knowingly, wilfully and by overt acts
affiliat[ing] [oneself] with, becom[ing] or remain[ing] a member of the Communist Party
of the Philippines and/or its successor or of any subversion association as defined in
sections two and three hereof. . . . " 6 Logically, the military could not have known that
Dural, at the time he was taken, was a member of the New People's Army because he
was not performing any over act that he was truly, a rebel. Indeed, it had to take a
"verification"6 before he could be identified as allegedly a member of the underground
army. Under these circumstances, I am hard put to say that he was committing
subversion when he was arrested, assuming that he was guilty of subversion, for
purposes of a warrantless arrest.

"Overt act" is made up of "[e]very act, movement, deed and word of the [accused]," 7
indicating intent to accomplish a criminal objective. Dural, at the time he was arrested,
was lying in a hospital bed. This is not the overt act contemplated by law.

Under the Rule above-quoted, the person must have either been apprehended in
flagranti (first paragraph) or after the act, provided that the peace officer has "personal
knowledge" that he, the suspect, is guilty. (second paragraph.) As I stated, Dural was
not caught in the act. Moreover, what the Regional Intelligence Operations Unit of the
Capital Command (RIOU-CAPCOM) had in its hands was a mere "confidential
information." I do not think that this is the personal knowledge referred to by the second
paragraph. 8 Plainly and simply, it is hearsay.

The rule, furthermore, on warrantless arrest is an exceptional one. By its language, it


may be exercised only in the most urgent cases and when the guilt of an offender is
plain and evident. What I think we have here is purely and simply, the military taking
the law in its hands.

By stamping validity to Rolando Dural's warrantless arrest, I am afraid that the majority
has set a very dangerous precedent. With all due respect, my brethren has accorded the
military a blanket authority to pick up any Juan, Pedro, and Maria without a warrant
for the simple reason that subversion is supposed to be a continuing offense.

That Rolando Dural was arrested for being a member of the New People's Army" 9 is
furthermore to me, a hasty statement. It has yet to be established that Dural is indeed
a member of the Communist Party's military arm. And unless proven guilty, he is
presumed, and must be presumed most of all by this Court, to be innocent.
Page 356 of 377

The majority also says that habeas corpus is moot and academic because Dural has
been convicted and is serving sentence. I likewise take exception. It has been held that:
"The writ may be granted upon a judgment already final." 10

The writ of liberty is a high prerogative writ. 11 Vindication of due process is its historic
office. 12

G.R. Nos. 84581-82

In the case of Wilfredo Buenaobra, the majority avers that he had "manifested his desire
to stay in the PC-INP stockade," 13 for which habeas corpus has supposedly become
moot and academic. I am not convinced that that is reason enough to dismiss habeas
corpus as moot and academic. It is the duty of this Court, in my opinion, to make sure
that Buenaobra has made his choice freely and voluntarily. Personally, I find it indeed
strange why he should prefer to stay in jail than go scot-free.

There is further no doubt that Buenaobra's petition is one impressed with a public
interest. In one case 14 we denied a motion to withdraw a petition for habeas corpus in
view of its far-reaching importance to the motion, I do not see how we should act
differently, perhaps even insouciantly, here, especially since it involves persons who
think and believe differently from the rest of us.

Both Buenaobra and Amelia Roque supposedly admitted that they were ranking officers
of the Communist Party of the Philippines. According to the majority, Buenaobra and
Roque are bound by their admissions. 15

That both parties had admitted to be members of the Communist Party of the Philippines
(the National United Front Commission) is a naked contention of the military. The fact
that it has not been controverted, in my view, does not justify the couple's arrest without
warrant. Worse, by relying on the bare word of the military, this very Court has, to all
intents and purposes, condemned the duo for a crime (subversion and/or illegal
possession of firearms) the bone of contention, precisely, below.

G.R. Nos. 84583-84

I also find the warrantless arrests of Domingo Añonuevo and Ramon Casiple to be
contrary to law. That they are "admittedly members of the standing committee of the
Page 357 of 377

NUFC" 16 and that "subversive materials" 17 and unlicensed firearms were found in
their possession, are, like Buenaobra's and Roque's cases, barren claims of the military.
I also fear that by the majority's strong language (that Añonuevo and Casiple are
admitted NUCF officers) the majority has pronounced the petitioners guilty, when the
lower courts have yet to sit in judgment. I think we should be the last to preempt the
decision of the trial courts. We would have set to naught the presumption of innocence
accused persons enjoy.

G.R. No. 83162

With respect to the case of Vicky Ocaya, I am afraid that I am inclined towards the same
conclusion. There was basis — at the outset — to say that Ocaya was probably guilty of
illegal possession of firearms. As I have observed, a warrantless arrest must be
predicated upon the existence of a crime being actually committed or having been
committed. What I find here, rather, is nothing less than a successful fishing expedition
conducted by the military upon an unwary citizen. I am quite distressed to note that
this is still possible under a supposed democracy.

G.R. No. 85727

Deogracias Espiritu was fast asleep in his house when he was placed under arrest. For
the life of me, I can not figure out how one can be picked upon in one's own home and
held moments later without a warrant of arrest.

Espiritu was allegedly guilty of inciting to sedition as a result of a speech delivered in a


press conference at the National Press Club on November 21, 1988. He was, however,
arrested the day after, November 22, 1988. Under these circumstances, it eludes me
how an arrest without a warrant could be justified, either under paragraph (a) or
paragraph (b) of the Rule on warrantless arrests.

The majority avers that since an information had been filed with the court, Espiritu's
detention, is allegedly justifiable. The question is whether or not an information is an
authority to hold a person in custody. Under the Rules, an information means "an
accusation in writing charging a person with an offense subscribed by the fiscal and
filed with the court." 18 It is not, however, an order to keep one under detention.

G.R. No. 86332


Page 358 of 377

The offense for which Narciso Nazareno is being held — the fatal shooting of Romulo
Bunye II — was committed on December 14, 1988. It was, however, only on December
28, 1988 that the police collared a suspect, Ramil Regala, who subsequently pointed to
Nazareno as his accomplice. It also escapes me how Nazareno, under these
circumstances, could have been validly put under arrest without a warrant or the
existence of the circumstance described under either paragraph (a) or (b) of the Rule
above-quoted: The crime had long been committed prior to the arrest.

G.R. Nos. 81567; 84581-82; 84583-84; 83162;


85727 & 86332; Postscripts

The majority has disposed of these cases on the bedrock of what I view as doctrines that
have lost their luster:

1. The teaching of Garcia-Padilla v. Enrile, 19 which held that subversion is a continuing


offense;

2. The ruling in Ilagan v. Enrile. 20

I also find, for reasons to be set forth hereinafter, a glossing over of the fundamental
rights of the petitioners under the Constitution in the authorities' handling of the
petitioners' cases.

I hold that Garcia-Padilla is no longer good law under the present Constitution. Two
reasons persuade me. First, it is repugnant to due process of law. ("The arrest, therefore,
need not follow the usual procedure in the prosecution of offenses which require the
determination by a judge of the existence of probable cause before the issuance of a
judicial warrant of arrest and the granting of bail if the offense is bailable." 21 Under
the 1987 Constitution, not even "[a] state of martial law suspend[s] the operation of [the
Charter]. . ." 22 Second, it leaves the liberty of citizens to the whim of one man ("On
these occasions [the existence of a state of emergency], the President takes absolute
command, for the very life of the Nation and its government, which, incidentally,
includes the courts, is in grave peril. In so doing, the President is answerable only to his
conscience, the people and to God. For their part, in giving him the supreme mandate
as their President, the people can only trust and pray that, giving him their own loyalty
and without patriotism, the President will not fail them." 23 ) Under the Charter now
Page 359 of 377

prevailing, the Chief Executive shares, to a certain extent, the exercise of emergency
powers, with Congress. 24

As a law advocate under the regime of Marcos, I had challenged the soundness of Garcia-
Padilla. I doubted whether it could stand up under the aegis of the 1973 Constitution.
I still doubt whether it can withstand scrutiny under the 1987 Constitution.

The majority also fails to point out that six days after Garcia-Padilla was handed down,
the Court promulgated Morales, Jr. v. Enrile, 25 a case that in my view has significantly
whittled down Garcia-Padilla's very esse. In that case, Mr. Justice Hermogenes
Concepcion, Jr. wrote for the majority:

xxx xxx xxx

16. After a person is arrested . . . without a warrant . . . the proper complaint or


information against him must be filed with the courts of justice within the time
prescribed by law. . .

17. Failure of the public officer to do so without any valid reason would constitute a
violation of Art. 125, Revised Penal Code, as amended. And the person detained would
be entitled to be released on a writ of habeas corpus, unless he is detained under
subsisting process issued by a competent court. 26

I also gather from the records that none of the petitioners had been: (1) informed of their
right to remain silent; and (2) to have competent and independent counsel. 27

As I said, the majority is denying habeas corpus on self-serving claims of the military
that the petitioners (Dural, Buenaobra, Roque, Añonuevo, and Casiple) are members of
the Communist Party of the Philippines — and that they have supposedly confessed to
be in fact members of the outlawed organization. The question that has not been
answered is whether or not these supposed confessions are admissible, for purposes of
a warrantless arrest, as evidence of guilt, in the absence of any showing that they were
apprised of their constitutional rights. I am perturbed by the silence of the majority. I
am distressed because as we held in one case, violation of the Constitution divests the
court of jurisdiction and entitles the accused to habeas corpus. 28

According to the majority, a "re-examination or re-appraisal . . . of


Page 360 of 377

the Ilagan doctrine is not the answer." 29 In my considered opinion, Ilagan v. Enrile 30
does not rightfully belong in the volumes of Philippine jurisprudence. In that case, the
petitioners, three Davao-based lawyers, were held by virtue of a simple information ("the
petition herein has been rendered moot and academic by virtue of the filing of an
Information against them for Rebellion . . . and the issuance of a Warrant of Arrest
against them" 31 ) without any preliminary investigation (examination) having been
previously conducted (to justify the issuance of a warrant).i•t•c-aüsl As I have stated,
an information is not a warrant of arrest. The fact that an information exists does not
mean that a warrant will be issued.

Accused persons have the right of preliminary investigation (examination). 32 It forms


part and parcel of due process of law .33

I find the majority's reliance on U.S. v. Wilson, 34 an ancient (1905) decision, inapt and
untenable. In that case, the accused had been served with a warrant and thereafter
taken into custody. The question that faced the Court was whether or not the warrant
was valid, amid the accused's charges that the judge who issued it did not examine the
complainant under oath. We held that the query was academic, because the accused
had already pleaded, and the case had entered the trial stage.

The cases at bar are not on all fours. Here, no warrant has been issued. I submit that
in that event, the petitioners are entitled to freedom by way of the writ of liberty.

xxx xxx xxx

The apprehensions in question chronicle in my mind the increasing pattern of arrests


and detention in the country without the sanction of a judicial decree. Four years ago
at "EDSA", and many years before it, although with much fewer of us, we valiantly
challenged a dictator and all the evils his regime had stood for: repression of civil
liberties and trampling on of human rights. We set up a popular government, restored
its honored institutions, and crafted a democratic constitution that rests on the
guideposts of peace and freedom. I feel that with this Court's ruling, we have frittered
away, by a stroke of the pen, what we had so painstakingly built in four years of
democracy, and almost twenty years of struggle against tyranny.

It also occurs to me that I am interposing what looms as a quixotic outlook of Philippine


law on warrantless arrests and its implications on liberty. It is an impression that does
not surprise me. Quixotic as they may seem, and modesty aside, my views reflect a
strong bias on my part — forged by years of experience and sharpened by a painful and
Page 361 of 377

lonely struggle for freedom and justice — toward men and women who challenge settled
beliefs. If this dissent can not gain any adherent for now, let it nevertheless go on record
as a plea to posterity and an appeal for tolerance of opinions with which we not only
disagree, but opinions we loathe.

I feel it is my duty to articulate this dissent.

\
Page 362 of 377

G.R. No. 101837 February 11, 1992

ROLITO GO y TAMBUNTING, petitioner, vs. THE COURT OF APPEALS, THE


HON. BENJAMIN V. PELAYO, Presiding Judge, Branch 168, Regional Trial
Court, NCJR Pasig, M.M., and PEOPLE OF THE PHILIPPINES, respondents.

FELICIANO, J.:

According to the findings of the San Juan Police in their Investigation Report, 1 on 2
July 1991, Eldon Maguan was driving his car along Wilson St., San Juan, Metro Manila,
heading towards P. Guevarra St. Petitioner entered Wilson St., where it is a one-way
street and started travelling in the opposite or "wrong" direction. At the corner of Wilson
and J. Abad Santos Sts., petitioner's and Maguan's cars nearly bumped each other.
Petitioner alighted from his car, walked over and shot Maguan inside his car. Petitioner
then boarded his car and left the scene. A security guard at a nearby restaurant was
able to take down petitioner's car plate number. The police arrived shortly thereafter at
the scene of the shooting and there retrieved an empty shell and one round of live
ammunition for a 9 mm caliber pistol. Verification at the Land Transportation Office
showed that the car was registered to one Elsa Ang Go.

The following day, the police returned to the scene of the shooting to find out where the
suspect had come from; they were informed that petitioner had dined at Cravings Bake
Shop shortly before the shooting. The police obtained a facsimile or impression of the
credit card used by petitioner from the cashier of the bake shop. The security guard of
the bake shop was shown a picture of petitioner and he positively identified him as the
same person who had shot Maguan. Having established that the assailant was probably
the petitioner, the police launched a manhunt for petitioner.

On 8 July 1991, petitioner presented himself before the San Juan Police Station to verify
news reports that he was being hunted by the police; he was accompanied by two (2)
lawyers. The police forthwith detained him. An eyewitness to the shooting, who was at
the police station at that time, positively identified petitioner as the gunman. That same
day, the police promptly filed a complaint for frustrated homicide 2 against petitioner
with the Office of the Provincial Prosecutor of Rizal. First Assistant Provincial Prosecutor
Dennis Villa Ignacio ("Prosecutor") informed petitioner, in the presence of his lawyers,
that he could avail himself of his right to preliminary investigation but that he must first
sign a waiver of the provisions of Article 125 of the Revised Penal Code. Petitioner
refused to execute any such waiver.
Page 363 of 377

On 9 July 1991, while the complaint was still with the Prosecutor, and before an
information could be filed in court, the victim, Eldon Maguan, died of his gunshot
wound(s).

Accordingly, on 11 July 1991, the Prosecutor, instead of filing an information for


frustrated homicide, filed an information for murder 3 before the Regional Trial Court.
No bail was recommended. At the bottom of the information, the Prosecutor certified
that no preliminary investigation had been conducted because the accused did not
execute and sign a waiver of the provisions of Article 125 of the Revised Penal Code.

In the afternoon of the same day, 11 July 1991, counsel for petitioner filed with the
Prosecutor an omnibus motion for immediate release and proper preliminary
investigation,4 alleging that the warrantless arrest of petitioner was unlawful and that
no preliminary investigation had been conducted before the information was filed.
Petitioner also prayed that he be released on recognizance or on bail. Provincial
Prosecutor Mauro Castro, acting on the omnibus motion, wrote on the last page of the
motion itself that he interposed no objection to petitioner being granted provisional
liberty on a cash bond of P100,000.00.

On 12 July 1991, petitioner filed an urgent ex-parte motion for special raffle 5 in order
to expedite action on the Prosecutor's bail recommendation. The case was raffled to the
sala of respondent Judge, who, on the same date, approved the cash bond 6 posted by
petitioner and ordered his release. 7 Petitioner was in fact released that same day.

On 16 July 1991, the Prosecutor filed with the Regional Trial Court a motion for leave
to conduct preliminary investigation8 and prayed that in the meantime all proceedings
in the court be suspended. He stated that petitioner had filed before the Office of the
Provincial Prosecutor of Rizal an omnibus motion for immediate release and preliminary
investigation, which motion had been granted by Provincial Prosecutor Mauro Castro,
who also agreed to recommend cash bail of P100,000.00. The Prosecutor attached to
the motion for leave a copy of petitioner's omnibus motion of 11 July 1991.

Also on 16 July 1991, the trial court issued an Order 9 granting leave to conduct
preliminary investigation and cancelling the arraignment set for 15 August 1991 until
after the prosecution shall have concluded its preliminary investigation.

On 17 July 1991, however, respondent Judge motu proprio issued an Order, 10


embodying the following: (1) the 12 July 1991 Order which granted bail was recalled;
petitioner was given 48 hours from receipt of the Order to surrender himself; (2) the 16
Page 364 of 377

July 1991 Order which granted leave to the prosecutor to conduct preliminary
investigation was recalled and cancelled; (3) petitioner's omnibus motion for immediate
release and preliminary investigation dated 11 July 1991 was treated as a petition for
bail and set for hearing on 23 July 1991.

On 19 July 1991, petitioner filed a petition for certiorari, prohibition and mandamus
before the Supreme Court assailing the 17 July 1991 Order, contending that the
information was null and void because no preliminary investigation had been previously
conducted, in violation of his right to due process. Petitioner also moved for suspension
of all proceedings in the case pending resolution by the Supreme Court of his petition;
this motion was, however, denied by respondent Judge.

On 23 July 1991, petitioner surrendered to the police.

By a Resolution dated 24 July 1991, this Court remanded the petition for certiorari,
prohibition and mandamus to the Court of Appeals.

On 16 August 1991, respondent Judge issued an order in open court setting the
arraignment of petitioner on 23 August 1991.

On 19 August 1991, petitioner filed with the Court of Appeals a motion to restrain his
arraignment.

On 23 August 1991, respondent judge issued a Commitment Order directing the


Provincial Warden of Rizal to admit petitioner into his custody at the Rizal Provincial
Jail. On the same date, petitioner was arraigned. In view, however, of his refusal to enter
a plea, the trial court entered for him a plea of not guilty. The Trial court then set the
criminal case for continuous hearings on 19, 24 and 26 September; on 2, 3, 11 and 17
October; and on 7, 8, 14, 15, 21 and 22 November 1991. 11

On 27 August 1991, petitioner filed a petition for habeas corpus 12 in the Court of
Appeals. He alleged that in view of public respondent's failure to join issues in the
petition for certiorari earlier filed by him, after the lapse of more than a month, thus
prolonging his detention, he was entitled to be released on habeas corpus.
Page 365 of 377

On 30 August 1991, the Court of Appeals issued the writ of habeas corpus. 13 The
petition for certiorari, prohibition and mandamus, on the one hand, and the petition for
habeas corpus, upon the other, were subsequently consolidated in the Court of Appeals.

The Court of Appeals, on 2 September 1991, issued a resolution denying petitioner's


motion to restrain his arraignment on the ground that that motion had become moot
and academic.

On 19 September 1991, trial of the criminal case commenced and the prosecution
presented its first witness.

On 23 September 1991, the Court of Appeals rendered a consolidated decision 14


dismissing the two (2) petitions, on the following grounds:

a. Petitioner's warrantless arrest was valid because the offense for which he was arrested
and charged had been "freshly committed." His identity had been established through
investigation. At the time he showed up at the police station, there had been an existing
manhunt for him. During the confrontation at the San Juan Police Station, one witness
positively identified petitioner as the culprit.

b. Petitioner's act of posting bail constituted waiver of any irregularity attending his
arrest. He waived his right to preliminary investigation by not invoking it properly and
seasonably under the Rules.

c. The trial court did not abuse its discretion when it issued the 17 July 1991 Order
because the trial court had the inherent power to amend and control its processes so as
to make them conformable to law and justice.

d. Since there was a valid information for murder against petitioner and a valid
commitment order (issued by the trial judge after petitioner surrendered to the
authorities whereby petitioner was given to the custody of the Provincial Warden), the
petition for habeas corpus could not be granted.

On 3 October 1991, the prosecution presented three (3) more witnesses at the trial.
Counsel for petitioner also filed a "Withdrawal of Appearance" 15 with the trial court,
with petitioner's conformity.
Page 366 of 377

On 4 October 1991, the present Petition for Review on Certiorari was filed. On 14
October 1991, the Court issued a Resolution directing respondent Judge to hold in
abeyance the hearing of the criminal case below until further orders from this Court.

In this Petition for Review, two (2) principal issues need to be addressed: first, whether
or not a lawful warrantless arrest had been effected by the San Juan Police in respect
of petitioner Go; and second, whether petitioner had effectively waived his right to
preliminary investigation. We consider these issues seriatim.

In respect of the first issue, the Solicitor General argues that under the facts of the case,
petitioner had been validly arrested without warrant. Since petitioner's identity as the
gunman who had shot Eldon Maguan on 2 July 1991 had been sufficiently established
by police work, petitioner was validly arrested six (6) days later at the San Juan Police
Station. The Solicitor General invokes Nazareno v. Station Commander, etc., et al., 16
one of the seven (7) cases consolidated with In the Matter of the Petition for Habeas
Corpus of Roberto Umil, etc., v. Ramos, et al. 17 where a majority of the Court upheld
a warrantees arrest as valid although effected fourteen (14) days after the killing in
connection with which Nazareno had been arrested. Accordingly, in the view of the
Solicitor General, the provisions of Section 7, Rule 112 of the Rules of Court were
applicable and because petitioner had declined to waive the provisions of Article 125 of
the Revised Penal Code, the Prosecutor was legally justified in filing the information for
murder even without preliminary investigation.

On the other hand, petitioner argues that he was not lawfully arrested without warrant
because he went to the police station six (6) days after the shooting which he had
allegedly perpetrated. Thus, petitioner argues, the crime had not been "just committed"
at the time that he was arrested. Moreover, none of the police officers who arrested him
had been an eyewitness to the shooting of Maguan and accordingly none had the
"personal knowledge" required for the lawfulness of a warrantees arrest. Since there had
been no lawful warrantless arrest. Section 7, Rule 112 of the Rules of Court which
establishes the only exception to the right to preliminary investigation, could not apply
in respect of petitioner.

The reliance of both petitioner and the Solicitor General upon Umil v. Ramos is, in the
circumstances of this case, misplaced. In Umil v. Ramos, by an eight-to-six vote, the
Court sustained the legality of the warrantless arrests of petitioners made from one (1)
to fourteen days after the actual commission of the offenses, upon the ground that such
offenses constituted "continuing crimes." Those offenses were subversion, membership
in an outlawed organization like the New People's Army, etc. In the instant case, the
offense for which petitioner was arrested was murder, an offense which was obviously
Page 367 of 377

commenced and completed at one definite location in time and space. No one had
pretended that the fatal shooting of Maguan was a "continuing crime."

Secondly, we do not believe that the warrantees "arrest" or detention of petitioner in the
instant case falls within the terms of Section 5 of Rule 113 of the 1985 Rules on Criminal
Procedure which provides as follows:

Sec. 5 Arrest without warrant; when lawful. — A peace officer or a private person may,
without warrant, arrest a person:

(a) When, in his presence, the person to be arrested has committed, is actually
committing, or is attempting to commit an offense;

(b) When an offense has in fact just been committed, and he has personal knowledge of
facts indicating that the person to be arrested has committed it; and

(c) When the person to be arrested is a prisoner who has escaped from a penal
establishment or place where he is serving final judgment or temporarily confined while
his case is pending, or has escaped while being transferred from one confinement to
another.

In cases falling under paragraphs (a) and (b) hereof, the person arrested without a
warrant shall be forthwith delivered to the nearest police station or jail, and he shall be
proceed against in accordance with Rule 112, Section 7.

Petitioner's "arrest" took place six (6) days after the shooting of Maguan. The "arresting"
officers obviously were not present, within the meaning of Section 5(a), at the time
petitioner had allegedly shot Maguan. Neither could the "arrest" effected six (6) days
after the shooting be reasonably regarded as effected "when [the shooting had] in fact
just been committed" within the meaning of Section 5(b). Moreover, none of the
"arresting" officers had any "personal knowledge" of facts indicating that petitioner was
the gunman who had shot Maguan. The information upon which the police acted had
been derived from statements made by alleged eyewitnesses to the shooting — one
stated that petitioner was the gunman; another was able to take down the alleged
gunman's car's plate number which turned out to be registered in petitioner's wife's
name. That information did not, however, constitute "personal knowledge." 18
Page 368 of 377

It is thus clear to the Court that there was no lawful warrantless arrest of petitioner
within the meaning of Section 5 of Rule 113. It is clear too that Section 7 of Rule 112,
which provides:

Sec. 7 When accused lawfully arrested without warrant. — When a person is lawfully
arrested without a warrant for an offense cognizable by the Regional Trial Court the
complaint or information may be filed by the offended party, peace officer or fiscal
without a preliminary investigation having been first conducted, on the basis of the
affidavit of the offended party or arresting office or person

However, before the filing of such complaint or information, the person arrested may
ask for a preliminary investigation by a proper officer in accordance with this Rule, but
he must sign a waiver of the provisions of Article 125 of the Revised Penal Code, as
amended, with the assistance of a lawyer and in case of non-availability of a lawyer, a
responsible person of his choice. Notwithstanding such waiver, he may apply for bail as
provided in the corresponding rule and the investigation must be terminated within
fifteen (15) days from its inception.

If the case has been filed in court without a preliminary investigation having been first
conducted, the accused may within five (5) days from the time he learns of the filing of
the information, ask for a preliminary investigation with the same right to adduce
evidence in his favor in the manner prescribed in this Rule. (Emphasis supplied)

is also not applicable. Indeed, petitioner was not arrested at all. When he walked into
San Juan Police Station, accompanied by two (2) lawyers, he in fact placed himself at
the disposal of the police authorities. He did not state that he was "surrendering"
himself, in all probability to avoid the implication he was admitting that he had slain
Eldon Maguan or that he was otherwise guilty of a crime. When the police filed a
complaint for frustrated homicide with the Prosecutor, the latter should have
immediately scheduled a preliminary investigation to determine whether there was
probable cause for charging petitioner in court for the killing of Eldon Maguan. Instead,
as noted earlier, the Prosecutor proceed under the erroneous supposition that Section
7 of Rule 112 was applicable and required petitioner to waive the provisions of Article
125 of the Revised Penal Code as a condition for carrying out a preliminary investigation.
This was substantive error, for petitioner was entitled to a preliminary investigation and
that right should have been accorded him without any conditions. Moreover, since
petitioner had not been arrested, with or without a warrant, he was also entitled to be
released forthwith subject only to his appearing at the preliminary investigation.
Page 369 of 377

Turning to the second issue of whether or not petitioner had waived his right to
preliminary investigation, we note that petitioner had from the very beginning demanded
that a preliminary investigation be conducted. As earlier pointed out, on the same day
that the information for murder was filed with the Regional Trial Court, petitioner filed
with the Prosecutor an omnibus motion for immediate release and preliminary
investigation. The Solicitor General contends that that omnibus motion should have
been filed with the trial court and not with the Prosecutor, and that the petitioner should
accordingly be held to have waived his right to preliminary investigation. We do not
believe that waiver of petitioner's statutory right to preliminary investigation may be
predicated on such a slim basis. The preliminary investigation was to be conducted by
the Prosecutor, not by the Regional Trial Court. It is true that at the time of filing of
petitioner's omnibus motion, the information for murder had already been filed with the
Regional Trial Court: it is not clear from the record whether petitioner was aware of this
fact at the time his omnibus motion was actually filed with the Prosecutor. In Crespo v.
Mogul, 19 this Court held:

The preliminary investigation conducted by the fiscal for the purpose of determining
whether a prima facie case exists to warranting the prosecution of the accused is
terminated upon the filing of the information in the proper court. In turn, as above
stated, the filing of said information sets in motion the criminal action against the
accused in Court. Should the fiscal find it proper to conduct a reinvestigation of the
case, at such stage, the permission of the Court must be secured. After such
reinvestigation the finding and recommendations of the fiscal should be submitted to
the Court for appropriate action. While it is true that the fiscal has the quasi-judicial
discretion to determine whether or not a criminal case should be filed in court or not,
once the case had already been brought to Court whatever disposition the fiscal may
feel should be proper in the case thereafter should be addressed for the consideration
of the Court. The only qualification is that the action of the Court must not impair the
substantial rights of the accused., or the right of the People to due process of law.

xxx xxx xxx

The rule therefore in this jurisdiction is that once a complaint or information is filed in
Court any disposition of the case [such] as its dismissal or the conviction or acquittal of
the accused rests in the sound discretion of the Court. Although the fiscal retains the
direction and control of the prosecution of criminal cases even while the case is already
in Court he cannot impose his opinion on the trial court. The Court is the best and sole
judge on what to do with the case before it. . . . 20 (Citations omitted; emphasis supplied)

Nonetheless, since petitioner in his omnibus motion was asking for preliminary
investigation and not for a re-investigation (Crespo v. Mogul involved a re-investigation),
Page 370 of 377

and since the Prosecutor himself did file with the trial court, on the 5th day after filing
the information for murder, a motion for leave to conduct preliminary investigation
(attaching to his motion a copy of petitioner's omnibus motion), we conclude that
petitioner's omnibus motion was in effect filed with the trial court. What was crystal
clear was that petitioner did ask for a preliminary investigation on the very day that the
information was filed without such preliminary investigation, and that the trial court
was five (5) days later apprised of the desire of the petitioner for such preliminary
investigation. Finally, the trial court did in fact grant the Prosecutor's prayer for leave
to conduct preliminary investigation. Thus, even on the (mistaken) supposition
apparently made by the Prosecutor that Section 7 of Rule 112 of the Revised Court was
applicable, the 5-day reglementary period in Section 7, Rule 112 must be held to have
been substantially complied with.

We believe and so hold that petitioner did not waive his right to a preliminary
investigation. While that right is statutory rather than constitutional in its fundament,
since it has in fact been established by statute, it is a component part of due process in
criminal justice. 21 The right to have a preliminary investigation conducted before being
bound over to trial for a criminal offense and hence formally at risk of incarceration or
some other penalty, is not a mere formal or technical right; it is a substantive right. The
accused in a criminal trial is inevitably exposed to prolonged anxiety, aggravation,
humiliation, not to speak of expense; the right to an opportunity to avoid a process
painful to any one save, perhaps, to hardened criminals, is a valuable right. To deny
petitioner's claim to a preliminary investigation would be to deprive him the full measure
of his right to due process.

The question may be raised whether petitioner still retains his right to a preliminary
investigation in the instant case considering that he was already arraigned on 23 August
1991. The rule is that the right to preliminary investigation is waived when the accused
fails to invoke it before or at the time of entering a plea at arraignment. 22 In the instant
case, petitioner Go had vigorously insisted on his right to preliminary investigation
before his arraignment. At the time of his arraignment, petitioner was already before the
Court of Appeals on certiorari, prohibition and mandamus precisely asking for a
preliminary investigation before being forced to stand trial.

Again, in the circumstances of this case, we do not believe that by posting bail petitioner
had waived his right to preliminary investigation. In People v. Selfaison, 23 we did hold
that appellants there had waived their right to preliminary investigation because
immediately after their arrest, they filed bail and proceeded to trial "without previously
claiming that they did not have the benefit of a preliminary investigation." 24 In the
instant case, petitioner Go asked for release on recognizance or on bail and for
preliminary investigation in one omnibus motion. He had thus claimed his right to
preliminary investigation before respondent Judge approved the cash bond posted by
petitioner and ordered his release on 12 July 1991. Accordingly, we cannot reasonably
Page 371 of 377

imply waiver of preliminary investigation on the part of petitioner. In fact, when the
Prosecutor filed a motion in court asking for leave to conduct preliminary investigation,
he clearly if impliedly recognized that petitioner's claim to preliminary investigation was
a legitimate one.

We would clarify, however, that contrary to petitioner's contention the failure to accord
preliminary investigation, while constituting a denial of the appropriate and full
measure of the statutory process of criminal justice, did not impair the validity of the
information for murder nor affect the jurisdiction of the trial court. 25

It must also be recalled that the Prosecutor had actually agreed that petitioner was
entitled to bail. This was equivalent to an acknowledgment on the part of the Prosecutor
that the evidence of guilt then in his hands was not strong. Accordingly, we consider
that the 17 July 1991 order of respondent Judge recalling his own order granting bail
and requiring petitioner to surrender himself within forty-eight (48) hours from notice,
was plainly arbitrary considering that no evidence at all — and certainly no new or
additional evidence — had been submitted to respondent Judge that could have justified
the recall of his order issued just five (5) days before. It follows that petitioner was
entitled to be released on bail as a matter of right.

The final question which the Court must face is this: how does the fact that, in the
instant case, trial on the merits has already commenced, the Prosecutor having already
presented four (4) witnesses, impact upon, firstly, petitioner's right to a preliminary
investigation and, secondly, petitioner's right to be released on bail? Does he continue
to be entitled to have a preliminary investigation conducted in respect of the charge
against him? Does petitioner remain entitled to be released on bail?

Turning first to the matter of preliminary investigation, we consider that petitioner


remains entitled to a preliminary investigation although trial on the merits has already
began. Trial on the merits should be suspended or held in abeyance and a preliminary
investigation forthwith accorded to petitioner. 26 It is true that the Prosecutor might, in
view of the evidence that he may at this time have on hand, conclude that probable
cause exists; upon the other hand, the Prosecutor conceivably could reach the
conclusion that the evidence on hand does not warrant a finding of probable cause. In
any event, the constitutional point is that petitioner was not accorded what he was
entitled to by way of procedural due process. 27 Petitioner was forced to undergo
arraignment and literally pushed to trial without preliminary investigation, with
extraordinary haste, to the applause from the audience that filled the courtroom. If he
submitted to arraignment at trial, petitioner did so "kicking and screaming," in a manner
of speaking. During the proceedings held before the trial court on 23 August 1991, the
date set for arraignment of petitioner, and just before arraignment, counsel made very
Page 372 of 377

clear petitioner's vigorous protest and objection to the arraignment precisely because of
the denial of preliminary investigation. 28 So energetic and determined were petitioner's
counsel's protests and objections that an obviously angered court and prosecutor dared
him to withdraw or walkout, promising to replace him with counsel de oficio. During the
trial, before the prosecution called its first witness, petitioner through counsel once
again reiterated his objection to going to trial without preliminary investigation:
petitioner's counsel made of record his "continuing objection." 29 Petitioner had
promptly gone to the appellate court on certiorari and prohibition to challenge the
lawfulness of the procedure he was being forced to undergo and the lawfulness of his
detention.30 If he did not walk out on the trial, and if he cross-examined the
prosecution's witnesses, it was because he was extremely loath to be represented by
counsel de oficio selected by the trial judge, and to run the risk of being held to have
waived also his right to use what is frequently the only test of truth in the judicial
process.

In respect of the matter of bail, we similarly believe and so hold that petitioner remains
entitled to be released on bail as a matter of right. Should the evidence already of record
concerning petitioner's guilt be, in the reasonable belief of the Prosecutor, strong, the
Prosecutor may move in the trial court for cancellation of petitioner's bail. It would then
be up to the trial court, after a careful and objective assessment of the evidence on
record, to grant or deny the motion for cancellation of bail.

To reach any other conclusions here, that is, to hold that petitioner's rights to a
preliminary investigation and to bail were effectively obliterated by evidence
subsequently admitted into the record would be to legitimize the deprivation of due
process and to permit the Government to benefit from its own wrong or culpable
omission and effectively to dilute important rights of accused persons well-nigh to the
vanishing point. It may be that to require the State to accord petitioner his rights to a
preliminary investigation and to bail at this point, could turn out ultimately to be largely
a ceremonial exercise. But the Court is not compelled to speculate. And, in any case, it
would not be idle ceremony; rather, it would be a celebration by the State of the rights
and liberties of its own people and a re-affirmation of its obligation and determination
to respect those rights and liberties.

ACCORDINGLY, the Court resolved to GRANT the Petition for Review on Certiorari. The
Order of the trial court dated 17 July 1991 is hereby SET ASIDE and NULLIFIED, and
the Decision of the Court of Appeals dated 23 September 1991 hereby REVERSED.

The Office of the Provincial Prosecutor is hereby ORDERED to conduct forthwith a


preliminary investigation of the charge of murder against petitioner Go, and to complete
such preliminary investigation within a period of fifteen (15) days from commencement
Page 373 of 377

thereof. The trial on the merits of the criminal case in the Regional Trial Court shall be
SUSPENDED to await the conclusion of the preliminary investigation.

Meantime, petitioner is hereby ORDERED released forthwith upon posting of a cash bail
bond of One Hundred Thousand Pesos (P100,000.00). This release shall be without
prejudice to any lawful order that the trial court may issue, should the Office of the
Provincial Prosecutor move for cancellation of bail at the conclusion of the preliminary
investigation.

No pronouncement as to costs. This Decision is immediately executory.

SO ORDERED.

Narvasa, C.J., Bidin, Medialdea, Romero and Nocon, JJ., concur.

Separate Opinions

GUTIERREZ, JR., J., concurring:

I concur in the majority decision penned by Mr. Justice Florentino P. Feliciano but am
at a loss for reasons why an experienced Judge should insist on proceeding to trial in a
sensational murder case without preliminary investigation inspite of the vigorous and
continued objection and reservation of rights of the accused and notwithstanding the
recommendations of the Prosecutor that those rights must be respected. If the Court
had faithfully followed the Rules, trial would have proceeded smoothly and if the
accused is really guilty, then he may have been convicted by now. As it is, the case has
to go back to square one.

I agree with Justice Isagani Cruz "that the trial court has (apparently) been moved by a
desire to cater to public opinion to the detriment of the impartial administration of
justice." Mass media has its duty to fearlessly but faithfully inform the public about
events and persons. However, when a case has received wide and sensational publicity,
the trial court should be doubly careful not only to be fair and impartial but also to give
the appearance of complete objectivity in its handling of the case.

The need for a trial court to follow the Rules and to be fair, impartial, and persistent in
getting the true facts of a case is present in all cases but it is particularly important if
Page 374 of 377

the accused is indigent; more so, if he is one of those unfortunates who seem to spend
more time behind bars than outside. Unlike the accused in this case who enjoys the
assistance of competent counsel, a poor defendant convicted by wide and unfavorable
media coverage may be presumed guilty before trial and be unable to defend himself
properly. Hence, the importance of the court always following the Rules.

While concurring with Justice Feliciano's ponencia, I am constrained to add the


foregoing observations because I feel they form an integral part of the Court's decision.

CRUZ, J., concurring:


I was one of the members of the Court who initially felt that the petitioner had waived
the right to preliminary investigation because he freely participated in his trial and his
counsel even cross-examined the prosecution witnesses. A closer study of the record,
however, particularly of the transcript of the proceedings footnoted in the ponencia,
reveals that he had from the start demanded a preliminary investigation and that his
counsel had reluctantly participated in the trial only because the court threatened to
replace him with a counsel de oficio if he did not. Under the circumstances, I am
convinced that there was no waiver. The petitioner was virtually compelled to go to trial.
Such compulsion and unjustified denial of a clear statutory right of the petitioner
vitiated the proceedings as violative of procedural due process.

It is true that the ruling we lay down here will take the case back to square one, so to
speak, but that is not the petitioner's fault. He had the right to insist that the procedure
prescribed by the Rules of Court be strictly observed. The delay entailed by the
procedural lapse and the attendant expense imposed on the Government and the
defendant must be laid at the door of the trial judge for his precipitate and illegal action.

It appears that the trial court has been moved by a desire to cater to public opinion to
the detriment of the impartial administration of justice. The petitioner as portrayed by
the media is not exactly a popular person. Nevertheless, the trial court should not have
been influenced by this irrelevant consideration, remembering instead that its only
guide was the mandate of the law.

GRIÑO-AQUINO, J., dissenting:

I regret that I cannot agree with the majority opinion in this case. At this point, after
four (4) prosecution witnesses have already testified, among them an eyewitness who
identified the accused as the gunman who shot Eldon Maguan inside his car in cold
blood, and a security guard who identified the plate number of the gunman's car, I do
not believe that there is still need to conduct a preliminary investigation the sole purpose
Page 375 of 377

of which would be to ascertain if there is sufficient ground to believe that a crime was
committed (which the petitioner does not dispute) and that he (the petitioner) is probably
guilty thereof (which the prosecutor, by filing the information against him, presumably
believed to be so).

In the present stage of the presentation of the prosecution's evidence, to return the case
to the Prosecutor to conduct a preliminary investigation under Rule 112 of the 1985
Rule on Criminal Procedure would be supererogatory.

This case did not suffer from a lack of previous investigation. Diligent police work, with
ample media coverage, led to the identification of the suspect who, seven (7) days after
the shooting, appeared at the San Juan police station to verify news reports that he was
the object of a police manhunt. Upon entering the station, he was positively identified
as the gunman by an eyewitness who was being interrogated by the police to ferret more
clues and details about the crime. The police thereupon arrested the petitioner and on
the same day, July 8, 1991, promptly filed with the Provincial Prosecutor of Rizal, a
complaint for frustrated homicide against him. As the victim died the next day, July 9,
1991, before an information could be filed, the First Assistant Prosecutor, instead of
filing an information for frustrated homicide, filed an information for murder on July
11, 1991 in the Regional Trial Court, with no bail recommended.

However, the Provincial Prosecutor, acting on the petitioner's omnibus motion for
preliminary investigation and release on bail (which was erroneously filed with his office
instead of the court), recommended a cash bond of P100,000 for his release, and
submitted the omnibus motion to the trial court for resolution.

Respondent Judge Benjamin Pelayo must have realized his impetuosity shortly after he
had issued: (a) his order of July 12, 1991 approving the petitioner's cash bail bond
without a hearing, and (b) his order of July 16, 1991 granting the Prosecutor leave to
conduct a preliminary investigation, for he motu propio issued on July 17, 1991 another
order rescinding his previous orders and setting for hearing the petitioner's application
for bail.

The cases cited in page 15 of the majority opinion in support of the view that the trial of
the case should be suspended and that the prosecutor should now conduct a
preliminary investigation, are not on all fours with this case. In Doromal vs.
Sandiganbayan, 177 SCRA 354 and People vs. Monton, 23 SCRA 1024, the trial of the
criminal case had not yet commenced because motions to quash the information were
filed by the accused. Lozada vs. Hernandez, 92 Phil. 1053; U.S. vs. Banzuela, 31 Phil.
565; San Diego vs. Hernandez, 24 SCRA 110 and People vs. Oandasan, 25 SCRA 277
Page 376 of 377

are also inapplicable because in those cases preliminary investigations had in fact been
conducted before the informations were filed in court.

It should be remembered that as important as is the right of the accused to a preliminary


investigation, it is not a constitutional right. Its absence is not a ground to quash the
information (Doromal vs. Sandiganbayan, 177 SCRA 354). It does not affect the court's
jurisdiction, nor impair the validity of the information (Rodis vs. Sandiganbayan, 166
SCRA 618), nor constitute an infringement of the right of the accused to confront
witnesses (Bustos vs. Lucero, 81 Phil. 640).

The petitioner's motion for a preliminary investigation is not more important than his
application for release on bail, just as the conduct of such preliminary investigation is
not more important than the hearing of the application for bail. The court's hearing of
the application for bail should not be subordinated to the preliminary investigation of
the charge. The hearing should not be suspended, but should be allowed to proceed for
it will accomplish a double purpose. The parties will have an opportunity to show not
only: (1) whether or not there is probable cause to believe that the petitioner killed Eldon
Maguan, but more importantly (b) whether or not the evidence of his guilt is strong. The
judge's determination that the evidence of his guilt is strong would naturally foreclose
the need for a preliminary investigation to ascertain the probability of his guilt.

The bail hearing may not be suspended because upon the filing of an application for
bail by one accused of a capital offense, "the judge is under a legal obligation to receive
evidence with the view of determining whether evidence of guilt is so strong as to warrant
denial of bond." (Payao vs. Lesaca, 63 Phil. 210; Hadhirul Tahil vs. Eisma, 64 SCRA
378; Peralta vs. Ramos and Provincial Fiscal of Isabela, 71 Phil. 271; Padilla vs. Enrile,
121 SCRA 472; Ilagan vs. Ponce Enrile, 139 SCRA 349; People vs. Albofera, 152 SCRA
123)

The abolition of the death penalty did not make the right to bail absolute, for persons
charged with offenses punishable by reclusion perpetua, when evidence of guilt is
strong, are not bailable (Sec. 3, Art. III, 1987 Constitution). In People vs. Dacudao, 170
SCRA 489, we called down the trial court for having granted the motion for bail in a
murder case without any hearing and without giving the prosecution an opportunity to
comment or file objections thereto.

Similarly this Court held in People vs. Bocar, 27 SCRA 512:


Page 377 of 377

. . . due process also demands that in the matter of bail the prosecution should be
afforded full opportunity to present proof of the guilt of the accused. Thus, if it were true
that the prosecution in this case was deprived of the right to present its evidence against
the bail petition, or that the order granting such petition was issued upon incomplete
evidence, then the issuance of the order would really constitute abuse of discretion that
would call for the remedy of certiorari. (Emphasis supplied.)

The petitioner may not be released pending the hearing of his petition for bail for it
would be incongruous to grant bail to one who is not in the custody of the law (Feliciano
vs. Pasicolan, 2 SCRA 888).

I respectfully take exception to the statements in the ponencia that the "petitioner was
not arrested at all" (p. 12) and that "petitioner had not been arrested, with or without a
warrant" (p. 130). Arrest is the taking of the person into the custody in order that he
may be bound to answer for the commission of an offense (Sec. 1, Rule 113, Rules of
Court). An arrest is made by an actual restraint of the person to be arrested, or by his
submission to the custody of the person making the arrest (Sec. 2, Rule 113, Rules of
Court). When Go walked into the San Juan Police Station on July 8, 1991, and placed
himself at the disposal of the police authorities who clamped him in jail after he was
identified by an eyewitness as the person who shot Maguan, he was actually and
effectively arrested. His filing of a petition to be released on bail was a waiver of any
irregularity attending his arrest and estops him from questioning its validity (Callanta
vs. Villanueva, 77 SCRA 377; Bagcal vs. Villaraza, 120 SCRA 525).

I vote to dismiss the petition and affirm the trial court's order of July 17, 1991.

Melencio-Herrera, Paras, Padilla, Regalado and Davide, Jr., JJ., concur.

Das könnte Ihnen auch gefallen